EXAM 5

¡Supera tus tareas y exámenes ahora con Quizwiz!

Fatigue and Nutrition associated with Leukemia

Well balanced diet Small meals that require little chewing May need parental nutrition or enteral nutrition Assist in self care and mobility activities Allow rest periods

What is the average age of second permanent molar eruption in the child? 6 to 7 years 7 to 8 years 10 to 11 years 12 to 13 years

12 to 13 years Rationale Tooth eruption occurs at specific ages. Second molar eruption in the upper jaw occurs at the age of 12 to 13 years old. First molar eruption occurs between 6 and 7 years of age. Central incisor eruption occurs between 7 and 8 years of age. First bicuspid eruption occurs between 10 and 11 years of age.

mumps

12-15 mo no need before 12 m because of moms antibodies second dose 4-6yo along with MMR

rubella

12-15 mo second dose 4-6 yo not to become pregnant 28 days after vaccine because attenuates live vaccine can cross placenta

measles

12-15mo second dose 4-6 yo attenuates live vaccine not to give to HIV kids vit A - effective in decresing morbidity and mortality of measles

a particular area of concern for the adolescent receiving radiotherapy is: a. frequent vomiting b. altered sexual function c. high risk for sterility d. precocious puberty

c. high risk for sterility

in an attempt to prevent central nervous system invasion of malignant cells, children with leukemia usually receive prophylactic: a. cranial spinal irridation b. intravenous steroid therapy c. intrathecal chemotherapy d. intravenous methotrexate and cytrabine

c. intrathecal chemotherapy

which one of the following does not contribute to sleep disturbances during preschool years? a. evening media use b. inconsistent bedtime routines c.growing pains d. nighttime fears

c.growing pains

general guidelines for emergency poison treatment

assess pt stop exposure to poison call poison control to identify poison and seek advice

types of gastric decontaminatin

ipecac, charcoal, gastric lavage routinely not recommended ( complications - GI perforation, hypoxia, aspiration; little help after 1 h; requires protected airways, sedation, largest diameter tube

heavy metal poisoning

leadm mercury ( inhalation)

incest

any physical sexual activity between family members

complications of cancer therapy

tumor lysis syndrome hyperleukocytosis superior vena cava syndrome spinal cord compression disseminated intravascular coagulation

airborn precaution

type of precaution used if droplet is =<5 mm type of precautions using AIIR (airborn infection isolation rooM) rooms, N95 masks b/c of measles, varicella, TB

targeted therapies

interfere with specific molecules involved in cancer 1. act on molecular targets 2. selected and designed to affect their target, compared with chemotherapy drugs

HAI preventable

meticulous hand washing cleaning disposable techniques

bone marrow test

used to determine the presence or absence or tumor or response to therapy in this specific location

Postoperative interventions (Wilms' tumor)

Monitor temp and bp closely Monitor for signs of hemorhage and infection Monitor strict intake and output Monitor for abdominal distention monitor bowel sounds nd other signs of GI activity because of the risk for intestinal obstruction

Preoperative interventions (Wilms' tumor)

Monitor vital signs, particularly BLOOD PRESSURE AVOID PALPATION OF THE ABDOMEN Measure abdominal girth once daily

pneumococcal disease

PCV13 administered at 2,4,6,12-15 mo PPSV23 not under 24 mo

Neuroblastoma

Tumor that originates form the embryonic neural crest cells that normally give rise to the adrenal medulla and the sympathetic ganglia

a child with hodgkin disease who has lesions in both the left and right supraclaviluar area, the mediastinum, and the lungs would be classified as: a. stage 1 b. stage 2 c. stage 3 d. stage 4

b. stage 2

Most children present with neuroblastoma before age __

10 years

peak age SIDS

2-3 months

the severe cellular damage that is caused by chemotherapy drugs infiltrating into surrounding tissue occurs when the chemotherapeutic agent is a: a. hormone b. steroid c. vesicant d. antimetabolite

c. vesicant

middle childhood represents a. ages 5-13 b. 4-14 c. 6-12 d. 6-16

c. 6-12

polio

4 doses - 2,4,6-18m, 4-6 yo

Pertussis

Cough occurs at night, and inspirations sound like crowing.

Infection associated with Leukemia

Infection can occur through self-contamination or cross-contamination The most common sites for infection are the skin, respiratory tract, and GI tract

Hospital-acquired infection (HIA)

Infections occur when there is interaction among patients, health care personnel, equipment, and bacteria.

Universal Precautions

Blood and body fluid precautions designed to reduce the risk of transmission of blood-borne pathogens

Treatment of osteosarcoma

Surgical resection (limb salvage procedure) Amputation Chemotherapy

Osteosarcoma

The most common bone cancer in children; also known as osteogenic sarcoma

prevent spread

prevention of the desease ( exclusively immunization) control of its spread if child is in the hospital with undiagnosed exanthema, institute strict transmission-ased precautions (contact, airborn, droplet) and standard precaution

Toxoid

Modified bacterial toxin that has been made nontoxic but retains its ability to stimulate the formation of antitoxin

leukemia

a broad term given to a group of malignant diseases of the bone marrow and lymphatic system

the most common bone cancer in children is: a. ewing b. osteosarcoma

b. osteosarcoma

children who receive induction therapy for acute lymphocytic leukemia are likely to a. receive cyclophosphamide, cytarabine, and mercaptopurine b. receive vincristine and prednisone, l-asparaginase c. receive a bone marrow transplant d. relapse 5 years after a complete remission

b. receive vincristine and prednisone, l-asparaginase

what are the 3 transmission based precautions

airborne droplet contract

Checkpoint inhibitors

block pathways that allow cancer cells to escape the immune system

which of the following parental behavioral responses should alert the nurse to the possibility of maltreatment of the child a. parent displays extreme care for the child not wanting to leave the child's side b. parent dispays signs of guilt, not being able to eat or sleep c. parents displays abnormal interest in the incident going over each detail repeatedly d. parent displays anger at the child for being injured

d. parent displays anger at the child for being injured

The condition that often occurs when an infant is placed supine which results in an oblique or asymmetric head is: a. microcephaly. b. cranisosynostosis. c. hydrocephaly. d. postitional plagiocephaly.

d. postitional plagiocephaly.

The general contraindication for all immunizations is: a. minor illness such as common cold. b. breastfeeding. c. pregnancy. d. severe febrile illness

d. severe febrile illness.

the surgical technique that used computed tomography and magnetic resonance imaging is called a. sclerotherapy b. microsurgery c. laser surgery d. stereotactic surgery

d. stereotactic surgery

reorganization after the death of a child means that the a. loved one is forgotten b. pain is gone c. survivors have "let go" d. survivors have recovered from their loss

d. survivors have recovered fromtheir loss

Hyperleukocytosis

defigned as a peripheral WBC count greater than 100,000 mm, leading to: -capillary obstruction -microinfarction -organ dysfunction What does hyperleukocytosis cause? -respiratory distress -cyanosis -altered LOC -visual disturbance -confusion; delirium Tx for hyperleukocytosis: -rapid cytoreduction with chemotherapy -hydration urinary alkalization -allopurinlol

physical abuse

deliberate infliction of physical injury on a child

physical neglect

deprivation of necessities such as food and clothing

in relation to body image school age children a. are not aware of physical disabilities in others b. pay little attn to their own body capabilities c. seldom express concerns about their bodies to their families d. do not model themselves after their parents or compare themselves with images observed in the media

c. seldom express concerns about their bodies to their families

Standard Precautions involve the use of barrier protection to prevent contamination from: a. blood. b. body fluids. c. mucous membranes. d. all of the above.

d. all of the above.

Gram-negative bacteria

examples are pseudomonas aeruginosa escherichia coli klebsiella

nightmares

scary dream taking place during REM sleep elicits crying and fright in children even when awake return to sleep delayed because of persistent fear

Which sleeping disorder is appropriately described as the child appears awake but not aware of people in the near vicinity, cannot be comforted, and pushes away anyone who tries to hold or restrain the child? Nightmares Night waking Sleep terrors Delayed sleep onset

sleep terrors

prescool

some understanding of death might see it as sleeping, temporary and reversible may see illness as punishment may do regression helpful to minimize separation from parents

factors to reduce risk of SIDS

supine back position ( including preterms and infants with GERD) room sharing with parents, but not cosleeping

abdominal mass

typical finding in children with wilms tumor and neuroblastoma

A school-aged child with a seizure disorder asks the nurse, "Will I have this for a long time?" What is the nurse's best response? "All chronic illnesses are lifelong conditions." "It just depends. Hopefully your disorder will go away as you get older." "A seizure disorder is a chronic illness, which means it will require long-term care." "Seizures are sometimes just a one-time occurrence. You will likely not have to deal with seizures your entire life."

"A seizure disorder is a chronic illness, which means it will require long-term care." Any chronic illness will require long-term care, although the severity of the illness can vary over time.

A parent of a six- and four-year-old, the youngest one with juvenile arthritis, states, "I feel terrible when I have to discipline the youngest one." What is the nurse's best response? "A child with a chronic illness should not be disciplined because it causes unnecessary stress." "Permissive parenting of children with a chronic illness can cause long-term damage to the child." "Although disciplining a child with a chronic illness can be hard, it is important for the development of the child." "It is important that your older child sees that you still discipline the four-year-old so that there is not any resentment."

"Although disciplining a child with a chronic illness can be hard, it is important for the development of the child." This is the best response since it is important for children to have discipline in order to maintain age-appropriate expectations and development.

The student nurse, under the guidance of a registered nurse, is explaining organ donation to the family of a child receiving end-of-life care. Which statement made by the student nurse is correct? "Any number of organs can be donated." "Removal of organs might disfigure the body." "Funeral service might be delayed due to organ donation." "Organ donation can be done without the family's consent."

"Any number of organs can be donated." Rationale Any number of organs or body tissues can be donated. The skin, kidneys, liver, heart, bones, corneas, and pancreas can be donated. The removal of organs from the body does not mutilate or disfigure the body and there will not be a delay of the funeral service. Written consent from the family is always necessary before organ donation.

The nurse teaches a patient about brain structure and function. Which statement by the nurse is true regarding the child's brain anatomy and physiology? "Cerebral spinal fluid reduces injury to the brain in the case of a fall." "The subarachnoid space maintains a constant pressure of the cerebral spinal fluid." "The epidural space contains a blood brain barrier to reduce pain signals that reach the brain." "The blood brain barrier (BBB) prevents harmful medications from entering the brain."

"Cerebral spinal fluid reduces injury to the brain in the case of a fall." One of the functions of CSF is to reduce trauma to the brain by providing a cushion.

The registered nurse is teaching a student nurse about how parents should tell a child that the child is terminally ill. Which statement made by the student nurse requires correction? "Allow the child's questions to guide the discussion." "Encourage the parents to approach the discussion gently." "Encourage the parents not to speak openly about death." "Use nonthreatening examples to begin the conversation with the child."

"Encourage the parents not to speak openly about death." Rationale The nurse should encourage the parents and members of the child's family to discuss the child's condition and impending death openly. The nurse should advise the parents to start with basic information and let the child's questions lead the discussion further. The discussion should be approached gently, without showing aggression or anxiety. The nurse should encourage the parents to start with nonthreatening examples, such as the life span of houseflies.

The nurse is caring for a patient two weeks after surgery for Ewing sarcoma. Which statement by the patient requires an immediate action by the nurse? "I feel like only half a person." "I can still feel my amputated leg!" "Although my leg hurt worse last week, the pain is not entirely gone." "Although I am able to care for myself, I am not able to get around as well as I used to."

"I feel like only half a person." An important outcome for patients with bone and muscle tumors is having a positive body image, as evidenced the ability to adapt to the new appearance. The patient's comment suggests that this adaptation is not occurring. This would require immediate action by the nurse before the patient becomes depressed or socially withdrawn

A nurse is providing general guidelines and teaching to a group of parents of 10-year-old children on how to best help their children in school. What statement made by a parent indicates effective learning? "I should encourage my child to ask questions." "I should immediately point out whenever my child makes a mistake." "I must encourage my child to focus on schoolwork, because too many hobbies can be overwhelming." "I must arrange for activities that occupy the whole of my child's free time so that my child does not get bored."

"I should encourage my child to ask questions." Rationale Parents should encourage their child to ask questions so that the child can discover sources for information or places to explore and investigate. The parents should not be quick to chastise children for their mistakes; children should be encouraged to try out new solutions to problems without the fear of making mistakes, because this stimulates creative thinking and problem solving. While pursuing too many hobbies can definitely be stressful for the child, the parents must also foster the development of hobbies and collections by their children and not just make them focus on schoolwork. Parents should encourage their children to wonder and reflect during free time.

The registered nurse is educating trainee nurses about strategies that will help them reduce stress when caring for a dying child. Which statement made by a trainee nurse working in the cancer unit indicates effective understanding of the information presented? "I should remain detached from the child." "I should visit the child on off days if I want to." "I should not take time off or go on vacation to relieve stress." "I should organize a staff remembrance service to share my feelings."

"I should organize a staff remembrance service to share my feelings." Rationale The nurse should organize staff remembrance services to share personal feelings and experiences with the team and reduce stress. Many nurses think that professionals must remain detached while caring for a dying child. However, involvement allows nurses to understand patients' feelings. The nurse should not visit the child on off days. The nurse should take time off or go on vacation if required to reduce stress while caring for a dying child.

The nurse is assessing a terminally ill adolescent. Which statement regarding death does the nurse anticipate from this patient? "I will not die at a young age." "I know death is a type of sleep." "I am afraid because I know death is irreversible." "I know I would return in a few months if I died."

"I will not die at a young age." Rationale Adolescents have a clear picture of what death is. Even though they have a mature understanding, they usually think they will not die at a young age. Preschoolers see death as a type of sleep. School-age preadolescent children understand that death is universal and irreversible. Infants and toddlers have a vague separation between facts and fantasy, which makes the concept of death difficult to understand.

Parents of a 12-year-old child with cancer ask the nurse about the policy on discussions of death with their child. Which is the most appropriate response by the nurse? "I will not initiate any discussion regarding dying." "I will be contacting the ethics committee because your request is unethical." "I will not lie to the child as lying may harm our nurse-patient relationship." "My Nursing Code of Ethics will not allow me to be dishonest with the child."

"I will not lie to the child as lying may harm our nurse-patient relationship." The needs of the patient come first so this response provides the parents with an honest response regarding their question.

The mother of a young child diagnosed with neuroblastoma asks the nurse how this could have happened to her baby. Which is the best response from the nurse to explain the etiology of neuroblastoma? "It could have resulted from your mother's work in the chemical plant." "It could be possibly associated with your previous work as a farmer." "It could have resulted from exposure of your infant to electronic waves shortly after birth." "There are no known causative factors. Development of disease is completely random."

"It could be possibly associated with your previous work as a farmer." Maternal exposure to certain environmental sources (e.g., farm products) is possibly associated with the development of neuroblastoma in children.

A 6-year-old child's parent tells the nurse that the child has a loose tooth. The parents are concerned about the child having a loose tooth. What is the appropriate response by the nurse? "The child has a chronic disease." "It is a normal finding at this age." "The child has calcium deficiency." "It occurs due to vitamin A deficiency."

"It is a normal finding at this age." Rationale At the middle childhood stage, the process of shedding deciduous teeth begins. The nurse tells the child's parent that a loose tooth is a normal finding at this age. It does not indicate that the child has chronic disease, calcium deficiency, or vitamin A deficiency. Calcium deficiency affects the bones and does not cause tooth shedding. Vitamin A deficiency does not affect teeth or cause tooth loss.

colony stimulating factors

a family of glycoprotein hormones that regulate the reproduction, maturation, and function of blood cells; used as a supportive measure to prevent the side effects caused by low blood counts

according to freud, middle childhood is described as which of the following periods? a. anal b. latency c. oral d. oedipal

b. latency

corrosives clinical manifestations

coughing, hemoptysis, drooling and inability to clear secretions, severe burning pain in mouth, throat, and stomach; signs of shock, white swollen mucus membranes, edema of the lips, tongue, and pharynx

absolute neutrophil count

if lower than 500, risk for infection and major complications

munchausen syndrome by proxy

illness that one person fabricates or induces in another person

Special needs of the family facing the unexpected death of a child

include support while awaiting news of the child's status; a sensitive pronouncement of death; acknowledgment of feelings of denial, guilt, and anger; an opportunity to view the body; and referrals for support.

major cause of death in infancy

injuries

Supporting the child terminal illness

involves encouraging self-expression, alleviating feelings of being different, and strengthening the child's self-image. Alleviating the child's feeling of being different and normalizing his or her life as much as possible is one of the most important interventions nurses can provide.

Preadolescence

is the period that begins toward the end of middle childhood and ends with the 13th birthday. Puberty signals the beginning of the development of secondary sex characteristics, and prepubescence, the 2-year period that precedes puberty, typically occurs during preadolescence. A major task during the middle school years is developing a sense of industry or accomplishment, according to Erikson. School-age children are eager to develop skills and participate in meaningful and socially useful work. Children gain a great deal of satisfaction from independent behavior in exploring and manipulating their environment and from interaction with peers. Piaget's period of concrete operations occurs between the ages of 7 and 11 years. During this stage, children develop an understanding of relationships between things and ideas. They progress from making judgments based on what they see (perceptual thinking) to making judgments based on what they reason (conceptual thinking). School-age children develop a conscience and are able to understand and adhere to rules and standards set by others. Older school-age children are able to judge an act by the intentions that prompted it rather than just by the consequences. Rules and judgments become less absolute and authoritarian and instead begin to be founded more on the needs and desires of others. Entertaining different points of view, becoming sensitive to social norms, and forming peer friendships are important features of social development during the school years. Children enter middle childhood with remarkably efficient language skills, but they make many important linguistic achievements during the school-age years. During the elementary school years, they learn to correct previous syntactic errors and begin to use more complex grammatical forms, such as correct past tenses for irregular verbs, correct plurals for irregular nouns, and correct personal pronouns. Identification and association with the peer group are essential to a child's socialization. Cooperative play, team activities, and the acquisition of skills are prime elements of play during the school years, and rules and rituals assume greater importance. Relationships with age mates provide the most important social interactions for school-age children. For the first time, children are able to join in group activities with unrestrained enthusiasm and steady participation. Previously, interactions were limited to short periods under considerable adult supervision. One of the outstanding characteristics of middle childhood is the formation of formalized groups or clubs. Membership in groups provides children with a comfortable place in society. The groups often impose rigid rules on the members, and conforming to these rules provides children with feelings of security. Closely associated with developing a sense of industry is developing a concept of one's value and worth. With the emphasis on skill building and broadened social relationships, children are continually occupied in the process of self-evaluation. Self-concept refers to a conscious awareness of a variety of self-perceptions, such as one's physical characteristics, abilities, values, and self-ideals and one's idea of self in relation to others.

Which heavy metal is appropriate when considering a commonly ingested toxic substance? Iron Lead Silver Mercury

lead Rationale Lead is a heavy metal that is a commonly ingested toxic substance. Mercury toxicity is a rare form of heavy metal poisoning. Iron and silver are not among the commonly ingested toxins.

rotavirus

leading cause of severe diarrhea transmitted fecal-oral route RotaTeq® (RV5) is given in 3 doses at ages 2 months, 4 months, and 6 months. Rotarix® (RV1) is given in 2 doses at ages 2 months and 4 months. oral vaccine

skin assessment in cancer

may show signs of low platelet count, ecchymosis, petichiae

To counsel families and children regarding death

nurses need to understand children's perceptions of death, the fears in each age-group, and personal meanings of death and bereavement during developmental stages.

Ewing sarcomas

or the Ewing sarcoma family of tumors (which includes primitive neuroectodermal tumor of the bone), are the second most common malignant bone tumor (after osteosarcoma) in childhood. Ewing sarcoma arises in the marrow spaces of the bone rather than from osseous tissue, and treatment includes radiation and chemotherapy

3 strategies for providers to manage differences between parents and child preferences

parental decision - deference strategy child preference - advocative strategy differences between parents and children - arbitrative strategy

bone tumors

produce localized pain in the affected site, which may be severe or dull and may be attributed to trauma or the vague complaint of "growing pains." The pain is often relieved by a flexed position, which relaxes the muscles overlying the stretched periosteum. Frequently, it draws attention when the child limps, curtails physical activity, or is unable to hold heavy objects.

principle in dealing with a poisoning

treat the child first, not the poison. This requires an immediate concern for life support. The major principles are assessment (including vital signs), airway and cardiovascular supportive measures, minimization of poison absorption, prevention of complications, family support, and prevention of recurrence.

sexual abuse

use, persuasion, coercion of any child to engage in sexually explicit content

diagnostics of lead poisoning

venous blood sample of 5 mcg/dl or higher screening of lead poisoning at 1 and 2 years

pertussis

2 months, 4 months, 6 months, 12-15 months, 4-6 yrs When do we give DTap immunizations? 2 forms of pertussis vaccine - purified acellular vaccine has less reaction than whole cell vaccine first 3 vaccine from same manufacturer clinical manifestation of pertussiss - gagging and gasping, followed by emesis, apnea, cyanosis, typical whoop is absent

What statement is characteristic of type 1 diabetes mellitus? a. Onset is usually gradual. b. Ketoacidosis is infrequent. c. Peak age incidence is 10 to 15 years. d. Oral agents are available for treatment.

ANS: C Type 1 diabetes mellitus typically usually has its onset before the age of 20 years, with a peak incidence between ages 10 and 15 years. Type 1 has an abrupt onset, in contrast to type 2, which has a more gradual appearance. Ketoacidosis occurs when insulin is unavailable and the body uses sources other than glucose for cellular metabolism. Ketoacidosis is more common in type 1 diabetes than in type 2. At this time, oral agents are available only for type 2 diabetes.

The nurse is taking care of a child who had a thyroidectomy. The nurse recognizes what as a positive Chvostek sign? a. Paresthesia occurring in feet and toes b. Frequent sharp flexion of wrist and ankle joints c. Carpal spasm elicited by pressure applied to the nerves of the upper arm d. Facial muscle spasm elicited by tapping the facial nerve in the region of the parotid gland

ANS: D A positive Chvostek sign is a facial muscle spasm that is elicited by tapping the facial nerve in the region of the parotid gland. Paresthesia occurring in the feet and toes and frequent sharp flexion of the wrist and ankle joints can be signs of hypoparathyroidism but are not part of a positive Chvostek sign. Carpal spasm elicited by pressure applied to nerves of the upper arm is called a positive Trousseau sign.

The nurse suspects that a child has ingested some type of poison. What clinical manifestation would be most suggestive that the poison was a corrosive product? a. Tinnitus b. Disorientation c. Stupor, lethargy, and coma d. Edema of the lips, tongue, and pharynx

ANS: D Edema of the lips, tongue, and pharynx indicates a corrosive ingestion. Tinnitus is indicative of aspirin ingestion. Corrosives do not act on the central nervous system.

What clinical manifestation is considered a cardinal sign of diabetes mellitus? a. Nausea b. Seizures c. Impaired vision d. Frequent urination

ANS: D Hallmarks of diabetes mellitus are glycosuria, polyuria, and polydipsia. Nausea and seizures are not clinical manifestations of diabetes mellitus. Impaired vision is a long-term complication of the disease.

What do inflicted immersion burns often appear as? a. Partial-thickness, asymmetrical burns b. Splash pattern burns on hands or feet c. Any splash burn with dry linear marks d. Sharply demarcated, symmetrical burns

ANS: D Immersion burns are sharply demarcated symmetrical burns. Asymmetrical burns and splash burns are often accidental.

The nurse outlines short- and long-term goals for a 10-year-old child with many complex health problems. Who should agree on these goals? a. Family and nurse b. Child, family, and nurse c. All professionals involved d. Child, family, and all professionals involved

ANS: D In the home, the family is a partner in each step of the nursing process. The family priorities should guide the planning process. Both short- and long-term goals should be outlined and agreed on by the child, family, and professionals involved. Elimination of any one of these groups can potentially create a care plan that does not meet the needs of the child and family.

A child will start treatment for central precocious puberty. What synthetic hormone will be injected? a. Thyrotropin b. Gonadotropins c. Somatotropic hormone d. Luteinizing hormone-releasing hormone

ANS: D Precocious puberty of central origin is treated with monthly subcutaneous injections of luteinizing hormone-releasing hormone, which regulates pituitary secretions. Thyrotropin, gonadotropins, and somatotropic hormone are not the appropriate therapies for precocious puberty.

Secondary prevention for cognitive impairment includes what activity? a. Genetic counseling b. Avoidance of prenatal rubella infection c. Preschool education and counseling services d. Newborn screening for treatable inborn errors of metabolism

ANS: D Secondary prevention involves activities that are designed to identify the condition early and initiate treatment to avert cerebral damage. Inborn errors of metabolism such as hypothyroidism, phenylketonuria, and galactosemia can cause cognitive impairment. Genetic counseling and avoidance of prenatal rubella infections are examples of primary prevention strategies to preclude the occurrence of disorders that can cause cognitive impairment. Preschool education and counseling services are examples of tertiary prevention. These are designed to include early identification of conditions and provision of appropriate therapies and rehabilitation services.

The nurse is discussing with a child and family the various sites used for insulin injections. What site usually has the fastest rate of absorption? a. Arm b. Leg c. Buttock d. Abdomen

ANS: D The abdomen has the fastest rate of absorption but the shortest duration. The arm has a fast rate of absorption but a short duration. The leg has a slow rate of absorption but a long duration. The buttock has the slowest rate of absorption and the longest duration.

When only one child is abused in a family, the abuse is usually a result of what? a. The child is the firstborn. b. The child is the same gender as the abusing parent. c. The parent abuses the child to avoid showing favoritism. d. The parent is unable to deal with the child's behavioral style.

ANS: D The child unintentionally contributes to the abuse. The "fit" or compatibility between the child's temperament and the parent's ability to deal with that behavior style is an important predictor. Birth order and gender can contribute to abuse, but there is not a specific birth order or gender relationship that is indicative of abuse. Being the firstborn or the same gender as the abuser is not linked to child abuse. Avoidance of favoritism is not usually a cause of abuse.

An immunocompromised child has been exposed to chickenpox. What should the nurse anticipate to be prescribed to the exposed child? a. Acyclovir (Zovirax) b. Valacyclovir (Valtrex) c. Amantadine (Symmetrel) d. Varicella-zoster immune globulin

ANS: D The use of varicella-zoster immune globulin or immune globulin intravenous (IGIV) is recommended for children who are immunocompromised, who have no previous history of varicella, and who are likely to contract the disease and have complications as a result. The antiviral agent acyclovir (Zovirax) or valacyclovir may be used to treat varicella infections in susceptible immunocompromised persons. It is effective in decreasing the number of lesions; shortening the duration of fever; and decreasing itching, lethargy, and anorexia. Symmetrel is an antiviral used to treat influenza.

Total-body irradiation is indicated for what reason? a. Palliative care b. Lymphoma therapy c. Definitive therapy for leukemia d. Preparation for bone marrow transplant

ANS: D Total-body irradiation is used as part of the destruction of the child's immune system necessary for a bone marrow transplant. The child is at great risk for complications because there is no supportive therapy until engraftment of the donor marrow takes place. Irradiation for palliative care is done selectively. The area that is causing pain or potential obstruction is irradiated. Lymphoma and leukemia are treated through a combination of modalities. Total-body irradiation is not indicated.

The camp nurse is choosing a toy for a child with cognitive impairment to play with during swimming time. What toy should the nurse choose to encourage improvement of developmental skills? a. Dive rings b. An inner tube c. Floating ducks d. A large beach ball

ANS: D Toys are selected for their recreational and educational value. For example, a large inflatable beach ball is a good water toy; encourages interactive play; and can be used to learn motor skills such as balance, rocking, kicking, and throwing. Dive rings, an inner tube, and floating ducks are not interactive toys.

Which factor determines the initial treatment of a neuroblastoma? Child's mental health Presence of an infection Accurate clinical staging Type of health insurance

Accurate clinical staging Rationale The initial treatment of a neuroblastoma is determined with accurate clinical staging. Surgery is used both to remove as much of the tumor as possible and to obtain biopsies. Treatment will depend on the clinical stage of the cancer. The child's mental health, presence of an infection, or type of health insurance is not associated with the treatment of a neuroblastoma.

The nurse is caring for a child in hospice. While listening to the child's breathing, the nurse observes "death rattle." Which intervention is appropriate? Administer an opioid medication. Administer an anticholinergic medication. Administer an antihistamine medication. Provide a nonpharmacologic intervention.

Administer an anticholinergic medication. Rationale Hospice is a health care organization for caring for terminally ill patients. Noisy chest sounds, known as death rattle, are common in patients at the end of life due to the accumulation of fluid in the pulmonary cavity. Anticholinergic drugs are effective in reducing such noisy chest sounds by preventing the secretions from oozing from the cavities. Opioid medication is administered for treating severe pain in a dying patient but will not reduce death rattle. Diphenhydramine can help manage constipation-related problems, such as nausea and vomiting. Nonpharmacologic interventions are the choice of the patient or family members, though they are not specifically intended to reduce death rattle. p. 619

The nurse is caring for a pediatric patient admitted with seizure activity related to cerebral palsy (CP). Which interventions should the nurse perform immediately? Select all that apply. Administer intravenous diazepam (Valium). Institute safety measures such as seizure padding. Assess gait disturbances and muscle coordination. Consult speech therapy for a swallowing evaluation. Evaluate gross motor development and muscle tone.

Administer intravenous diazepam (Valium). Administering a benzodiazepine will decrease spasticity and seizure activity. This is an immediate need. Institute safety measures such as seizure padding. Safety is a primary goal, and the nurse must prevent the child from sustaining unnecessary injuries.

An 18-year-old patient with human immunodeficiency virus (HIV) presents to the clinic for a second meningococcal vaccine. The nurse notes that the patient received the first Menactra vaccine nine weeks ago. Which is most appropriate nursing action? Tell the patient that the meningococcal vaccine only requires one shot. Ask the patient to return in 7 weeks for the next dose. Administer the meningococcal vaccine immediately. Schedule an appointment for the final dose next year.

Administer the meningococcal vaccine immediately. The nurse should administer the second dose of the meningococcal vaccine (Menactra) to this high-risk patient. Adolescents aged 11 through 18 years with human immunodeficiency virus (HIV) infection should receive a two-dose primary series of Menactra or Menveo with at least eight weeks between doses. The patient received the first dose nine weeks ago and the second dose may be given now.

A child diagnosed with leukemia reports nausea, vomiting, and anorexia related to chemotherapy treatment. The nurse notes a hemoglobin count of 10, WBC count of 8, platelet count of 103,000, and an albumin level of 2.1. Which prescription would the nurse anticipate? Decrease dietary protein Administer PO allopurinol Administer platelet transfusion Administer total parenteral nutrition

Administer total parenteral nutrition An albumin level of 2.1 indicates malnutrition. The nurse would administer total parenteral nutrition via a central line to improve the child's nutritional status.

A child is brought to the emergency department with a suspected spinal cord injury at the level of C2. What is the immediate priority in the nursing care of a patient with this injury? Administer ventilatory support Assess child's visual field for spots Manage hypertension and bradycardia Provide permanent cervical stabilization

Administer ventilatory support A spinal cord injury at the level of C2 will cause the patient to be apneic and require ventilator support which is the immediate priority in the nursing care plan in this patient.

Which effect does increased lead absorption have on a patient? Anemia Rise in vitamin D levels Decrease in intracranial pressure Disintegration of alternative metabolites

Anemia Rationale Increased lead absorption leads to a rise in erythrocyte protoporphyrin levels, causing anemia. Lead affects the renal system by damaging the proximal tubules leading to glycosuria, proteinuria, and ketonuria, resulting in a fall in vitamin D levels. An increase in lead absorption affects the neurologic system by increasing the membrane permeability, thereby ensuing in a rise in intracranial pressure. Lead affects the hematologic system by interfering with the synthesis of heme, leading to an accumulation of alternative metabolites.

Which are the most common signs and symptoms of leukemia related to bone marrow involvement? Anemia, infection, bleeding Headache, papilledema, irritability Muscle wasting, weight loss, fatigue Decreased intracranial pressure, psychosis, confusion

Anemia, infection, bleeding Rationale Anemia, infection, and bleeding resulting from decreased platelet production are signs of infiltration of the bone marrow. Petechiae occur as a result of a lowered platelet count, infection occurs because of the depressed number of effective leukocytes, and fatigue occurs as a result of the anemia. Headache, papilledema, and irritability are not signs of bone marrow involvement. Muscle wasting, weight loss, and fatigue are not signs of bone marrow involvement. Decreased intracranial pressure, psychosis, and confusion are not signs of bone marrow involvement.

The nurse is caring for a child who is about to undergo enucleation for retinoblastoma. After surgery, what is the nurse's priority action? Apply an eye patch Fit the patient for a prosthesis Provide caregiver teaching on proper care for the eye Schedule a follow-up retinal examination for the patient

Apply an eye patch Immediately at post-op, the nurse should apply the eye patch over the affected eye. The patch will remain in place for one week.

The school nurse is teaching bicycle safety to a group of school-age children. What should the nurse include in the session? (Select all that apply.) a. Ride double file when possible. b. Watch for and yield to pedestrians. c. Only ride double with someone your own size. d. Ride bicycles with traffic away from parked cars. e. Keep both hands on the handlebars except when signaling.

B, D, E Bicycle safety includes watching for and yielding to pedestrians, riding bicycles with traffic away from parked cars, and keeping both hands on handlebars except when signaling. It is best to ride single file, not double file, and never to ride double on a bicycle.

A two-year-child has been recently diagnosed with retinoblastoma and undergoes enucleation of the right eye. Genetic testing on the child reveals the patient has a familial type of retinoblastoma. Why might this child need a bone scan later in life? To identify retinoblastoma seeding To follow long-term response to treatment To assess for recurrence of the retinoblastoma Because familial retinoblastoma is associated with osteosarcoma

Because familial retinoblastoma is associated with osteosarcoma Familial retinoblastoma is associated with osteosarcoma. The child would need a bone scan to assess for development of bone disease.

Which test is essential for the definitive diagnosis of leukemia? Blood culture Platelet count Complete blood count Bone marrow aspiration

Bone marrow aspiration Rationale A bone marrow aspiration is not a routine hematologic test but is essential for definitive diagnosis of leukemia. A blood culture is not essential for diagnosing leukemia. The platelet count is part of a complete blood count, which indicates the number of fragments in the body that help clotting to occur. A complete blood count is a routine test that helps to identify a potential problem.

What would the nurse include when giving parents guidelines about helping their children in school? Punish children who fail to perform adequately. Help children as much as possible with their homework. Communicate with teachers if there appears to be a problem. Accept responsibility for children's successes and failures.

Communicate with teachers if there appears to be a problem. Correct Communication between the parent and teachers is essential. It demonstrates that parents consider school important and that education is a shared responsibility. Excessive pressure or lack of encouragement from parents can inhibit the development of the child's maximum potential. Children need to be responsible for their schoolwork. By being responsible, children learn to meet deadlines and be successful.

A four-year-old patient receiving chemotherapy reports gum inflammation, nausea, and a burning sensation in the stomach. Which other symptom might the nurse expect? Hair loss Polyphagia Mood swings Excessive thirst

Hair loss Gum inflammation, nausea and gastroesophageal reflux disease (GERD) are side effects of chemotherapy treatment. Alopecia, or hair loss, is also a side effect of chemotherapy.

Standard Precautions

Interventions that synthesize the major features of universal (blood and body fluid) precautions and body substance isolation; involve the use of barrier protection; designed for the care of all patients to reduce the risk for transmission of microorganism from both recognized and unrecognized sources of infection

Direct contact transmission

Involves skin-to-skin contact and physical transfer of microorganisms to a susceptible host from an infected or colonized person

Which finding will the nurse expect to see in a 5-year-old child whom the mother suspects to have ingested the acetaminophen 3 hours ago? Jaundice Malaise, pallor, weakness Right upper quadrant (RUQ) pain Recovery from physical symptoms

Malaise, pallor, weakness During the first 24 hours, the nurse would expect to see malaise, nausea, vomiting, sweating, pallor, and weakness.

Warning for chemotherapy

Monitor a child receiving chemotherapy closely for signs of infection. Infection is a major cause of death in the immunosuppressed child

Infratentorial tumor

Most common brain tumor, located in the posterior third of the brain (primarily in the cerebellum or brainstem) and accounts for the frequency of symptoms resulting from increased intracranial pressure

Combination vaccine

Multiple vaccines into one parenteral form

Which medication is appropriate to administer as an antidote for acetaminophen overdose? Naloxone Vitamin K N-acetylcysteine Activated charcoal

N-acetylcysteine

Which substance is appropriate to administer to prevent further effects of acetaminophen overdose? Naloxone Flumazenil N-acetylcysteine Activated charcoal

N-acetylcysteine Rationale N-acetylcysteine is the antidote for acetaminophen. It blocks the activity of acetaminophen and forms a complex preventing its absorption. N-acetylcysteine is given orally by diluting it in fruit juice or soda because of its offensive odor. It is given as one loading dose and usually 17 maintenance doses in different dosages. Naloxone is an antidote for opioid overdose. Flumazenil is an antidote for benzodiazepines. Activated charcoal is not helpful in this situation; it is used to treat aspirin poisoning.

The nurse recognizes that bullying is a common occurrence during the school-age years. How are the victims of bullies affected? Select all that apply. Nightmares Social isolation Increased self-esteem Psychologic distress Increased self-harm behaviors

Nightmares Social isolation Psychologic distress Increased self-harm behaviors Rationale Bullying can cause victims to suffer psychologic distress, increased self-harm behaviors, social isolation, nightmares, depression, anxiety, worry, sadness, and violent behaviors. Increased self-esteem is not correlated with being a victim of a bully

Which characteristics differentiate nightmares and sleep terrors? Nightmares tend to result in no memory of the dream, and sleep terrors tend to result in description of the dream. Nightmares tend to result in a rapid return to sleep, and sleep terrors tend to result in a delay to sleep because of persistent fear. Nightmares tend to occur during the first half of the night; sleep terrors generally occur 5 or 6 hours after the child falls asleep. Nightmares take place during rapid eye movement sleep and are followed by full waking, whereas sleep terrors are a partial arousal from a very deep sleep.

Nightmares take place during rapid eye movement sleep and are followed by full waking, whereas sleep terrors are a partial arousal from a very deep sleep. Rationale Nightmares take place during rapid eye movement sleep and are followed by full waking, whereas sleep terrors are a partial arousal from very deep sleep (stage IV, or non-REM) sleep. Nightmares tend to result in a memory of the dream, rather than no memory of the dream; sleep terrors do not tend to result in description of the dream. Nightmares do not tend to result in a rapid return to sleep, whereas sleep terrors do. Nightmares tend to occur during the second half of the night, rather than the first half of the night; sleep terrors tend to occur after 1 to 4 hours of sleep rather than 5 to 6 hours.

The parents of a child with seizure disorder tell the nurse that the child's illness has changed their routine and is creating stress. Which nursing interventions are most important? Select all that apply. Normalize the life of the child. Identify effective coping strategies. Identify appropriate support systems. Encourage the parents to meditate often. Ask the parents to avoid thinking of the future.

Normalize the life of the child. Identify effective coping strategies. Identify appropriate support systems. Rationale The nurse identifies effective coping strategies to help the family adjust with the stress of caring for a chronically ill child. The nurse also helps to normalize the child by encouraging the child to pursue hobbies and attend school. The nurse also identifies appropriate support systems such as community support services to help the parents adjust with the new changes. The nurse avoids giving personal advice such as encouragement to meditate often, because it may interfere with the family's decision making, unless the family requests that type of spiritual guidance. The nurse empowers the parents by helping them to plan realistically for the future.

Munchausen syndrome by proxy

Parents make up or produce physical illness in their children in an effort to get sympathy/attention and be admired for their dedication to their children most common illness parents make up for munchausen seizures perpetrator of munchausen is usually mom consequences of munchausen may undergo needless and painful procedures and treatments considerations when determining whether a child is a victim of munchausen -is condition consistent with history -does diagnostic evidence support reported history -has anyone other than caregiver witnessed the symptoms -is tx being provided primarily bc of caregiver demands -will child offer any info when caregiver not in room

The nurse notes anxiety in the parents of a child diagnosed with neuroblastoma. After educating the parents on the treatment plan and available resources, which actions would indicate to the nurse that the parents are coping well? Select all that apply. Parent does not verbalize treatment plan Parent is tearful when speaking to the nurse Parent defers treatment decisions to the provider Parents request that family's pastor visit the child Parents are asking questions about the child's disease and treatment

Parents request that family's pastor visit the child Seeking the help of a support system is a sign of effective coping. Parents are asking questions about the child's disease and treatment Asking questions about the child's disease and treatment is a sign of effective coping.

A child is brought to the emergency department for altered mental status. Which would be the most concerning assessment finding? The child reacts angrily to being awakened. The child does not recall the trip to the hospital. The child believes the triage room is the bedroom. Parents state that the child does not recognize them.

Parents state that the child does not recognize them. A child who does not recognize the parents is classified a disoriented level of consciousness. More serious than confusion and delirium, disorientation indicates that the child is suffering from a more severe decrease in level of consciousness, and this would be the most concerning assessment.

Interventions for Hodgkins disease

Radiation Radiation and multidrug chemotherapy Monitor for drug induced pancytopenia Monitor for signs of infection and bleeding Preotect child from infection Monitor for side/adverse effects related to chemo/rad Monitor for nausea and vomiting

When the nurse suspects a communicable disease, it is important to assess:

Recent exposure to a known case Prodromal symptoms (symptoms that occur between early manifestations of the disease and its overt clinical syndrome) or evidence of constitutional symptoms, such as a fever or rash Immunization history History of having the disease

how can we control the spread of disease to others

Reduce the risk of cross-transmission of organisms Infection control policies Hand washing Sneeze and cough etiquette Hygiene measures in home and school

The nurse is reviewing the patient education handout with the parent of a patient. The parent asks, "Can you tell me if any of these risks are really serious?" Which does the nurse identify as an example of a rare, but serious, reaction a patient may experience following vaccination? Seizure Vomiting Joint stiffness Redness at the injection site

Seizure A seizure is a rare, but severe and potentially dangerous reaction that can lead to permanent brain damage. In very rare instances, seizure may be long-lasting and lead to coma. This reaction is a medical emergency.

Primary locations for Hodgkin's disease

Spleen, liver, bone marrow, lungs, mediastinum

How should the nurse maintain fluid regulation in a child after a supratentorial procedure? Select all that apply. Start with clear water at first. Stop oral liquids if the child vomits. Infuse fluids as per the maintenance rate. Avoid administration of a hypertonic solution of mannitol. Continue intravenous fluids until the child becomes well tolerant to oral fluids

Start with clear water at first. Stop oral liquids if the child vomits. Continue intravenous fluids until the child becomes well tolerant to oral fluids. Rationale After a supratentorial brain surgery, when the child is alert, the child can be given clear water. There is always a chance of aspiration; thus, clear water is the safest liquid to ingest. If the child vomits, oral liquids should be stopped. Until oral fluids are well tolerated, intravenous fluids should be continued. Postoperatively, fluids should be infused at less than the maintenance rate. A hypertonic solution of mannitol may be administered to remove the excess fluid.

A 10-year-old child with leukemia is dying after a failed bone marrow transplant. The child is anxious when approached by the nurses, expresses fear about going to sleep at night, and cries when the parents are not present in the room. The nurse is concerned with the child's ability to cope with dying. Which actions should the nurse take next? Select all that apply. Stay in the room with the child while they fall asleep. Have the child life specialist play games with the child. Give the child a stuffed animal to cuddle while they go to sleep. Have a volunteer stay with the child while the parents are gone. Talk with the parents about being present at the hospital around the clock.

Stay in the room with the child while they fall asleep. A child who is dying wants to feel safe and not be alone. Staying in the room with the child while they fall asleep assists with the patient accepting death. Have a volunteer stay with the child while the parents are gone. A child who is dying will not want to be alone. Having a volunteer stay with the child while the parents are gone prevents the child from being alone.

Assessment of Wilms' tumor

Swelling or mass within the abdomen Urinary retention or incontinence Anemia Pallor, anorexia, lethargy Hypertension Weight loss and fever Symptoms of lung involvement (shortness of breath, pain in chest)

The mother of a child presents to the clinic with concerns that the child is running into things more often, and telling her "I didn't see that." The nurse notes 20/30 and 20/50 vision in the right and left eyes, respectively, along with nystagmus and strabismus. Which additional information should the nurse obtain to support a suspicion of a brain tumor in the posterior fossa? History of facial tics Patient history of seizures Present muscular weakness Symptom severity during the day

Symptom severity during the day Symptoms related to a tumor in the posterior fossa are often more intense when rising in the morning. This additional information would provide more information about the type of tumor the child may have.

A child who plays soccer is brought to the clinic by the mom who suggests her child is not acting right. Which associated finding does the nurse evaluate further? The child cannot recall yesterday's events. The child requests a drink of water and a popsicle. The mother is pacing in the triage room continuously. The mother states that the child did not eat after the game.

The child cannot recall yesterday's events. Memory loss can be associated with postconcussion syndrome, and therefore the patient may have difficulty remembering yesterday's events. This information helps the nurse direct further care.

Assessment of Leukemia

-Infiltration of the bone marrow by malignant cells causing fever, pallor, fatigue, anorexia, hemorrhage, and bone joint pain -Signs of infection -Hepatosplenomegaly and lymphadenopathy -Normal, elevated, or low white blood cell count depending on the presence of infection -Decreased hemoglobin and hematocrit levels -Decreased platelet count -Positive bone marrow biopsy -Increased intracranial pressure (due to CNS involvement) -Invasion of Leukemic cells to the kidneys, testes, prostate, ovaries, GI tract, lungs - signs of cranial nerve or spinal nerve involvement

Syrup of ipecac

an emetic that exerts its action through irritation of the gastric mucosa and by stimulation of the vomiting center, is no longer recommended for immediate treatment of poison ingestion.

most common cause of severe accidental injury

motor vehicle accident - misinterpret traffic signs disobey common trafic fafety rules - cross the street against the red light - cross in places not for crossing - walk in the same direction as traffic booster used until kid is 57" tall rear seat for kids under 13 yo

the nurse counsels the family of a 4 year old child that in the period immediately following remission he must: a. consume a large amount of food items containing potassium b. consume a large amount of foods containing iron c. practice meticulous hand washing and monitor foe hemrrhage d. all of the above

c. practice meticulous hand washing and monitor foe hemrrhage

using present treatment protocols, prognosis for hodgkin disease may be estimated with: a. the ann arbor staging classification b. histologic staging c. degree of tumor burden d. initial leukocyte count

a. the ann arbor staging classification

Hep A

fecal oral route contaminated food watr first dose around 12 months, second 6 months later hep A symptoms - fever, malaise, anorexia, nausea, abdominal discomfort, dark urine, jaundice

Granulocyte colony-stimulating factor (G-CSF)

filgrastim; pegfilgrastim; direct granulocyte development and can decrease the duration of neuropenia following immunosuppresive therapy

poisoning assessment

first deal with the child, then with poison - check vitsals - assess mental status - institute respiratory and circulation support - measures to reduce shock - establish vascular access for rapid IV expansion

factors that predispose child to physical abuse

-parental low self esteem -inadequate maternal functioning -single parents with limited support -unrelated partners -child aged 0-12 mo -premature infant -infant that was unwanted -disabled child -stessful home environment - poverty, unemployment, poor housing, alcoholis, frequent relocation, drug addition

A child has been diagnosed with a Wilms tumor. What should preoperative nursing care include? a. Careful bathing and handling b. Monitoring of behavioral status c. Maintenance of strict isolation d. Administration of packed red blood cells

ANS: A Careful bathing and handling are important in preventing trauma to the Wilms tumor site.

Which developmental characteristics predispose children to poisoning? Select all that apply. Identity Imitation Initiative Autonomy Sensory motor

Imitation Initiative Autonomy Sensory motor Rationale Developmental characteristics that predispose children to poisoning include imitation, initiative, autonomy, and sensory motor tendencies. Identity does not predispose children to poisonous ingestion.

tumor suppressor genes

genes that keep tumor growth in check

Wills tumor associated

genetic inheritance several congenital anomalies

zika virus

transfers - transplacental, with urine, blood, semen, vaginal fluid no evidence through breastmilk symptoms - arthralgia, fever, maculopapular rash, conjuctivitis 3-12 days after infection sirious manifestations - fetal brain defects, microcephaly no therapy just supportive measures - rest , adequate hydration, analgesics, antipyretics no aspirin to avoid reye syndrome prevent with insect repellent and clothing, screen window and doors, and incect repellent treatment net windows

nurse's role in the end of life decision making

1. as information brokers - provide info about the child and family to the health care team and to the child and family about treatment plans 2. source of emotional support 3. advocate for palliative care planning

Calculate the absolute neutrophil count for a child with a WBC = 10,000/mm3, neutrophils = 25%, and nonsegmented neutrophils (bands) = 22%. Record your answer in a whole number.

ANS: 4700 Perform the calculation Determine the total percentage of neutrophils ("polys," or "segs," and "bands"). Multiply white blood cell (WBC) count by percentage of neutrophils. WBC = 10,000/mm3, neutrophils = 25%, and nonsegmented neutrophils (bands) = 22% Step 1: 25% + 22% = 47% Step 2: 0.47 ´ 10,000 = 4700/mm3 ANC

The nurse is performing an assessment on a 10-year-old child suspected to have Hodgkin's disease. The nurse understands that which assessment findings are specifically characteristic of this disease? Select all that apply. 1. Abdominal pain 2. Fever and malaise 3. Anorexia and weight loss 4. Painful, enlarged inguinal lymph nodes 5. Painless, firm, and movable adenopathy in the cervical area

1. Abdominal pain 5. Painless, firm, and movable adenopathy in the cervical area Hodgkin's disease (a type of lymphoma) is a malignancy of the lymph nodes. Specific clinical manifestations associated with Hodgkin's disease include painless, firm, and movable adenopathy in the cervical and supraclavicular areas and abdominal pain as a result of enlarged retroperitoneal nodes. Hepatosplenomegaly also is noted. Although fever, malaise, anorexia, and weight loss are associated with Hodgkin's disease, these manifestations are seen in many disorders.

Active immunity

Immune bodies are actively formed against specific antigens by having had the disease or by introducing the antigen into the individual

standard precautions

universal precautions ( blood and body fluids) and body substance isolation involve barrier protection - gloves, goggles, gowns, masks to prevent contamination from blood, all body fluids, excretions, secretions, except sweat nonintact skin, mucous membranes

Treatment Modalities

- maltimodal therapy - surgery, chemo, radiation, blood or marrow transplant)) - enrollement of large number of children in cooperative group clinical trials or protocols - improvements in supportive care radiaation - proton bean that doesnt damage nearby tissues; curative or palliative to releive symptoms

risk factors putting children at risk for sexual abuse

-parental unavailability -lack of emotional closeness and flexibility -social isolation -emotional deprivation -communication difficulties

varisella

cell free live-attenuated vaccine 0.5 ml SQ A child should have the first shot at ages 12-15 months. The second shot should be given at ages 4-6 years.

tetanus

in tdap vaccine 11-12 yo booster every 10 y 2 months, 4 months, 6 months, 12-15 months, 4-6 yrs When do we give DTap immunizations?

bedbugs

result of bites: - infection - cellulitis -folliculitis - intense urticaria - impetigo - alaphylactic reaction - sleep loss

Lymphangitis (streaking) is frequently seen in what? a. Cellulitis b. Folliculitis c. Impetigo contagiosa d. Staphylococcal scalded skin

ANS: A Lymphangitis is frequently seen in cellulitis. If it is present, hospitalization is usually required for parenteral antibiotics. Lymphangitis is not associated with folliculitis, impetigo, or staphylococcal scalded skin.

What immunization should not be given to a child receiving chemotherapy for cancer? a. Tetanus vaccine b. Inactivated poliovirus vaccine c. Diphtheria, pertussis, tetanus (DPT) d. Measles, mumps, rubella (MMR)

ANS: D The vaccine used for MMR is a live virus and can cause serious disease in immunocompromised children. The tetanus vaccine, inactivated poliovirus vaccine, and DPT are not live vaccines and can be given to immunosuppressed children. The immune response is likely to be suboptimum, so delaying vaccination is usually recommended.

The nurse is caring for a child who is post-op after a recent nephrectomy. Which assessment/study is priority to ensure the patient meets the expected outcomes? CBC Respiratory rate Renal ultrasound Fluid intake and output

Fluid intake and output It is important to assess fluid intake and urine output to determine if the patient is demonstrating appropriate fluid balance, which is an expected outcome for the patient with Wilms tumor.

A drug used to treat children with giardiasis is: a. metronidazole (Flagyl). b. amoxicillin. c. erythromycin. d. tetracycline.

a. metronidazole (Flagyl).

Which of the following are warning signs of physical abuse in children? Select all that apply. Multiple fractures Delay in seeking medical care Similar histories given by different caregivers Inappropriate affect in relation to the child's injury History incompatible with the pattern or degree of injury

Multiple fractures Delay in seeking medical care Inappropriate affect in relation to the child's injury History incompatible with the pattern or degree of injury Rationale Warning signs of physical abuse in children include multiple fractures, delay in seeking medical care, inappropriate affect in relation to the injury, and incompatible history with the pattern or degree of injury. Histories are similar, not inconsistent, among caregivers.

A 7-month-old infant is brought to the emergency department with a femur fracture. The parents report that the child fell off a couch. Which form of abuse does the nurse suspect? Neglect Physical abuse Emotional abuse Munchausen syndrome by proxy

Physical abuse A lower extremity fracture in a nonambulatory child is a cause for further evaluation for signs of physical abuse.

which one of the following statements is FALSE? a. most reporting of abuse has been from the middle socioeconomic population b. one child is usually the victim in an abusive family; removal of this child often places the other siblings at risk c. the abusive family environment is one of chronic stress, including problems of divorce, poverty, unemployment, and poor housing d. child abuse is problem of all socioeconomic groups

a. most reporting of abuse has been from the middle socioeconomic population

what finding should teh nurse expect to when assessing physical growth in the school aged child? a. weight increase 2-3kg/year b. height increase 3cm/year c. little change in refined coordination d. decrease in body fat and muscle tissue

a. weight increase 2-3kg/year

the nurse is administering a dose of intravenous cisplatin when the child suddenly starts complaining of difficulty breathing and is markedly flushed. The nurses highest priority is to: a. administer an oral antihistamine b. stop the infusion immediately c. slow down the infusion rate d. call the practitioner

b. stop the infusion immediately

Leukemia is more common in

boys than girls over the age of 1yr

The nurse is caring for a child with suspected cancer. Which signs and symptoms should the nurse expect to note on assessment? Select all that apply. Fatigue Headache Polydipsia Weight loss Lymphadenopathy

Fatigue Although fatigue is a nonspecific symptom, it is considered a covert manifestation of pediatric cancer and should therefore be noted by the nurse. Headache Although headache is a nonspecific symptom, it is considered a covert manifestation of pediatric cancer and would be expected by the nurse during an assessment. Weight loss Weight loss is an overt sign of pediatric cancer, and would be an expected finding in a child with cancer. Lymphadenopathy Although lymphadenopathy is a nonspecific symptom, it is considered a covert manifestation of pediatric cancer and therefore expected by the nurse during the assesement.

Fever, low blood counts, lymph node enlargement, and enlarged liver and spleen are diagnostic criteria for what illness? Hepatitis Leukemia Wilms tumor Cystic fibrosis

Leukemia Rationale Fever, low blood counts, lymph node enlargement, and enlarged liver and spleen are diagnostic criteria for leukemia, not hepatitis, Wilms tumor, or cystic fibrosis.

Identification of abuse

entails securing evidence of maltreatment, taking a history pertaining to the incident, and assessing parental and child behaviors. After abuse is identified, it is important to remove the child from the situation or environment to prevent further injury.

gastrointestinal decontamination

is activated charcoal, an odorless, tasteless, fine black powder that adsorbs many compounds, creating a stable complex.

Wilms tumor, or nephroblastoma,

is the most common kidney tumor of childhood. The most common presenting sign is painless swelling or a mass within the abdomen. The mass is characteristically firm, nontender, confined to one side, and deep within the flank. Combined treatment with surgery and chemotherapy, with or without irradiation, is based on the clinical stage and histologic pattern.

Reed-sternberg cells

Characterizes Hodgkins disease, noted in a lymph node biopsy specimen

identified characteristics of parents who are at higher risk for abusing their children do NOT include a. older parents who waited until their mid30s before having their first child b. isolated parents with few supportive relationships c. parents who grew up with poor role models d. single-parent family

a. older parents who waited until their mid30s before having their first child

a limb salvage procedure is the treatment of choice with which bone cancer: a. ewing b. osteosarcoma

b. osteosarcoma

the cancer that occurs with the most frequency in children is: a. lymphoma b. neuroblastoma c. leukemia d. melanoma

c. leukemia

Scarlett Fever

infection caused by group A Strep; diffuse rash and sloughing of the skin

Staphylococcal infections

occur most often in children in the younger age-groups; the incidence decreases with advancing age. All of these factors point up the importance of careful hand washing and cleanliness when caring for infected children and their lesions to prevent spread of the infection and as an essential prophylactic measure when caring for infants and small children.

Acute Myeloid Leukemia (AML)

predisposing - chromosomal anomalies, inherited single gene mutation, inherited cytopenias clinical manifestation - fever with or without infection, night sweats, shortness of breath, weakness or fatigue, easy bleeding bruising, petechiae, bone or joint pain, eczema type sin rash, painless bluish lumps diagnosis - physical examination, CBC, blood chemistries, chest x-ray, bone marrow, biopsy, cytogenetic analysis, molecular testing, LP chemo -induction phase - postremission consolidation /intensification after remission SCT performed

Osteosarcoma (osteogenic sarcoma)

the most common bone cancer in children and most commonly affects patients in the second decade of life during their growth spurt. Optimum treatment of osteosarcoma includes surgery and chemotherapy.

cancer surgery

1. obtain biopsy 2. remove tumor 3. restore normal body functioning better when tumor is encapsulated and localized

causes of accidental death injury in infants

1. suffocation 2. motor vihicle related 3. drowning

The nurse is assessing a child with leukemia. The white blood cell (WBC) count is 2500 with 10% neutrophils and 10% nonsegmented neutrophils. What is the absolute neutrophil count (ANC)? Record your answer using a whole number.

500 Rationale The WBC count is multiplied by the percent of neutrophils. WBC = 2500; neutrophils = 10%; nonsegmented neutrophils = 10% Step 1: 10% + 10% = 20% Step 2: 0.20 × 2500 = 500 ANC.

alopecia

Cause of alopecia: -chemo agents -cranial irradiation Does everyone with cancer lose their hair? No -may just experience thinning Does hair grow back after tx of cancer? Yes -3 to 6 months -different color and texture is possible

Monovalent vaccine

Designed to vaccinate against a single antigen or organism

Treatment of Hodgkin's disease

Radiation and Chemotherapy

prime time for sleep problems to occur

preschool

To avoid a fall from a crib, the nurse recommends to parents that their toddler should sleep in a bed rather than a crib when reaching what height? a. 30 in b. 35 in c. 40 in d. 45 in

ANS: B When children reach a height of 89 cm (35 in), they should sleep in a bed rather than a crib.

What chemotherapeutic agent can cause an anaphylactic reaction? a. Prednisone (Deltasone) b. Vincristine (Oncovin) c. L-Asparaginase (Elspar) d. Methotrexate (Trexall)

ANS: C A potentially fatal complication is anaphylaxis, especially from L-asparaginase, bleomycin, cisplatin, and etoposide (VP-16).

What clinical manifestation occurs with hypoglycemia? a. Lethargy b. Confusion c. Nausea and vomiting d. Weakness and dizziness

ANS: D Some of the clinical manifestations of hypoglycemia include weakness; dizziness; difficulty concentrating, speaking, focusing, and coordinating; sweating; and pallor. Lethargy, confusion, and nausea and vomiting are manifestations of hyperglycemia.

When preparing a child to receive chemotherapy, the nurse can expect to see which values in the patient's electronic health record? ABG levels WBC levels Serum glucose level Resting metabolic rate

WBC levels WBC levels should be obtained prior to chemotherapy. This provides a baseline for comparison once treatment has begun.

leading causes of death children 1-19

accidents assault/homicide cancer intentional self-harm

monoclonal antibodies

attach to protein on cancer cell so the immune system can find and destroy them

Which of the following is not considered a characteristic with ALTE? a. Apnea. b. Cyanosis. c. Coughing. d. Hypotonia. e. Acrocyanosis.

e. Acrocyanosis.

oncogenes

genes that activate tumor growth

side effects of treatment

infection hemorrhage anemia nausea vomiting altered nutrition mucosal ulceration neurologic problems hemorrhagic cystitis steroid effects

Peak incidence of Hodgkin's disease is during.

mid-adolescence

transmission precautions

for patients with documented or suspected infection or colonization - presence of microorgansms in or on patient

Leukemia is characterized by A. high leukocyte count b. destruction of normal cells by abnormal cells c. low numbers of blast cells d. overproduction of blast cells

d. overproduction of blast cells

school aged children: a. have little interest in complex board, card, or computer games b. rarely collect items c. tire of having stories read aloud d. participate in hero worship

d. participate in hero worship

Cocooning

Strategy of protecting infants from pertussis by vaccinating all persons who come in close contact with the infant

A nurse is teaching student nurses about cognitive milestones reached by average American children at different stages of development. What statement made by a student nurse indicates effective learning? "A 5-year-old child can repeat three numbers backward." "An 8-year-old child can describe common objects in detail." "A 7-year-old child can give similarities and differences between two things from memory." "A 6-year-old child can read an ordinary clock or watch correctly to the nearest quarter hour."

"An 8-year-old child can describe common objects in detail." Rationale An 8-year-old child can describe common objects in detail, and not merely enumerate their use. A 7-year-old child can repeat three numbers backward; a 5-year-old child usually cannot. Children in the age group of 8 to 9 years can give similarities and differences between two things from memory; a 7-year-old child usually cannot. A 7-year-old child can read an ordinary clock or watch correctly to the nearest quarter hour; a 6-year-old child usually cannot.

History Assessment/cancer

- History of present illness (onset of symptoms, severity and duration, alleviating or potentiating factors) - History of previous illnesses (communicable diseases, infections, medication history, previous hospitalizations or surgeries, exposure to blood products, and immunization status) - Family history (family members with prior cases of cancer as well as types, ages at diagnosis, treatments, and outcomes) - Present health status of family members (history of illnesses or diseases in others) - Developmental factors (milestones obtained and recent regressions in any milestones) - Psychosocial factors (include family concerns or problems)

Biotherapy (immunotherapy) types

1. do not target cancer cells directly but stimulate the body's immune system to act against cancer cells, and are colectively referred to as immunotherpay or biologic response modifier therapy 2. those that use anibodies or segments or genetic material to target cancer cells directly 3. therapies that interfere with specific molecules involved in tumor growth and progression and are referred to as targeted therapies

Which patients may receive the measles, mumps and rubella (MMR) vaccine? Select all that apply. A breastfeeding mother A 12-month-old baby with flu A pregnant woman in first trimester A pregnant woman in third trimester A 14-month-old baby allergic to eggs

A breastfeeding mother A 12-month-old baby with flu A 14-month-old baby allergic to eggs Rationale A nurse should be aware of some of the common contraindications to all the vaccines. Common cold or flu is not a contraindication to immunization, although severely ill clients should not be given any vaccines. Measles and mumps vaccines, which are grown in chick embryo tissue cultures, do not contain a significant amount of egg cross-reacting proteins. Therefore egg allergy is not a contraindication to MMR vaccine. Similarly, breastfeeding is also not a contraindication to MMR vaccine. MMR is a live virus vaccine and should not be given to pregnant women, irrespective of their gestational age.

Conjugate vaccine

A carrier protein with proven immunologic potential combined with a less antigenic polysaccharide antigen to enhance the type and magnitude of the immune response; Hib

What are the goals of organized athletics for preadolescent children? (Select all that apply.) a. Physical fitness b. Basic motor skills c. A positive self-image d. Commitment to winning

A, B, C The goals of organized athletics for preadolescent children include physical fitness, basic motor skills, and a positive self-image. The commitment is to the values of teamwork, fair play, and sportsmanship, not to winning.

Parents are concerned about their child riding an all-terrain vehicle. What should the nurse tell the parents about safe use of all-terrain vehicles? (Select all that apply.) a. Restrict riding to familiar terrain. b. Limit street use to the neighborhood. c. Nighttime riding should not be allowed. d. Vehicles should not carry more than two persons. e. Vehicles should include seat belts, roll bars, and automatic headlights.

A, C, E Safe use of all-terrain vehicles includes restricting riding to familiar terrain; not allowing nighttime riding; and assuring the vehicle has seat belts, roll bars, and automatic headlights. Street use should not be allowed, and the vehicle should not carry more than one person.

A health care provider prescribes ondansetron (Zofran) 0.15 mg/kg intravenously (IV) 30 minutes before chemotherapy for a child with acute lymphoblastic leukemia. The child weighs 22 kg. The medication label states: "Ondansetron (Zofran) 2 mg/1 ml." The nurse prepares to administer the dose. How many milliliters will the nurse prepare to administer the dose? Fill in the blank. Round your answer to one decimal place.

ANS: 1.7 Follow the formula for dosage calculation. 22 ´ 0.15 = 3.3 mg as the dose Desired ———- ´ Volume = ml per dose Available 3.3 mg ———- ´ 1 ml = 1.65 ml round to 1.7 ml 2 mg

The health care provider has prescribed propylthiouracil (PTU) 7 mg/kg/day PO divided every 12 hours for a child with Graves disease. The child weighs 66 lb. The nurse is preparing to administer the 0800 dose. Calculate the dose the nurse should administer in milligrams. Record your answer below in a whole number.

ANS: 105 The correct calculation is: 66 lb/2.2 kg = 30 kg Dose of PTU is 7 mg/kg/day divided every 12 hours 7 mg ´ 30 = 210 mg 210 mg/2 = 105 mg

What technique facilitates lip reading by a hearing-impaired child? a. Speak at an even rate. b. Avoid using facial expressions. c. Exaggerate pronunciation of words. d. Repeat in exactly the same way if child does not understand.

ANS: A Help the child learn and understand how to read lips by speaking at an even rate. Avoiding using facial expressions, exaggerating pronunciation of words, and repeating in exactly the same way if the child does not understand interfere with the child's understanding of the spoken word.

What does impetigo ordinarily results in? a. No scarring b. Pigmented spots c. Atrophic white scars d. Slightly depressed scars

ANS: A Impetigo tends to heal without scarring unless a secondary infection occurs.

A child has a slight (26-40 dB) degree of hearing loss. The nurse recognizes this amount of hearing loss can have what effect? (Select all that apply.) a. No speech defects b. Difficulty hearing faint speech c. Usually is unaware of the hearing difficulty d. Can distinguish vowels but not consonants e. Unable to understand conversational speech

ANS: A, B, C A child with a slight degree of hearing loss has no speech defects, may have difficulty hearing faint speech, and is usually unaware of the hearing difficulty. The ability to distinguish vowels but not consonants is an effect of severe hearing loss and being unable to understand conversational speech is an effect of moderately severe hearing loss.

The nurse is teaching coping strategies to parents of a child with a chronic illness. What coping strategies should the nurse include? (Select all that apply.) a. Listen to the child. b. Accept the child's illness. c. Establish a support system. d. Learn to care for the child's illness one day at a time. e. Do not share information with the child about the illness.

ANS: A, B, C, D Coping strategies for parents caring for a child with a chronic illness include listening to the child, accepting the child's illness, establishing a support system, and learning to care for the child's illness one day at a time. Information should be shared with the child about the illness.

The nurse should plan which actions to facilitate lipreading for a child with a hearing impairment? (Select all that apply.) a. Face the child directly. b. Speak at eye level. c. Keep sentences short. d. Speak at a fast, even-paced rate. e. Establish eye contact and show interest.

ANS: A, B, C, E To facilitate lipreading, the nurse should plan to face the child directly, speak at eye level, keep sentences short, and establish eye contact and show interest. The nurse should plan to speak at a slow rate, not a fast one.

The nurse is assessing coping behaviors of a family with a child with a chronic illness. What indicates approach coping behaviors? (Select all that apply.) a. Plans realistically for the future b. Verbalizes possible loss of the child c. Uses magical thinking and fantasy d. Realistically perceives the child's condition e. Does not share the burden of the disorder with others

ANS: A, B, D Approach coping behaviors include planning realistically for the future, verbalizing possible loss of a child, and realistically perceiving the child's behavior. Using magical thinking and fantasy is an avoidance behavior. The family should share the burden of the disorder with others as an approach behavior.

What can the nurse suggest to families to reduce blood lead levels? (Select all that apply.) a. Do not store food in open cans. b. Ensure the child eats regular meals. c. Mix formula with hot water from the tap. d. Vacuum hard-surfaced floors and window wells. e. Wash and dry the child's hands and face frequently.

ANS: A, B, E To reduce blood lead levels, the family should ensure the child eats regular meals because more lead is absorbed on an empty stomach. The child's hands and face should be washed and dried frequently, especially before eating. Food should not be stored in open cans, particularly if cans are imported. Hot water dissolves lead more quickly than cold water and thus contains higher levels of lead. Hot water should not be used to mix formula. Hard-surfaced floors or window sills or wells should not be vacuumed because this spreads dust.

The clinic nurse is reviewing the immunization guidelines for hepatitis B. Which are true of the guidelines for this vaccine? (Select all that apply.) a. The hepatitis B vaccination series should be begun at birth. b. The adolescent not vaccinated at birth does not have a need to be vaccinated. c. Any child not vaccinated at birth should receive two doses at least 4 months apart. d. An unimmunized 10-year-old child should receive three doses administered 4 weeks apart.

ANS: A, D Current immunization guidelines for hepatitis B vaccination recommend beginning the hepatitis B vaccine series at birth or, in unimmunized children, as soon as possible. Children younger than 11 years of age may be vaccinated with a three-dose series, administered 4 weeks apart. Children 11 years and older may receive the two-dose adult formulation given at least 4 months apart.

The nurse is teaching parents of a child with a cognitive impairment signs that indicate the child is developmentally ready for dressing training. What signs should the nurse include that indicate the child is developmentally ready for dressing training? (Select all that apply.) a. Can follow verbal commands b. Can sit quietly for 1 to 2 minutes c. Can master every task of dressing d. Can follow physical gestures or cues e. Can relate clothing to the appropriate body part

ANS: A, D, E Children are considered developmentally ready for dressing training if they can sit quietly for 3 to 5 minutes (not 1 to 2) while working on a task; can follow physical gestures or cues; can follow verbal commands; and can relate clothing to the appropriate body part, such as socks to feet. As with other self-help skills, the child may not be able to master every task but should be evaluated for evidence of willingness to participate at his or her level of readiness.

The nurse is caring for a 6-year-old child with acute lymphoblastic leukemia (ALL). The parent states, "My child has a low platelet count, and we are being discharged this afternoon. What do I need to do at home?" What statement is most appropriate for the nurse to make? a. "You should give your child aspirin instead of acetaminophen for fever or pain." b. "Your child should avoid contact sports or activities that could cause bleeding." c. "You should feed your child a bland, soft, moist diet for the next week." d. "Your child should avoid large groups of people for the next week."

ANS: B A child with a low platelet count needs to avoid activities that could cause bleeding such as playing contact sports, climbing trees, using playground equipment, or bike riding. The child should be given acetaminophen, not aspirin, for fever or pain; the child does not need to be on a soft, bland diet or avoid large groups of people because of the low platelet count.

The clinic nurse is instructing parents about caring for a toddler with ascariasis (common roundworm). Which statement made by the parents indicates a need for further teaching? a. "We will wash our hands often, especially after diaper changes." b. "We know that roundworm can be transmitted from person to person." c. "We will be sure to continue the nitazoxanide (Alinia) orally for 3 days." d. "We will bring a stool sample to the clinic for examination in 2 weeks."

ANS: B Ascariasis (common roundworm) is transferred to the mouth by way of contaminated food, fingers, or toys. Further teaching is needed if parents state it is transmitted from person to person. Frequent handwashing, especially after diaper changes, continuing the Alinia for 3 days, and reexamining the stool in 2 weeks are appropriate actions.

What is the single most prevalent cause of disability in children and responsible for the recent increase in childhood disability? a. Cancer b. Asthma c. Seizures d. Heart disease

ANS: B Asthma is the single most prevalent cause of disability in children and has been largely responsible for much of the recent increase in childhood disability.

A child, age 10 years, has a neuroblastoma and is in the hospital for additional chemotherapy treatments. What laboratory values are most likely this child's? a. White blood cell count, 17,000/mm3; hemoglobin, 15 g/dl b. White blood cell count, 3,000/mm3; hemoglobin, 11.5 g/dl c. Platelets, 450,000/mm3; hemoglobin, 12 g/dl d. White blood cell count, 10,000/mm3; platelets, 175,000/mm3

ANS: B Chemotherapy is the mainstay of therapy for extensive local or disseminated neuroblastoma. The drugs of choice are vincristine, doxorubicin, cyclophosphamide, cisplatin, etoposide, ifosfamide, and carboplatin. These cause immunosuppression, so the laboratory values will indicate a low white blood cell count and hemoglobin.

Pertussis vaccination should begin at which age? a. Birth b. 2 months c. 6 months d. 12 months

ANS: B The acellular pertussis vaccine is recommended by the American Academy of Pediatrics beginning at age 6 weeks. Infants are at greater risk for complications of pertussis. The vaccine is not given after age 7 years, when the risks of the vaccine become greater than those of pertussis. The first dose is usually given at the 2-month well-child visit. Infants are highly susceptible to pertussis, which can be a life-threatening illness in this age group.

The nurse is administering activated charcoal to a preschool child with acetaminophen (Tylenol) poisoning. What potential complications from the use of activated charcoal should the nurse plan to assess for? (Select all that apply.) a. Diarrhea b. Vomiting c. Fluid retention d. Intestinal obstruction

ANS: B, D Potential complications from the use of activated charcoal include vomiting and possible aspiration, constipation, and intestinal obstruction. Diarrhea and fluid retention are not potential complications of activated charcoal administration.

What are characteristics of diabetic ketoacidosis? (Select all that apply.) a. Pallor b. Acidosis c. Bradypnea d. Dehydration e. Electrolyte imbalance

ANS: B, D, E Characteristics of diabetic ketoacidosis include acidosis, dehydration, and electrolyte imbalance. Respirations are rapid (Kussmaul respirations), not slow, and flushing, not pallor, would occur.

The nurse is caring for a child with an anterior pituitary tumor. What hormones are secreted by the anterior pituitary? (Select all that apply.) a. Oxytocin b. Luteinizing hormone c. Antidiuretic hormone d. Thyroid-stimulating hormone e. Adrenocorticotrophic hormone

ANS: B, D, E The anterior pituitary is responsible for secreting the following hormones: growth hormone, thyroid-stimulating hormone, adrenocorticotrophic hormone, follicle-stimulating hormone, luteinizing hormone, and prolactin. The posterior pituitary secretes antidiuretic hormone and oxytocin.

To help an adolescent deal with diabetes, the nurse needs to consider which characteristic of adolescence? a. Desire to be unique b. Preoccupation with the future c. Need to be perfect and similar to peers d. Awareness of peers that diabetes is a severe disease

ANS: C Adolescence is a time when the individual has a need to be perfect and similar to peers. Having diabetes makes adolescents different from their peers. Adolescents do not wish to be unique; they desire to fit in with the peer group. An adolescent is usually not future oriented. Awareness of peers that diabetes is a severe disease would further alienate the adolescent with diabetes. The peer group would focus on the differences.

The parents of a child with cognitive impairment ask the nurse for guidance with discipline. What should the nurse's recommendation be based on? a. Discipline is ineffective with cognitively impaired children. b. Cognitively impaired children do not require discipline. c. Behavior modification is an excellent form of discipline. d. Physical punishment is the most appropriate form of discipline.

ANS: C Discipline must begin early. Limit-setting measures must be clear, simple, consistent, and appropriate for the child's mental age. Behavior modification, especially reinforcement of desired behavior and use of time-out procedures, is an appropriate form of behavior control. Aversive strategies should be avoided in disciplining the child.

A child with acetylsalicylic acid (aspirin) poisoning is being admitted to the emergency department. What early clinical manifestation does the nurse expect to assess on this child? a. Hematemesis b. Hematochezia c. Hyperglycemia d. Hyperventilation

ANS: D An early clinical manifestation of acetylsalicylic acid (aspirin) poisoning is hyperventilation. Hematemesis, hematochezia, and hyperglycemia are clinical manifestations of iron poisoning.

What is a common clinical manifestation of Hodgkin disease? a. Petechiae b. Bone and joint pain c. Painful, enlarged lymph nodes d. Nontender enlargement of lymph nodes

ANS: D Asymptomatic, enlarged cervical or supraclavicular lymphadenopathy is the most common presentation of Hodgkin disease. Petechiae are usually associated with leukemia. Bone and joint pain are not likely in Hodgkin disease. The enlarged nodes are rarely painful.

What is the role of the peer group in the life of school-age children? a. Decreases their need to learn appropriate sex roles b. Gives them an opportunity to learn dominance and hostility c. Allows them to remain dependent on their parents for a longer time d. Provides them with security as they gain independence from their parents

ANS: D Peer group identification is an important factor in gaining independence from parents. Through peer relationships, children learn ways to deal with dominance and hostility. They also learn how to relate to people in positions of leadership and authority and how to explore ideas and the physical environment. A child's concept of appropriate sex roles is influenced by relationships with peers.

Many of the clinical features of Down syndrome present challenges to caregivers. Based on these features, what intervention should be included in the child's care? a. Delay feeding solid foods until the tongue thrust has stopped. b. Modify the diet as necessary to minimize the diarrhea that often occurs. c. Provide calories appropriate to the child's mental age. d. Use a cool-mist vaporizer to keep the mucous membranes moist and secretions liquefied.

ANS: D The constant stuffy nose forces the child to breathe by mouth, drying the mucous membranes and increasing the susceptibility to upper respiratory tract infections. A cool-mist vaporizer will keep the mucous membranes moist and liquefy secretions. Respiratory tract infections combined with cardiac anomalies are the primary cause of death in the first years. The child has a protruding tongue, which makes feeding difficult. The parents must persist with feeding while the child continues the physiologic response of the tongue thrust. The child is predisposed to constipation. Calories should be appropriate to the child's weight and growth needs, not mental age.

Herpes zoster is caused by the varicella virus and has an affinity for which? a. Sympathetic nerve fibers b. Parasympathetic nerve fibers c. Lateral and dorsal columns of the spinal cord d. Posterior root ganglia and posterior horn of the spinal cord

ANS: D The herpes zoster virus has an affinity for posterior root ganglia, the posterior horn of the spinal cord, and the skin. The zoster virus does not involve the nerve fibers listed.

The nurse is assessing a three-year-old child and notes leukocoria, redness, and inflammation of the left eye. Which action would the nurse take next? Assess for strabismus Listen to lung sounds Inspect skin for breakdown Palpate cervical lymph nodes

Assess for strabismus Leukocoria and redness of the left eye are signs of retinoblastoma. The nurse would assess for strabismus as another indication of the condition.

The nurse is caring for a 16-year-old patient after amputation of the left tibia related to osteosarcoma. The patient appears tearful and refuses to speak to the nurse. Which nursing action should take priority? Administer an IV sedative Assess for suicidal ideation Encourage peer visitation Refer patient and family to support group

Assess for suicidal ideation Withdrawal and crying are signs of depression after amputation. IT is important to asses any patient with signs of depression or inappropriate coping for suicidal ideation so that appropriate actions can be taken.

During assessment of a 7-month-old infant in the emergency department, the nurse notes papilledema, vomiting, and lethargy. The mother reports that the child has been colicky and that she brought the child in because of an altered mental status after the child had been left with a babysitter for 4 hours. Which type of abuse would the nurse suspect? Sexual abuse Emotional abuse Abusive head trauma Munchausen syndrome by proxy

Abusive head trauma The nurse would suspect abusive head trauma, specifically shaken baby syndrome, in a colicky child whose mental status suddenly changed after being left with a babysitter.

Warning for Wilms' tumor

AVOID PALPATION OF THE ABDOMEN. Be cautious when bathing, moving or handling the child. It is important to keep the tumor in tact. Rupture of the tumor can cause the cancer cells to spread throughout the abdomen, lymph system, and bloodstream

What is the similarity between neuroblastoma and nephroblastoma? Abdominal mass Tumor across midline Prognosis is generally poor Spontaneous regression

Abdominal mass Rationale Both neuroblastoma and nephroblastoma present as an abdominal mass. Nephroblastoma tend to affect only one side while neuroblastoma often crosses the midline. The prognosis of neuroblastoma is poor but that of nephroblastoma is good. Neuroblastomas may regress spontaneously, possibly due to maturity of the embryonic cell or development of an active immune system. Such regression does not occur in nephroblastoma.

Which clinical manifestation of scarlet fever is present in the prodromal stage of the disease? Enlarged tonsils Abrupt high fever Rash over the body Red "strawberry tongue"

Abrupt high fever Rationale Abrupt high fever, vomiting, headache, chills, malaise, abdominal pain, and halitosis are clinical manifestations of scarlet fever in its prodromal stage. Enlarged tonsils and red "strawberry tongue" (enanthema) occur during the first 1 to 2 days. Body rash (exanthema) appears within 12 hours after the onset of prodromal signs.

anemia

Anemia is a consequence of: myelosuppression What may be needed with anemia? blood therapy with PRBcs to raise the hgb to 10 g/dL

A child with a terminal illness has died following complications. The nurse is following-up with the child's parents. What can the nurse expect to observe in response to their child's death? Select all that apply. Any emotions that the parents are feeling may be expressed. The grief response will be different for each parent, and that is normal. The grief response could last months but should resolve within a year. Symptoms that might indicate one or both parents need a referral to a mental health care provider. A support group must be attended so that the parents can learn from other parents who have been in similar circumstances.

Any emotions that the parents are feeling may be expressed. Expression of emotion is important to help the parents grieve and not hold their emotions in. Correct The grief response will be different for each parent, and that is normal. Everyone grieves in an individual way and that is OK. The parents need to know that they may react differently to the loss of the same child. Symptoms that might indicate one or both parents need a referral to a mental health care provider. There is a normal expected grief response, but depending on the individual, that normal response can lead to more serious emotional problems that may require a referral. Incorrect

What are risk factors for sudden infant death syndrome? (Select all that apply.) a. Postterm b. Female gender c. Low Apgar scores d. Recent viral illness e. Native American infants

C, D, E Infant risk factors for sudden infant death syndrome include those with low Apgar scores and recent viral illness and Native American infants. Preterm, not postterm, birth and male, not female, gender are other risk factors.

The nurse observes that the ten-year-old patient is becoming increasingly restless. Knowing that the child suffered a concussion playing football, what does the nurse do next? Select all that apply. Calculate Glasgow coma score. Perform bilateral pupil examination. Inspect child's skull for size and shape. Ask patient about nausea and headache. Check vital signs and oxygen saturation.

Calculate Glasgow coma score. The child's GCS will offer information about the increased restlessness and whether it is neurologic in nature. Correct Perform bilateral pupil examination. The nurse assesses reasons for restlessness including signs of increased ICP: diplopia, papilledema, and poor pupillary response to light. Ask patient about nausea and headache. Nausea and headaches are common, early signs of increased intracranial pressure. If patient does have these, it helps direct the nurse's assessment. Correct Check vital signs and oxygen saturation. Hypoxia can be a cause of restlessness in any population. Anytime there is a new finding in a patient, vital signs should be reevaluated as indicators of overall cardiorespiratory status.

Which type of poisoning might the nurse expect for a 6-year-old who has presented with a cherry-red mucosa and a history of altered mental status after playing in the garage with the car running? Lead Corrosives Hydrocarbons Carbon monoxide

Carbon monoxide Carbon monoxide (CO) binds tightly to hemoglobin, preventing the binding of oxygen. The CO makes the hemoglobin appear bright red, causing the patient to look rosy-cheeked and to have cherry-red lips.

Treatment of Leukemia

Chemotherapy and radiation therapy. Also hematopoietic stem cell transplantation

Which substances are frequently ingested poisons by children? Select all that apply. Coins Tobacco Cologne Mouthwash Self-rising flour

Coins Cologne Mouthwash Rationale Coins, cologne, and mouthwash are among the most frequently ingested poisons. Tobacco and self-rising flour are not on the list of most frequently ingested poisons for children

The nurse is performing a neurologic assessment on a patient with cerebral palsy (CP). The nurse notes bilateral arm spasticity and the child is unable to grip the nurse's fingers. What action should the nurse perform? Notify health care provider Assess child's gait for ataxia Complete neurologic assessment Inquire about learning difficulties

Complete neurologic assessment Spasticity is an expected finding in a child with cerebral palsy. The nurse should continue the assessment.

Which structure is most likely to be disrupted if the left cerebral hemisphere is pushed across the cranium toward the right cerebral hemisphere? Fontanel Tentorium Cerebellum Corpus callosum

Corpus callosum The corpus callosum is a band of commissural fibers that connects the right and left cerebral hemispheres and is the most likely to be damaged when the left cerebral hemisphere is pushed toward the right cerebral hemisphere (herniation).

Which signs in a child with whooping cough may indicate airway obstruction? Select all that apply. Skin pallor Cyanosis Elevated temperature Increased restlessness Apprehension in the child

Cyanosis Increased restlessness Apprehension in the child Rationale Whooping cough is a respiratory infection caused by Bordetella pertussis. Airway obstruction can occur due to the infection. Therefore, nurses are instructed to assess for signs of airway obstruction, including cyanosis, apprehension, increased restlessness, and retractions. The nurse would immediately report any of these signs. Pallor is seen in cases of shock. An increased temperature indicates infection. p. 182

after bone marrow aspiration is performed on a child, the nurse should: a. apply an adhesive bandage b. place the child in the trendelenburg position c. ask the child to remain in the supine position d. apply a pressure bandage

d. apply a pressure bandage

an assessment finding that is consistent with the presence of a brain tumor is increased: a. headaches b. appetite c. pulse d. blood pressure

d. blood pressure

How do are sleep problems in school-age children often demonstrated? Night terrors that awaken them Delaying tactics because they do not wish to go to bed Somatic illness that awakens them Increasing need for sleep time as they get older

Delaying tactics because they do not wish to go to bed Correct Children in middle childhood must be reminded to go to sleep. Older children up to age 11 years are particularly resistant, and they may demonstrate delaying tactics because they do not wish to go to bed. Night terrors are common in the preschool-age child. If somatic illness is present, a thorough assessment is indicated. The amount of sleep needed decreases as children get older

If a child does not understand the universality of death, the nurse understands that the child likely believes which things about death? Select all that apply. All living things eventually die. Death is inevitable, unavoidable. Death is avoidable if you are clever. Death occurs in the remote future only. Children, in general, may be excluded from death.

Death is avoidable if you are clever. Death occurs in the remote future only. Children, in general, may be excluded from death. Rationale If a child does not understand the universality of death, the child likely believes if a person is clever, death is avoidable; death occurs in the remote future only; and children, in general, may be excluded from death. Universality of death includes the belief that all living things eventually die and death is inevitable, unavoidable.

Administration of diphenhydramine can help with which physiologic response expected near the time of death? Apnea Bradycardia Death rattle Cyanosis of the hands and feet

Death rattle Diphenhydramine may be given to decrease the amount of secretions found in the oral airway of the child, which will decrease the death rattle heard with respiration.

Polyvalent vaccine

Designed to vaccinate against muliple antigens or organisms; meningococcal polysaccharide vaccine

A nurse is caring for a child and notices that as the child's last days are approaching, the family is becoming more withdrawn. Which action is most important to ensure the child is not left alone during the dying process? Discuss utilizing a one-to-one staffing model so the nurse only cares for the child. Discuss the withdrawn behavior observed by the nurse and how it impacts the child's last days. Discuss the need to talk to the child, touch the child, and remain at the bedside of the child during the last days. Discuss the situation with the child life specialist and see if volunteers are available to provide coverage if the parents are gone.

Discuss the need to talk to the child, touch the child, and remain at the bedside of the child during the last days. The dying child needs to experience complete love and acceptance and know that they will not be left alone to die. Discussing the expectations with the parents provides them with how to interact with their child before the child dies and may decrease their withdrawnness.

Risk factors for Leukemia

Genetic, viral, immunological, and environmental factors and exposure to radiation, chemicals, and medications

Which vaccinations are appropriate to administer at a one-month well visit? Hep B #1 PCV #1 IPV #1 Rotavirus #1

Hep B #1 Hep B can be given at a one-month visit. Administer Hep B #1 at one-month visit if no prior dose was given at birth. It is permissible to give Hep B #2 at the one-month visit if Hep B #1 was given at birth.

Which profession is common among mothers with Munchausen syndrome by proxy? Banker Teacher Accountant Home health aide

Home health aide Rationale Biologic mothers with some degree of health care knowledge and training are more likely to have Munchausen syndrome by proxy (MSBP) than are biologic mothers with no health care knowledge. Mothers who are teachers, bankers, or accountants are less likely to perpetuate MSBP than a biologic mother who is a home health aide.

Which causative agent is responsible for erythema infectiosum (fifth disease)? Paramyxovirus Human parvovirus B19 Human herpesvirus type 6 Group A β-hemolytic streptococcus

Human parvovirus B19 Rationale Human parvovirus B19 is the causative agent of fifth disease. Paramyxovirus causes mumps. Human herpesvirus types 1 and 2 are the major causes of herpetic infections in human beings. Group A β-hemolytic streptococcus is the causative agent for scarlet fever.

Preoperative interventions of neuroblastoma

Monitor for signs and symptoms related to the location of the tumor Provide emotional support

A child is admitted to the hospital with Non-Hodgkin lymphoma, and reports having lower abdominal pain of 6/10. The nurse notes a temperature of 102.3° F, heart rate 107 and respiratory rate 14. Which actions are most important for the nurse to take? Select all that apply. Assess lung sounds Obtain urine culture Obtain blood culture Obtain uric acid level Administer pain medication

Obtain urine culture A temperature of 102.3 indicates a fever and possible infection. The nurse should obtain a urine culture to identify the cause of infection. Correct Obtain blood culture A temperature of 102.3 indicates a fever and possible infection. The nurse should obtain a blood culture to identify the cause of infection

Acquired immunity

Occurs from exposue to the invading agent

A nurse is tending to the emotional needs of the parents who have just lost a child. What is an example of the type of communication necessary during this time? The nurse tells the family that the chaplain has been called to come and pray with them. The nurse promises the family to be back in an hour to bring the body to the morgue. The nurse tells the family of the plan to remain in the room with them until the family goodbyes are done being said. The nurse tells the family members that the nurse is available to talk if they desire and will return when needed or requested.

The nurse tells the family members that the nurse is available to talk if they desire and will return when needed or requested. The nurse should provide privacy for the family but remain close by and return as needed or requested.

The peak incidence of nephroblastoma

The peak incidence is 3 years of age

Double Effect Principle

The principle that recognizes that ethical choices may result in untoward outcomes.

The prognosis of Hodgkins disease

The prognosis is excellent, with long term surival rates depending on the stage of the disease

Wilms tumor (nephroblastoma)

Wilm's tumor (nephroblastoma) Sx of Wilm's tumor: -mass that's firm, nontender, painless(parents usually discover during bath) -swelling in one side of abdomen -hematuria 25% of the time -anemia -pallor -anorexia -lethargy - hypertension weight loss fever Dx of Wilms tumor: history and physical examination -CT -MRI of the abdomen DO NOT PALPATE ABDOMEN Prognosis of Wilms tumor: 90% survival rate Nursing care for Wilm's tumor: -Do NOT palpitate abdomen -I&O -fluid balance -prevent infection

Separation is one of the greatest threats to seriously ill children of which age-group? a. Toddlers b. Preschoolers c. School-age children d. Adolescents

a. Toddlers

sleep problems in the school aged child are often demonstrated by: a. delaying tactics because the child does not wish to go to bed b. night terrors that awaken the child c. the development of somatic illness that awakens the child during the night d. the increasing need for larger amounts of sleep compared with preschool or adolescent children

a. delaying tactics because the child does not wish to go to bed

neuroblastoma is often classified as a silent tumor because: a. diagnosis is not usually made until after metastasis b. the primary site is intracranial c. the primary site is the bone marrow d. diagnosis is made based on the location of the primary site

a. diagnosis is not usually made until after metastasis

prognosis of leukemia depends on

age at diagnosis initial WBC count type of cells involved sex of the child

What antiviral agent is used to treat varicella infections in children at increased risk for complications associated with varicella? a. Varicella-zoster immune globulin. b. Acyclovir. c. Salicylates. d. Steroids.

b. Acyclovir.

which of the following statements about ipecac use for poisonous substance ingestion is true a. ipecac helps to absorb toxin b. ipecac is no longer recommended for routine home treatment of poisoning c. ipecac is useful when corrosive substance has been ingested d. ipecac useful when overdose of Ca channel blocker is ingested

b. ipecac is no longer recommended for routine home treatment of poisoning

Mumps immunization: a. is recommended for children ages 4 years of age or older. b. is typically given in combination with measles and rubella. c. can be administered to infants as young as 6 months of age. d. All of the above.

b. is typically given in combination with measles and rubella.

an 8 year old is to receive a chemotherapeutic drug in the outpatient clinic. His mother comments that he is not eating well because he gets so nauseous with the chemo. one way to minimize these effects is to: a. administer an antiemetic when the child complains of nausea b. avoid foods in the room that may make him nauseous c. administer an antiemetic 30 minutes to 1 hour before the chemo infusion d.omit the chemo therapeutic drug from his regimen

c. administer an antiemetic 30 minutes to 1 hour before the chemo infusion

Which of the following may be used to treat positional plagiocephaly in the first few months of life? a. Serial casting. b. Molded helmet. c. alternating head position when sleeping d. Ventricular shunting

c. alternating head position when sleeping

complicated grief reactions a. usually take about a year b. are accompanied by support of the family at the funeral c. may extend over years d. can be eleminated if the family is well prepared

c. may extend over years

Which one of the following symptoms would be considered normal grief behavior? a. Depression b. Anger c. Hearing the dead person's voice d. All of the above

d. All of the above

Certain groups of children are at risk for serious complications from communicable diseases. These children do not include which of the following groups? a. Children with an immunodeficiency or immunologic disorder. b. Children receiving steroid therapy. c. Children with leukemia. d. Children who have recently undergone a surgical procedure.

d. Children who have recently undergone a surgical procedure.

When instructing the parents caring for an infant with conjunctivitis, the nurse will include which of the following in the plan? a. Accumulated secretions are removed by wiping from out canthus inward. b. Hydrogen peroxide placed on cotton swabs is helpful in removing crusts from eyelids. c. Compresses of warm tap water are kept in place on the eye to prevent crusting. d. Washcloth and towel used by the infant are kept separate and not used by others.

d. Washcloth and towel used by the infant are kept separate and not used by others.

parents sometimes view the child's moon face from steroids as an appearance of: a. an anorexic, undernourished child b. a malnourished child with a swollen abdomen c. an overweight child but undernourished d. a well nourished, healthy child

d. a well nourished, healthy child

in identification of the abused child the nurse knows a. physical abuse can be readily identified during the physical examination b. specific behavioral problems can be seen in the abused child c. maltreated children easily admit to the abuse they received from their parents d. incompatability between the hx and the injury is probably the most important criterion on which to base the decision to report suspected abuse

d. incompatability between the hx and the injury is probably the most important criterion on which to base the decision to report suspected abuse

according to erikson, what is the developmental goal of middle childhood? a. autonomy b. trust c. initiative d. industry

d. industry

Nursing goals in the treatment of a communicable disease are

identification, prevention of transmission, provision of comfort, and prevention of complications.

major factor in fatal accidents/ motor vihicle/infants

lack of proper restraints

Antitumor antibiotics

natural products that interfere with cell division by reacting with DNA to prevent further replication of DNA and transcription of RNA doxorubicin daunomycin

hydrocarbons poisoning treatment

no inducing vomiting may be gastric lavage ( cuffed endotracheal tube to prevent aspiration) symptomatic treatmenbt of chemical pneumonia - high humidity, oxygen, hydration, acitaminophen

symptoms of cancer in children

pain fever swelling and tenderness mass low platelet count ecchymosis and petechiae spontaneous gum or nose bleed palor, lethargy, weight loss, malaise swallen lympth nodes

lead stays in body how

settles in bones and teeth where it remains in storage lead poisoning effects what systems renal, hematologic, and neurologic what is of most concern in kids with lead poisoning developing brain and nervous systmes why is there a relationship between lead poisoning and iron deficiency anemia they absorb the lead more easily children usually do or do not die from lead poisoning

SUEND and SUID

similar features but differ in regard of timing: SUID - postneonatal period SUEND - first week of life

administration vaccination

- protect potency - institution of recommended procedures - refrigirated vaccine should be stored on middle shelf - wrap in aluminum foil to protect from light - use before expiration date - rotate sites for multiple doses - inject less painful first - accurate documentation

osteosarcoma

-arises from bone-forming mesenchyme Osteosarcoma occurs.. in wider part of bone shaft, adjacent to epiphyseal growth plate of long bones -more than 50% occur in femur risks: - ionizing radiation exposure, genetic predisposition, history of retinoblastoma Tx of osteosarcoma: -surgery -chemo -limb salvage with prosthetic bone implanted What might occur with osteosarcoma? amputation -provide stump care -teach about prosthetics -phantom limb pain

Bone tumors:

-osteosarcoma -Ewing sarcoma Bone tumors occur around.. age 15 Bone tumors more common in.. males Sx of bone tumors: -growing pain type, discomfort at site -lump -become inactive or avoid carrying objects Dx bone tumors: -CT -MRI -needle bx ewing sarcoma mostly - pelvis, long bones, lowerextremities, chest wall osteosarcoma - metapheseal part of the bone, long bones

Although puberty can begin in either gender after the age of 8 years, at what age does puberty generally begin for girls and boys? 8 years for girls, 10 years for boys 9 years for girls, 11 years for boys 12 years for girls, 10 years for boys 10 years for girls, 12 years for boys

10 years for girls, 12 years for boys Rationale Puberty generally begins in girls at 10 years of age and at 12 years of age in boys.

A parent asks the nurse about the "characteristics of a sleep terror." What response should the nurse give to the parent? (Select all that apply.) a. The child screams during the sleep terror. b. Return to sleep is delayed because of persistent fear. c. The night terror occurs during the second half of night. d. The child has no memory of the dream with a sleep terror. e. The child is not aware of another's presence during a sleep terror.

A, D, E During sleep terrors, the child screams and has no memory of the dream. The child is not aware of another's presence during a sleep terror. Return to sleep is usually rapid with a sleep terror, but it is delayed with a nightmare. The sleep terror occurs usually within 1 to 4 hours of sleep, but nightmares occur during the second half of night.

The school nurse is teaching female school-age children about the average age of puberty. What is the average age of puberty for girls? a. 10 years b. 11 years c. 12 years d. 13 years

ANS: C The average age of puberty is 12 years in girls.

What are classified as hydrocarbon poisons? (Select all that apply.) a. Bleach b. Gasoline c. Turpentine d. Lighter fluid e. Oven cleaners

B, C, D Gasoline, turpentine, and lighter fluid are classified as hydrocarbon poisons. Bleach and oven cleaners are classified as corrosive poisons.

The nurse is interviewing the parents of a 4-month-old boy brought to the hospital emergency department. The infant is dead, and no attempt at resuscitation is made. The parents state that the baby was found in his crib with a blanket over his head, lying face down in bloody fluid from his nose and mouth. The nurse might initially suspect his death was caused by what? a. Suffocation b. Child abuse c. Infantile apnea d. Sudden infant death syndrome (SIDS)

D The description of how the child was found in the crib is suggestive of SIDS. The nurse is careful to tell the parents that a diagnosis cannot be confirmed until an autopsy is performed.

most chemotherapeutic drugs act to suppress cancer cells by interfering with the cells function or production of ___ and ___

DNA RNA

Common places for osteosarcoma

Found in the metaphysis of long bones, especially in the lower extremities, with most tumors occurring in the femur

Which term best explains the ability of an organism to resist a particular agent by the action of specific antibodies? Toxin Vector Virulence Immunity

Immunity Immunity refers to the ability of an organism to resist a particular agent by the action of specific antibodies.

diagnostic procedures for cancer

LP bone marrow bone marrow biopsy CT, MRI, PET, MIBG pathologic and molecular evaluation

Fifth disease

Rash appears in three stages; stage I is erythema on face, chiefly on cheeks.

meds for pertussis

azithromycin (younger than 1 month) and erythromycin clarithromycin

dental care for a child whose platelet count is 32,000 and granulocyte count is 450 should include daily: a. toothbrushing with flossing b. toothbrushing without flossing c. flossing without toothbrushing d. wiping with moistened sponges

d. wiping with moistened sponges

The toddler stage

extending from 12 to 36 months, is a period of intense exploration of the environment.

exhibitionism

indecent exposure

palliative care advantages

prolongs life improves quality of care dicreases hospital costs dicreases caregiver burden

causes of sids

sleep apnea is not a cause genetic predisposition maternal smoking young maternal age substance abuse

immunotherapy

stimulates the activity of the immune system against cancer cells or by counteracting signals produced by cancer cells that supress immune responses momoclonal antibodies checkpoint inhibitors cytokines - interferons, interleukins oncolytic virus chimeric antigen receptor

tylenol poisoning

tylenol overdose clinical manifestations occurs in how many stages 4 Stage 1 acetaminophen toxicity 0 to 24 hours; minor, nonspecific signs and symptoms; anorexia, nausea, vomiting, pallor, malaise Stage 2 acetaminophen toxicity day 2 to 3; right upper quadrant pain, patient improves Stage 3 acetaminophen toxicity 3-4 days; vomiting, confusion, stupor, coagulation abnormalities, renal failure, pancreatitis Stage 4 acetaminophen toxicity more than 5 days; resolution or progress to multiple organ failure Toxic dose of acetaminophen 150 mg/kg within 24 hours

Body substance isolation

Designed to reduce the risk of transmission of pathogens from moist body substances

Supratentorial tumor

Located within the anterior two thirds of the brain mainly the cerebellum

Scarlet fever

Tonsils are enlarged, edematous, reddened, and covered with patches of exudate; rash is absent on face; desquamation occurs.

HPV vaccine

cutaneous and genital risk of cancer gardasil 9 IM - deltoid - in 3 doses - 11-12 uo, 1-2 months after first, 6 moths after first boys girls 9-26

infsants and toddlers

vague separation fact impossible to comprehent absence of life behaviorsal response might include regression perceive seriousness of situation from parents reaction

Teaching a child the steps to hand washing is an intervention to minimize the risk for which nursing assessment? Fatigue Risk for infection Isolation from friends Risk for fever and chills

Risk for infection Teaching a child the steps to hand washing is an appropriate intervention for risk of infection. An appropriate intervention for this assessment is to encourage the parent and child to take steps to break the chain of infection.

What statement accurately describes physical development during the school-age years? The child's weight almost triples. The child grows an average of 5 cm (2 inches) per year. Few physical differences are apparent among children of different genders at the end of middle childhood. Fat gradually increases, which contributes to the child's heavier appearance.

The child grows an average of 5 cm (2 inches) per year. Correct In middle childhood, growth in height and weight occurs at a slower pace. Between the ages of 6 and 12 years, children grow 5 cm (2 inches) per year. In middle childhood, children's weight will almost double; they gain 3 kg (6.6 pounds) per year. At the end of middle childhood, girls grow taller and gain more weight than boys. Children take on a slimmer look with longer legs in middle childhood.

Mumps

This condition results in earache that is aggravated by chewing.

lymphomas

a group of neoplastic diseases that arise from the lymphoid and hematopoietic systems, are divided into Hodgkin disease and non-Hodgkin lymphoma (NHL). These diseases are further subdivided according to tissue type and extent of disease (staging). In children, NHL is more common than Hodgkin disease.

nightmare

a scary dream in REM sleep that wakes up the child

bacterial infection

accounts for most instances of acute conjunctivitis in children. Diagnosis is made primarily from the clinical manifestations. Treatment of conjunctivitis depends on the cause. Viral conjunctivitis is self-limiting, and treatment is limited to removal of the accumulated secretions. Bacterial conjunctivitis has traditionally been treated with topical antibacterial agents such as polymyxin and bacitracin (Polysporin), sodium sulfacetamide (Sulamyd), or trimethoprim and polymyxin (Polytrim).

child maltreatment

intentional physical abuse or neglect, emotional abuse or neglect, sexual abuse of children

The parents of infant twins, both with developmental delays, ask the nurse about resources for caring for the children as they get a little older. Which information should the nurse share? "Your family will find a pattern of normalcy that will make caring for the children easier." "Medical homes can offer holistic and centralized care that include the patient and family as the primary focus." "There are support groups and community resources available to help cope with caring for children with special health care needs." "Having two children with developmental delays likely means that the children will need to be institutionalized to receive adequate care."

"Medical homes can offer holistic and centralized care that include the patient and family as the primary focus." With two children needing care, the family will need support and care that is centered on the family. A medical home can provide the care and community resources needed to care for two children with developmental delays.

Leukemia:

-broad term given to a group of malignant diseases of bone marrow and lymphatic system -unrestricted proliferation of immature WBCs in blood 2 forms of leukemia recognized in children: 1. acute lymphocytic leukemia (ALL) 2. acute myelogenous leukemia (AML) Is leukemia a tumor? no but demonstrates properties of solid cancers What's most severely effected from leukemia? spleen liver Although leukemia is an overproduction of WBCs.. the WBC count is low with large amount of immature cells What occurs with leukemia? cellular destruction through the process of infiltration and subsequent competition for metabolic elements Proliferating cells depress bone marrow production by competing with normal cells for essential nutrients of metabolism causing: -anemia from decreased erythrocytes -infection from neutropenia -bleeding from decreased platelet production

warning signs of abuse

-physical evidence -history doesn't match injury -explanation of injury is vague -brought in with minor complaint and significant trauma is found out -saying child hurt themselves when not possible developmentally -brought in by unrelated adult -delay in seeking care -affect is inappropriate

how to talk to children who reveal abuse

-privately -do not promise not to tell- required by law -do not express shock or judgement -use their vocab for body parts -reassure they did right thing by telling -abuse is not their fault -determine immediate need for safety -let them know what will happen after report

neurologic problems

-severe constipation caused by decreased bowel innervation ( physical activity and stool softener, laxatives, fiber -foot drop (weakness and numbness) -weakness ( adjust school, activities) -numbness of extremities -severe jaw pain neurologic syndrome, posriiradiation somnolence - somnolence, fever, anorexia, nausea, vomiting

The mother of a 4-year-old child tells the pediatric nurse that the child's abdomen seems to be swollen. During further assessment of subjective data, the mother tells the nurse that the child is eating well and that the activity level of the child is unchanged. The nurse, suspecting the possibility of Wilms' tumor, should avoid which during the physical assessment? 1. Palpating the abdomen for a mass 2. Assessing the urine for the presence of hematuria 3. Monitoring the temperature for the presence of fever 4. Monitoring the blood pressure for the presence of hypertension

1. Palpating the abdomen for a mass Wilms' tumor is the most common intraabdominal and kidney tumor of childhood. If Wilms' tumor is suspected, the tumor mass should not be palpated by the nurse. Excessive manipulation can cause seeding of the tumor and spread of the cancerous cells. Hematuria, fever, and hypertension are clinical manifestations associated with Wilms' tumor.

A child with leukemia is found to have 1000 white blood cells with 6% segmented and 8% nonsegmented neutrophils. What is the absolute neutrophil count (ANC) of this child? Record your answer using a whole number.

140 Rationale A child with leukemia is at a greater risk of life-threatening infections. Therefore, the child is allowed to go to school only when the white blood cell count (WBC) is found to be optimal. An ANC greater than 500/mm 3 is usually satisfactory. The formula used to determine ANC is as follows: ANC = WBC x % of neutrophils (including segmented, nonsegmented, and polys). Therefore, in this scenario ANC = 1000 x (6% + 8%) = 1000 x 14% = 1000 x 0.14 = 140

The nurse is assessing a child suspected of having pinworms. Which is the most common symptom the nurse expects to assess? a. Restlessness b. Distractibility c. Rectal discharge d. Intense perianal itching

ANS: D Intense perianal itching is the principal symptom of pinworms. Restlessness and distractibility may be nonspecific symptoms. Rectal discharge is not a symptom of pinworms.

Which specific nursing interventions are implemented in the care of a child with leukemia who is at risk for infection? Select all that apply. 1. Maintain the child in a semiprivate room. 2. Reduce exposure to environmental organisms. 3. Use strict aseptic technique for all procedures. 4. Ensure that anyone entering the child's room wears a mask. 5. Apply firm pressure to a needle stick area for at least 10 minutes

2. Reduce exposure to environmental organisms. 3. Use strict aseptic technique for all procedures. 4. Ensure that anyone entering the child's room wears a mask. Leukemia is a malignant increase in the number of leukocytes, usually at an immature stage, in the bone marrow. It affects the bone marrow, causing anemia from decreased erythrocytes, infection from neutropenia, and bleeding from decreased platelet production (thrombocytopenia). A common complication of treatment for leukemia is overwhelming infection secondary to neutropenia. Measures to prevent infection include the use of a private room, strict aseptic technique, restriction of visitors and health care personnel with active infection, strict hand-washing, ensuring that anyone entering the child's room wears a mask, and reducing exposure to environmental organisms by eliminating raw fruits and vegetables from the diet and fresh flowers from the child's room and by not leaving standing water in the child's room. Applying firm pressure to a needle stick area for at least 10 minutes is a measure to prevent bleeding.

What are classified as corrosive poisons? (Select all that apply.) a. Batteries b. Paint thinner c. Drain cleaners d. Mineral seed oil e. Mildew remover

A, C, E Batteries, drain cleaners, and mildew removers are classified as corrosive poisons. Paint thinner and mineral seed oil are classified as hydrocarbon poisons.

Which common plants have toxic parts that could be poisonous to children? Select all that apply. Boston fern Rose leaves Apple seeds Holly berries Azalea flowers Buttercup petals

Apple seeds Holly berries Azalea flowers Buttercup petals Rationale Common plants that have toxic parts that could be poisonous to children include apple seeds, holly berries, azalea flowers, and buttercup petals. Nonpoisonous plants include Boston ferns and roses.

The nurse knows that the hyperventilating patient is experiencing which intracranial processes to maintain proper oxygenation? Select all that apply. Autoregulation Cerebral vasodilation Cerebral vasoconstriction Decreased cerebral blood flow Decreased intracranial pressure

Cerebral vasoconstriction A patient who is hyperventilating will have decreased PaCO2 and therefore have vasoconstriction. The endothelial cells of the vasculature will constrict in the presence of increased levels of oxygen but not carbon dioxide. Decreased cerebral blood flow A patient who is hyperventilating will have decreased cerebral blood flow because of decreased PaCO2. The decrease in cerebral blood flow occurs because as the vasculature vasoconstricts, it decreases the caliber of the vessel for blood to flow through. Autoregulation A patient who is hyperventilating will autoregulate to maintain a steady blood flow to the brain.

The nurse is working in the emergency department when a disheveled 4-year-old patient is brought in with an injured arm that is likely fractured. Documentation reveals a prior visit for a fractured tibia and fibula. The child's parents report the injury occurred 3 days ago and are anxious to leave. Which findings would lead the nurse to initiate the abuse reporting protocol? Select all that apply. Arm fracture Parents anxious to leave Delay in seeking treatment Child's disheveled appearance The patient states that she feels isolated from her friends.

Delay in seeking treatment The fact that the parents waited 3 days to seek treatment for the child's arm indicates neglect and the need to initiate the abuse reporting protocol. Correct Child's disheveled appearance The child's disheveled appearance indicates neglect and the need to initiate the abuse reporting protocol. Correct The patient states that she feels isolated from her friends. Feelings of being isolated from friends can be a sign that the child is living in an abusive household. Arm fracture The patient's current fracture is a sign of physical abuse, since it is a repeat occurrence, indicating the need to initiate the abuse reporting protocol.

Transmission-Based Precautions

Designed for patients documented or suspected to be infected or colonized with highly transmissible or epidemiologically important pathogens for which interventions beyond Standard Precautions are needed to interrupt transmission in hospitals

A nurse is studying rates of immunization among immigrant families in a large city. The nurse plans to gather data on how improved communication affects the rate of immunization in this population. Which are examples of communication that the nurse would expect to improve immunization rates? Select all that apply. Documentation of vaccinations administered Communication to patients and families about vaccine costs Conversation among patients about personal vaccination experiences Use of interpreters for teaching patients and families with a language barrier Providing vaccine information sheets prior to obtaining consent to vaccinate

Documentation of vaccinations administered Timely and accurate documentation of all vaccinations administered improves communication between health care providers and facilitates the efficient delivery of immunizations according to the recommended schedule. Use of interpreters for teaching patients and families with a language barrier Health care providers should use an interpreter to teach patients and families with a language barrier about the importance and risks associated with vaccines. This allows providers to answer questions and to be the teaching has been effective. Correct Providing vaccine information sheets prior to obtaining consent to vaccinate Providing the patient and family with vaccine information sheets prior to obtaining consent to vaccinate will help answer some of the common questions parents have about the vaccines their child will receive.

Which preventative measure is appropriate regarding protection from lead exposure, specifically for a child living in an environment with considerable lead content? Eat regular meals. Eat a high-fat diet. Drink only warm water. Eat only from ceramic ware.

Eat regular meals. Rationale When a child is exposed to toxic levels of lead in the immediate environment, it is necessary to ensure that the child eats regular meals. This acts as a preventive measure, because lead is less absorbed on a full stomach. A child prone to lead exposure should have sufficient iron and calcium supplements in the diet, not a high-fat diet. Since hot water dissolves lead faster, the child should be always provided with cool water. It is advisable to refrain from using ceramic and earthenware that may not be adequately fired, because they lead to direct lead exposure.

Which methods are common causes of lead exposure for a child? Select all that apply. Eating junk food Eating loose paint Ingestion of lead dust Eating food from aluminum pans Drinking water from old plastic pipes

Eating loose paint Ingestion of lead dust Rationale Common methods of lead exposure for a child are eating loose paint and ingestion of lead dust. Eating junk food, eating from aluminum pans, or drinking from old plastic pipes do not expose a child to lead.

A seven-year-old child with a brain tumor appears irritable and refuses to eat. The nurse notes that the patient is exhibiting bulging eyes. The parents report recent personality changes and a drop in grades. Which provider order would the nurse implement first to address the patient's symptoms? Elevate head of bed Administer IV fluid bolus Identify academic support Give oxygen via nonrebreather

Elevate head of bed The patient is experiencing signs of increased intracranial pressure related to the brain tumor. Elevating the head of the bed promotes drainage of CSF and should be implemented for this patient.

The school nurse is teaching dental health to a class of first graders. What would be included in the teaching? Teach how to floss teeth properly. Recommend a toothbrush with hard nylon bristles. Emphasize the importance of brushing before bedtime. Recommend nonfluoridated toothpaste approved by the American Dental Association.

Emphasize the importance of brushing before bedtime. Correct Brushing before bedtime is especially important because there is more time overnight for interaction between bacteria and unremoved substrate on the tooth surfaces. Flossing is important, but most first graders do not have the manual dexterity to floss properly. Soft nylon bristles are recommended for school-age children. Fluoridated toothpastes are recommended

What instructions would the nurse include when teaching the parents of school-age children ways to reduce the risk of injury during sports activities? Select all that apply. Encourage lots of competitive activities. Discourage participation in sports at this age. Encourage physical activity outside of sports. Have children wear helmets and other protective equipment. Have children compete against kids who are about the same size.

Encourage physical activity outside of sports. Have children wear helmets and other protective equipment. Have children compete against kids who are about the same size. Rationale When instructing the parents of school-age children in ways to reduce the risk of injury in sports, the nurse should stress the importance of wearing helmets and proper equipment, competition with children who are close to the same size, and regular physical activities outside of sports to keep the children in shape. Discouraging sports participation at this age is not helpful because most school-age children enjoy sports. Encouraging lots of competitive activities is not recommended because too much competition at a young age can lead to sport-related injuries.

Match the listed cancer treatment with its corresponding purpose.

Enhance cell recovery Biologic agents An adjunct for nausea control Steroid therapy Replace conventional therapy based on current scientific research CAM therapies

A four-month-old infant is in the clinic to receive vaccinations. The infant experienced redness and swelling at the injection site following the first dose of diphtheria, tetanus, & acellular pertussis (DTaP). The infant's parent asks whether the infant should receive the second dose. How should the nurse respond? Tell the parent that the infant must have the next dose of DTaP. Advise the parent not to repeat the DTaP after this serious reaction. Explain that this was a mild reaction and is not a contraindication for the next dose. Inform the parent that the infant should not receive another dose until age 12 months

Explain that this was a mild reaction and is not a contraindication for the next dose. The nurse should explain that localized redness and swelling are common mild reactions and these are not considered contraindications for the next dose of DTaP.

The nurse informs the family about which symptoms they might see as their child passes? Select all that apply. Fatigue Seizures Tardive dyskinesia Sleep disturbances Restlessness, agitation

Fatigue Seizures Sleep disturbances Restlessness, agitation Rationale Common central nervous symptoms experienced by dying children include fatigue, seizures, sleep disturbances, restlessness, and agitation. Tardive dyskinesia is a side effect associated with certain medication use but not directly a symptom experienced by dying children.

Failure to wash one's hands before eating and/or cooking facilitates which mode of pathogen transmission? Airborne Fecal-Oral Vector-borne Sexual/Saliva/Blood

Fecal-Oral Failure to wash one's hands before eating and/or cooking is an example of what may facilitate fecal-oral transmission. Fecal-oral transmission is usually associated with organisms, such as Salmonella, that infect the digestive system. Poor hand washing is a common cause.

An 8-year-old patient is having problems adjusting to daily radiation treatments. Which is the most appropriate action for the nurse to take to help with the adjustment? Collaborate with the oncology radiologist. Encourage the parents to comfort the child. Have a discussion with the child life specialist. Educate the child on the importance of the treatments.

Have a discussion with the child life specialist. Use of the child life specialist uses methods that are age-appropriate, educational, supportive, and therapeutic and may include medical play and art and therapeutic play and art. Therefore, this would be the most appropriate action.

The parents of a seven-year-old child report swelling in the child's neck and recent fatigue. They are apprehensive about the diagnosis and ask the nurse about the progression of Hodgkin lymphoma. Which information should the nurse include in the response? Hodgkin Lymphoma begins in the lymph nodes. Hodgkin Lymphoma is an unpredictable condition that spreads rapidly. Hodgkin lymphoma can begin in any site involved in WBC production or storage. Hodgkin lymphoma affects the maturation of WBC and eventually leads to obliteration of the patient's immune function.

Hodgkin Lymphoma begins in the lymph nodes. Hodgkin lymphoma originates in a single lymph node or a group of lymph nodes in the same anatomic region.

An adolescent female comes to the clinic and the nurse finds out the patient has been having unprotected intercourse. Which nursing assessment might the nurse include in the plan of care? Fatigue Isolation from friends Risk for fever and chills Inability to meet routine health care needs

Inability to meet routine health care needs An adolescent female lacking understanding of the importance of barrier methods of contraception supports the assessment of inability to meet routine health care needs. This assessment supports healthy behaviors related to STI and health maintenance.

What is a source of stress for both 7-year-olds and 9-year-olds? Moodiness Interruptions Rebelliousness Aversion to the opposite sex

Interruptions Rationale Interruptions are a source of stress for both 7-year-old and 9-year-old children. The 7-year-old hates to be disturbed when intensely involved in an activity. The 9-year-old continues to dislike interruptions; however, the 9-year-old will usually resume an activity after an interruption. Moodiness is a source of stress for the 7-year-old; this child is often moody, unhappy, or pensive. This is not a particular problem faced by 9-year-olds. Rebelliousness is a source of stress for the 9-year-old; this child occasionally tests independence by rebelling. A 7-year-old child has not yet started to rebel. Aversion to the opposite sex is a source of stress in the 9-year-old who prefers to engage in sex-segregated play. A 7-year-old starts to become more selective about playmates; however, aversion to the opposite sex is not specifically observed in children of this age.

An adolescent with a seizure disorder is admitted to the hospital after a fall. The child is increasingly withdrawn and depressed. Which action should the nurse take to best care for this adolescent? Sit with the patient on the nurse's breaks and lunch period to decrease the teen's withdraw Accept the changes in behavior as a temporary regression in an adolescent patient that will quickly resolve on their own Encourage the family to visit with the adolescent more often and help make decisions for the teen to decrease the teen's stress level Make arrangements for more peer interaction opportunities that might include increased visitation, social media, or other opportunities to connect

Make arrangements for more peer interaction opportunities that might include increased visitation, social media, or other opportunities to connect The child is showing signs of depression, which could be from missed socialization with peers. If the nurse can accommodate extra visitors safely, then it should be done with permission from administration. If peers are not allowed in the hospital unit, a phone may be provided to allow the child to speak with friends.

A child receiving radiotherapy for cancer has nausea and vomiting. Which interventions should the nurse consider in such a case? Select all that apply. Limit fluid intake. Make the food appealing. Measure the amount of emesis. Give antiemetic when the child has nausea. Give the child a light, soft diet and small frequent meals.

Make the food appealing. Measure the amount of emesis. Give the child a light, soft diet and small frequent meals. Rationale Side effects of radiotherapy include nausea and vomiting and the child may not want to take food and fluids. The nurse should encourage the child to eat by making any meals offered as appealing as possible. The amount of emesis is measured to monitor the hydration status of the body. The child should be given a light, soft diet and small, frequent meals. The child must be given enough fluids to maintain a healthy hydration level. Antiemetics should be administered on schedule around the clock and should not be given only when the child has nausea.

A 2-year-old child comes to the emergency department with a substantial acetaminophen overdose. Which drug-specific medication should the nurse anticipate administering to this patient? Naloxone N-acetylcysteine Activated charcoal Diluted oil of wintergreen

N-acetylcysteine N-acetylcysteine is an antidote used for significant acetaminophen ingestion.

What is the leading cause of death from disease in children past infancy? Seizure disorders Endocrine disorders Neoplastic disorders Cardiovascular disorders

Neoplastic disorders Rationale Neoplastic disorders are the leading cause of death from disease in children past infancy, and almost half of all childhood cancers involve the blood or blood-forming organs. Seizure disorders are not the leading cause of death from disease in children past infancy; neither are endocrine or cardiovascular disorders.

A child with a brain tumor has undergone craniotomy. Which are the postoperative nursing care measures to undertake once the child is alert? Select all that apply. Note vital signs every two hours. Position the child in the Trendelenburg position. Observe the dressings for evidence of drainage. Restrain the child from touching the head. Check handgrip reflexes and functioning of the cranial nerves.

Observe the dressings for evidence of drainage. Restrain the child from touching the head. Check handgrip reflexes and functioning of the cranial nerves. Rationale The nurse should observe the dressings for evidence of drainage; any presence of colorless drainage requires immediate medical attention. Once alert, the child should be put in an elbow restraint and other additional restraints to prevent the hands from reaching the head. The restraints will also preserve the infusion line and maintain a specific position. The nurse should check the muscle strength, reflexes, and functioning of cranial nerves as soon as possible. Vital signs should be taken as often as every 15 to 30 minutes, because the child may develop sudden hypothermia postoperatively. The Trendelenburg position increases intracranial pressure and the risk of hemorrhage, and should be avoided.

Which substances contributed to high-lead blood levels in the early 1900s? Select all that apply. Glue Paint Gasoline Play dough Paint thinner

Paint Gasoline Rationale Paint and gasoline posed the greatest risk for high-lead blood levels for the public in the early 1900s. Glue, play dough, and paint thinner were not and are not sources of concern for lead poisoning.

grief

Parental grief after the death of a child has been found to be the most intense, complex, long-lasting, and fluctuating grief experience when compared with that of other bereaved individuals. Grief is not an event, but a process of experiencing physiologic, psychologic, behavioral, social, and spiritual reactions to the loss of a child. Acute grief is a syndrome with intense and distressing psychologic and somatic symptoms that appear at the time of death. Acute grief is a syndrome with psychologic and somatic symptomatology that may appear after the crisis or may be delayed, exaggerated, or apparently absent. "Distorted" reactions may represent one aspect of the syndrome and can be transformed into normal grief work. Mourning, or grief work, consists of four phases that do not necessarily proceed in sequence and may recur at any time: shock and disbelief, expression of grief, disorganization and despair, and reorganization. Formal bereavement programs may assist families in coping with the loss of a child and in the process of grief resolution.

Surgery nephroblastoma

Performed to remove as much of the tumor as possible and to obtain biopsy specimens; in the early stages, complete surgical removal of the tumor is the treatment of choice in later stages surgery only for biopsy due to the extensive metastasis

Which factors can contribute to the child's risk for lead poisoning? Select all that apply. Poverty Living in rural areas Younger than 6 years old Living in older rental homes Use of Ba-Baw-San for colic

Poverty Younger than 6 years old Living in older rental homes Use of Ba-Baw-San for colic Rationale Risk factors that can contribute to lead poisoning are poverty, age younger than 6 years old, living in older rental homes, and use of Ba-Baw-San for colic. Living in urban areas (not rural) is a risk factor for lead poisoning.

Which system is appropriate to monitor when the child is being treated with intravenous ethylenediaminetetraacetic acid (EDTA) for lead poisoning? Lipid Renal Cardiac Pulmonary

Renal Rationale The clinical presentation and history of the child show that the child is suffering from lead poisoning. Lead and EDTA are toxic to the kidneys; therefore the nurse must keep records of intake and output and send the child's urine for urinalysis. The urinalysis results should be monitored closely to detect for any change in renal functioning. Monitoring of lipid profile, cardiac function, and lung function is not required, because lead and EDTA do not cause related toxicities

What best describes a neuroblastoma? The diagnosis is usually made after metastasis occurs. It is the most common brain tumor in young children. Older children have an improved likelihood of survival. Early diagnosis is usually possible because of the obvious clinical manifestations.

The diagnosis is usually made after metastasis occurs. Correct Neuroblastoma is considered a "silent" tumor. In more than 70% of cases, the diagnosis is made after metastasis occurs. The neuroblastoma is usually located in the abdomen. Children younger than 1 year of age have a 75% survival rate; those older than age 1 year, only 50%. The diagnosis is not usually made until other sites have been invaded by the tumor.

What is the similarity between acute lymphoblastic leukemia (ALL) and acute myelogenous leukemia (AML)? High number of leukocyte cells Incidence more frequent in boys than in girls Formation of tumor in the blood-forming tissues Unrestricted proliferation of immature white blood cells

Unrestricted proliferation of immature white blood cells Rationale Acute leukemia is an unrestricted proliferation of immature white blood cells in the blood-forming tissues in the body. This is the cause of both acute lymphoblastic leukemia (ALL) and acute myelogenous leukemia (AML). In leukemia, there is an overproduction of white blood cells, but most often the leukocyte count is low. ALL is more common in boys than in girls. However, the incidence of AML is similar for both boys and girls. The leukemic cells of both AML and ALL do not form a tumor as such, but they have neoplastic properties of solid cancers.

Which conjunctivitis condition is likely to be found in the patient with an upper respiratory tract infection, serous drainage, inflamed conjunctiva, and swollen eyelids? Viral Allergic Bacterial Caused by a foreign body

Viral Rationale Conjunctivitis is inflammation of the conjunctiva. Upper respiratory tract infection, serous drainage, inflamed conjunctiva, and swollen eyelids are symptoms suggestive of viral conjunctivitis. Itching in the eye, stringy discharge, inflamed conjunctiva, and swollen eyelids are symptoms of allergic conjunctivitis. Purulent drainage, crusting of eyelids (especially on awakening), inflamed conjunctiva, and swollen eyelids are symptoms of bacterial conjunctivitis. Tearing, pain, and inflamed conjunctiva are symptoms of conjunctivitis caused by a foreign body. In foreign body conjunctivitis, usually only one eye is affected.

Which nursing actions are appropriate for a patient with red and edematous conjunctivae with the presence of secretions? Select all that apply. Wipe the eyes from inner canthus to outer canthus. Instill corticosteroid eye drops to prevent infections. Use eye drops at night and ointments during the day. Instill the prescribed eye drops before cleaning the eyes. Do not apply any type of continuous compresses to the eyes

Wipe the eyes from inner canthus to outer canthus. Do not apply any type of continuous compresses to the eyes.

When a child dies suddenly, which one of the following interventions would be least beneficial? a. Avoid having the family view the body of a disfigured child. b. Inform the family of what to expect when they see the disfigured body of their child. c. Offer the parents the opportunity to see the child's body even after resuscitation was performed. d. Arrange to have a health care worker with bereavement training meet with the family.

a. Avoid having the family view the body of a disfigured child.

children with pinworm infection are seen with the principle symptom of: a. perianal itching b. diarrhea with blood c. evidence of small, ricelike worms in their stool and urine d. abdominal pain

a. perianal itching

Which one of the following techniques is recommended to provide atraumatic care for immunization administration to infants? a. Select a 25-mm needle to deposit vaccine deep into the muscle mass. b. Use an air bubble to clear the needle before injection. c. Use the deltoid muscle. d. Use the EMLA patch before administration.

a. Select a 25-mm needle to deposit vaccine deep into the muscle mass.

Which one of the following regarding palliative care is false? a. Serves to hasten death b. Provides pain and symptom management c. Promotes optimal functioning and quality of life d. Addresses issues faced by the family regarding death and dying

a. Serves to hasten death

Which of the following statements about Haemophilus influenzae type b (Hib) vaccine is true? a. The vaccine protects against a number of infections including bacterial meningitis, epiglotties, bacterial pneumonia, septic arthritis, and sepsis. b. The vaccine protects against the virus that produces influenza. c. Only two doses of Hib vaccine should be given to children 15 months of age or older who have not been previously vaccinated. d. Hiberix is a conjugate vaccine licenses for use in infants over the age of 2 months.

a. The vaccine protects against a number of infections including bacterial meningitis, epiglotties, bacterial pneumonia, septic arthritis, and sepsis.

gastric lavage for pediatric poison ingestion a. can be associated with serious complications of gi perforation, hypoxia, and aspiration b. is recommended in the emergency depatment for all cases of ingestion c. has been proven to decrease morbidity d. is most useful when the child comes to the emergency department within 3hrs of ingestion of toxin

a. can be associated with serious complications of gi perforation, hypoxia, and aspiration

according to piaget, what is the stage of development for middle childhood a. concrete operational b. preoperational c. formal operational d. sensorimotor

a. concrete operational

the nurse plans to conduct a sex education class for 10 year old. which of the following does the nurse identify as most appropriate for this age group? a. present sex information as a normal part of Growth & Development b. discourage question and answer sessions c. because sex info supplied by parents usually produces feelings of guilt and anxiety in children, avoid parental assistance in conducting the program d. segregate boys from girls and include info related only to same sex in the discussion

a. present sex information as a normal part of G & D

biologic response modifiers (BRMs)

agents that modify that relationship between tumor and host by therapeuticaly changing the host's reaction to tumor cells

influenza

annually for 6mo-18 yo inactivated vaccine IM 2 doses 4 weeks apart in first time younger than 9 yo intradermal for 18-64yo

When administering the pertussis vaccine the nurse recognizes which of the following as incorrect? a. Can be given any time during pregnancy. b. Cannot be given to the postpartum mother who is breastfeeding. c. The acellular pertussis vaccine, from the same manufacturer, is recommended for the first three immunizations and is given along with diphtheria and tetanus at 2, 4, and 6 months of age. d. Is recommended for health care workers having close contact with infants under the age of 12 months.

b. Cannot be given to the postpartum mother who is breastfeeding.

Fear of punishment is one of the greatest threats to seriously ill children of which age-group? a. Toddlers b. Preschoolers c. School-age children d. Adolescents

b. Preschoolers

The following statements are about hepatitis A illness or vaccination. Which one is correct? a. The illness has a gradual onset ,with often dark urine and jaundice being the only symptoms. b. The vaccine is recommended for all children between the ages of 12 months. c. The vaccine consists of a series of 3 injections timed 2 months apart. d. The illness is spread by all body secretions.

b. The vaccine is recommended for all children between the ages of 12 and 23 months.

Pinworm infection (enterobiasis)

begins when the eggs are ingested or inhaled (the eggs float in the air). The typical hand-to-mouth activity of youngsters makes them especially prone to reinfection. Pinworm eggs persist in the indoor environment for 2 to 3 weeks, contaminating anything they contact, such as toilet seats, doorknobs, bed linen, underwear, and food. Except for the intense rectal itching associated with pinworms, the clinical manifestations are nonspecific. Direct nursing care for pinworms at identifying the parasite, eradicating the organism, and preventing reinfection. Parents need clear, detailed instructions for the tape test. A loop of transparent (not "frosted" or "magic") tape, sticky side out, is placed around the end of a tongue depressor, which is then firmly pressed against the child's perianal area. The drugs available for treatment of pinworms include mebendazole (Vermox), pyrantel pamoate (Pin-Rid, Antiminth), and albendazole.

BRUE

brief resolved unexplained event infant younger than 1 yo during which an observer reports sudden but breif less than 1 min episode including at least 1 of the following: - cyanosis or pallor - absent, decreased, or irregular breathing - marked change i muscle tone ( hypertonia, or hypotonia) - altered responsiveness diagnosed only when there is no exlanation for the event

Fear of the unknown is one of the greatest threats to seriously ill children of which age-group? a. Toddlers b. Preschoolers c. School-age children d. Adolescents

c. School-age children

physical assessment for child sexual abuse a. includes documentation of only abnormal genital findings b. includes collecting forensic evidence obtained directly from a prepubertal victim's body as much as 3 days after the incident c. includes examination of the anal area d. includes documentation of the size of the hymenal opening, since it is predictive of sexual abuse

c. includes examination of the anal area

a major difference in moral development between young school age children and older school age children is best described as: a. younger children believe that standards of behavior come from within themselves b. children 6-7 y/o know the rules and understand the reasons behind the rules c. older school age children are able to judge an act by the intentions that prompted it and not only by the consequences d. rewards and punishments guide older school aged children behavior

c. older school age children are able to judge an act by the intentions that prompted it and not only by the consequences

DMSA administration

capsule opened and sprinkled on food or swollowed whole adverse effects - nausea, vomiting, diarrhea, loss of appetite, rash, elevated liver function test, neutropenia hydration essential ( because excreted via kidneys)

antimetabolites

chemicals that prevent cell division by inhibiting formation of substances necessary to make DNA; used in cancer chemotherapy methtrexate mercaptopurine

Nursing care during blood or bone marrow transplant:

child hospitalized for several weeks -extremely susceptible to infections-social isolation -any infection can be life threatening -skin breakdown and delayed wound healing

germ-line mutation

chromosome abnormalities that are not confined to the tumor alone but are presnet alsewhere; may be present in all cells

the most common bone cancer is most likely to occur in a child age: a. birth - 4/ infancy b. 4-8/ preschool c. 8-10/ school age d. 11 or older/ adolescence

d. 11 or older/ adolescence

emotional abuse

deliberate attempt to destroy child's self-estreem

hydrocabons agents of poisoning

gasoline kerosine lamp oil mineral seal oil lighter fluid turpentine paint thinnner and remver

body image toddler

occurs with increasing motor ability, at which point toddlers recognize the importance and capacity of body parts. A sense of maleness or femaleness— gender identity—is formed by age 3 years. The two phases of differentiation of self from significant others are separation and individuation.

sleep terrors

partial arousal from very deep REM sleep occurs 1-4hrs after falling asleep return to sleep is fast child usually doesn't stay awake but initially may sit up, scream, talk, bizarre behavior, which disappears when child is awake

alkylating agents

synthetic chemicals containing alkyl groups that attack DNA, causing strand breaks cyclophosphamide ifosfamide cisplatin dacarbazine

A nurse is teaching student nurses about the adaptive behaviors exhibited by children at different stages of development. What statement made by a student nurse indicates the need for further teaching? "A 12-year-old child can raise a pet." "A 7-year-old child can use a table knife for cutting meat." "A 5-year-old child can brush and comb hair acceptably without help." "A 9-year-old child can help with routine household tasks such as dusting and sweeping."

"A 5-year-old child can brush and comb hair acceptably without help." Rationale In general, a 7-year-old child can brush and comb hair acceptably without help; a 5-year-old child is too young to display such behavior. This statement by the student nurse indicates a need for further teaching. Children ages 10 to 12 can raise pets. A 7-year-old child can use a table knife for cutting meat, even though help may be needed with tough or difficult pieces. Children in the age group of 8 to 9 years can help with routine household tasks such as dusting and sweeping.

The mother of a 10-year-old questions the nurse regarding the method of chemotherapy administration and is concerned the child will require frequent IV insertions. With which statement should the nurse respond? "The injections will not hurt." "The peripheral IV will remain in place for up to 3 months." "Eventually, the child will get accustomed to the frequent injections." "A central venous catheter (CVC) may be implanted for long-term chemotherapy administration."

"A central venous catheter (CVC) may be implanted for long-term chemotherapy administration." A CVC will likely be placed for long-term chemotherapy administration and to prevent the possibility of infection related to frequent needle sticks.

The mother of a 14-year-old child who recently died tells the nurse that the 10-year-old sibling has become very withdrawn, angry, and aggressive. The child has said to the mother, "It should have been me." The mother asks the nurse how to respond to the child. Which responses by the nurse are appropriate? Select all that apply. "Your child's aggression is concerning. Is there a history of behavioral problems?" "A child's response to death is varied because of age and developmental level." "Your child is working through the stages of grief. Sometimes a child stays in one stage longer than others." "The child's response demonstrates the need for psychologic counseling. I can provide you with a referral." "Your child may be experiencing survivor's guilt related to the death of your child and needs to express feelings."

"A child's response to death is varied because of age and developmental level." Children respond to a sibling's death in a variety of ways based on their age and developmental level. Correct "Your child is working through the stages of grief. Sometimes a child stays in one stage longer than others." Children usually experience all 5 stages of grief, not always in order. They can also experience one stage several times as a way of coping with their feelings. This child is in the anger stage of grief. "Your child may be experiencing survivor's guilt related to the death of your child and needs to express feelings." Siblings often feel guilt for being the child who survived. Allowing the child to express feelings provides a way for those feelings to resolve.

The parents of a child newly diagnosed with a chronic illness ask the nurse how long the illness will last. How should the nurse respond? Select all that apply. "A chronic illness may have residual aspects that can last a lifetime." "A chronic illness is incurable, so the child will always struggle with the illness." "A chronic illness usually affects activities of daily living (ADLs) and lasts a long time." "A chronic illness can occur during a wide range of times and often spontaneously resolves." "A chronic illness lasts at least three months and up to a lifetime, depending on the illness."

"A chronic illness may have residual aspects that can last a lifetime." A chronic illness can have effects that last a lifetime, even if the active symptoms of the illness have resolved. "A chronic illness lasts at least three months and up to a lifetime, depending on the illness." Lasting at least three months is part of the definition of a chronic condition. Many chronic illnesses are incurable and therefore can last a lifetime.

A nurse is providing specific guidelines to parents for helping their children who are starting school this year. What information will the nurse include? "Accept the separation readily as your child starts attending school for the first time." "Express openly in front of the child that school has a different culture from home that you have little power to affect." "Remember that viewing television makes it difficult for the child to adjust by giving an unrealistic picture of what to expect in school." "Realize that a preschool program that focuses on developing social aspects is more effective in facilitating later academic achievement."

"Accept the separation readily as your child starts attending school for the first time." Rationale Successful adjustment of a child is directly related with the parents' readiness to accept the separation associated with school entrance. Clinging behavior demonstrated by the parents might be unconscious attempts to delay the child's maturity. Parents who view the school as an alien culture and one that they have little, if any, power to affect may unknowingly teach their children to be fearful and resentful toward it. This can occur even though the parents agree with school's purposes and objectives. It is more effective for parents to view school as a place that they have helped to create and support. Children receive information regarding the role of pupils in school from different sources, including the media, before they even start school. Television influences the acquisition of information and attitudes and provides anticipatory socialization to the children. It has been found that later academic achievement is facilitated by childhood programming that stresses cognitive development more than social aspects.

A nurse is teaching student nurses about the cognitive milestones reached by children at different stages of development. What statement made by a student nurse indicates effective learning? "A 7-year-old child can use the telephone for practical purposes." "An 8-year-old child reads for practical information or own enjoyment." "A 6-year-old child understands concepts of space and cause and effect." "An 11-year-old child can write occasional short letters to friends on self-initiative."

"An 11-year-old child can write occasional short letters to friends on self-initiative." Rationale An 11-year-old child can write occasional short letters to friends or relatives on self-initiative. This statement made by the nurse indicates effective learning. Children in the age group of 10 to 12 years can use the telephone for practical purposes; a 7-year-old usually cannot do so. Children in the age group of 10 to 12 years read stories or library books for practical information or enjoyment; an 8-year-old usually cannot do so. Children in the age group of 8 to 9 years understand the concepts of space, cause and effect, nesting puzzles, and conservation, that is, permanence of mass and volume; a 6-year-old usually does not.

Which question is appropriate to ask when admitting a child with a diagnosis of suspected meningitis? "Do you have a cough?" "Has the child had a recent injury?" "Are the immunizations up to date?" "Are there any pets in the household?"

"Are the immunizations up to date?" Rationale Immunizations are one of two public health interventions, clean public drinking water being the other, that have had the greatest impact on world health. Nurses should review individual immunization records at every clinical visit and hospitalization. Meningitis can be viral or bacterial, but is not caused by injury or transmitted from pets. Meningitis symptoms include headache, neck pain, fever, lethargy, nausea, vomiting, tachycardia, decreased appetite, and a rash.

Parents are devastated that their child is dying of cancer and express to the nurse that "they are going to do everything to keep their child alive." Which are the most appropriate responses by the nurse? Select all that apply. "Have you talked with your child about this decision? You must talk with your child soon." "The health care team will look at everything and revise the treatment plan accordingly." "Your actions may increase the suffering experienced by the child. You should think about this a little more." "As the child's advocate, the health care team must work together with the family to provide reasonable treatments for the child." "Together the family and the health care team must look at the child's needs to ensure we are not prolonging or worsening the child's condition with the treatments."

"As the child's advocate, the health care team must work together with the family to provide reasonable treatments for the child." The family and the health care team work together to remain the child's advocate with a focus on what the treatment is doing for the child versus what it is doing to the child. "Together the family and the health care team must look at the child's needs to ensure we are not prolonging or worsening the child's condition with the treatments." The health care team works with the family to be the child's advocate with a focus on what the treatment is doing for the child versus what it is doing to the child.

The nurse is explaining end-of-life care to the family of a dying child. Which statement by the nurse needs correction? "Encourage the child to talk." "Answer questions honestly." "Encourage family members to stay near the child." "Avoid explaining the physical effects the child will experience."

"Avoid explaining the physical effects the child will experience." Rationale The nurse should have the family members explain to the child in simple language about the physical effects of the end-of-life care on the child. The nurse should tell the family to encourage the dying child to talk and express feelings. The nurse should advise the family to honestly answer the child's questions and encourage the family members to stay close to provide support to the child.

The nurse is explaining end-of-life care to the family of a dying child. Which statement by the nurse needs correction? "Encourage the child to talk." "Answer questions honestly." "Encourage family members to stay near the child." "Avoid explaining the physical effects the child will experience."

"Avoid explaining the physical effects the child will experience." Rationale The nurse should have the family members explain to the child in simple language about the physical effects of the end-of-life care on the child. The nurse should tell the family to encourage the dying child to talk and express feelings. The nurse should advise the family to honestly answer the child's questions and encourage the family members to stay close to provide support to the child.

A fifteen-year-old child presents to the emergency department with a severe head trauma, including herniation of the brain into the spinal column. The child is non-responsive and intubated. How will the nurse most correctly explain to the parents what is happening? "While the hydrocephalus persists, your child will not be responsive to your voice or painful stimuli." "Eventually your child will be able to squeeze your hand and will gradually begin to respond to touch." "The injury is severe, but with surgery and extended medical care in, a complete recovery can be expected." "Because of the type of injury sustained, the brain will not be able to maintain respiratory effort without the ventilator."

"Because of the type of injury sustained, the brain will not be able to maintain respiratory effort without the ventilator." The brain stem is essential to respiratory and cardiac function. Once it is compressed into the spinal column it can no longer recover the ability to maintain those vital functions.

The mother of a 7-year-old with a brain tumor has just learned that cancer is now also found in her child's spine. Which response by the nurse addresses the mother's concerns? Select all that apply. "Cancer cells do not typically spread." "Cancer cells spread by invasion of other tissues." "Cancer cells spread through the endocrine system." "Cancer cells spread through the gastrointestinal (GI) tract." "Cancer cells spread by metastasis to distant body sites."

"Cancer cells spread by invasion of other tissues." Cancer cells spread by invasion, in which cells grow in unrestricted, disorderly fashion at the site of origin; therefore, this is the response expected by the nurse. "Cancer cells spread by metastasis to distant body sites." Cancer cells spread by metastasis, in which the cells grow in sites other than the site of the primary cancer.

The nurse is educating parents of a child with Wilms tumor who is scheduled to begin chemotherapy. The child's mother is upset because she thinks the surgery should be done first. Which statement made by the nurse is the best response? "Radiation is an alternative treatment that is faster." "Parents can opt for surgery first, then chemotherapy." "Chemotherapy will shrink the tumor and make surgery safer." "Surgical intervention will not be necessary after chemotherapy."

"Chemotherapy will shrink the tumor and make surgery safer." Chemotherapy is given to patients with Wilms Tumor to shrink the tumor and make surgery safer to remove without rupture or other complications.

The parent of a 6-year-old child is worried because the child frequently suffers from diarrhea and stomach upset. What is an accurate statement for the nurse to respond to the parent? "Children at age 6 eat less due to reduced appetite." "The immune system in 6-year-old children is not well developed." "The digestive system in 6-year-old children is not well developed." "Children tend to acquire infections during the first two years of schooling."

"Children tend to acquire infections during the first two years of schooling." Rationale During early middle childhood, children are more prone to infections as they start going to school and are exposed to other children who may have infections. The immune system is well developed, and 6-year-old children are able to combat localized infections. The digestive system is well developed, so there are fewer stomach upsets. Six-year-old children begin eating more as their appetites increase.

A nurse is providing education to parents who have a child recently diagnosed with a chronic illness. Which statement made by the nurse should be helpful for this family trying to cope with the changes the family is experiencing? "Coping with a chronic illness is not easy, but can be done if the family works to achieve cohesiveness." "Coping with a chronic illness can be very difficult and unfortunately leads to a weakened family unit." "A chronic illness will impact the daily life of your family and will require a lot of change for everyone involved." "Change impedes the process of normalization. Achieving normalcy will not be possible while dealing with a chronic condition."

"Coping with a chronic illness is not easy, but can be done if the family works to achieve cohesiveness." Cohesiveness is an important part of obtaining resiliency. The nurse will need to provide specific ways to obtain cohesiveness, but this answer provides hope and is a starting point for a family with a child newly diagnosed with a chronic illness.

The healthy sister of a 9-year-old male with cystic fibrosis approaches the nurse and states, "I feel so guilty my brother is sick and I'm not." Which response by the nurse is appropriate? "Could you tell me a little more about how you are feeling?" "Feeling guilty is normal for someone your age, it will go away." "I know you feel bad, but you should be thankful that you aren't sick." "It's okay to feel guilty, you are able to do things your brother can't."

"Could you tell me a little more about how you are feeling?" This response provides the sibling with the ability to express feelings to the nurse and work through the guilt being experienced. This will assist the sibling in resolving the feelings and not carrying them forward into the adult years.

The mother of a child who was recently diagnosed with retinoblastoma asks if it is possible to delay treatment until the child is out of school for the summer. Which is the appropriate response by the nurse? "Delaying treatment may lead to extraocular manifestations and worse prognosis." "Delaying treatment can lead to loss of both eyes rather than just the affected eye." "Because this is a genetic disease, your child is at risk for other tumors if not treated quickly." "It is possible to delay treatment for a short time as retinoblastoma is a focal disease."

"Delaying treatment may lead to extraocular manifestations and worse prognosis." If diagnosis and treatment of retinoblastoma are delayed, the cells can break off and form independent tumors. These tumors can extend down the optic nerve and spread to the CNS and to site outside of the eye, leading to extraocular disease and worse prognosis.

The parents of a toddler newly diagnosed with a chronic illness ask, "Will our little girl develop just like her peers?" How should the nurse respond? "Children with chronic illnesses often lag behind their peers developmentally." "Because your child was diagnosed with a chronic illness at such a young age, the outlook for the child is poor." "Development is unique to each child, but can be supported through consistency at home and community resources to help minimize any developmental delays." "A chronic illness does not usually impact the social or behavioral development, so your child might only struggle with physical development because of the chronic illness."

"Development is unique to each child, but can be supported through consistency at home and community resources to help minimize any developmental delays." This information provides accurate and reasonable expectations and gives the parents some control over how they can best assist the development of their child.

The nurse administers morphine to a child in hospice care, but the child does not verbalize relief from pain. The primary health care provider instructs the nurse to increase the drug dosage. The parents express their concern about addiction. Which statement made by the nurse regarding the pain medication is appropriate in this situation? "Increasing the dosage of morphine will hasten death." "Other pain medications are no longer useful for your child." "Don't worry. Increasing the dosage of morphine will not cause addiction." "An increased dosage of morphine will be administered only when the child has unbearable pain."

"Don't worry. Increasing the dosage of morphine will not cause addiction." Rationale Pain management in children in the end stage of life is given high priority. Occasionally, children at the end stage of life may require a high dosage of opioids, such as morphine, to control pain. Increasing the dosage of the pain medication will not result in addiction. A child might develop a tolerance to the medication, but tolerance should not be confused with addiction (a psychological dependence). Note that morphine does not hasten death; the medication is used only to relieve pain. Morphine is not given only when other medications are no longer useful for the patient. The administration of opiates should occur when the child needs relief and not be reserved for when the child is in unbearable pain.

The nurse administers morphine to a child in hospice care, but the child does not verbalize relief from pain. The primary health care provider instructs the nurse to increase the drug dosage. The parents express their concern about addiction. Which statement made by the nurse regarding the pain medication is appropriate in this situation? "Increasing the dosage of morphine will hasten death." "Other pain medications are no longer useful for your child." "Don't worry. Increasing the dosage of morphine will not cause addiction." "An increased dosage of morphine will be administered only when the child has unbearable pain."

"Don't worry. Increasing the dosage of morphine will not cause addiction." Rationale Pain management in children in the end stage of life is given high priority. Occasionally, children at the end stage of life may require a high dosage of opioids, such as morphine, to control pain. Increasing the dosage of the pain medication will not result in addiction. A child might develop a tolerance to the medication, but tolerance should not be confused with addiction (a psychological dependence). Note that morphine does not hasten death; the medication is used only to relieve pain. Morphine is not given only when other medications are no longer useful for the patient. The administration of opiates should occur when the child needs relief and not be reserved for when the child is in unbearable pain.

The nurse is caring for a newborn with Down syndrome. The parents state, "The health care provider just informed us that our daughter has Down syndrome. How long will my child be sick?" How should the nurse respond? "All chronic conditions are lifelong conditions." "I am very sorry that your daughter was born with this lifelong illness." "Down syndrome is a chronic condition, so the child will always have this condition." "Your child is not sick but has a developmental disability that will require her to need care her entire life."

"Down syndrome is a chronic condition, so the child will always have this condition." The nurse knows that many chronic illnesses are lifelong and that Down syndrome is a type of chronic illness that lasts a lifetime.

A nurse preceptor is working with a new school nurse. The preceptor is discussing cognitive development in middle childhood. What statement made by the new nurse indicates effective teaching? "During middle childhood, children cannot memorize symbols." "During middle childhood, children have a rigid and egocentric view." "During middle childhood, children learn from their past experiences." "During middle childhood, children have very low perceptual thinking."

"During middle childhood, children learn from their past experiences." Rationale During middle childhood, children learn from their experiences. They use behaviors adopted in the past to evaluate and interpret present situations. Children age 6 to 12 years (middle childhood) lose the rigid and egocentric view of preschoolers and begin to develop empathy and to perceive the situation from the other's point of view. During middle childhood, children can memorize symbols and develop problem-solving abilities. They also begin making judgments on what they see and gradually develop perceptual thinking.

During a well-child visit, a three-year-old patient being examined exhibits unsteady gait and poorly developed speech. The health care provider suspects cerebral palsy (CP). How should the nurse guide the parents? Select all that apply. "Ensure that your home is free of sharp edges to protect the child in case of falling while walking." "Administer pain medication whenever the child shows signs of pain such as grimacing or moaning." "While we get further testing arranged, keep notes regarding any unusual behaviors or actions." "The speech-language pathologist will work with you to evaluate reasons for the poorly developed speech." "The physical therapist will show you some exercises to improve coordination and strengthen the child's muscles."

"Ensure that your home is free of sharp edges to protect the child in case of falling while walking." A three-year-old child with suspected cerebral palsy would be expected to have unsteadiness when walking due to abnormal development of the motor system. "The speech-language pathologist will work with you to evaluate reasons for the poorly developed speech." A three-year-old child with suspected cerebral palsy would be expected to have speech difficulties due to abnormal central nervous system development. Speech development can also be related to cognitive issues, autism, and hearing loss. Correct "The physical therapist will show you some exercises to improve coordination and strengthen the child's muscles." Physical therapy and occupational therapy will work with the child to further evaluate gait and develop a plan to prevent or reduce declining strength and coordination.

The nurse is caring for an 8-year-old child with lymphoma undergoing extensive radiotherapy. The parent tells the nurse that the child has been falling asleep at the dinner table. What is the best response to the parent's concerns? "She may need to be homeschooled." "Fatigue is a common side effect of irradiation." "Most kids don't get tired as a result of radiotherapy." "She needs to be reevaluated so we can determine whether more treatments are necessary."

"Fatigue is a common side effect of irradiation." Rationale Fatigue is the most common side effect of irradiation. Parents need to be instructed about this before treatment so they can observe the child for behaviors indicating extreme fatigue. It is better to schedule more rest time during the day or an earlier bedtime than to take the child out of school or the normal routine. Most children do get fatigued by radiotherapy. The child does not need to be reevaluated; fatigue is the most common side effect.

The parents of three children, including a toddler with hemophilia, are talking with the nurse and admit that the child with hemophilia is the center of attention in the family since they are more protective of the child who is ill. Which statements by the nurse should be an appropriate response to the parent's actions toward their child? Select all that apply. "Favoritism toward your toddler can negatively affect important boundaries and should be avoided." "Making the toddler the center of attention can actually cause some harm by decreasing age-appropriate development." "It is important for you as parents to spend time together to maintain your relationship as well so that the foundation of your family is strong." "Let's talk about ways to ensure that all of your children feel equal attention so that there are not negative feelings towards the ill child or parents." "The toddler needs extra support because of the chronic condition, so it is important to ensure that the toddler's required care is the focus of family activities."

"Favoritism toward your toddler can negatively affect important boundaries and should be avoided." Maintaining boundaries in the family with a child who has a chronic condition is important so that all members of the family feel cared for equally and development can be fostered. "Making the toddler the center of attention can actually cause some harm by decreasing age-appropriate development." Overprotection can slow development, and therefore this is the appropriate response for the nurse to make. "It is important for you as parents to spend time together to maintain your relationship as well so that the foundation of your family is strong." Parents do need to foster a strong relationship between each other and with their children in order to cope with a chronic condition. Making the child who is ill the center of attention can cause problems in the family. "Let's talk about ways to ensure that all of your children feel equal attention so that there are not negative feelings towards the ill child or parents." Overindulging one child can cause animosity among the children and cause problems in sibling relationships.

The nurse manager instructs a nurse to conduct a group activity for assessing physical and cognitive development in 6- to 12-year-old children. What suggestion given by the nurse manager is appropriate? "Give grades and material rewards to the children after performing the task." "Compare the performance of one child with the other children based on grades." "Do not allow the children to interact with each other while doing the task." "A child who performs well in all the tasks can be considered as mentally fit."

"Give grades and material rewards to the children after performing the task." Rationale Children 6 to 12 years of age are eager to participate and effectively complete the given tasks. The nurse can conduct group activities to assess the physical and mental abilities of the children, because they are motivated in the presence of peers and effectively participate in the task. The nurse should adopt the reward system, such as giving grades and material rewards to encourage the children. In order to avoid having children feel inferior, the nurse should not compare them with their peers. The children must be encouraged to interact with their peers and help each other in order to develop the ability to cooperate and to compete with other children effectively. No one has the ability to do everything perfectly, so children should not be expected to perform well in all tasks.

A 14-year-old child with terminal brain cancer has months left to live. The child's pain is increasing, along with problems ambulating due to unsteady gait and increasing difficulty in swallowing. The parents ask the nurse about hospice care. Which response by the nurse is appropriate? Select all that apply. "Hospice care can be provided in your home." "With hospice care, your family is able to receive respite care." "With hospice care, your child doesn't have to be admitted to the hospital." "Hospice care allows for a comfortable death while ensuring the dignity of your child." "With hospice care, some of the physical and emotional burdens of caring for your child are taken care of."

"Hospice care can be provided in your home." Hospice care can be provided in the patient's home or in a hospital setting, and therefore this is the best response for the nurse to make. "With hospice care, your family is able to receive respite care." A child may be admitted to the hospital when in hospice care for pain control and respite care, and therefore this is the best response for the nurse to make. "Hospice care allows for a comfortable death while ensuring the dignity of your child." Hospice care allows for a less-chaotic, home-like environment that enhances the ability to have a comfortable death. The dignity of the child is enhanced as well.

A preschool-aged child with cancer has begun crying for her pacifier, which she stopped using 6 months ago. The parents voice concerns to the nurse. How should the nurse respond to the parents? "How do you feel about allowing her to use the pacifier while in the hospital?" "Explain to the child that she is a 'big girl' and does not need a pacifier anymore." "Do not give her the pacifier because this could cause her not to develop appropriately." "Your daughter may not be dealing well with the illness and treatment. She may need to meet with a psychologist."

"How do you feel about allowing her to use the pacifier while in the hospital?" Regression is typical and expected in young children who are undergoing a major illness. The child is trying to cope with stress related to the hospitalization and/or cancer, so allowing a pacifier is acceptable and can decrease the child's stress. The nurse should investigate how the parents feel about the child using the pacifier during the hospital admission.

Which nursing response is appropriate for a mother that is apprehensive about giving her child vaccines in fear of developing autism? "What you've heard is bogus. There's no evidence that immunizations cause autism." "The law says you must immunize your children. Refusing to immunize is breaking the law." "You must really be afraid. Autism is a horrible condition for any parent to have to deal with." "I can understand your concerns. Would you like more information about the safety of immunizations?"

"I can understand your concerns. Would you like more information about the safety of immunizations?" Rationale Expressing understanding of the mother's concerns and then asking whether the woman would like more information about the safety of immunizations is an empathic way of addressing her fears and concerns. Stating that what she has heard is bogus is belittling and certainly not helpful for this mother. Telling the mother that by refusing to immunize she is breaking the law is not an empathic response to her concerns. Noting that the mother must be fearful because autism is a horrible condition for any parent to have to deal with is not helpful and does not offer the mother support or guidance.

Which nursing response is appropriate for a mother that is apprehensive about giving her child vaccines in fear of developing autism? "What you've heard is bogus. There's no evidence that immunizations cause autism." "The law says you must immunize your children. Refusing to immunize is breaking the law." "You must really be afraid. Autism is a horrible condition for any parent to have to deal with." "I can understand your concerns. Would you like more information about the safety of immunizations?"

"I can understand your concerns. Would you like more information about the safety of immunizations?" Rationale Expressing understanding of the mother's concerns and then asking whether the woman would like more information about the safety of immunizations is an empathic way of addressing her fears and concerns. Stating that what she has heard is bogus is belittling and certainly not helpful for this mother. Telling the mother that by refusing to immunize she is breaking the law is not an empathic response to her concerns. Noting that the mother must be fearful because autism is a horrible condition for any parent to have to deal with is not helpful and does not offer the mother support or guidance.

A child has recently been diagnosed with Guillain- Barré syndrome. Which patient statements require follow-up by the nurse? Select all that apply. "I had a nasty cold two weeks ago, but mom said it was not a big deal." "I had a red rash a while back with a fever. It only lasted about three days." "When I get well, I'm not shaking hands with anyone, not even the pastor!" "I haven't felt well for a few days and my toes are kind of numb feeling today." "I hate getting the flu-shot, but dad insisted this year since I've been sick so much."

"I had a nasty cold two weeks ago, but mom said it was not a big deal." Enterovirus causes cold and flu-type symptoms and is associated with the development of Guillain- Barré syndrome. The child needs teaching about hand-washing techniques to prevent respiratory illnesses. Correct "I had a red rash a while back with a fever. It only lasted about three days." A rubella infection has been associated with Guillain-Barré syndrome. The nurse follows up as the health care team works to determine a cause that could be prevented in the future. Also, if "a while back" means more than a month or so, the health care provider can rule it out as a cause. "I hate getting the flu-shot, but dad insisted this year since I've been sick so much." The influenza vaccine has a demonstrated link to the onset of Guillain-Barré. If this is the suspected cause, the child needs to avoid the flu-vaccine in the future.

The 12-year-old patient with severe headaches is being discharged from the emergency department. Which statement by the patient indicates to the nurse the need for further teaching? Select all that apply. "I hear my friends use marijuana for stress relief. I will try that." "If ibuprofen doesn't work at first, I take another one in two hours." "Since chocolate seems to trigger my headaches, I will eat less of it." "In my diary, lack of sleep is a trigger. I am going to go to bed earlier." "Football and the Spanish club are too much. I think I will drop them both."

"I hear my friends use marijuana for stress relief. I will try that." Smoke can increase headache occurrence, therefore this statement would indicate a need for further teaching. Also the nurse will want to teach about illegal substance use. Correct "If ibuprofen doesn't work at first, I take another one in two hours." Ibuprofen will cause bleeding and other injury if taken this way, therefore this statement indicates a need for further teaching. Correct "Since chocolate seems to trigger my headaches, I will eat less of it." The child needs to avoid, not just decrease, any trigger that has been identified. This statement would therefore indicate a need for more education on dietary habits and their connection to headaches. "Football and the Spanish club are too much. I think I will drop them both." Encourage the child to consider ways to reduce fatigue and stress without increasing isolation by dropping both activities at the same time. Perhaps there is a way to reduce involvement with one of the activities or maybe consider dropping one to evaluate headache response.

Which response made by the parent indicates a need for further teaching about prevention of infant burns at home? "I have installed smoke detectors in my home." "I do not leave my child in a parked car." "I heat baby food in a microwave." "I check bath water temperature before using."

"I heat baby food in a microwave." Rationale The infant's food should not be heated using a microwave oven, because it can lead to uneven heating of the food and may cause burns. The parents should always check the temperature of fluids before feeding. Not leaving the child in a parked car, installing a smoke detector, and checking the bath water temperature before using it will prevent the infant from experiencing burns.

A nurse is providing care for a thirteen-year-old patient with a history of headaches. Which statements by the patient indicate further teaching is required? Select all that apply. "I just try to stay away from everyone." "I keep a diary of what I eat and how I sleep." "I dropped one of my extracurricular groups." "When I feel stressed out, I play video games." "I alternate ibuprofen and acetaminophen all day."

"I just try to stay away from everyone." Isolation is not an effective therapeutic strategy for the long-term management of headache, and therefore this statement indicates a need for further teaching. "When I feel stressed out, I play video games." The use of stress management techniques is a therapeutic strategy for the long-term nursing management of headache. However, visual stimuli can induce headaches. Alternative stress management techniques must be taught, and therefore this statement indicates a need for further teaching. "I alternate ibuprofen and acetaminophen all day." The administration of medication is a therapeutic strategy for the long-term nursing management of headache. Constant self-medicating will lead to rebound headaches. This statement indicates a need for further teaching.

A mother of a chronically ill child tells the nurse that her other child is angry with the sick child. Which statement made by the mother would describe the reason for the child's anger? "I make the sibling sleep in a different room." "I appreciate the sibling helping the ill child." "I have explained the child's illness to the sibling." "I read stories to the sibling along with the chronically ill child."

"I make the sibling sleep in a different room." Rationale If the sibling was made to sleep in a separate room, that sibling may feel that the parents are showing special attention to the ill child and subsequently feel angry. The parents should appreciate when the sibling shows positive behaviors. The sibling should be informed about the illness. The parents should read stories to the ill child along with the sibling to show that the sibling is also important.

Which response made by the parent requires further teaching about the safety measures to be taken at home? "I keep the oven door closed." "I do not cover the mattress with a plastic cover." "I store plastic bags in a place my child can't reach." "I provide attractive small toys for my child to play with."

"I provide attractive small toys for my child to play with." Rationale Parents should not provide small toys to infants. Infants identify objects by putting them in their mouth, which may lead to suffocation. All appliance doors should be kept closed to avoid electric shocks. Mattresses and pillows should not be covered with plastic, because this can lead to suffocation. Plastic bags, which cause suffocation, should be kept out of the reach of infants.

Which response made by the parent indicates a need for further education about sanitary and safety practices for preventing infection in the home? "I rinse reusable diapers in the toilet." "I use disposable paper diapers for my child." "I change the diapers as soon as they are soiled." "I wash my hands after changing my child's diaper."

"I rinse reusable diapers in the toilet." Rationale Reusable diapers should not be rinsed in the toilet, although fecal contents can be flushed down. This will spread infection. Using disposable paper diapers will help in preventing infection, because the diapers are discarded after one use. Diapers should be changed when they are soiled to prevent infection. Children and parents should wash their hands after changing diapers; this will help in controlling infection.

Which statements made by the parent indicate effective learning about preventing intestinal parasitic diseases? Select all that apply. "I rinsed my child's diapers in the toilet." "I taught my child to defecate in the toilet." "I teach my child to wash hands before eating." "I rinse raw fruits and vegetables before eating." "I allow my child to swim in the public swimming pool."

"I taught my child to defecate in the toilet." "I teach my child to wash hands before eating." "I rinse raw fruits and vegetables before eating." Rationale To prevent intestinal parasitic diseases, parents should teach their children to defecate in the toilet. Parents should teach their children to wash their hands before eating. Raw fruits and vegetables should be washed before using them for cooking or eating. These measures will help prevent children from developing intestinal parasitic diseases. Parents should not rinse diapers in the toilet; this would increase the risk of developing intestinal parasitic diseases. Parents should avoid public swimming pools due to an increased chance of diapered children in the pools.

Which statement made by the parent indicates a need for further learning about ways of preventing the spread and recurrence of pediculosis? "I will vacuum the carpets." "I will hand wash my child's clothes." "I will wash combs in boiling water for 10 minutes." "I will store my child's belongings in a separate cupboard."

"I will hand wash my child's clothes." Rationale The clothes of the child with pediculosis should be machine washed in hot water and dried using a hot dryer for at least 20 minutes. The carpets should be vacuum cleaned thoroughly to prevent the spread of pediculosis to household people. The comb of the child with pediculosis should be washed with boiling water for 10 minutes. The belongings of the child with pediculosis should be stored in a separate cabinet in order to prevent the spread of pediculosis.

A 4-year-old patient has died and the nurse is extremely distraught because of the strong bond the nurse developed with the child. Which statement provides an example of how the nurse can cope better in the future? "In the future I must try to get more sleep each day." "In the future I will attend weekly grief support meetings." "In the future I will spend equal amounts of time with each patient." "In the future I will wait to visit my patients until they are discharged to home."

"In the future I will spend equal amounts of time with each patient." Spending equal amounts of time with every patient is an example of setting professional boundaries, which prevents the formation of strong bonds with patients.

Which question is appropriate to ask the parent before giving influenza vaccination to a child? "Is your child allergic to cow's milk?" "Does your child have celiac disease?" "Is your child allergic to any antibiotics?" "Is your child allergic to eggs of any kind?"

"Is your child allergic to eggs of any kind?" Rationale In the preparation of the influenza vaccine, eggs are used. Therefore, if the child is allergic to eggs, the child may develop an allergic reaction after receiving influenza vaccine. Therefore, the nurse would ask the parent whether the child is allergic to eggs. The influenza vaccine is not contraindicated for children who have celiac disease. Celiac disease is a condition in which the patient is either allergic or intolerant to gluten, which is not related to egg allergy. It is safe to administer the influenza vaccine to children allergic to cow's milk and antibiotics, because neither is a component of the vaccine.

A dying child, who has been unresponsive for the past two days in hospice at the hospital, opens his eyes and softly talks to his mother. He asks his mother how she is and where his siblings are and can he see them. The mother tells the nurse, "I think he is getting better." Which response by the nurse is appropriate? "Your child is not going to get better. He is dying." "Your child may be improving; we will have to wait and see." "Your child is showing signs of strength and may live for a few more weeks." "It is not uncommon to be more alert and interested in the family right before a child dies."

"It is not uncommon to be more alert and interested in the family right before a child dies." Children have been noted to have a surge of energy and alertness immediately before they die. Parents must be educated that this is not a sign of improvement.

A 12-year-old patient has been in hospice for one month and expresses feelings of discontent and would like to go back to the hospital. The patient states missing being with other children and liking the food there better. How will the nurse respond? Select all that apply. "It is okay to change your mind." "Tell me more about how you are feeling." "I'll let your parents know you want to go back to the hospital." "I'll call the hospital and see how soon you can be transferred back." "Why don't you give this place a little more time before going back to the hospital"

"It is okay to change your mind." The child should know that they have the right to change health care environments and that there won't be trouble for wanting to change. Correct "Tell me more about how you are feeling." It is important to allow the child to express emotions regarding hospice care, and therefore this is the best response for the nurse to make. Correct "I'll let your parents know you want to go back to the hospital." This demonstrates to the child that you are working on accommodating the change from hospice to the hospital setting.

The parents of four children, one of which has a chronic illness, state, "My spouse and I spend as much time as we can with our sick child, but our family just seems to be stressed all the time." What should be the nurse's best response? "The children just need an outlet for their frustration. You could send them to a support group." "The other children are probably just reacting to the stress of having a sibling with a chronic illness." "It is possible to spend too much time with a child who is ill and not enough time with your other children." "Educate your other children on the need for you to spend extra time with the child who is ill to decrease their stress."

"It is possible to spend too much time with a child who is ill and not enough time with your other children." This statement is factual and provides an opportunity for the nurse to educate on the need for spending time equally among all the children.

The parents of a terminally ill child are very anxious as they ask the home care nurse about the child's progress. The nurse knows that any improvement in the child's condition can be determined only after 3 weeks. Which statement by the nurse is most therapeutic? "The health care provider can tell you that." "I am seeing no evidence of improvement yet." "We can't tell you when the child will be better." "It may take a couple of weeks to determine progress."

"It may take a couple of weeks to determine progress." Rationale The nurse should answer the parents' queries in a straightforward manner to alleviate their anxiety. Telling the parents that it may take a couple of weeks may help the parents feel better. Telling the parents that there is no improvement will increase their anxiety. The parents might feel frustrated if they are passed on to get the information from the health care provider. Telling the parents that there is no evidence of improvement may make the parents feel more anxious.

An adolescent who has had diabetes mellitus for ten years is able to check his blood glucose and maintain glucose levels while still participating in sports and maintaining a 'B' average. The adolescent tells the nurse, "I am so glad that I am finally able to deal with having diabetes." What is the nurse's best response? "You seem to be taking on a lot for someone with a chronic health care condition." "It seems that you have finally worked through your anger and frustration related to your illness. Keep up the good work." "You might be in denial that you have a chronic illness that requires you to make some changes in your life. This is common at your age." "It sounds like you feel as though you are coping well with diabetes and have reached acceptance of the illness. Just remember that at times you might struggle, and that it is OK to ask for help."

"It sounds like you feel as though you are coping well with diabetes and have reached acceptance of the illness. Just remember that at times you might struggle, and that it is OK to ask for help." This teen seems to be doing well, but setting unreasonable expectations can be a set-up for distress in the future. Acknowledging the hard work is important, but also pointing out that there are resources available to help if needed is equally important.

A 4-year-old child with chronic sickle cell crisis is terminal and unable to walk, requires continuous oxygen administration, is moaning and crying with position changes, and has refused to eat meals for the last 2 days. The parents ask the nurse how they will care for their child and what they should do now. Which response by the nurse is appropriate? "Learning about hospice care services would be appropriate at this time." "Initiating home health nursing services will help you care for your child at home." "Initiating palliative care services for your child would be important right now." "Learning how to care for your child at home would be appropriate at this time."

"Learning about hospice care services would be appropriate at this time." The goal of hospice care is to improve the quality of life and decrease the stress on the child and family members and can be introduced at any time during the child's treatment.

A single parent of a child with cancer comes into the care provider's office and states, "I am so frustrated. Lately the appointments are too much and my child's behavior has been terrible." Which statement by the nurse is appropriate? "Why exactly are you frustrated?" "You should be firmer when you discipline the child." "All the appointments are necessary for the child's health." "Let's talk about some helpful techniques that can assist you in working through the frustration you are having."

"Let's talk about some helpful techniques that can assist you in working through the frustration you are having." Teaching the parent relaxation techniques, such as deep breathing, provides an effective outlet for emotional tension and support for the current problem, and therefore this statement by the nurse would be appropriate.

A teenager with leukemia states, "I thought everything about this was going to be terrible, but at least my parents have stopped making me do chores and homework." The next time the nurse sees the parents, what should be the nurse's best response? "Your teen seems to be responding well to a decrease in stress at home from the elimination of the need to do homework and chores." "Let's talk about the importance of maintaining age-appropriate expectations with your teenager, even while your teen is battling leukemia." "Allowing your teen to neglect homework and chores is going to make it harder on going back to usual expectations after the battle with leukemia is over." "Decreasing expectations for a little while will minimize the behavioral and psychologic issues that often develop in children and teens experiencing a chronic health condition."

"Let's talk about the importance of maintaining age-appropriate expectations with your teenager, even while your teen is battling leukemia." Maintaining expectations and rules is important to the development of any child with a chronic illness. Although modifications may need to be made, eliminating chores and expectations can actually hurt the overall development of the teen.

The parent of a school-aged child states, "I am so proud of my daughter. She was diagnosed with asthma and yet she still does everything she was doing before. She processed this diagnosis more quickly than I thought she would." How should the nurse respond? "Let's talk to your daughter and see how she is doing." "It is great to hear that your daughter is coping so well." "She might not be doing as well as you think. There is a chance she is in denial that there is a problem." "Your daughter might be hiding her feelings from you and should speak with a mental health specialist for support."

"Let's talk to your daughter and see how she is doing." The nurse needs to assess the child to see if she is really reaching acceptance or if she is in denial with the asthma diagnosis. Having a discussion with the daughter will be the best action to take.

Which statement made by the child supports the nurse's suspicion of emotional neglect? "My father has not paid my school fees on time for months." "My mother does not give me a blanket when I feel cold at night." "My mother didn't take me to the doctor when I broke my ankle." "My father does not praise me even when my teacher praises my drawing.

"My father does not praise me even when my teacher praises my drawing." Rationale Emotional neglect involves a disregard for the child's need for emotional nurturance. When the child says that the father didn't praise the child's drawing, it indicates emotional neglect. Not providing a blanket when the child feels cold at night is physical neglect. Not seeking treatment for a broken ankle indicates physical neglect. Not catering to the educational requirements of the child in terms of paying school fees on time also amounts to physical neglect. This is because the lack of payment may interfere with the child's education.

Which statement by the parent suggests the need for additional information about injury prevention in early childhood? "We keep all of the medicines out of reach." "We have a fence around the swimming pool." "We've put gates at the top and bottom of our stairs." "Our 2-year-old takes a bath with a sibling while we cook dinner."

"Our 2-year-old takes a bath with a sibling while we cook dinner." Rationale Allowing a 2-year-old toddler to take a bath unsupervised suggests that the parent needs additional information about injury prevention in early childhood. Keeping all medicines out of reach or locked, placing gates at the top and bottom of the stairs, and having a fence around the swimming pool are all ways to prevent injury and therefore do not warrant additional teaching about injury prevention.

A child is dying of sickle cell disease and parents are expressing concern over the large amounts of opiates needed to control the child's pain. The father states, "I am worried that my child will become addicted to the medication." Which statement by the nurse addresses this parent's concern? "If we observe signs of addiction, we will use other medications to provide comfort to your child." "Addiction is a common concern when using opiates, but we don't worry about it in these situations." "Addiction occurs over a long period of time. Your child's prognosis is shorter than that time frame." "Our goal with your child is to effectively manage the pain and maintain an acceptable comfort level which may include using larger doses of opiates."

"Our goal with your child is to effectively manage the pain and maintain an acceptable comfort level which may include using larger doses of opiates." Parents are often concerned about addiction problems, but when educated on pain management techniques and drug dosages, an understanding related to why large doses of opiates are utilized occurs and the concern about addiction is resolved.

The nurse is assessing a female child diagnosed with Hodgkin lymphoma. When taking a patient history, which information from the patient's parents would the nurse expect? Select all that apply. "She has a poor appetite recently." "She has been more colds than usual this year." "She has had a lot of unexplained fevers lately." "She just doesn't have much energy for play anymore." "She has a lump in her neck that is painful when touched."

"She has a poor appetite recently." Anorexia is a clinical manifestation of Hodgkin lymphoma. Loss of appetite for food is expected. "She has had a lot of unexplained fevers lately." Fever is a constitutional symptom of Hodgkin lymphoma and would be an expected symptom. "She just doesn't have much energy for play anymore." Fatigue is a clinical manifestation of Hodgkin lymphoma. Loss of energy or disinterest in physical play is expected.

A 15-year-old patient in the middle of his growth spurt is diagnosed with a tumor affecting the distal femur. The patient reports insidious pain and a progressively limited range of motion in the affected limb. The nurse is educating the family on the disease and will include which information as it pertains to metastasis? "Headaches are a likely symptom of metastasis, and should be reported to the HCP." "Abdominal distention could be a sign of metastasis and should be immediately report to the HCP." Monitor your urine output for blood. "Frequent metastasis occurs to the kidneys causing blood in the urine." "Shortness of breath is concerning and could be a sign of metastasis. This should be immediately reported to the provider."

"Shortness of breath is concerning and could be a sign of metastasis. This should be immediately reported to the provider." The patient is experiencing signs of osteosarcoma. Patients with this disease should be told that the lungs are the primary site if metastasis and to report any respiratory symptoms to the provider.

The nurse is discussing the surgical closure of a myelomeningocele with the parents of a newborn patient. Which statement by the parents indicates the need for further teaching? "Surgically closing this defect will ensure my baby can walk at the right age." "This surgery makes me feel less like my baby is too fragile to hold and feed." "This surgery minimizes the problems my baby has over the course of a lifetime." "Surgically closing this defect reduces the chance my baby will develop infections."

"Surgically closing this defect will ensure my baby can walk at the right age." Surgery will decrease cord deterioration and allow for earlier physical therapy and developmental interventions. The child with spina bifida corrected by surgery still may require additional surgeries.

A graduate nurse was caring for a child in the intensive care unit under the guidance of a registered nurse. After the child's death, the graduate nurse admits to feeling low and relates this to the registered nurse. Which advice given by the registered nurse is the most appropriate in this situation? "Spend some time alone until you feel better." "Do not express grief for a patient. It is unprofessional." "Focus on the positive aspects of your chosen profession." "Talk to your colleagues and supervisor about your feelings."

"Talk to your colleagues and supervisor about your feelings." Rationale The registered nurse should advise the graduate nurse to talk to colleagues and to the supervisor to get professional support, because the death of a patient can be stressful for less-experienced nurses. Talking with colleagues and senior nurses may provide support and assistance to the graduate nurse while developing coping strategies. Telling the graduate nurse to spend some time alone might increase the graduate nurse's stress, because solitude limits social interaction. The registered nurse should not advise the graduate nurse to avoid expressing grief. Nurses may fear that showing their emotions is unprofessional; however, that is not true. Telling the graduate nurse to focus on the positive aspects of the profession is correct; however, in this situation, advising the graduate nurse to talk with colleagues and the supervisor is more appropriate than focusing on the positives of the profession.

A graduate nurse was caring for a child in the intensive care unit under the guidance of a registered nurse. After the child's death, the graduate nurse admits to feeling low and relates this to the registered nurse. Which advice given by the registered nurse is the most appropriate in this situation? "Spend some time alone until you feel better." "Do not express grief for a patient. It is unprofessional." "Focus on the positive aspects of your chosen profession." "Talk to your colleagues and supervisor about your feelings."

"Talk to your colleagues and supervisor about your feelings." Rationale The registered nurse should advise the graduate nurse to talk to colleagues and to the supervisor to get professional support, because the death of a patient can be stressful for less-experienced nurses. Talking with colleagues and senior nurses may provide support and assistance to the graduate nurse while developing coping strategies. Telling the graduate nurse to spend some time alone might increase the graduate nurse's stress, because solitude limits social interaction. The registered nurse should not advise the graduate nurse to avoid expressing grief. Nurses may fear that showing their emotions is unprofessional; however, that is not true. Telling the graduate nurse to focus on the positive aspects of the profession is correct; however, in this situation, advising the graduate nurse to talk with colleagues and the supervisor is more appropriate than focusing on the positives of the profession.

An adolescent going through cancer treatment lost her hair and missed prom six months ago. The teen is coming in for her usual check-up and starts crying. The nurse states, "All of your blood work looks good. You are fine." Which statement reflects how the nurse should have responded better? "Tell me what you are thinking about right now." "There is nothing to be concerned about right now. Everything looks good." "I think you are misunderstanding what your test results show. You are getting better." "I thought you had made it past your grief. You seemed so happy during your last appointment."

"Tell me what you are thinking about right now." The nurse needs to understand that anyone, especially an adolescent, may fluctuate through the grieving process. This teen has undergone a change in body image and has missed out on a significant milestone in her life. Allowing the child to talk openly would be best in this situation.

The hospice nurse is caring for a school-aged child who has a glioblastoma with metastasis throughout the body. The toddler has been declining in health for the past 7 days, has had no oral intake, has a labored, irregular respiratory rate of 6 breaths per minute and is bradycardic at 50 bpm. The parents ask the nurse what they can do to help the child through the process. Which is an appropriate response by the nurse? "Tell the child it is ok to die." "Your child's body is wearing out and will die soon." "It's hard to know why your child hasn't died. It should be soon." "Your child's condition has plateaued; your child could last for weeks like this."

"Tell the child it is ok to die." The child may be hanging on to life because the child cannot die without permission by the parents. The parents must convey feelings of love to child, a sense that child will be missed, and finally give the child permission to die.

A nurse is teaching a group of parents about the growth and developmental stages in school-age children. What statement does the nurse include on tasks that can be performed by a 6-year-old child? "The child can identify missing numbers." "The child can count 13 pennies without effort." "The child can count numbers backward from 20 to 1." "The child knows the days of the week and months in order."

"The child can count 13 pennies without effort." Rationale A physically and mentally fit 6-year-old child should be able to count 13 pennies and know the concept of numbers. An 8- to 9-year-old child should be able to count numbers backward from 20 to 1 and know the days of the week and months in order. A 7-year-old child should be able to identify the missing numbers and pictures in a story or cartoon book.

The nurse is performing an assessment of a 9-year-old child. The nurse finds that the child's mother is very concerned about the child's appearance because the child is very thin. What is the appropriate response by the nurse? "Because your child is thin, you need to provide more fatty foods in the child's diet." "Encourage your child to play indoor games so that too much exercise won't lead to additional weight loss." "Don't worry, many children are underweight during this age, and this problem corrects itself as they become better eaters." "The child looks thinner now because of the reduction and redistribution of fat throughout the body. This is normal."

"The child looks thinner now because of the reduction and redistribution of fat throughout the body. This is normal." Rationale School children may look thinner than they did as toddlers and preschoolers because of the reduction of fat and its redistribution to different parts of the body. During these years, the "baby fat" gradually diminishes, giving the children a slimmer appearance. This apparent loss of weight is not necessarily due to poor eating, so the nurse should not give false assurances of this type. The nurse should advise the mother to feed the child a diet that is high in calories and rich in protein but to avoid fatty foods that can eventually lead to childhood obesity. High-calorie and high-protein foods are preferred because they promote proper growth and development. The child must be encouraged to play both indoor and outdoor games. Physical activity helps in skeletal lengthening and fat diminution.

A child with osteosarcoma (bone cancer) rates her pain 10/10 despite narcotic administration, has lost 20 lb since starting chemotherapy, and cries with every movement and position change. The parents approach the nurse and ask how the pain can have an emotional impact on the child. Which response by the nurse is appropriate? "Your child has given up on living and wants to die." "Your child is in so much pain and doesn't know what is being said." "Your child is speaking out because of pain. Try not to worry about what is being said." "The child's thoughts of having to stay alive while living in pain are worse than the thoughts regarding dying."

"The child's thoughts of having to stay alive while living in pain are worse than the thoughts regarding dying." When a dying child works through the stages of grief, the child may reach a point where being ill is worse than dying.

Which statement pertaining to the rotavirus vaccine RotaTeq is appropriate when teaching a parent? "The dose is 1 mL during each administration." "The vaccine is given through the subcutaneous route." "The first dose is administered between 6 to 12 weeks of age." "The vaccine should not be given beyond 32 months of age."

"The first dose is administered between 6 to 12 weeks of age." Rationale RotaTeq vaccine administration should be initiated between 6 to 12 weeks of age. The vaccine has to be administered in two additional doses at 4- to 10-week intervals following the initial dose of the vaccine. The dose of the vaccine is 2 mL, not 1 mL. The vaccine is administered through the oral route. The vaccine should not be administered beyond 32 weeks of age.

What statement is true regarding peer groups formed by children during the early school years? "The groups are loosely organized and have little formal structure." "The groups are composed predominantly of children of the same sex." "The groups can contribute to bullying when there is excessive peer identification." "The groups invite membership from children who are secure enough to function independently."

"The groups are loosely organized and have little formal structure." Rationale During the early school years, groups are small and loosely organized, with changing membership and little formal structure. Groups formed in the later school years are composed predominantly of children of the same sex. In the earlier school years, there may be a mixture of both sexes in a group. Poor relationships with peers and a lack of group identification can contribute to bullying behavior. Excessive identification with the peers in a group does not do so. Children in a peer group merge their identities with the identities of their peers. They substitute conformity to a peer-group pattern for conformity to a family pattern while they are still too insecure to function independently.

Parents of a child with a terminal illness are choosing hospice care for their 7-year-old child. The child has difficulty swallowing, is bedridden, and requires frequent morphine administration to keep the pain at a 4/10 level. An interdisciplinary team meeting has been scheduled with hospice and the parents to discuss the child. The parents ask the nurse what to expect at this meeting. Which response by the nurse is appropriate? "The provider and hospice RN will only discuss pain management and home care options for your child." "This will be just like other care meetings we have had. The providers and case manager will discuss treatment options." "We discuss when we can deliver the hospital bed, prescriptions, and other medical equipment to your house for your child." "The health care providers, child life specialist, social workers, nurses, home health aides, case managers, and grief counselors meet to discuss your child's plan of care."

"The health care providers, child life specialist, social workers, nurses, home health aides, case managers, and grief counselors meet to discuss your child's plan of care." The hospice team is an interdisciplinary team that focuses on all areas of care for the terminal patient. This is therefore the most appropriate response by the nurse.

A 12-month-old infant is brought to the clinic for a well visit. The parents ask if their infant can receive the diphtheria, tetanus, & acellular pertussis (DTaP) vaccine today. The nurse reviews the chart and notes that the infant's DTaP #3 was administered at age eight months. Which is the best response by the nurse? "It is dangerous to administer another dose of the DTaP to your baby so soon." "The last DTaP was very recent. We will schedule the next dose in two more months." "The fourth dose of DTaP is not scheduled to be administered until the age of 15 months." "If we give the dose this early, your baby will need to repeat the dose to complete the series."

"The last DTaP was very recent. We will schedule the next dose in two more months." The fourth dose of (DTaP) may be given as early as age 12 months, provided that at least six months have elapsed since the last dose. In two more months, the next dose should be administered, but it is not advised to administer the dose before then. See catch-up immunization schedule.

A 10-year-old patient with scoliosis comes to the clinic and asks the nurse a question about an uncomfortable medical procedure. Which is the nurse's most appropriate response? "The procedure isn't too bad; you'll be fine." "The procedure may be uncomfortable, but we all feel things differently." "The procedure is somewhat uncomfortable, but you have nothing to worry about." "The procedure can be uncomfortable; let's talk about things you can do to help you through it."

"The procedure can be uncomfortable; let's talk about things you can do to help you through it." An honest answer allows the patient to trust the nurse and staff and therefore would be the most appropriate response for the nurse to make. Additionally, it is important to involve the patient as much as possible.

A nurse discusses the maturation of systems in a school-age child with student nurses. What statement made by a student nurses indicates effective learning? "The heart grows faster in the school-age child than at any other period of life." "The school-age child can have several infections in the first 1 to 2 years of school." "Physical maturity in the school-age child correlates with emotional and social maturity." "The school-age child needs to be fed more carefully, promptly, and frequently than a preschooler."

"The school-age child can have several infections in the first 1 to 2 years of school." Rationale School-age children can have several infections in the first 1 to 2 years of school because of increased exposure to others in school classes while immunity develops. The heart grows more slowly during the middle years and is smaller in relation to the rest of the body than at any other period of life. Physical maturity in the school-age child is not necessarily correlated with emotional and social maturity. Children will generally have the emotional and social maturity that corresponds to their age. Because the gastrointestinal system in a school-age child is more mature than that in a preschooler, the school-age child does not need to be fed as carefully, as promptly, or as frequently as before.

A parent of a chronically ill child is concerned about how the child's sibling will react to the illness. Which statements by the nurse should be included in teaching the parent about the sibling's reactions toward the illness? Select all that apply. "The sibling experiences the loss of a playmate." "The sibling will be happy with a care provider." "The sibling may harbor feelings of loneliness and jealousy." "The sibling is not impacted by the chronically ill child." "The sibling may fear contracting the same condition."

"The sibling experiences the loss of a playmate." "The sibling may harbor feelings of loneliness and jealousy." "The sibling may fear contracting the same condition." Rationale Losing a playmate adds to the loneliness of the sibling and can cause boredom and frustration. With all attention focused on the ill child, the sibling may feel alone and less supported. Depending on the age, the sibling may or may not fully understand the nature of the chronic condition and will harbor fears of catching the same disease.

A mother states that she is worried about how a preschool-age sibling will react to the hospitalization of her dying infant. What does the nurse advise about the most likely reaction of the preschooler? "The sibling will not react at all." "The sibling will enjoy the freedom." "The sibling will know that you will return home." "The sibling will complain of an ailment, like headache."

"The sibling will complain of an ailment, like headache." Rationale The nurse should explain that the sibling will complain of stomach aches, headaches, or other physical symptoms to attract the parents' attention. The sibling will most likely have some sort of reaction and is not likely to enjoy any freedom. The sibling will develop a mindset that the parents will not ever return home.

Parents of a child with type 1 diabetes tell the nurse, "Our child will never have the life we dreamed for him." Which response by the nurse is appropriate? "Your child will be able to live whatever life they choose." "You must remain positive or your child will become discouraged." "You are grieving the loss of your 'perfect' child. Tell me more about how you feel." "There have been great advances in treating type 1 diabetes. Let me share them with you."

"There have been great advances in treating type 1 diabetes. Let me share them with you." Education decreases fears and misconceptions and encourages appropriate interactions between family members, the child, and health care providers. This is an appropriate response by the nurse.

A four-year-child with 6 months to live tells parents about angels who come to visit her at night. The parents are concerned and ask the nurse how they should handle this. What is the nurse's best response? a. "This response is based on your family's belief in God and should bring you comfort." b. "This response is concerning; you should talk to your child about fantasy versus reality." c. "This response demonstrates that your child may be declining faster than we thought. Let us know if the child says anything else." d. "This is a normal response for a 4-year-old child who is terminally ill. Continue to allow the child to share the experiences with you."

"This is a normal response for a 4-year-old child who is terminally ill. Continue to allow the child to share the experiences with you." The responses and actions of the dying child are affected by such things as visions of angels and are considered to be normal responses for the child.

A parent tells the nurse, "I'm worried about my 13-year-old son. He hasn't started puberty, but my daughter did when she was 11 years old." What is the appropriate explanation by the nurse? "This is unusual and requires further evaluation of your son." "This is unusual because the onset of pubescence is usually the same in siblings." "This is normal because the onset of pubescence is usually earlier in girls than it is in boys." "This is abnormal because the onset of pubescence is usually earlier in boys than it is in girls."

"This is normal because the onset of pubescence is usually earlier in girls than it is in boys." Rationale Girls begin puberty on average approximately 2 years before boys. Puberty usually begins no earlier than age 12 years in boys, with an average age of onset of 14 years; therefore onset of puberty is not abnormal in this case, and no further evaluation is necessary at this time.

A parent tells the nurse, "I'm worried about my 13-year-old son. He hasn't started puberty, but my daughter did when she was 11 years old." What is the appropriate explanation by the nurse? "This is unusual and requires further evaluation of your son." "This is unusual because the onset of pubescence is usually the same in siblings." "This is normal because the onset of pubescence is usually earlier in girls than it is in boys." "This is abnormal because the onset of pubescence is usually earlier in boys than it is in girls."

"This is normal because the onset of pubescence is usually earlier in girls than it is in boys." Rationale Girls begin puberty on average approximately 2 years before boys. Puberty usually begins no earlier than age 12 years in boys, with an average age of onset of 14 years; therefore onset of puberty is not abnormal in this case, and no further evaluation is necessary at this time.

The parents of a 3-year-old patient who is undergoing radiation therapy ask the nurse why radiation is necessary. What is the best response from the nurse? Select all that apply. "To eliminate cancer cells." "To prevent nausea and vomiting." "To replenish immunologic function." "To promote bone marrow suppression." "To prevent further growth of the cancer cells."

"To promote bone marrow suppression." Radiation therapy is used to suppress bone marrow prior to bone marrow and stem cell transplantation. "To prevent further growth of the cancer cells." Radiation therapy can be used as palliative therapy to prevent further tumor growth. "To eliminate cancer cells." Radiation therapy is used to eradicate the tumor by killing cancer cells.

A mother calls the nurse and reports that her child has ingested a toxin. Which statement by the nurse explains why inducing vomiting is contraindicated? "Vomiting can increase the toxicity of the agent." "Vomiting may cause additional damage to the esophagus." "Vomiting can increase the absorption in the oral mucosa." "Vomiting may cause bowel rupture due to the increase in pressure."

"Vomiting may cause additional damage to the esophagus." As the ingested agents makes a second trip through the esophagus, it may cause additional damage or burning of the tissue.

After interacting with the parents of a 10-year-old child, the nurse understands that they are effectively following guidelines to help their child with school work and grades. What statement made by the parents supports the conclusion of the nurse? "We have arranged home tutoring sessions for our child twice a day." "We are strict and allow our child to watch television only on achieving good grades." "We discuss all our concerns with the teacher at the annual parent-teacher conference." "We help our child with homework by explaining the questions and avoid giving answers."

"We help our child with homework by explaining the questions and avoid giving answers." Rationale The parents should help the child complete homework by explaining the concepts and encouraging them to find the answers. If the parents give answers, the child may not read the complete material and will not gain complete knowledge. Parents should communicate with the child's teachers regularly instead of meeting once during an annual conference. The child may feel stressed due to excessive sessions of tutoring. The parents should arrange tutoring sessions only when they are recommended by the child's teachers. Parents should encourage the child to focus on studies rather than on grades. The child may feel stressed if strict rules are imposed for not getting good grades.

The nurse cares for a five-year-old patient involved in a motor vehicle accident. The paralysis extends from the naval downward. In performing discharge teaching, the nurse knows further teaching is needed when the parents make which statements? Select all that apply. "We need to catheterize him every 8 hours for urine." "We need to turn him in the bed at least every 2 hours." "We need to make sure he has a bowel movement often." "He will need to eat every meal that we prepare for him." "He will enjoy sitting outside all morning in his wheelchair."

"We need to catheterize him every 8 hours for urine." The child should not hold urine in his bladder for an extended period of time. Since the child cannot feel the urge to urinate, catheterization does need to occur on a schedule. If urine is left in the bladder, causing bladder distention, a kidney infection may result. Catheterization should occur instead every 4-6 hours. "We need to make sure he has a bowel movement often." The parents need to make sure that the child has a bowel movement every 1-2 days, rather than stating "often." The stool should be softly formed rather than hard or loose. With little innervation to the bowels, peristalsis will be limited and constipation quickly becomes an obstruction. Correct "He will need to eat every meal that we prepare for him." This is not the best way to make sure the child gets adequate nutrition. Five-year-olds normally ingest small portions and they can love a food one day and hate it the next. Nutritional teaching includes information about snacks, supplements, and nutrient-dense foods. Correct "He will enjoy sitting outside all morning in his wheelchair." The child should not sit up in the wheelchair for an extended period of time. This statement needs to be clarified. Sitting for more than an hour or two can produce pressure ulcers.

Which statement by the parents indicates the need for further teaching by the nurse about the use of medicine in bacterial conjunctivitis? "We can use sodium sulfacetamide." "We should apply eye ointment before bedtime." "We should use corticosteroids to treat conjunctivitis." "We can administer eye drops anytime during the day."

"We should use corticosteroids to treat conjunctivitis." Rationale Inflammation of the conjunctiva due to bacterial infection is called bacterial conjunctivitis. Corticosteroids should be avoided because they reduce ocular resistance to bacteria and worsen the infection. Eye drops should be used during the day and an ointment at bedtime because the ointment preparation remains in the eye for longer duration. Sodium sulfacetamide is a topical antibacterial agent and is used to treat bacterial conjunctivitis.

The nurse is educating the parents of a child after surgery related to retinoblastoma. Which statement, made by the parents, indicates the need for further teaching? "We will bring her back here to have the prosthesis cleaned." "My child will be able to compensate with the unaffected eye." "Our other children need to be checked regularly for signs of the disease." "It is okay for my child to play sports, but protective eye wear is necessary."

"We will bring her back here to have the prosthesis cleaned." This statement, made by the parent, indicates a need for further teaching. The child and parents should be taught to maintain the prosthesis at home.

The nurse is providing education to the parents of a child experiencing spinal shock after a spinal cord injury. Which statements by the nurse are correct? Select all that apply. "We will not know what permanent injuries exist for one to two months." "Currently, the child appears to have no function below the level of injury." "Limbs that are currently flaccid will remain that way for about six months." "Nurses will perform passive range of motion exercises daily to maintain muscles." "Some complications, such as low blood pressure, will resolve within a few weeks."

"We will not know what permanent injuries exist for one to two months." The injuries associated with spinal shock can cause temporary loss of function, which means regaining function can occur. This statement could therefore be used by the nurse to address concerns regarding the recovery phase. Correct "Currently, the child appears to have no function below the level of injury." Spinal shock can cause deceased function of everything below the level of injury including musculoskeletal abilities, bladder and bowel control, and so on. When the shock resolves, some function may return. "Some complications, such as low blood pressure, will resolve within a few weeks." Complications related to spinal shock, such as inability to maintain blood pressure, control temperature, and manage heart rate should resolve when the spinal shock resolves, and therefore this information can be relayed to the parents.

During the home visit, the parents of a 10-year-old child ask the nurse about safety guidelines because the child wants a bicycle. What statement by the parents indicates the nurse's teaching has been effective? "Replace your helmet every 10 years." "Give hand signals just before turning the bicycle." "Wear light-colored clothes while riding the bicycle at night." "Carry heavy objects at the back of the bicycle instead of the front."

"Wear light-colored clothes while riding the bicycle at night." Rationale The nurse teaches bicycle safety guidelines to the child to prevent injuries due to accidents. The child should be instructed to wear light-colored clothes and attach a florescent cloth to the bicycle at night to increase visibility and prevent accidents. The child should be instructed to buy properly fitted helmets that are approved by the Consumer Product Safety Commission. The helmets should be replaced every 5 years or sooner as recommended by the manufacturer. Old helmets may not protect from head injuries. The child should be instructed not to carry heavy objects either on front or back of the bicycle, because the child may not be able to balance and can fall. Hand signals should be given well in advance so that others can regulate their speed and change their direction.

A mother of a terminally ill child tells the nurse, "Sometimes I wonder if I am doing the right thing." What is the most therapeutic response? "Why do you feel this way?" "What are your concerns right now?" "I would do the same thing if I were you." "Your doctor is a really good doctor; you are in good hands."

"What are your concerns right now?" Rationale The most therapeutic response to a parent stating "Sometimes I wonder if I am doing the right thing" is to use an open-ended, nonjudgmental question such as, "What are your concerns right now?" The question "Why do you feel this way?" is a communication block because it is judgmental and makes the person feel defensive. "I would do the same thing if I were you," may appear supportive but is also blocking communication, because it is giving an opinion and not encouraging communication. "Your doctor is a really good doctor; you are in good hands," is somewhat defensive of the doctor and does not encourage communication either.

Which nursing instruction about injury prevention would be appropriate during a 4-month-old well baby checkup? "Keep the doors of appliances closed at all times." "Never shake baby powder directly on the baby, because it can be aspirated into the lungs." "Don't let the baby chew paint from window ledges, because lead may be absorbed or ingested." "When the baby learns to roll over, you'll need to continue to supervise especially if the baby is at risk of falling from a surface."

"When the baby learns to roll over, you'll need to continue to supervise especially if the baby is at risk of falling from a surface." Rationale Rolling over from the abdomen to the back occurs between 4 and 7 months of age. This statement is the appropriate anticipatory guidance for this age related to the prevention of injuries. "Never shake baby powder directly on the baby, because it can be aspirated into the lungs," is appropriate guidance for a first month well baby checkup related to injury prevention. Information on lead, and lead sources, should be included at the 9-month visit, when the child is beginning to crawl and can move to a standing position. Guidance regarding appliances and keeping doors of appliances closed should also be included at this time.

A child has been diagnosed with a brain tumor. The parents tell the nurse, "We have been taking good care of our child, but we do not know how this happened." Which is the nurse's best response in this situation? "It is not your fault that your child has a brain tumor." "Would you like to tell me more about how you are feeling?" "Do you think you could have prevented your child's brain tumor?" "Many children who have been diagnosed with brain tumors have been cured."

"Would you like to tell me more about how you are feeling?" Rationale Parents of children with life-threatening conditions require emotional support. Therefore, the nurse should ask the parents if they would like to talk more about how they are feeling. The nurse should acknowledge all of the parents' reactions using the emphatic response technique and respond appropriately. While parents may feel responsible for their child's conditions, they will not necessarily feel better by being reassured that they are not responsible for the child's condition. Talking about why the parents think they could have prevented the child's brain tumor is not appropriate. Telling the parents that many children who have been diagnosed with brain tumors have been cured is not a supportive response.

The nurse is counseling parents who lost their child 5 months ago to a terminal illness. Which statement made by the nurse is correct and appropriate in this situation? "You should be feeling better by now." "You are still feeling all the pain of losing a child." "You are still young enough to have another baby." "You should be glad that your child is not suffering anymore."

"You are still feeling all the pain of losing a child." Rationale While supporting grieving parents, the nurse should focus on the parents' feelings by using words such as "feeling." The nurse should accept the parents' grief reactions and avoid judgmental statements such as, "You should be feeling better now." The nurse should avoid artificial consolation such as, "You are still young enough to have another baby." The nurse should avoid statements offering rationalization for the child's death such as, "You should be glad that your child is not suffering anymore."

A parent is taking care of her child dying of cancer and asks the nurse about oral care since the child often complains of a dry mouth. What education on oral care can the nurse provide to this parent? Select all that apply. "You can moisten her lips with this sponge swab." "Use the artificial saliva drops as needed to provide comfort." "A mint flavored mouth spray may decrease the dry sensation." "Using the mouth swabs every shift will provide comfort for the child." "Petroleum jelly on the lips provides moisture for a longer period of time."

"You can moisten her lips with this sponge swab." Sponge swabs provide moisture for the lips and mouth of the dying child. Correct "Use the artificial saliva drops as needed to provide comfort." Artificial saliva drops add moisture to the mouth and provide comfort. "Petroleum jelly on the lips provides moisture for a longer period of time." Petroleum jelly or lip balm is used to treat dry, chapped lips.

The parent of a 15-year-old with terminal cancer approaches the nurse and asks what care options are available for when the child is dying. The parent states the child is very close to family and siblings and loves to be around the pet dogs. Which is the best response? "Your child could spend the last weeks in the hospital." "You could care for your child at home with your family helping." "You could use palliative care services to make your child comfortable." "You could use home hospice care so the child is in a comfortable and relaxed environment.

"You could use home hospice care so the child is in a comfortable and relaxed environment." Hospice care can be provided in the home and allows for a more comfortable and relaxed environment than the hospital setting.

Which nursing instruction is appropriate when teaching a pregnant woman that has a child with erythema infectiosum? "The child needs two doses of vitamin A injections on consecutive days." "It is a bacterial infection. Both you and the child should take antibiotics." "It spreads by contaminated water; hence, you should drink safe water." "You have to have fetal ultrasonography for detection of fetal hydrops."

"You have to have fetal ultrasonography for detection of fetal hydrops." Rationale Pregnant women in contact with erythema infectiosum should be advised to undergo fetal ultrasonography for the detection of fetal hydrops. A child with measles needs vitamin A injections on consecutive days. Administering antibiotics is not required because erythema infectiosum is a viral infection. It is transmitted by respiratory secretions, blood, and blood products, not by contaminated water.

The parents of a dying child tell the nurse that they would prefer not to tell their child that the condition is terminal. They also avoid the subject of death when their child speaks about it. What does the nurse advise the parents? "You may answer the child's questions honestly." "It is wrong to hide such information from the child." "You may avoid speaking about it as long as you want." "You must hide your sorrow when speaking to the child."

"You may answer the child's questions honestly." Rationale The nurse advises the parents to speak honestly to their child about dying, because it helps to promote the emotional health of the parents and the child. The nurse encourages the parents to express their sorrow while speaking to the child. This helps to allow the parents to speak about dying. The nurse does not make critical statements about the parents' behavior, because this is unethical. The nurse asks the parents to speak to the child instead of telling them to avoid the subject.

The parents of a dying child tell the nurse that they would prefer not to tell their child that the condition is terminal. They also avoid the subject of death when their child speaks about it. What does the nurse advise the parents? "You may answer the child's questions honestly." "It is wrong to hide such information from the child." "You may avoid speaking about it as long as you want." "You must hide your sorrow when speaking to the child."

"You may answer the child's questions honestly." Rationale The nurse advises the parents to speak honestly to their child about dying, because it helps to promote the emotional health of the parents and the child. The nurse encourages the parents to express their sorrow while speaking to the child. This helps to allow the parents to speak about dying. The nurse does not make critical statements about the parents' behavior, because this is unethical. The nurse asks the parents to speak to the child instead of telling them to avoid the subject.

A 6-month-old infant died suddenly in the crib during the winter. There was no identifiable reason for the death, even after conducting a postmortem examination. What would the nurse instruct the mother to do to decrease the possibility of sudden infant death syndrome (SIDS) in another infant? Select all that apply. "You should not smoke while you are pregnant." "Do not allow the infant to sleep with a pacifier." "Your husband should not ever smoke in the home." "The baby should always sleep in the supine position." "You should have the baby sleep with you in your bed."

"You should not smoke while you are pregnant." "Your husband should not ever smoke in the home." "The baby should always sleep in the supine position." Rationale SIDS may be one of the possible reasons for the unexplained death of a baby. The parents should be advised to place the baby on the back when sleeping. The prone position increases the risk for SIDS because pharyngeal obstruction can occur or the baby may rebreathe exhaled carbon dioxide. Smoking by the pregnant mother or others in the home during the pregnancy or after the delivery should be strictly discouraged. Co-sleeping with the infant increases the risk for SIDS because it can lead to suffocation. Studies have shown that when the baby sleeps with a pacifier there is a decreased incidence of SIDS.

The student nurse is teaching the parents of a child who is receiving palliative care how to explain death to the child in a developmentally appropriate manner. Which statement made by the student nurse needs correction? "You should accept the emotions your child expresses." "You should ask your child to repeat what was discussed." "You should use phrases such as 'pass away' instead of 'die'." "You should use trees and leaves as examples to explain death to your child."

"You should use phrases such as 'pass away' instead of 'die'." Rationale The nurse should instruct the parents to use age-appropriate words such as "die" instead of "pass away" while explaining death to their child. Parents should accept the emotions expressed by their child as well as ask their child to repeat what has been discussed to clarify any misunderstandings. The nurse should suggest that the parents use examples of trees and leaves and how long they live while explaining death to their child.

A nurse is caring for a child with a chronic illness who was recently hospitalized due to complications of the illness. After leaving the patient's room, the nurse notices a sibling standing outside the room looking very angry. The sibling says, "Today is my birthday and nobody even cares." Which response from the nurse is best? "You sound upset. Tell me more about how you are feeling." "Happy Birthday! Let's find you something to celebrate with." "Your parents care about you; let's try to be positive in this situation." "Let's go see your brother since he is sick, and then we can celebrate your birthday."

"You sound upset. Tell me more about how you are feeling." This statement acknowledges the feelings of the sibling and allows the opportunity for discussion of those feelings.

The nurse administers a dose of the human papillomavirus (HPV) vaccine to an 11-year-old patient in the clinic. The nurse should provide which instruction to the patient's parents? "Your child needs the second dose in one to two months." "The HPV vaccine is given annually to provide effective protection." "HVP is a one-dose vaccine. There are no more HPV shots required." "A booster dose of HPV should be given to sexually active individuals at age 18."

"Your child needs the second dose in one to two months." Human papillomavirus (HPV) is a three-dose series. The first dose was given today. The second dose should be administered in one to two months. The third dose is administered 16 weeks after dose #2.

The nurse is preparing a 3-year-old patient for a positron emission tomography (PET) scan to diagnose a neck mass. The child's caregiver asks what will be done to prevent potential discomfort and restlessness of the child during the procedure. What should the nurse respond? "Your child will be sedated prior to the procedure." "We will administer opioid analgesia 2 hours prior to the procedure." "You will be allowed to remain in the room at all times with your child." "Soft wrist restraints during the procedure will help the procedure go faster."

"Your child will be sedated prior to the procedure." Young children may need to be sedated prior to a PET scan because they will need to remain still during the procedure

A parent of a child who has terminal cancer is concerned with the child's weight loss due to a decrease in appetite. The parent becomes frustrated and expresses to the nurse, "My child needs to be fed; she is starving!" What is the best response? "Tell me why you feel your child is starving." "Your child's lack of interest in food is a normal part of the dying process." "We can use liquid nutritional supplements with your child to help them gain weight." "We will have the dietician work with you to create food options that will help your child gain weight."

"Your child's lack of interest in food is a normal part of the dying process." The lack of interest in food and liquids is a normal part of the dying process and a source of great distress for family members. Educating the parent on the reason for the weight loss dispels the misconception that the child is starving to death.

A 4-year-old child is dying of an inoperable brain tumor. The child is conscious for only a few hours out of every day and doesn't talk much when awake. The parents tell the nurse what the child has told them. "Don't be sad when I die. I will go to sleep and then wake up and be here with you." The parents ask the nurse how to respond to the child's statements. Which response by the nurse is appropriate? "Your child's perception of death is that death is reversible and temporary and is appropriate for the developmental age." "Your child perceives death as a sad but reversible separation. This is an inappropriate perception for the child's developmental age." "Your child's perception of death is abnormal and concerning for the developmental age. Your child should be focused on death as a distant event." "Your child's perception of death is that death is temporary with the loss of you as the care provider. This is appropriate for the developmental age."

"Your child's perception of death is that death is reversible and temporary and is appropriate for the developmental age." In early childhood, children are at the preoperational cognitive stage and perceive death as reversible and temporary.

cancer/ review of systems

- Skin (history of bruising or bleeding, lesions, lumps, or open sores) - HEENT (history of trauma, vision disturbances, proptosis, pupil discoloration, unequal pupils, eye muscle weakness, infection, difficulty swallowing) - Heart (history of murmur of thrill) - Lungs (history of infection, asthma or reactive airway disease, cough, wheezing, dyspnea) - Abdomen (history of abdominal swelling, pain, mass, changes in bowel or bladder problems) - Musculoskeletal (history of weakness in extremities, limited ROM, tenderness, swelling, joint pain) - Neurological (loss of developmental milestones, altered LOC, decreased sensations, abnormal reflexes, abnormal cerebellar functions, headaches, seizures) - Lymphatic (history of enlarged lymph nodes or frequent infections) - Hematological (history of bruising, epistaxis, bleeding gums, paleness, fatigue, bloody or tarry stools)

nursing intervention related to parasitic infection

- assistance with identification of the parasite - treatment of the infection - prevention of initial infection or reinfection for eradication: - drug compliance - throurough hand washing

activated charcoal administration

- child ingested large amount of carbamazepine, dapsone, phenobarbital, quinine, theophylline - time to activate within 1 h - child has intact or protected airway - mixed with water or saline - mix with chocolate milk, fruit syrop, cola drink, give through straw in opaque container with a cover -charcoal through NG for small children complications - potential aspiration, constipation, intestinal obstruction

preoperapive interventions brain tumor

- perform a neurological assessment Q4H - institute seizure precautions and safety measure - assess weight loss and nutritional - shave the child's head - prepare child as much as possible

toddlers injury general

- unintentional childhood injury - major cause of death - males have higher rate - leading cause of accidental death - suffocation and drowning - cause of nanfatal injuries - falls - major factor in increased incidence of injuries - increased mobility, locomotion, curiosity, poor perception of depth and danger

Retinoblastoma:

-arises from retina -usually occurs in infants -may be caused by genetic alterations Sx of retinoblastoma: -parents notice whitish glow of pupil -"cats eye reflex" -may notice in photograph with flash -strabismus -blindness Cats eye reflex also known as... leukoconia Dx of retinoblastoma: -opthalmoscopic exam -CT -MRI blood 7 tumor samples Prognosis of retinoblastoma: 90% survival rate Tx of retinoblastoma: -surgery -radiation -chemo (remove eye, replace with prosthetic)

Mucosal ulceration:

-ulcer anywhere along alimentary tract -reddened, eroded areas mouth and pharynx, esophagus, rectal -makes eating extremely uncomfortable, leads to anorexia What diet may help mucosal ulceration? -bland, soft moist diet soft sponge toothbrush rinse mouth witn chlorhexidine or sodium bicarbonate What may also help mucosal ulceration? -salt water rinsesbaking soda, salt, water sucralfate solution of diphehindramine and maalox avoid lemon glycerin swabs and hydrogen peroxide -diphenhydramine and Maalox swab -vicious lidocaine not for children, can depress gag reflex and increase risk of aspiration and seizures can happen toddlers - finger with gauze soaked in saline or water, Q2-4H drink with straw kids prefer salty food for rectal ulcers - toilet hygiene, warm sitz bath after each bowel movement, occlusive ointment, stool softeners

Which age in years did more than 60% of exposures to plants occur, according to the American Association of Poison Control Centers? Select all that apply. 1 to 2 years 4 to 5 years 6 to 7 years 9 to 11 years 12 to 14 years

1 to 2 years 4 to 5 years Rationale In a recent review of the American Association of Poison Control Centers, more than 60 percent of exposures to plants occurred in children aged 5 years and under, not 6 years or older

The hepatitis A vaccine is now recommended for the first dose started at what age? 1 year 1 month 12 years It is not recommended at any age

1 year Correct Hepatitis A has been recognized as a significant child health problem, particularly in communities with unusually high infection rates. Hepatitis A virus is spread by the fecal-oral route and from person-to-person contact, by ingestion of contaminated food or water, and rarely by blood transfusion. HepA vaccine is now recommended for all children beginning at age 1 year (i.e., 12 to 23 months). The second dose in the 2-dose series may be administered no sooner than 6 months after the first dose

The nurse analyzes the laboratory values of a child with leukemia who is receiving chemotherapy. The nurse notes that the platelet count is 19,500 cells/mm3. On the basis of this laboratory result, which intervention should the nurse include in the plan of care? 1. Initiate bleeding precautions. 2. Monitor closely for signs of infection. 3. Monitor the temperature every 4 hours. 4. Initiate protective isolation precautions.

1. Initiate bleeding precautions. Leukemia is a malignant increase in the number of leukocytes, usually at an immature stage, in the bone marrow. It affects the bone marrow, causing anemia from decreased erythrocytes, infection from neutropenia, and bleeding from decreased platelet production (thrombocytopenia). If a child is severely thrombocytopenic and has a platelet count less than 20,000 cells/mm3, bleeding precautions need to be initiated because of the increased risk of bleeding or hemorrhage. Precautions include limiting activity that could result in head injury, using soft toothbrushes, checking urine and stools for blood, and administering stool softeners to prevent straining with constipation. In addition, suppositories, enemas, and rectal temperatures are avoided. Options 2, 3, and 4 are related to the prevention of infection rather than bleeding.

Which patient is the best candidate for organ donation? A child who died of sepsis A child who died of metastatic cancer A child who died of prolonged cardiac arrest A child who died in a motor vehicle accident

A child who died in a motor vehicle accident Rationale A child who died unexpectedly, like in a motor vehicle accident, is the best candidate for organ donation because the organs are more likely to be healthy than are those of a child who died of metastatic cancer, sepsis, or a prolonged cardiac arrest. p. 622

The nurse is monitoring a 3-year-old child for signs and symptoms of increased intracranial pressure (ICP) after a craniotomy. The nurse plans to monitor for which early sign or symptom of increased ICP? 1. Vomiting 2. Bulging anterior fontanel 3. Increasing head circumference 4. Complaints of a frontal headache

1. Vomiting The brain, although well protected by the solid bony cranium, is highly susceptible to pressure that may accumulate within the enclosure. Volume and pressure must remain constant within the brain. A change in the size of the brain, such as occurs with edema or increased volume of intracranial blood or cerebrospinal fluid without a compensatory change, leads to an increase in intracranial pressure (ICP), which may be life-threatening. Vomiting, an early sign of increased ICP, can become excessive as pressure builds up and stimulates the medulla in the brainstem, which houses the vomiting center. Children with open fontanels (posterior fontanel closes at 2 to 3 months; anterior fontanel closes at 12 to 18 months) compensate for ICP changes by skull expansion and subsequent bulging fontanels. When the fontanels have closed, nausea, excessive vomiting, diplopia, and headaches become pronounced, with headaches becoming more prevalent in older children.

help parents to communicate with dying child

1. encourage to approach the discussion gently ( what and how said) 2. begin discussion with non threatening topic like trees and leaves 3 allow child's questions to guide the discussion, speak simple 4, clarify misconseptions that she didnt cause illness, accept any emotions, provide warmth 5 ask to repeat to clarify that child understoood 6. encourage parents to discuss the child's impeding death openly and honestly with child, siblings

The pediatric nurse specialist provides a teaching session to the nursing staff regarding osteosarcoma. Which statement by a member of the nursing staff indicates a need for information? 1. "The femur is the most common site of this sarcoma." 2. "The child does not experience pain at the primary tumor site." 3. "Limping, if a weight-bearing limb is affected, is a clinical manifestation." 4. "The symptoms of the disease in the early stage are almost always attributed to normal growing pains."

2. "The child does not experience pain at the primary tumor site." Osteosarcoma is the most common bone cancer in children. Cancer usually is found in the metaphysis of long bones, especially in the lower extremities, with most tumors occurring in the femur. Osteosarcoma is manifested clinically by progressive, insidious, and intermittent pain at the tumor site. By the time these children receive medical attention, they may be in considerable pain from the tumor. Options 1, 3, and 4 are accurate regarding osteosarcoma.

A 4-year-old child is admitted to the hospital for abdominal pain. The mother reports that the child has been pale and excessively tired and is bruising easily. On physical examination, lymphadenopathy and hepatosplenomegaly are noted. Diagnostic studies are being performed on the child because acute lymphocytic leukemia is suspected. The nurse understands that which diagnostic study should confirm this diagnosis? 1. Lumbar puncture showing no blast cells 2. Bone marrow biopsy showing blast cells 3. platelet count of 350000 4. White blood cell count 4500

2. Bone marrow biopsy showing blast cells Leukemia is a malignant increase in the number of leukocytes, usually at an immature stage, in the bone marrow. The confirmatory test for leukemia is microscopic examination of bone marrow obtained by bone marrow aspirate and biopsy. A lumbar puncture may be done to look for blast cells in the spinal fluid that indicate central nervous system disease. The white blood cell count may be normal, high, or low in leukemia. An altered platelet count occurs as a result of the disease, but also may occur as a result of chemotherapy and does not confirm the diagnosis.

The nurse is monitoring a child for bleeding after surgery for removal of a brain tumor. The nurse checks the head dressing for the presence of blood and notes a colorless drainage on the back of the dressing. Which intervention should the nurse perform immediately? 1. Reinforce the dressing. 2. Notify the health care provider (HCP). 3. Document the findings and continue to monitor. 4. Circle the area of drainage and continue to monitor.

2. Notify the health care provider (HCP). Colorless drainage on the dressing in a child after craniotomy indicates the presence of cerebrospinal fluid and should be reported to the HCP immediately. Options 1, 3, and 4 are not the immediate nursing intervention because they do not address the need for immediate intervention to prevent complications.

A 6-year-old child with leukemia is hospitalized and is receiving combination chemotherapy. Laboratory results indicate that the child is neutropenic, and protective isolation procedures are initiated. The grandmother of the child visits and brings a fresh bouquet of flowers picked from her garden and asks the nurse for a vase for the flowers. Which response should the nurse provide to the grandmother? 1. "I have a vase in the utility room, and I will get it for you." 2. "I will get the vase and wash it well before you put the flowers in it." 3. "The flowers from your garden are beautiful, but should not be placed in the child's room at this time." 4. "When you bring the flowers into the room, place them on the bedside stand as far away from the child as possible."

3. "The flowers from your garden are beautiful, but should not be placed in the child's room at this time." Leukemia is a malignant increase in the number of leukocytes, usually at an immature stage, in the bone marrow. It affects the bone marrow, causing anemia from decreased erythrocytes, infection from neutropenia, and bleeding from decreased platelet production (thrombocytopenia). For a hospitalized neutropenic child, flowers or plants should not be kept in the room because standing water and damp soil harbor Aspergillus and Pseudomonas aeruginosa, to which the child is susceptible. In addition, fresh fruits and vegetables harbor molds and should be avoided until the white blood cell count increases.

A diagnosis of Hodgkin's disease is suspected in a 12-year-old child seen in a clinic. Several diagnostic studies are performed to determine the presence of this disease. Which diagnostic test result will confirm the diagnosis of Hodgkin's disease? 1. Elevated vanillylmandelic acid urinary levels 2. The presence of blast cells in the bone marrow 3. The presence of Epstein-Barr virus in the blood 4. The presence of Reed-Sternberg cells in the lymph nodes

4. The presence of Reed-Sternberg cells in the lymph nodes Hodgkin's disease (a type of lymphoma) is a malignancy of the lymph nodes. The presence of giant, multinucleated cells (Reed-Sternberg cells) is the classic characteristic of this disease. Elevated levels of vanillylmandelic acid in the urine may be found in children with neuroblastoma. The presence of blast cells in the bone marrow indicates leukemia. Epstein-Barr virus is associated with infectious mononucleosis.

A patient is born with spina bifida, a neural tube defect. The nurse understands that this defect occurred during what week of gestation? 4th week of gestation 15th week of gestation 20th week of gestation 30th week of gestation

4th week of gestation The 4th week of gestation is the stage in which the neural tube closes. Lack of maternal folic acid during first 28 days of gestation is considered a teratogenic cause of spinal cord malformations.

Which erythrocyte protoporphyrin (EP) blood level is a good indicator of lead toxicity? >7 mcg/dl >16 mcg/dl >25 mcg/dl >35 mcg/dl

>35 mcg/dl Rationale Elevation of the erythrocyte protoporphyrin (>35 mcg/dl of whole blood) is a good indicator of toxicity from lead and reflects the length of exposure and body burden of lead in an individual child.

to assist school aged children in coping with stress in their lives, the nurse should: (select all) a. be able to recognize signs that indicate the child is undergoing stress b. teach the child how to recognize signs of stress in himself c. help the child plan a means for dealing with any stress through problem solving d. reassure the child that the stress is only temporary

A B C

middle childhood is a time when children: (select all) a. learn the value of doing things with others b. learn the benefits derived from division of labor in accomplishing goals c. acheive a sense of industry and accomplishment d. expand interests and engage in tasks that can be carried to completion

A B C D

Leukemia

A malignant increase in the number of leukocytes, usually at an immature stage, in bone marrow In Leukemia, proliferating white blood cells depress bone marrow, causing anemia from decreased erythrocytes, infection from neutropenia, and bleeding from decreased platelet production (thrombocytopenia)

The parents of a school-aged child newly diagnosed with asthma are receiving discharge education. What information should the nurse emphasize to address the need for follow-up appointments? Asthma is an illness that is unique because it requires close monitoring. The child with asthma will receive routine follow-up care during the annual check-up. The illness trajectory of asthma indicates that the child will need to follow-up more than other chronic illnesses usually require. A child with a chronic illness needs regular care monitoring and follow-up and should work to develop a positive relationship with the health care team.

A child with a chronic illness needs regular care monitoring and follow-up and should work to develop a positive relationship with the health care team. The need for regular assessment, monitoring, and follow-up is much greater in children with chronic illnesses than it is for the general population of children. Detecting issues in this population early is important to decrease long-term issues.

Why is it important to identify the special needs for children with chronic and terminal illness? The definition helps to define the illness trajectory that a chronic illness might take. The definition describes the specific types of support and interventions that a child and family will need. A specific definition of chronic and terminal illness allows for easier billing and obtaining insurance coverage for treatments. A clear definition can help families and caregivers obtain the support necessary to adequately care for children with special health care needs.

A clear definition can help families and caregivers obtain the support necessary to adequately care for children with special health care needs. A clear definition of special health needs allows for enhanced advocacy and planning care specific to this category of chronic conditions

The nurse is caring for a four-year-old patient diagnosed with fungal meningitis. What is the most likely etiology in this child? Parents refuse to vaccinate the child Attending a child care center during the week Child has been recently diagnosed with type II diabetes A diagnosis of acquired immunodeficiency syndrome (AIDS)

A diagnosis of acquired immunodeficiency syndrome (AIDS) Infection with the virus that causes AIDS is the most likely concomitant infection in a child who has been diagnosed with fungal meningitis. Fungal meningitis occurs in those who are immunocompromised, not in children who are otherwise healthy.

approach to care of the child and family facing life-threatening illness or death

A family-centered approach to care of the child and family facing life-threatening illness or death respects the central role of the family and emphasizes communication, collaboration, and cultural sensitivity. Trends in the treatment of children with chronic illness or disability focus on developmental age, the child's strengths and uniqueness, family-centered care, normalization, early discharge, home care, mainstreaming, and early intervention. Using a developmental approach rather than one based on chronologic age when caring for children with special health care needs helps the nurse determine the children's current status and understand their response to the chronic illness or disability. Developmental changes in the child continue despite the added stress of coping with a chronic illness. In response to the child with chronic illness or disability, parents may be affected by feelings of inadequacy and failure; excessive demands on their time, energy, and financial resources; and strain on the marital relationship. Families' reactions to disability or chronic illness are manifested in the following stages: shock and denial, adjustment, reintegration, and acknowledgment. The child's reaction to illness or disability depends on the child's developmental level, coping mechanisms, reactions of others, and the illness itself. A child's conceptual understanding of his or her own illness is based not only on age and developmental level but also on the duration and type of experience accumulated with the disease.

Which scenarios may reduce the transmission of infectious agents? Select all that apply. A health care worker uses standard precautions with all patients. A health care worker removes gloves and gives a patient medication. A health care worker cleans, disinfects, or sterilizes shared care equipment. A health care worker wears a gown and gloves in the room of the patient with tuberculosis (TB). A health care worker wears gloves to minimize contamination and discards them after each patient.

A health care worker uses standard precautions with all patients. Using standard precautions with all patients may reduce the transmission of infectious agents. Standard precautions are a set of infection control practices used to prevent transmission of diseases that can be acquired by contact with blood, body fluids, non-intact skin (including rashes), and mucous membranes. A health care worker cleans, disinfects, or sterilizes shared care equipment. Cleaning, disinfecting, and sterilizing shared care equipment may reduce the transmission of infectious agents. A health care worker wears gloves to minimize contamination and discards them after each patient. Wearing gloves to minimize contamination and discarding them after each patient may reduce the transmission of infectious agents. Gloves are to be changed and proper hand hygiene is performed between patients.

The nurse assesses vital signs for the 1-year-old patient with a severe head trauma. Which findings are most concerning? Select all that apply. An oxygen saturation of 90% A temperature of 99.9 Fahrenheit A heart rate of 52 beats per minute Cheyne-Stokes respiratory pattern Blood pressure of 210/132 mm Hg

A heart rate of 52 beats per minute Bradycardia is a manifestation of Cushing's response, which can indicate an impending brain herniation. Correct Cheyne-Stokes respiratory pattern A change in the respiratory rate and pattern is a manifestation of Cushing's response, which can indicate an impending brain herniation. Correct Blood pressure of 210/132 mm Hg Increased systolic blood pressure and a widening pulse pressure is a manifestation of Cushing's response, which can indicate an impending brain herniation.

Rubella (German Measles)

A highly contagious viral disease, especially affecting children, that causes swelling of the lymph glands and a reddish pink rash; may be harmful to the unborn baby of a pregnant woman who contracts it

One nursing goal is to assess which families are at lesser or greater risk for succumbing to the effects of the crisis.

A number of variables, including available support systems, reactions to the child, perception of the illness or disability, coping mechanisms, available financial resources, and concurrent stresses within the family, may affect the resolution of a crisis. Although most families do cope, the needs of families at risk are considerable. If they receive guidance and emotional support early in the crisis, there is an increased possibility that they will cope successfully.

The nurse is caring for several hospitalized children. Which patient is most likely to be experiencing Munchausen syndrome by proxy? A school-age child with vaginal discharge and a diagnosis of chlamydia. A preschool-age child with a low albumin level who is underweight for age. A school-age child admitted because of vomiting and dehydration for the sixth time this year. A preschool-age child with second-degree burns on both lower extremities and the buttocks.

A school-age child admitted because of vomiting and dehydration for the sixth time this year. Frequent hospitalizations for nonspecific symptoms are a sign of Munchausen syndrome by proxy.

Antitoxin

A solution of antibodies derived from the serum of animals immunized with specific antigens and used to confer passive immunity

Immunoglobulin

A sterile solution containing antibodies from large pools of human blood plasma

Hodgkin's disease

A type of lymphoma. Malignancy of the lymph nodes that originates in a single lymph node or a single chain of nodes

Erythema Infectiosum (Fifth Disease)

A viral disease characterized by a face that appears as "slapped cheeks," a fiery red rash on the cheeks.

The nurse is teaching parents of preschool-aged children strategies to prevent sexual abuse. What should the nurse include in the teaching session? (Select all that apply.) a. Back up a child's right to say no. b. Don't take what your child says too seriously. c. Take a second look at signals of potential danger. d. Don't be too detailed about examples of sexual assault. e. Remind children that even "nice" people sometimes do mean things.

A, C, E To provide protection and preparation from sexual abuse, parents should back up a child's right to say no, take a second look at signals of potential danger, and remind children that even "nice" people sometimes do mean things. Parents should take what children say seriously and they should give specific definitions and examples of sexual assault.

A parent tells the nurse, "My toddler tries to undo the car seat harness and climb out of the seat." What strategies should the nurse recommend to the parent to encourage the child to stay in the seat? (Select all that apply.) a. Allow your child to hold a favorite toy. b. Allow your child out of the seat occasionally. c. Avoid using rewards to encourage cooperative behavior. d. When child tries to unbuckle the seat harness, firmly say, "No." e. It may be necessary to stop the car to reinforce the expected behavior.

A, D, E Strategies to encourage a child to stay in a car seat include allowing the child to hold favorite toy, firmly saying "No" if the child begins to undo the harness, and stopping the car to reinforce the expected behavior. Rewards, such as stars or stickers, can be used to encourage cooperative behavior. The child should stay in the car seat at all times, even for short trips.

The health care provider has prescribed desmopressin acetate (DDAVP) 0.05mg/kg/day PO divided twice daily for a child with diabetes insipidus. The child weighs 110 lb. The nurse is preparing to administer the 0900 dose. Calculate the dose the nurse should administer in milligrams. Record your answer in a whole number.

ANS: 1 The correct calculation is: 88 lb/2.2 kg = 40 kg Dose of DDAVP is 0.05 mg/kg/day divided bid 0.05 mg ´ 40 = 2 mg/day 2 mg/2 = 1 mg for one dose

The health care provider prescribes ceftazidime (Fortaz) 75 mg per intravenous piggyback (IVPB) every 8 hours for a child with cancer admitted with fever and neutropenia. The pharmacy sends the medication to the unit in a 50-ml bag with directions to run the medication over 30 minutes. What milliliters per hour will the nurse set the intravenous pump to run the medication over 30 minutes? Fill in the blank and record your answer in a whole number.

ANS: 100 Perform the calculation. 50 ml ________ ´ 60 minutes = 100 ml/hr 30 minutes

The health care provider prescribes vancomycin 200 mg per intravenous piggyback (IVPB) every 6 hours for a child with cancer admitted to the hospital for fever and neutropenia. The pharmacy sends the medication to the unit in a 240-ml bag with directions to run the medication over 120 minutes. What milliliters per hour will the nurse set the intravenous pump to run the medication over 120 minutes? Fill in the blank and record your answer in a whole number.

ANS: 120 Perform the calculation. Convert the minutes to hours = 120/60 = 2 hours 240 ml ________ = 120 ml/hr 2 hours

The health care provider has prescribed levothyroxine (Synthroid) 4 mcg/kg/day PO for a child with hypothyroidism. The child weighs 77 lb. The nurse is preparing to administer the daily dose. Calculate the dose the nurse should administer in micrograms. Record your answer below in a whole number.

ANS: 140 The correct calculation is: 77 lb/2.2 kg = 35 kg Dose of Synthroid is 4 mcg/kg/day 4 mcg ´ 35 = 140 mcg for the daily dose

A health care provider prescribes Osmitrol (mannitol) 0.25 g/kg intravenously (IV) every 6 hours for a child after a brain tumor removal. The child weighs 20 kg. The medication label states: "Osmitrol (Mannitol) 250 mg/1 ml." The nurse prepares to administer the dose. How many milliliters will the nurse prepare to administer the dose? Fill in the blank. Record your answer in a whole number.

ANS: 20 Calculate the dose. 0.25 g ´ 20 = 5 g convert to mg = 5000 mg Follow the formula for dosage calculation. Desired ———- ´ Volume = ml per dose Available 5000 mg ———- ´ 1 ml = 20 ml 250 mg

Calculate the absolute neutrophil count for a child with a WBC = 6000/mm3, neutrophils = 18%, and nonsegmented neutrophils (bands) = 20%. Record your answer below in a whole number.

ANS: 2280 Perform the calculation Determine the total percentage of neutrophils ("polys," or "segs," and "bands"). Multiply white blood cell (WBC) count by percentage of neutrophils. WBC = 6000/mm3, neutrophils = 18%, nonsegmented neutrophils (bands) = 20% Step 1: 18% + 20% = 38% Step 2: 0.38 ´ 6000 = 2280/mm3 ANC

A health care provider prescribes hydrocortisone (Solu-Cortef) 200 mg IV STAT for a child with acute adrenocortical insufficiency. The medication label states Solu-Cortef 100 mg/2 ml. The nurse prepares to administer one dose. How many milliliters will the nurse prepare to administer one dose? Fill in the blank. Record your answer below in a whole number.

ANS: 4 Follow the formula for dosage calculation. Desired ———- ´ Volume = ml per dose Available 200 mg ———- ´ 2 ml = 4 ml 100 mg

The health care provider has prescribed leuprolide acetate (Lupron Depot) 0.2 mg/kg IM every 4 weeks for a child with precocious puberty. The child weighs 55 lb. The nurse is preparing to administer the dose. Calculate the dose the nurse should administer in milligrams. Record your answer below in a whole number.

ANS: 5 The correct calculation is: 55 lb/2.2 kg = 25 kg Dose of Lupron Depot is 0.2 mg/kg 0.2 mg ´ 25 = 5 mg

Calculate the absolute neutrophil count for a child with a WBC = 3000/mm3, neutrophils = 10%, and nonsegmented neutrophils (bands) = 10%. Record your answer below in a whole number.

ANS: 600 Perform the calculation Determine the total percentage of neutrophils ("polys," or "segs," and "bands"). Multiply white blood cell (WBC) count by percentage of neutrophils. WBC = 3000/mm3, neutrophils = 10%, and nonsegmented neutrophils (bands) = 10% Step 1: 10% + 10% = 20% Step 2: 0.2 ´ 3000 = 600/mm3 ANC

Calculate the absolute neutrophil count for a child with a WBC = 15,000/mm3, neutrophils = 29%, and nonsegmented neutrophils (bands) = 29%. Record your answer in a whole number.

ANS: 8700 Perform the calculation Determine the total percentage of neutrophils ("polys," or "segs," and "bands"). Multiply white blood cell (WBC) count by percentage of neutrophils. WBC = 3000/mm3, neutrophils = 10%, and nonsegmented neutrophils (bands) = 10% Step 1: 29% + 29% = 58% Step 2: 0.58 ´ 15,000 = 8700/mm3 ANC

The school nurse needs to obtain authorization for a child who requires medications while at school. From whom does the nurse obtain the authorization? a. The parents b. The pharmacist c. The school administrator d. The prescribing practitioner

ANS: A A child who requires medication during the school day requires written authorization from the parent or guardian. Most schools also require that the medication be in the original container appropriately labeled by the pharmacist or physician. Some schools allow children to receive over-the-counter medications with parental permission. The pharmacist may be asked to appropriately label the medication for use at the school, but authorization is not required. The school administration should have a policy in place that facilitates the administration of medications for children who need them. The prescribing practitioner is responsible for ensuring that the medication is appropriate for the child. Because the child is a minor, parental consent is required.

A goiter is an enlargement or hypertrophy of which gland? a. Thyroid b. Adrenal c. Anterior pituitary d. Posterior pituitary

ANS: A A goiter is an enlargement or hypertrophy of the thyroid gland. Goiter is not associated with the adrenal, anterior pituitary, or posterior pituitary secretory organs.

In teaching parents about appropriate pacifier selection, the nurse should recommend which characteristic? a. Easily grasped handle b. Detachable shield for cleaning c. Soft, pliable material d. Ribbon or string to secure to clothing

ANS: A A good pacifier should be easily grasped by the infant. One-piece construction is necessary to avoid having the nipple and guard separate, posing a risk for aspiration. The material should be sturdy and flexible. If the pacifier is too pliable, it may be aspirated. No ribbon or string should be attached. This poses additional risks.

The nurse is planning care for a 3-year-old boy who has Down syndrome and is on continuous oxygen. He recently began walking around furniture. He is spoon fed by his parents and eats some finger foods. What goal is the most appropriate to promote normal development? a. Encourage mobility. b. Encourage assistance in self-care. c. Promote oral-motor development. d. Provide opportunities for socialization.

ANS: A A major principle for developmental support in children with complex medical issues is that it should be flexible and tailored to the individual child's abilities, interests, and needs. This child is exhibiting readiness for ambulation. It is an appropriate time to provide activities that encourage mobility, for example, longer oxygen tubing. Parents should provide decreasing amounts of assistance with self-care as he is able to develop these skills. The boy is receiving oral foods and is eating finger foods. He has acquired this skill. Mobility is a new developmental task. Opportunities for socialization should be ongoing.

A young child with leukemia has anorexia and severe stomatitis. What approach should the nurse suggest that the parents try? a. Relax any eating pressures. b. Firmly insist that the child eat normally. c. Serve foods that are either hot or cold. d. Provide only liquids because chewing is painful.

ANS: A A multifaceted approach is necessary for children with severe stomatitis and anorexia. First, the parents should relax eating pressures. The nurse should suggest that the parents try soft, bland foods; normal saline or bicarbonate mouthwashes; and local anesthetics. Insisting that the child eat normally is not suggested. For some children, not eating may be a way to maintain some control. This can set the child and caregiver in opposition to each other. Hot and cold foods can be painful on ulcerated mucosal membranes. Substitution of high-calorie foods that the child likes and can eat should be used.

After chemotherapy is begun for a child with acute leukemia, prophylaxis to prevent acute tumor lysis syndrome includes which therapeutic intervention? a. Hydration b. Oxygenation c. Corticosteroids d. Pain management

ANS: A Acute tumor lysis syndrome results from the release of intracellular metabolites during the initial treatment of leukemia. Hyperuricemia, hypocalcemia, hyperphosphatemia, and hyperkalemia can result. Hydration is used to reduce the metabolic consequences of the tumor lysis. Oxygenation is not helpful in preventing acute tumor lysis syndrome. Allopurinol, not corticosteroids, is indicated for pharmacologic management. Pain management may be indicated for supportive therapy of the child, but it does not prevent acute tumor lysis syndrome.

What is appropriate mouth care for a toddler with mucosal ulceration related to chemotherapy? a. Mouthwashes with plain saline b. Lemon glycerin swabs for cleansing c. Mouthwashes with hydrogen peroxide d. Swish and swallow with viscous lidocaine

ANS: A Administering mouth care is particularly difficult in infants and toddlers. A satisfactory method of cleaning the gums is to wrap a piece of gauze around a finger; soak it in saline or plain water; and swab the gums, palate, and inner cheek surfaces with the finger. Mouth rinses are best accomplished with plain water or saline because the child cannot gargle or spit out excess fluid. Avoid agents such as lemon glycerin swabs and hydrogen peroxide because of the drying effects on the mucosa. Lidocaine should be avoided in young children.

The nurse is teaching parents of preschoolers about plants that are poisonous. What plant should the nurse include in the teaching session? a. Azalea b. Begonia c. Boston fern d. Asparagus fern

ANS: A All parts of the azalea are poisonous. Begonias, Boston ferns, and asparagus ferns are nonpoisonous plants.

The nurse is preparing an airborne infection isolation room for a patient. Which communicable disease does the patient likely have? a. Varicella b. Pertussis c. Influenza d. Scarlet fever

ANS: A An airborne infection isolation room is the isolation for persons with a suspected or confirmed airborne infectious disease transmitted by the airborne route such as measles, varicella, or tuberculosis. Pertussis, influenza, and scarlet fever require droplet transmission precautions.

A toddler, age 16 months, falls down a few stairs. He gets up and "scolds" the stairs as if they caused him to fall. What is this an example of? a. Animism b. Ritualism c. Irreversibility d. Delayed cognitive development

ANS: A Animism is the attribution of lifelike qualities to inanimate objects. By scolding the stairs, the toddler is attributing human characteristics to them. Ritualism is the need to maintain sameness and reliability. It provides a sense of comfort to toddlers. Irreversibility is the inability to reverse or undo actions initiated physically. The toddler is acting in an age-appropriate manner.

The nurse is caring for an adolescent with anorexia nervosa. What pituitary dysfunction should the nurse assess for in the adolescent? a. Hypopituitarism b. Pituitary hyperfunction c. Hyperplasia of the pituitary cells d. Overproduction of the anterior pituitary hormones

ANS: A Anorexia nervosa can cause hypopituitarism. It does not cause the hyperfunction of the pituitary, hyperplasia of the pituitary cells, or overproduction of the anterior pituitary hormones.

The nurse is talking to the parent of a child with special needs. The parent has expressed worry about how to support the siblings at home. What suggestion is appropriate for the nurse to give to the parent? a. "You should help the siblings see the similarities and differences between themselves and your child with special needs." b. "You should explain that your child with special needs should be included in all activities that the siblings participate in even if they are reluctant." c. "You should give the siblings many caregiving tasks for your child with special needs so the siblings feel involved." d. "You should intervene when there are differences between your child with special needs and the siblings."

ANS: A Appropriate information to give to a parent who wants to support the siblings of a child with special needs includes helping the siblings see the differences and similarities between themselves and the child with special needs to promote an understanding environment. The parent should be encouraged to allow the siblings to participate in activities that do not always include the child with special needs, to limit caregiving responsibilities, and to allow the children to settle their own differences rather than step in all the time.

An adolescent will receive a bone marrow transplant (BMT). The nurse should explain that the bone marrow will be administered by which method? a. Bone grafting b. Intravenous infusion c. Bone marrow injection d. Intraabdominal infusion

ANS: B Bone marrow from a donor is infused intravenously, and the transfused stem cells migrate to the recipient's marrow and repopulate it.

The thyroid-stimulating hormone (TSH) increases secretion in response to which hormone? a. Low levels of circulating thyroid hormone b. High levels of circulating thyroid hormone c. Low levels of circulating adrenocorticotropic hormone d. High levels of circulating adrenocorticotropic hormone

ANS: A As blood concentrations of the target hormones reach normal levels, a negative message is sent to the anterior pituitary to inhibit release of the tropic hormone. For example, TSH responds to low levels of circulating TH. As blood levels of TH reach normal concentrations, a negative feedback message is sent to the anterior pituitary, resulting in diminished release of TSH. Adrenocorticotropic stimulates the adrenals to secrete glucocorticoids.

An 8-year-old girl tells the nurse that she has cancer because God is punishing her for "being bad." What should the nurse interpret this as? a. A common belief at this age b. Indicative of excessive family pressure c. Faith that forms the basis for most religions d. Suggestive of a failure to develop a conscience

ANS: A Children at this age may view illness or injury as a punishment for a real or imagined misbehavior. School-age children expect to be punished and tend to choose a punishment that they think "fits the crime." This is a common belief and not related to excessive family pressure. Many faiths do not include a God that causes cancer in response for "bad" behavior. This statement reflects the child's belief in what is right and wrong.

The nurse is teaching a preschool child with a cognitive impairment how to throw a ball overhand. What teaching strategy should the nurse use for this child? a. Demonstrate how to throw a ball overhand. b. Explain the reason for throwing a ball overhand. c. Show pictures of children throwing balls overhand. d. Explain to the child how to throw the ball overhand.

ANS: A Children with cognitive impairment have a deficit in discrimination, which means that concrete ideas are much easier to learn effectively than abstract ideas. Therefore, demonstration is preferable to verbal explanation, and the nurse should direct learning toward mastering a skill rather than understanding the scientific principles underlying a procedure. Demonstrating how to throw the ball is the best teaching strategy.

The nurse is planning care for a child recently diagnosed with diabetes insipidus (DI). What intervention should be included? a. Encourage the child to wear medical identification. b. Discuss with the child and family ways to limit fluid intake. c. Teach the child and family how to do required urine testing. d. Reassure the child and family that this is usually not a chronic or life-threatening illness.

ANS: A DI is a potentially life-threatening disorder if the voluntary demand for fluid is suppressed or the child does not have access to fluids. Medical alert identification should be worn. Fluid intake is not restricted in children with DI. The child is unable to concentrate urine and can rapidly become dehydrated. Fluid intake may be limited during diagnosis, when the lack of intake will result in decreased urinary output and dehydration. Urine testing is not required in DI. Changes in body weight provide information about approximate fluid balance. This is a lifelong disorder that requires supplemental vasopressin throughout life.

A 12-year-old girl is newly diagnosed with diabetes when she develops ketoacidosis. How should the nurse structure a successful education program? a. Essential information is presented initially. b. Teaching should take place in the child's semiprivate room. c. Education is focused toward the parents because the child is too young. d. All information needed for self-management of diabetes is taught at once.

ANS: A Diagnosis of type 1 diabetes can be traumatic for the child and family. Most families are not psychologically ready for the complex teaching that is needed for self-management. Most structured diabetes education programs begin with essential or survival information followed by the complex background material when the family is better able to learn. Teaching can take place either as an outpatient or as an inpatient. The actual teaching area should be free from distractions that would interfere with learning. A semiprivate room would have many individuals entering and leaving the room, causing distraction. A 12-year-old child who is cognitively age appropriate needs to be included in the educational process. Most children older than the age of 8 years can be involved in blood glucose monitoring and insulin administration. Teaching all information needed for self-management of diabetes at once would be too overwhelming for a family in crisis.

The parents of a child born with disabilities ask the nurse for advice about discipline. The nurse's response should be based on remembering that discipline is which? a. Essential for the child b. Not needed unless the child's behavior becomes problematic c. Best achieved with punishment for misbehavior d. Too difficult to implement with a special needs child

ANS: A Discipline is essential for the child. It provides boundaries on which she can test out her behavior and teaches her socially acceptable behaviors. The nurse should teach the parents ways to manage the child's behavior before it becomes problematic. Punishment is not effective in managing behavior.

The nurse is caring for a child with suspected ingestion of some type of poison. What action should the nurse take next after initiating cardiopulmonary resuscitation (CPR)? a. Empty the mouth of pills, plants, or other material. b. Question the victim and witness. c. Place the child in a side-lying position. d. Call poison control.

ANS: A Emptying the mouth of any leftover pills, plants, or other ingested material is the next step after assessment and initiation of CPR if needed. Questioning the victim and witnesses, calling poison control, and placing the child in a side-lying position are follow-up steps.

Which vitamin supplementation has been found to reduce both morbidity and mortality in measles? a. A b. B1 c. C d. Zinc

ANS: A Evidence suggests that vitamin A supplementation reduces both morbidity and mortality in measles.

During the summer many children are more physically active. What changes in the management of the child with diabetes should be expected as a result of more exercise? a. food intake b. ¯food intake c. risk of hyperglycemia d. ¯risk of insulin reaction

ANS: A Exercise is encouraged and never restricted unless indicated by other health conditions. Exercise lowers blood glucose levels, depending on the intensity and duration of the activity. Consequently, exercise should be included as part of diabetes management, and the type and amount of exercise should be planned around the child's interests and capabilities. However, in most instances, children's activities are unplanned, and the resulting decrease in blood glucose can be compensated for by providing extra snacks before (and, if the exercise is prolonged, during) the activity. In addition to a feeling of well-being, regular exercise aids in utilization of food and often results in a reduction of insulin requirements.

The nurse is teaching parents of a preschool child strategies to implement when the child delays going to bed. What strategy should the nurse recommend? a. Use consistent bedtime rituals. b. Give in to attention-seeking behavior. c. Take the child into the parent's bed for an hour. d. Allow the child to stay up past the decided bedtime.

ANS: A For children who delay going to bed, a recommended approach involves a consistent bedtime ritual and emphasizing the normalcy of this type of behavior in young children. Parents should ignore attention-seeking behavior, and the child should not be taken into the parents' bed or allowed to stay up past a reasonable hour.

What are the most common clinical manifestations of brain tumors in children? a. Headaches and vomiting b. Blurred vision and ataxia c. Hydrocephalus and clumsy gait d. Fever and poor fine motor control

ANS: A Headaches, especially on awakening, and vomiting that is not related to feeding are the most common clinical manifestations of brain tumors in children. Diplopia (double vision), not blurred vision, can be a presenting sign of brainstem glioma. Ataxia is a clinical manifestation of brain tumors, but headaches and vomiting are the most common. Hydrocephalus can be a presenting sign in infants when the sutures have not closed. Children at this age are usually not walking steadily. Poor fine motor coordination may be a presenting sign of astrocytoma, but headaches and vomiting are the most common presenting signs of brain tumors.

The parents of a child on a ventilator tell the nurse that their insurance company wants the child to be discharged. They explain that they do not want the child home "under any circumstances." What principle should the nurse consider when working with this family? a. Desire to have the child home is essential to effective home care. b. Parents should not be expected to care for a technology-dependent child. c. Having a technology-dependent child at home is better for both the child and the family. d. Parents are not part of the decision-making process because of the costs of hospitalization.

ANS: A Home care requires the family to manage the child's illness, including providing daily hands-on care, monitoring the child's medical condition, and educating others to care for the child. The child's home environment with the child's family is perceived as the best place for the child to be cared for. If the family does not want to or is not able to assume these responsibilities, other arrangements need to be investigated. The family is an essential part of the decision-making process. Without family involvement and support, the technology-dependent child will not be well cared for at home.

An adolescent is scheduled for a leg amputation in 2 days for treatment of osteosarcoma. What approach should the nurse implement? a. Answer questions with straightforward honesty. b. Avoid discussing the seriousness of the condition. c. Explain that although the amputation is difficult, it will cure the cancer. d. Help the adolescent accept the amputation as better than a long course of chemotherapy.

ANS: A Honesty is essential to gain the child's cooperation and trust. The diagnosis of cancer should not be disguised with falsehoods. The adolescent should be prepared for the surgery so there is time for reflection about the diagnosis and subsequent treatment. This allows questions to be answered. To accept the need for radical surgery, the child must be aware of the lack of alternatives for treatment. Amputation is necessary, but it will not guarantee a cure. Chemotherapy is an integral part of the therapy with surgery. The child should be informed of the need for chemotherapy and its side effects before surgery.

What causes warts? a. A virus b. A fungus c. A parasite d. Bacteria

ANS: A Human warts are caused by the human papillomavirus. Infection with fungus, parasites, or bacteria does not result in warts.

What observation in a child should indicate the need for a referral to a specialist regarding a communication impairment? a. At 2 years of age, the child fails to respond consistently to sounds. b. At 3 years of age, the child fails to use sentences of more than five words. c. At 4 years of age, the child has impaired sentence structure. d. At 5 years of age, the child has poor voice quality.

ANS: A If a 2-year-old child fails to respond consistently to sounds, it is an indication for referral to a specialist regarding communication impairment. At age 3 years, the child failing to use sentences of three words would be an indication for referral; impaired sentence structure would be seen in a 5-year-old child and poor voice quality in an older child who has a communication impairment.

A child with growth hormone (GH) deficiency is receiving GH therapy. When is the best time for the GH to be administered? a. At bedtime b. After meals c. Before meals d. After arising in morning

ANS: A Injections are best given at bedtime to more closely approximate the physiologic release of GH. After meals, before meals, and after arising in the morning do not parallel the physiologic release of the hormone.

What is the leading cause of death during the toddler period? a. Injuries b. Infectious diseases c. Childhood diseases d. Congenital disorders

ANS: A Injuries are the most common cause of death in children ages 1 through 4 years. It is the highest rate of death from injuries of any childhood age group except adolescence. Congenital disorders are the second leading cause of death in this age group. Infectious and childhood diseases are less common causes of death in this age group.

A school-age child with diabetes gets 30 units of NPH insulin at 0800. According to when this insulin peaks, the child should be at greatest risk for a hypoglycemic episode between when? a. Lunch and dinner b. Breakfast and lunch c. 0830 to his midmorning snack d. Bedtime and breakfast the next morning

ANS: A Intermediate-acting (NPH and Lente) insulins reach the blood 2 to 6 hours after injection. The insulins peak 4 to 14 hours later and stay in the blood for about 14 to 20 hours.

Which is usually the only symptom of pediculosis capitis (head lice)? a. Itching b. Vesicles c. Scalp rash d. Localized inflammatory response

ANS: A Itching is generally the only manifestation of pediculosis capitis (head lice). Diagnosis is made by observation of the white eggs (nits) on the hair shaft. Vesicles, scalp rash, and localized inflammatory response are not symptoms of head lice.

What is most important in the management of cellulitis? a. Burow solution compresses b. Oral or parenteral antibiotics c. Topical application of an antibiotic d. Incision and drainage of severe lesions

ANS: B Oral or parenteral antibiotics are indicated depending on the extent of the cellulitis. Warm water compresses may be indicated for limited cellulitis. The antibiotic needs to be administered systemically. Incision and drainage of severe lesions presents a risk of spreading infection or making the lesion worse.

A child is admitted with a suspected diagnosis of Munchausen syndrome by proxy (MSBP). What is an important consideration in the care of this child? a. Monitoring the parents whenever they are with the child b. Reassuring the parents that the cause of the disorder will be found c. Teaching the parents how to obtain necessary specimens d. Supporting the parents as they cope with diagnosis of a chronic illness

ANS: A MSBP refers to an illness that one person fabricates or induces in another. The child must be continuously observed for development of symptoms to determine the cause. MSBP is caused by an individual harming the child for the purpose of gaining attention. Nursing staff should obtain all specimens for analyzing. This minimizes the possibility of the abuser contaminating the sample. The child must be supported through the diagnosis of MSBP. The abuser must be identified and the child protected from that individual.

For case management to be most effective, who should be recognized as the most appropriate case manager? a. Nurse b. Panel of experts c. Multidisciplinary team d. Insurance company

ANS: A Nursing case managers are ideally suited to provide the care coordination necessary. Care coordination is most effective if a single person works with the family to accomplish the many tasks and responsibilities that are necessary. The family retains the role as primary decision maker. Most likely the insurance company will have a case manager focusing on the financial aspects of care. This does not include coordination of care to assist the family.

What manifestation observed by the nurse is suggestive of parental overprotection? a. Gives inconsistent discipline b. Facilitates the child's responsibility for self-care of illness c. Persuades the child to take on activities of daily living even when not able d. Encourages social and educational activities not appropriate to the child's level of capability

ANS: A Parental overprotection is manifested when the parents fear letting the child achieve any new skill, avoid all discipline, and cater to every desire to prevent frustration. Overprotective parents do not allow the child to assume responsibility for self-care of the illness. The parents prefer to remain in the role of total caregiver. The parents do not encourage the child to participate in social and educational activities.

The nurse is assisting with a growth hormone stimulation test for a child with short stature. What should the nurse monitor closely on this child during the test? a. Hypotension b. Tachycardia c. Hypoglycemia d. Nausea and vomiting

ANS: A Patients receiving clonidine (Catapres) for a growth hormone stimulation test require close blood pressure monitoring for hypotension. Tachycardia, hypoglycemia, and nausea and vomiting do not occur with Catapres administered for a growth hormone stimulation test.

The community nurse is planning prevention measures designed to avoid conditions that can cause cognitive impairment. Taking folic acid supplements during pregnancy to prevent neural tube defects is which type of prevention strategy? a. Primary b. Secondary c. Tertiary d. Rehabilitative

ANS: A Primary prevention strategies are those designed to avoid conditions that cause cognitive impairment. Use of folic acid supplements during pregnancy to prevent neural tube defects is a primary prevention strategy. Secondary prevention activities are those designed to identify the condition early and initiate treatment to avert cerebral damage. Tertiary prevention strategies are those concerned with treatment to minimize long-term consequences. Rehabilitation services is an example of tertiary prevention.

One of the supervisors for a home health agency asks the nurse to give a family of a child with a chronic illness a survey evaluating the nurses and other service providers. How should the nurse recognize this request? a. Appropriate to improve quality of care b. Improper because it is an invasion of privacy c. Inappropriate unless nurses and other providers agree to participate d. Not acceptable because the family lacks remembering necessary to evaluate professionals

ANS: A Quality assessment and improvement activities are essential for virtually all organizations. Family involvement in evaluating a home care plan can occur on several levels. The nurse can ask the family open-ended questions at regular intervals to assess their opinion of the effectiveness of care. Families should also be given an opportunity to evaluate the individual home care nurses, the home care agency, and other service providers periodically. Evaluation of the provision of care to the patient and family requires evaluation of the care provider, that is, the nurse. Quality-monitoring activities are required by virtually all health care agencies. During the evaluation process, the family is asked to provide their perceptions of care.

A child's parents ask the nurse many questions about their child's illness and its management. The nurse does not know enough to answer all the questions. What nursing action is most appropriate at this time? a. Tell them, "I don't know, but I will find out." b. Suggest that they ask the physician these questions. c. Explain that the nurse cannot be expected to know everything. d. Answer questions vaguely so they do not lose confidence in the nurse.

ANS: A Questions from parents should be answered in a straightforward manner. Stating "I don't know" or "I'll find out" is better than pretending to know or giving excuses. Suggesting that they ask the physician these questions is not supportive of the family. The nurse's role is to assist the parents in obtaining accurate information about their child's illness and its management. Although the nurse cannot be expected to know everything, it is an unprofessional attitude to state this. Nurses must provide accurate information to the extent possible. Vague answers are not helpful to the family.

One pediatric oncologic emergency is acute tumor lysis syndrome. Symptoms that this may be occurring include what? a. Muscle cramps and tetany b. Respiratory distress and cyanosis c. Thrombocytopenia and sepsis d. Upper extremity edema and neck vein distension

ANS: A Risk factors for development of tumor lysis syndrome include a high white blood cell count at diagnosis, large tumor burden, sensitivity to chemotherapy, and high proliferative rate. In addition to the described metabolic abnormalities, children may develop a spectrum of clinical symptoms, including flank pain, lethargy, nausea and vomiting, muscle cramps, pruritus, tetany, and seizures. Respiratory distress and cyanosis occur with hyperleukocytosis. Thrombocytopenia and sepsis occur with disseminated intravascular coagulation. Upper extremity edema and neck vein distention occur with superior vena cava syndrome.

A child is having tests done to determine parathyroid function. The clinic nurse knows that the parathyroid hormone (PTH) regulates the homeostasis of what in the serum? a. Sodium b. Calcium c. Potassium d. Magnesium

ANS: B The parathyroid glands secrete PTH. Along with vitamin D and calcitonin, PTH regulates the homeostasis of serum calcium concentrations.

What is an important consideration in preventing injuries during middle childhood? a. Achieving social acceptance is a primary objective. b. The incidence of injuries in girls is significantly higher than it is in boys. c. Injuries from burns are the highest at this age because of fascination with fire. d. Lack of muscular coordination and control results in an increased incidence of injuries.

ANS: A School-age children often participate in dangerous activities in an attempt to prove themselves worthy of acceptance. The incidence of injury during middle childhood is significantly higher in boys compared with girls. Motor vehicle collisions are the most common cause of severe injuries in children. Children have increasing muscular coordination. Children who are risk takers may have inadequate self-regulatory behavior.

A parent of a hospitalized child on chemotherapy asks the nurse if a sibling of the hospitalized child should receive the varicella vaccination. The nurse should give which response? a. The sibling can get a varicella vaccination. b. The sibling should not get a varicella vaccination. c. The sibling should wait until the child is finished with chemotherapy. d. The sibling should get varicella-zoster immune globulin if exposed to chickenpox.

ANS: A Siblings and other family members can receive the live measles, mumps, and rubella vaccine and the varicella vaccine without risk to the child who is immunosuppressed.

A 9-year-old girl often comes to the school nurse complaining of stomach pains. Her teacher says she is completing her school work satisfactorily but lately has been somewhat aggressive and stubborn in the classroom. The school nurse should recognize this as which? a. Signs of stress b. Developmental delay c. Lack of adjustment to school environment d. Physical problem that needs medical intervention

ANS: A Signs of stress include stomach pains or headache, sleep problems, bedwetting, changes in eating habits, aggressive or stubborn behavior, reluctance to participate, or regression to earlier behaviors. The child is completing school work satisfactorily; any developmental delay would have been diagnosed earlier. The teacher reports that this is a departure from the child's normal behavior. Adjustment issues would most likely be evident soon after a change. Medical intervention is not immediately required. Recognizing that this constellation of symptoms can indicate stress, the nurse should help the child identify sources of stress and how to use stress reduction techniques. The parents are involved in the evaluation process.

As part of the diagnostic evaluation of a child with cancer, biopsies are important for staging. What statement explains what staging means? a. Extent of the disease at the time of diagnosis b. Rate normal cells are being replaced by cancer cells c. Biologic characteristics of the tumor or lymph nodes d. Abnormal, unrestricted growth of cancer cells producing organ damage

ANS: A Staging is a description of the extent of the disease at the time of diagnosis. Staging criteria exist for most tumors. The stage usually relates directly to the prognosis; the higher the stage, the poorer the prognosis. The rate that normal cells are being replaced by cancer cells is not a definition of staging. Classification of the tumor refers to the biologic characteristics of the tumor or lymph nodes. Abnormal, unrestricted growth of cancer cells producing organ damage describes how cancer cells grow and can cause damage to an organ.

The mother of a young child with cognitive impairment asks for suggestions about how to teach her child to use a spoon for eating. The nurse should make which recommendation? a. Do a task analysis first. b. Do not expect this task to be learned. c. Continue to spoon feed the child until the child tries to do it alone. d. Offer only finger foods so spoon feeding is unnecessary.

ANS: A Successful teaching begins with a task analysis. The endpoint (self-feeding, toilet training, and so on) is broken down into the component steps. The child is then guided to master the individual steps in sequence. Depending on the child's functional level, using a spoon for eating should be an achievable goal. The child requires demonstration and then guided training for each component of the self-feeding. Feeding finger foods so spoon feeding is unnecessary eliminates some of the intermediate steps that are necessary to using a fork and spoon. For socialization purposes, it is desirable that a child use feeding implements.

The nurse should suspect a hearing impairment in an infant who fails to demonstrate which behavior? a. Babbling by age 12 months b. Eye contact when being spoken to c. Startle or blink reflex to sound d. Gesturing to indicate wants after age 15 months

ANS: A The absence of babbling or inflections in voice by at least age 7 months is an indication of hearing difficulties. Lack of eye contact is not indicative of a hearing loss. An infant with a hearing impairment might react to a loud noise but not respond to the spoken word. The child with hearing impairment uses gestures rather than vocalizations to express desires at this age.

The nurse should expect to care for which age of child if the admitting diagnosis is retinoblastoma? a. Infant or toddler b. Preschool- or school-age child c. School-age or adolescent child d. Adolescent

ANS: A The average age of the child at the time of diagnosis is 2 years, and bilateral and hereditary disease is diagnosed earlier than unilateral and nonhereditary disease.

In teaching parents how to minimize or prevent bleeding episodes when the child is myelosuppressed, the nurse includes what information? a. Meticulous mouth care is essential to avoid mucositis. b. Rectal temperatures are necessary to monitor for infection. c. Intramuscular injections are preferred to intravenous ones. d. Platelet transfusions are given to maintain a count greater than 50,000/mm3.

ANS: A The decrease in blood platelets secondary to the myelosuppression of chemotherapy can cause an increase in bleeding. The child and family are taught how to perform good oral hygiene to minimize gingival bleeding and mucositis. Rectal temperatures are avoided to minimize the risk of ulceration. Hygiene is also emphasized. Intramuscular injections are avoided because of the risk of bleeding into the muscle and of infection. Platelet transfusions are usually not given unless there is active bleeding or the platelet count is less than 10,000/mm3. The use of platelets when not necessary can contribute to antibody formation and increased destruction of platelets when transfused.

The nurse is teaching parents of preschool children consequences of inadequate sleep. What should the nurse include in the teaching session? (Select all that apply.) a. Behavior changes b. Increased appetite c. Difficulty concentrating d. Poor control of emotions e. Impaired learning ability

ANS: A, C, D, E Consequences of inadequate sleep include daytime tiredness, behavior changes, hyperactivity, difficulty concentrating, impaired learning ability, poor control of emotions and impulses, and strain on family relationships. Increased appetite is not a consequence of inadequate sleep.

A child has been diagnosed with cat scratch disease. The nurse explains which characteristics about this disease? a. "The disease is usually a benign, self-limiting illness." b. "The animal that transmitted the disease will also be ill." c. "The disease is treated with a 5-day course of oral azithromycin." d. "Symptoms include pruritus, especially at the site of inoculation."

ANS: A The disease is usually a benign, self-limiting illness that resolves spontaneously in 4 to 6 weeks. The animals are not ill during the time they transmit the disease. Treatment is primarily supportive. Antibiotics do not shorten the duration or prevent progression to suppuration. The usual manifestation is a painless, nonpruritic erythematous papule at the site of inoculation.

The parent of 16-month-old child asks, "What is the best way to keep my child from getting into our medicines at home?" What should the nurse advise? a. "All medicines should be locked securely away." b. "The medicines should be placed in high cabinets." c. "Your child just needs to be taught not to touch medicines." d. "Medicines should not be kept in the homes of small children."

ANS: A The major reason for poisoning in the home is improper storage. Toddlers can climb, unlatch cabinets, and obtain access to high-security places. For medications, only a locked cabinet is safe. Toddlers can climb using furniture. High places are not a deterrent to an exploring toddler. Toddlers are not able to generalize that all the different forms of medications in the home may be dangerous. Keeping medicines out of the homes of small children is not feasible because many parents require medications for chronic or acute illnesses. Parents must be taught safe storage for their home and when they visit other homes.

An 18-month-old child has been diagnosed with pediculosis capitis (head lice). Which prescription should the nurse question if ordered for the child? a. Malathion (Ovide) b. Permethrin 1% (Nix) c. Benzyl alcohol 5% lotion d. Pyrethrin with piperonyl butoxide (RID)

ANS: A The nurse should question malathion for an 18-month-old child. Malathion contains flammable alcohol, must remain in contact with the scalp for 8 to 12 hours, and is not recommended for children younger than 2 years of age. The drug of choice for infants and children is permethrin 1% cream rinse (Nix) or pyrethrin with piperonyl butoxide, which kill adult lice and nits. Benzyl alcohol 5% lotion has been approved by the Food and Drug Administration for the treatment of head lice in children as young as 6 months.

What nursing intervention is most appropriate in promoting normalization in a school-age child with a chronic illness? a. Give the child as much control as possible. b. Ask the child's peer to make the child feel normal. c. Convince the child that nothing is wrong with him or her. d. Explain to parents that family rules for the child do not need to be the same as for healthy siblings.

ANS: A The school-age child who is ill may be forced into a period of dependency. To foster normalcy, the child should be given as much control as possible. It is unrealistic for one individual to make the child feel normal. The child has a chronic illness, so it would be unacceptable to convince the child that nothing is wrong. The family rules should be similar for each of the children in a family. Resentment and hostility can arise if different standards are applied to each child.

The nurse is teaching parents about avoiding accidental burns with their toddler. What water heater setting should the nurse recommend to the parents? a. 120° F b. 130° F c. 140° F d. 150° F

ANS: A The water heater should be set to limit household water temperatures to less than 49° C (120° F). At this temperature, it takes 10 minutes for exposure to the water to cause a full-thickness burn. Conversely, water temperatures of 54° C (130° F), the usual setting of most water heaters, expose household members to the risk of full-thickness burns within 30 seconds.

A 4-month-old infant comes to the clinic for a well-infant checkup. Immunizations she should receive are DTaP (diphtheria, tetanus, acellular pertussis) and IPV (inactivated poliovirus vaccine). She is recovering from a cold but is otherwise healthy and afebrile. Her older sister has cancer and is receiving chemotherapy. Nursing considerations should include which? a. DTaP and IPV can be safely given. b. DTaP and IPV are contraindicated because she has a cold. c. IPV is contraindicated because her sister is immunocompromised. d. DTaP and IPV are contraindicated because her sister is immunocompromised.

ANS: A These immunizations can be given safely. Serious illness is a contraindication. A mild illness with or without fever is not a contraindication. These are not live vaccines, so they do not pose a risk to her sister.

The parents of a 3-month-old infant report that their infant sleeps supine (face up) but is often prone (face down) while awake. The nurse's response should be based on remembering what? a. This is acceptable to encourage head control and turning over. b. This is acceptable to encourage fine motor development. c. This is unacceptable because of the risk of sudden infant death syndrome (SIDS). d. This is unacceptable because it does not encourage achievement of developmental milestones.

ANS: A These parents are implementing the guidelines to reduce the risk of SIDS. Infants should sleep on their backs to reduce the risk of SIDS and then be placed on their abdomens when awake to enhance achievement of milestones such as head control. These position changes encourage gross motor, not fine motor, development.

The potential effects of chronic illness or disability on a child's development vary at different ages. What developmental alteration is a threat to a toddler's normal development? a. Hindered mobility b. Limited opportunities for socialization c. Child's sense of guilt that he or she caused the illness or disability d. Limited opportunities for success in mastering toilet training

ANS: A Toddlers are acquiring a sense of autonomy, developing self-control, and forming symbolic representation through language acquisition. Mobility is the primary tool used by toddlers to experiment with maintaining control. Loss of mobility can create a sense of helplessness. Toddlers do not socialize. They are sensitive to changes in family routines. A sense of guilt is more likely to occur in a preschooler. Toilet training is not usually mastered until the end of the toddler period.

A child with cancer being treated with chemotherapy is receiving a platelet transfusion. The nurse understands that the transfused platelets should survive the body for how many days? a. 1 to 3 days b. 4 to 6 days c. 7 to 9 days d. 10 to 12 days

ANS: A Transfused platelets generally survive in the body for 1 to 3 days. The peak effect is reached in about 1 hour and decreased by half in 24 hours.

What form of diabetes is characterized by destruction of pancreatic beta cells, resulting in insulin deficiency? a. Type 1 diabetes b. Type 2 diabetes c. Gestational diabetes d. Maturity-onset diabetes of the young (MODY)

ANS: A Type 1 diabetes is characterized by the destruction of the pancreatic beta cells, which leads to absolute insulin deficiency. Type 2 diabetes results usually from insulin resistance. The pancreatic beta cells are not destroyed in gestational diabetes. MODY is an autosomal dominant monogenetic defect in beta cell function that is characterized by impaired insulin secretion with minimum or no defects in insulin action.

The nurse notes that the parents of a critically ill child spend a large amount of time talking with the parents of another child who is also seriously ill. They talk with these parents more than with the nurses. How should the nurse interpret this situation? a. Parent-to-parent support is valuable. b. Dependence on other parents in crisis is unhealthy. c. This is occurring because the nurses are unresponsive to the parents. d. This has the potential to increase friction between the parents and nursing staff.

ANS: A Veteran parents share experiences that cannot be supplied by other support systems. They have known the stress related to diagnosis, have weathered the many transition times, and have a practical remembering of resources. The parents can be mutually supportive during times of crisis. Nursing staff cannot provide the type of support that is realized from other parents who are experiencing similar situations. Friction should not exist between the nursing staff and the family of the child who is critically ill.

What statement is correct about young children who report sexual abuse? a. They may exhibit various behavioral manifestations. b. In more than half the cases, the child has fabricated the story. c. Their stories should not be believed unless other evidence is apparent. d. They should be able to retell the story the same way to another person.

ANS: A Victims of sexual abuse have no typical profile. The child may exhibit various behavioral manifestations, none of which is diagnostic for sexual abuse. When children report potentially sexually abusive experiences, their reports need to be taken seriously. Other children in the household also need to be evaluated. In children who are sexually abused, it is often difficult to identify other evidence. In one study, approximately 96% of children who were sexually abused had normal genital and anal findings. The ability to retell the story is partly dependent on the child's cognitive level. Children who repeatedly tell identical stories may have been coached.

A child is receiving vincristine (Oncovin). The nurse should monitor for which side effect of this medication? a. Diarrhea b. Photosensitivity c. Constipation d. Ototoxicity

ANS: A Vincristine, and to a lesser extent vinblastine, can cause various neurotoxic effects. One of the more common neurotoxic effects is severe constipation caused from decreased bowel innervation.

The nurse is preparing to assist with a growth hormone provocative test for a child with short stature. The nurse recognizes that which pharmacologics should be used to provoke the release of growth hormone (GH)? (Select all that apply.) a. Larodopa (levodopa) b. Clonidine (Catapres) c. Propranolol (Inderal) d. Cortisone (hydrocortisone) e. Biosynthetic growth hormone

ANS: A, B, C GH stimulation, or provocative testing, involves the use of pharmacologics to provoke the release of GH either directly or indirectly. Provocative testing involves the use of neuromodulators such as levodopa or agents such as clonidine, arginine, insulin, propranolol, or glucagon followed by the measurement GH blood levels. Cortisone is given to replace hormone deficiencies that can occur with GH deficiency. Biosynthetic GH is used to treat GH deficiency.

A parent asks the nurse about the "characteristics of a nightmare." What response should the nurse give to the parent? (Select all that apply.) a. Nightmares are scary dreams. b. The child can describe the nightmare. c. The child is reassured by your presence. d. Nightmares occur usually 1 to 4 hours after falling asleep. e. Nightmares take place during non-rapid eye movement sleep

ANS: A, B, C Nightmares are scary dreams, the child can describe the nightmare, and the child is reassured by a parent's presence. Sleep terrors occur usually 1 to 4 hours after falling asleep, but nightmares occur in the second half of sleep. Sleep terrors occur during non-rapid eye movement sleep, but nightmares occur during rapid eye movement sleep.

The nurse understands that which gestational disorders can cause a cognitive impairment in the newborn? (Select all that apply.) a. Prematurity b. Postmaturity c. Low birth weight d. Physiological jaundice e. Large for gestational age

ANS: A, B, C Prematurity, postmaturity, and low birth weight can be causes of cognitive impairment in newborns. Physiological jaundice and large for gestational age are not associated causes of cognitive impairment in newborns.

What strategies should the nurse implement to increase nutritional intake for the child receiving chemotherapy? (Select all that apply.) a. Allow the child any food tolerated. b. Fortify foods with nutritious supplements. c. Allow the child to be involved in food selection. d. Encourage the parents to place pressure on the importance of eating. e. Encourage the child to eat favorite foods during infusion of chemotherapy medications.

ANS: A, B, C To increase nutritional intake for the child receiving chemotherapy, the nurse should allow the child any food tolerated, fortify foods with nutritious supplements, and allow the child to be involved in food selection. The parents should be encouraged to reduce pressure placed on eating. Some children develop aversions to certain foods if they are eaten during chemotherapy. It is best to refrain from offering the child's favorite foods while the child is receiving chemotherapy.

The clinic nurse is assessing a child with central precocious puberty. What conditions can cause central precocious puberty? (Select all that apply.) a. Trauma b. Neoplasms c. Radiotherapy d. Exogenous sex hormones e. Primary hypothyroidism

ANS: A, B, C Trauma, neoplasms, and radiotherapy can be the cause of central precocious puberty. Exogenous sex hormones and primary hypothyroidism can cause peripheral precocious puberty.

The nurse should teach the family that which residual disabilities can occur for a child being treated for a brain tumor? (Select all that apply.) a. Ataxia b. Anorexia c. Dysphagia d. Sensory deficits e. Crania nerve palsies

ANS: A, C, D, E Even with children who are long-term survivors after treatment for a brain tumor, residual disabilities, such as short stature, cranial nerve palsies, sensory defects, motor abnormalities (especially ataxia), intellectual deficits, dysphagia, dysgraphia, and behavioral problems, may occur. Anorexia is not a residual disability.

The nurse is planning strategies to assist difficult or easily distracted children when they participate in activities. What strategies should the nurse plan? (Select all that apply.) a. Role-play before the activity. b. Handle behavior with firmness. c. Acquaint them with what to expect. d. Be patient with inappropriate behavior. e. Don't give them much information about the activity.

ANS: A, B, C, D Difficult or easily distracted children may benefit from "practice" sessions in which they are prepared for a given event by role-playing, visiting the site, reading or listening to stories, or using other methods to acquaint them with what to expect. Nurses need to handle children with difficult temperaments with exceptional patience, firmness, and understanding so they can learn appropriate behavior in their interactions with others.

What are supportive interventions that can assist a preschooler with a chronic illness to meet developmental milestones? (Select all that apply.) a. Encourage socialization. b. Encourage mastery of self-help skills. c. Provide devices that make tasks easier. d. Clarify that the cause of the child's illness is not his or her fault. e. Discuss planning for the future and how the condition can affect choices.

ANS: A, B, C, D To encourage initiative, mastery of self-help skills should be encouraged, and devices should be provided that make tasks easier. To develop peer relationships, socialization should be encouraged. To develop body image, the fact that the cause of the child's illness is not the fault of the child should be emphasized. Discussing planning for the future and how the condition can affect choices is appropriate for an adolescent.

The nurse is assessing coping behaviors of a family with a child with a chronic illness. What indicates avoidance coping behaviors? (Select all that apply.) a. Refuses to agree to treatment b. Avoids staff, family members, or child c. Is unable to discuss possible loss of the child d. Recognizes own growth through a passage of time e. Makes no change in lifestyle to meet the needs of other family members

ANS: A, B, C, E Avoidance coping behaviors include refusing to agree to treatment; avoiding staff, family members, or child; unable to discuss possible loss of the child; and making no change in lifestyle to meet the needs of other family members. Recognizing one's own growth through a passage of time is an approach behavior.

A child on chemotherapy has developed rectal ulcers. What interventions should the nurse teach to the child and parents to relieve the discomfort of rectal ulcers? (Select all that apply.) a. Warm sitz baths b. Use of stool softeners c. Record bowel movements d. Use of an opioid for discomfort e. Occlusive ointment applied to the area

ANS: A, B, C, E If rectal ulcers develop, meticulous toilet hygiene, warm sitz baths after each bowel movement, and an occlusive ointment applied to the ulcerated area promote healing; the use of stool softeners is necessary to prevent further discomfort. Parents should record bowel movements because the child may voluntarily avoid defecation to prevent discomfort. Opioids would cause increased constipation.

The nurse is preparing an education program on hearing impairment for a group of new staff nurses. What concepts should be included? (Select all that apply.) a. A child with a slight hearing loss is usually unaware of a hearing difficulty. b. A clinical manifestation of a hearing impairment in children is avoidance of social interaction. c. A child with a severe hearing loss may hear a loud voice if nearby. d. Children with sensorineural hearing loss can benefit from the use of a hearing aid. e. A clinical manifestation of hearing impairment in an infant is lack of the startle reflex. f. Identification of a hearing loss after the first year is essential to facilitate language development in children.

ANS: A, B, C, E When discussing hearing impairment in children, the nurse should include information about differences in hearing losses, such as with a slight hearing loss, the child is usually unaware of a hearing difficulty, and with a severe loss, the child may hear a loud noise if it is nearby. An infant with a hearing loss may lack the startle response, and a hearing impaired child may avoid social interaction. Children with a sensorineural hearing loss would not benefit from a hearing aid. Identification of a hearing loss is imperative in the first 3 to 6 months to facilitate language and educational development for children.

The nurse is planning to admit an 8-year-old child with hypoparathyroidism. What clinical manifestations should the nurse expect to observe in this child? (Select all that apply.) a. Muscle cramps b. Positive Chvostek sign c. Emotional lability d. Laryngeal spasms e. Short attention span

ANS: A, B, D Clinical manifestations of hypoparathyroidism include muscle cramps, positive Chvostek sign, and laryngeal spasms. Emotional lability and short attention span are signs of Graves disease.

The nurse is planning to admit a 12-year-old with Graves disease (GD). What clinical manifestations should the nurse expect to observe in this child? (Select all that apply.) a. Insomnia b. Irritability c. Tonic rigidity d. Hyperactivity e. Muscle cramps

ANS: A, B, D Signs and symptoms of hyperthyroidism develop gradually, with an interval between onset and diagnosis of approximately 6 to 12 months. Clinical features include irritability, hyperactivity, short attention span, tremors, insomnia, and emotional lability. Tonic rigidity and muscle cramps are signs of hypoparathyroidism.

What are supportive interventions that can assist an infant with a chronic illness to meet developmental milestones? (Select all that apply.) a. Encourage consistent caregivers. b. Encourage periodic respite from demands of care. c. Encourage one family member to be the primary caretaker. d. Encourage parental "rooming in" during hospitalization. e. Withhold age-appropriate developmental tasks until the child is older.

ANS: A, B, D To develop trust, consistent caretakers and parents "rooming in" should be encouraged. To develop a sense of separateness from parents, periodic respites from caregiving should be encouraged. All members of the family, not one primary caretaker, should be encouraged to participate in care. Age-appropriate developmental tasks should be encouraged, not withheld until an older age.

The nurse is preparing to admit a 2-year-old child with rubella (German measles). Which clinical manifestations of rubella should the nurse expect to observe? (Select all that apply.) a. Sore throat b. Conjunctivitis c. Koplik spots d. Lymphadenopathy e. Discrete, pinkish red maculopapular exanthema

ANS: A, B, D, E The clinical manifestations of rubella include a sore throat; conjunctivitis; lymphadenopathy; and a discrete, pinkish red maculopapular exanthema. Koplik spots occur in measles but not rubella.

The nurse is providing anticipatory guidance to parents of a 4-month-old infant on preventing an aspiration injury. What should the nurse include in the teaching? (Select all that apply.) a. Keep baby powder out of reach. b. Inspect toys for removable parts. c. Allow the infant to take a bottle to bed. d. Teething biscuits can be used for teething discomfort. e. The infant should not be fed hard candy, nuts, or foods with pits.

ANS: A, B, E Anticipatory guidance to prevent aspiration for a 4-month-old infant takes into account that the infant will begin to be more active and place objects in the mouth. Toys should be checked for removable parts; baby powder should be kept out of reach; and hard candy, nuts, and foods with pits should be avoided. The infant should not go to bed with a bottle. Teething biscuits should be used with caution because large chunks may be broken off and aspirated.

What are indications for a referral regarding a communication impairment in a school-age child? (Select all that apply.) a. Barely audible voice quality b. Vocal pitch inappropriate for age c. Intonation noted during speaking d. Maintains a rhythm while speaking e. Distortion of sounds after age 7 years

ANS: A, B, E Barely audible voice quality, vocal pitch inappropriate for age, and distortion of sounds after age 7 years are indications for a referral regarding a communication impairment. Intonation noted while speaking and maintaining a rhythm while speaking are normal characteristics of speech.

What growth and development milestones are expected between the ages of 8 and 9 years? (Select all that apply.) a. Can help with routine household tasks b. Likes the reward system for accomplished tasks c. Uses the telephone for practical purposes d. Chooses friends more selectively e. Goes about home and community freely, alone or with friends f. Enjoys family time and is respectful of parents

ANS: A, B, E Children between the age of 8 and 9 years accomplish many growth and development milestones, including helping with routine household tasks, liking the reward system when a task is accomplished well, and going out with friends or alone more independently and freely. Using the telephone for practical reasons, choosing friends more selectively, and finding enjoyment in family with new-found respect for parents are tasks accomplished between the ages of 10 and 12 years.

What are symptoms of abusive head trauma (AHT) in the more severe form that may be present? (Select all that apply.) a. Seizures b. Posturing c. Tachypnea d. Tachycardia e. Altered level of consciousness

ANS: A, B, E In more severe forms, presenting symptoms of abusive head trauma may include seizures, posturing, alterations in level of consciousness, apnea, bradycardia, or death.

What are supportive interventions that can assist a toddler with a chronic illness to meet developmental milestones? (Select all that apply.) a. Give choices. b. Provide sensory experiences. c. Avoid discipline and limit setting. d. Discourage negative and ritualistic behaviors. e. Encourage independence in as many areas as possible.

ANS: A, B, E To encourage autonomy, choices should be given and independence encouraged in as many areas as possible. Sensory experiences should be encouraged to help the toddler to learn through sensorimotor experiences. Age-appropriate discipline and limit setting should be initiated. Negative and ritualistic behaviors are normal and should be allowed.

The nurse is planning to administer immunizations to a 6-month-old infant. Which interventions should the nurse implement to minimize local reactions from the vaccines? (Select all that apply.) a. Select a needle length of 1 inch. b. Administer in the deltoid muscle. c. Inject the vaccine into the vastus lateralis. d. Draw the vaccine up from a vial with a filter needle. e. Change the needle on the syringe after drawing up the vaccine and before injecting.

ANS: A, C To minimize local reactions from vaccines, the nurse should select a needle of adequate length (25 mm [1 inch] in infants) to deposit the antigen deep in the muscle mass and inject it into the vastus lateralis muscle. The deltoid may be used in children 18 months of age or older but not in a 6-month-old infant. A filter needle is not needed to draw the vaccine from a vial. Changing the needle on the syringe after drawing up the vaccine before injecting it has not been shown to decrease local reactions.

The nurse is preparing to admit a 7-year-old child with type 2 diabetes. What clinical features of type 2 diabetes should the nurse recognize? (Select all that apply.) a. Oral agents are effective. b. Insulin is usually needed. c. Ketoacidosis is infrequent. d. Diet only is often effective. e. Chronic complications frequently occur.

ANS: A, C, D The clinical features of type 2 diabetes include the following: oral agents are effective, ketoacidosis is infrequent, and diet only is often effective. Insulin is only needed in 20% to 30% of cases and chronic complications occur infrequently.

What are supportive interventions that can assist an adolescent with a chronic illness to meet developmental milestones? (Select all that apply.) a. Encourage activities appropriate for age. b. Avoid discussing planning for the future. c. Provide instruction on interpersonal and coping skills. d. Emphasize good appearance and wearing of stylish clothes. e. Understand that the adolescent will not have the same sexual needs.

ANS: A, C, D To achieve independence from family, instruction on interpersonal and coping skills should be provided. To promote heterosexual relationships, activities appropriate for age should be encouraged, and a good appearance and wearing of stylish clothes should be emphasized. Plans for the future should be discussed, and the adolescent will have the same sexual needs as adolescents without a chronic illness.

A school-age child has been a victim of bullying. What characteristics does the nurse assess for in this child? (Select all that apply.) a. Anxiety b. Outgoing c. Low self-esteem d. Psychosomatic complaints e. Good academic performance

ANS: A, C, D Victims of bullying are at increased risk for low self-esteem; anxiety; depression; feelings of insecurity and loneliness; poor academic performance; and psychosomatic complaints such as feeling tense, tired, or dizzy.

The nurse is providing anticipatory guidance to parents of an 8-month-old infant on preventing a drowning injury. Which should the nurse include in the teaching? (Select all that apply.) a. Fence swimming pools. b. Keep bathroom doors open. c. Eliminate unnecessary pools of water. d. Keep one hand on the child while in the tub. e. Supervise the child when near any source of water.

ANS: A, C, D, E Anticipatory guidance to prevent drowning for an 8-month-old infant takes into account that the child will begin to crawl, cruise around furniture, walk, and climb. Fences should be placed around swimming pools, unnecessary pools of water should be eliminated, one hand should be kept on the child when bathing, and the child should be supervised when near any source of water. The bathroom doors should be kept closed.

The nurse is providing anticipatory guidance to the parents of a 1-month-old infant on preventing a suffocation injury. Which should the nurse include in the teaching? (Select all that apply.) a. Do not place pillows in the infant's crib. b. Crib slats should be 4 inches or less apart. c. Keep all plastic bags stored out of the infant's reach. d. Plastic over the mattress is acceptable if it is covered with a sheet. e. A pacifier should not be tied on a string around the infant's neck.

ANS: A, C, E Anticipatory guidance for a 1-month-old infant to prevent a suffocation injury takes into account that the infant will have increased eye-hand coordination and a voluntary grasp reflex as well as a crawling reflex that may propel the infant forward or backward. Pillows should not be placed in the infant's crib, plastic bags should be kept out of reach, and a pacifier should not be tied on a string around the neck. Crib slats should be 2.4 inches apart (4 inches is too wide), and the mattress should not be covered with plastic even if a sheet is used to cover it.

The nurse is planning to admit a 14-year-old adolescent with Cushing syndrome. What clinical manifestations should the nurse expect to observe in this child? (Select all that apply.) a. Truncal obesity b. Decreased pubic hair c. Petechial hemorrhage d. Hyperpigmentation of elbows e. Facial plethora f. Headache and weakness

ANS: A, C, E Clinical manifestations of Cushing syndrome include truncal obesity, petechial hemorrhage, and facial plethora. Decreased pubic and axillary hair; hyperpigmentation of elbows, knees, and wrists; and headache and weakness are clinical manifestations of adrenocortical insufficiency.

What are supportive interventions that can assist a school-age child with a chronic illness to meet developmental milestones? (Select all that apply.) a. Encourage socialization. b. Discourage sports activities. c. Encourage school attendance. d. Provide instructions on assertiveness. e. Educate teachers and classmates about the child's condition.

ANS: A, C, E To develop a sense of accomplishment, school attendance should be encouraged, and teachers and classmates should be educated about the child's condition. To form peer relationships, socialization should be encouraged. Sports activities should be encouraged (e.g., Special Olympics), not discouraged. Providing instructions on assertiveness is appropriate for adolescence.

The nurse is preparing a community outreach program for adolescents about the characteristic differences between type 1 and type 2 diabetes mellitus (DM). What concepts should the nurse include? (Select all that apply.) a. Type 1 DM has an abrupt onset. b. Type 1 DM is often controlled with oral glucose agents. c. Type 1 DM occurs primarily in whites. d. Type 2 DM always requires insulin therapy. e. Type 2 DM frequently has a familial history. f. Type 2 DM occurs in people who are overweight.

ANS: A, C, E, F Characteristics of type 1 DM include having an abrupt onset, primarily occurring in whites, and not being controlled with oral glucose agents (insulin is required for therapy). Type 2 DM frequently has a familial history, occurs in people who are overweight, and does not always require insulin therapy (it is used in 20% to 30% of patients).

The nurse is precepting a new graduate nurse at an ambulatory pediatric hematology and oncology clinic. What cardinal signs of cancer in children should the nurse make the new nurse aware of? (Select all that apply.) a. Sudden tendency to bruise easily b. Transitory, generalized pain c. Frequent headaches d. Excessive, rapid weight gain e. Gradual, steady fever f. Unexplained loss of energy

ANS: A, C, F The cardinal signs of cancer in children include a sudden tendency to bruise easily; frequent headaches, often with vomiting; and an unexplained loss of energy. Other cardinal signs include persistent, localized pain; excessive, rapid weight loss; and a prolonged, unexplained fever.

The nurse is planning care for an infant with candidiasis (moniliasis) diaper dermatitis. Which topical ointments may be prescribed for the patient? (Select all that apply.) a. Nystatin b. Bactroban c. Neosporin d. Miconazole e. Clotrimazole

ANS: A, D, E Candidiasis diaper dermatitis skin lesions are treated with topical nystatin, miconazole, and clotrimazole. Bactroban and Neosporin are used to treat bacterial dermatitides.

The nurse is planning care for a child with chickenpox (varicella). Which prescribed supportive measures should the nurse plan to implement? (Select all that apply.) a. Administration of acyclovir (Zovirax) b. Administration of azithromycin (Zithromax) c. Administration of Vitamin A supplementation d. Administration of acetaminophen (Tylenol) for fever e. Administration of diphenhydramine (Benadryl) for itching

ANS: A, D, E Chickenpox is a virus, and acyclovir is ordered to lessen the symptoms. Benadryl and Tylenol are prescribed as supportive treatments. Vitamin A supplementation is used for treating rubeola. Zithromax is an antibiotic prescribed for bacterial infections such as pertussis.

The nurse is planning to admit a 14-year-old adolescent with hyperparathyroidism. What clinical manifestations should the nurse expect to observe in this patient? (Select all that apply.) a. Polyuria b. Diarrhea c. Hypotension d. Vague bone pain e. Paresthesia in extremities

ANS: A, D, E Clinical manifestations of hyperparathyroidism include polyuria, vague bone pain, and paresthesia in the extremities. Constipation, not diarrhea, and hypertension, not hypotension, are manifestations of hyperparathyroidism.

The clinic nurse is assessing an infant. What are early signs of cognitive impairment the nurse should discuss with the health care provider? (Select all that apply.) a. Head lag at 11 months of age b. No pincer grasp at 4 months of age c. Colicky incidents at 3 months of age d. Unable to speak two to three words at 24 months of age e. Unresponsiveness to the environment at 12 months of age

ANS: A, D, E Early signs of cognitive impairment include gross motor delay (head lag should be established by 6 months, and head lag still present at 11 months is a delay), language delay (normal language development is speaking two to three words by age 12 months; if unable to speak two to three words at 24 months, that is a delay), and unresponsiveness to the environment at 12 months. No pincer grasp at 4 months of age is normal (palmar grasp is the expected finding), and colicky incidents at 3 months of age is a normal finding.

A male school-age student asks the school nurse, "How much with my height increase in a year?" The nurse should give which response? a. "Your height will increase on average 1 inch a year." b. "Your height will increase on average 2 inches a year." c. "Your height will increase on average 3 inches a year." d. "Your height will increase on average 4 inches a year."

ANS: B Between the ages of 6 and 12 years, children grow an average of 5 cm (2 inches) per year.

The nurse is conducting discharge teaching to an adolescent with a methicillin-resistant staphylococcus aureus (MRSA) infection. What should the nurse include in the instructions? (Select all that apply.) a. Avoid sharing of towels and washcloths. b. Launder clothes and bedding in cold water. c. Use bleach when laundering towels and washcloths. d. Take a daily bath or shower with an antibacterial soap. e. Apply mupirocin (Bactroban) to the nares twice a day for 2 to 4 weeks.

ANS: A, D, E For MRSA infection, the adolescent should be provided with washcloths and towels separate from those of other family members. Daily bathing or showering with an antibacterial soap is also recommended. Mupirocin should be applied to the nares of those with MRSA infection twice daily for 2 to 4 weeks. Clothing should be laundered in warm to hot water, not cold, and bleach does not need to be used when laundering towels and washcloths.

The nurse is planning strategies to assist a slow-to-warm child to try new experiences. What strategies should the nurse plan? (Select all that apply.) a. Attend after-school activities with a friend. b. Suggest the child move quickly into a new situation. c. Avoid trying new experiences until the child is ready. d. Allow the child to adapt to the experience at his or her own pace. e. Contract for permission to withdraw after a trial of the experience.

ANS: A, D, E The nurse should encourage slow-to-warm children to try new experiences but allow them to adapt to their surroundings at their own speed. Pressure to move quickly into new situations only strengthens their tendency to withdraw. After-school activities can be a cause for reaction, but attending with a friend or contracting for permission to withdraw after a trial of a specified number of times may provide them with sufficient incentive to try.

The parent of an 8.2-kg (18-lb) 9-month-old infant is borrowing a federally approved car seat from the clinic. The nurse should explain that the safest way to put in the car seat is what? a. Front facing in back seat b. Rear facing in back seat c. Front facing in front seat with air bag on passenger side d. Rear facing in front seat if an air bag is on the passenger side

ANS: B A rear-facing car seat provides the best protection for an infant's disproportionately heavy head and weak neck. The middle of the back seat is the safest position for the child. Severe injuries and deaths in children have occurred from air bags deploying on impact in the front passenger seat.

A child with osteosarcoma is experiencing phantom limb pain after an amputation. What prescribed medication is effective for short-term phantom pain relief? a. Phenytoin (Dilantin) b. Gabapentin (Neurontin) c. Valproic Acid (Depakote) d. Phenobarbital (Phenobarbital)

ANS: B A recent Cochrane review reported that various medications have been used for phantom limb pain but complete pain relief has been unsuccessful. Morphine, gabapentin, and ketamine are effective for short-term pain relief.

A child with hypopituitarism is being started on growth hormone (GH) therapy. Nursing considerations should be based on which knowledge? a. Therapy is most successful if it is started during adolescence. b. Replacement therapy requires daily subcutaneous injections. c. Hormonal supplementation will be required throughout child's lifetime. d. Treatment is considered successful if children attain full stature by adolescence.

ANS: B Additional support is required for children who require hormone replacement therapy, such as preparation for daily subcutaneous injections and education for self-management during the school-age years. Young children, obese children, and those who are severely GH deficient have the best response to therapy. Replacement therapy is not needed after attaining final height. The children are no longer GH deficient. When therapy is successful, children can attain their actual or near-final adult height at a slower rate than their peers.

Essential postoperative nursing management of a child after removal of a brain tumor includes which nursing care? a. Turning and positioning every 2 hours b. Measuring all fluid intake and output c. Changing the dressing when it becomes soiled d. Using maximum lighting to ensure accurate observations

ANS: B After brain surgery, cerebral edema is a risk. Careful monitoring is essential. All fluids, including intravenous antibiotics, are included in the intake. Turning and positioning depend on the surgical procedure. When large tumors are removed, the child is usually not positioned on the operative side. The dressing is not changed. It is reinforced with gauze after the amount of drainage is marked and estimated. A quiet, dimly lit environment is optimum to decrease stimulation and relieve discomfort such as headaches.

An awake, alert 4-year-old child has just arrived at the emergency department after an ingestion of aspirin at home. The practitioner has ordered activated charcoal. The nurse administers charcoal in which manner? a. Giving half of the solution and then repeating the other half in 1 hour b. Mixing with a flavorful beverage in an opaque container with a straw c. Serving it in a clear plastic cup so the child can see how much has been drunk d. Administering it through a nasogastric tube because the child will not drink it because of the taste

ANS: B Although activated charcoal can be mixed with a flavorful sugar-free beverage, it will be black and resemble mud. When it is served in an opaque container, the child will not have any preconceived ideas about its being distasteful. The ability to see the charcoal solution may affect the child's desire to drink the solution. The child should be encouraged to drink the solution all at once. The nasogastric tube would be traumatic. It should be used only in children who cannot be cooperative or those without a gag reflex.

What is the major health concern of children in the United States? a. Acute illness b. Chronic illness c. Congenital disabilities d. Nervous system disorders

ANS: B An estimated 18% of children in the United States have a chronic illness or disability that warrants health care services beyond those usually required by children. Chronic illness has surpassed acute illness as the major health concern for children. Congenital disabilities exist from birth but may not be hereditary. These represent a portion of the number of children with chronic illnesses. Mental and nervous system disorders account for approximately 17% of chronic illnesses in children.

The nurse is caring for a child after a parathyroidectomy. What medication should the nurse have available if hypocalcemia occurs? a. Insulin b. Calcium gluconate c. Propylthiouracil (PTU) d. Cortisone (hydrocortisone)

ANS: B Because hypocalcemia is a potential complication after a parathyroidectomy, observing for signs of tetany, instituting seizure precautions, and having calcium gluconate available for emergency use are part of the nursing care.

A child is admitted to the hospital with lesions on his abdomen that appear like cigarette burns. What should accurate documentation by the nurse include? a. Two unhealed lesions are on the child's abdomen. b. Two round 4-mm lesions are on the child's lower abdomen. c. Two round symmetrical lesions are on the child's lower abdomen. d. Two round lesions on the child's abdomen that appear to be cigarette burns.

ANS: B Burn documentation should include the location, pattern, demarcation lines, and presence of eschar or blisters. The option that includes the size of the lesions is the most accurate.

The school nurse is providing guidance to families of children who are entering elementary school. What is essential information to include? a. Meet with teachers only at scheduled conferences. b. Encourage growth of a sense of responsibility in children. c. Provide tutoring for children to ensure mastery of material. d. Homework should be done as soon as child comes home from school.

ANS: B By being responsible for school work, children learn to keep promises, meet deadlines, and succeed in their jobs as adults. Parents should meet with the teachers at the beginning of the school year, for scheduled conferences, and whenever information about the child or parental concerns needs to be shared. Tutoring should be provided only in special circumstances in elementary school, such as in response to prolonged absence. The parent should not dictate the study time but should establish guidelines to ensure that homework is done.

What is the most common form of child maltreatment? a. Sexual abuse b. Child neglect c. Physical abuse d. Emotional abuse

ANS: B Child neglect, which is characterized by the failure to provide for the child's basic needs, is the most common form of child maltreatment. Sexual abuse, physical abuse, and emotional abuse are individually not as common as neglect.

Parents are switching their toddler, who has met the weight requirement, from a rear-facing car seat to a forward-facing seat. The nurse should recommend the parents place the seat where in the car? a. In the front passenger seat b. In the middle of the rear seat c. In the rear seat behind the driver d. In the rear seat behind the passenger

ANS: B Children 0 to 3 years of age riding properly restrained in the middle of the backseat have a 43% lower risk of injury than children riding in the outboard (window) seat during a crash.

What intervention should be included in the nursing care of a child with autism spectrum disorder (ASD)? a. Assign multiple staff to care for the child. b. Communicate with the child at his or her developmental level. c. Provide a wide variety of foods for the child to try. d. Place the child in a semiprivate room with a roommate of a similar age.

ANS: B Children with ASD require individualized care. The nurse needs to communicate with the child at the child's developmental level. Consistent caregivers are essential for children with ASD. The same staff members should care for the child as much as possible. Children with ASD do not adapt to changing situations. The same foods should be provided to allow the child to adjust. A private room is desirable for children with ASD. Stimulation is minimized.

The parents of a 7-year-old boy tell the nurse that lately he has been cruel to their family pets and actually caused physical harm. The nurse's recommendation should be based on remembering what? a. This is an expected behavior at this age. b. This is a warning sign of a serious problem. c. This is harmless venting of anger and frustration. d. This is common in children who are physically abused.

ANS: B Cruelty to family pets is not an expected behavior. Hurting animals can be one of the earliest symptoms of a conduct disorder. Abusing animals does not dissipate violent emotions; rather, the acts may fuel the abusive behaviors. Referral for evaluation is essential. This behavior may be seen in emotional abuse or neglect, not physical abuse

Intranasal administration of desmopressin acetate (DDAVP) is used to treat which condition? a. Hypopituitarism b. Diabetes insipidus (DI) c. Syndrome of inappropriate antidiuretic hormone (SIADH) d. Acute adrenocortical insufficiency

ANS: B DDAVP is the treatment of choice for DI. It is administered intranasally through a flexible tube. The child's response pattern is variable, with effectiveness lasting from 6 to 24 hours.

Exophthalmos (protruding eyeballs) may occur in children with which condition? a. Hypothyroidism b. Hyperthyroidism c. Hypoparathyroidism d. Hyperparathyroidism

ANS: B Exophthalmos is associated with hyperthyroidism. Hypothyroidism, hypoparathyroidism, and hyperparathyroidism are not associated with exophthalmos.

Families progress through various stages of reactions when a child is diagnosed with a chronic illness or disability. After the shock phase, a period of adjustment usually follows. This is often characterized by what response? a. Denial b. Guilt and anger c. Social reintegration d. Acceptance of the child's limitations

ANS: B For most families, the adjustment phase is accompanied by several responses, including guilt, self-accusation, bitterness, and anger. The initial diagnosis of a chronic illness or disability often is met with intense emotion and characterized by shock and denial. Social reintegration and acceptance of the child's limitations are the culmination of the adjustment process.

What description applies to fragile X syndrome? a. Chromosomal defect affecting only females b. Second most common genetic cause of cognitive impairment c. Most common cause of uninherited cognitive impairment d. Chromosomal defect that follows the pattern of X-linked recessive disorders

ANS: B Fragile X syndrome is the most common inherited cause of cognitive impairment and the second most common genetic cause of cognitive impairment after Down syndrome. Fragile X primarily affects males and follows the pattern of X-linked dominant inheritance with reduced penetrance.

What is a clinical manifestation of acetaminophen poisoning? a. Hyperpyrexia b. Hepatic involvement c. Severe burning pain in stomach d. Drooling and inability to clear secretions

ANS: B Hepatic involvement is the third stage of acetaminophen poisoning. Hyperpyrexia is a severe elevation in body temperature and is not related to acetaminophen poisoning. Acetaminophen does not cause burning pain in stomach and does not pose an airway threat.

A child with leukemia is receiving intrathecal chemotherapy to prevent which condition? a. Infection b. Brain tumor c. Central nervous system (CNS) disease d. Drug side effects

ANS: C Children with leukemia are at risk for invasion of the CNS with leukemic cells. CNS prophylactic therapy is indicated. Intrathecal chemotherapy does not prevent infection or drug side effects. A brain tumor in a child with leukemia would be a second tumor, and additional appropriate therapy would be indicated.

What is descriptive of the social development of school-age children? a. Identification with peers is minimum. b. Children frequently have "best friends." c. Boys and girls play equally with each other. d. Peer approval is not yet an influence for the child to conform.

ANS: B Identification with peers is a strong influence in children's gaining independence from parents. Interaction among peers leads to the formation of close friendships with same-sex peers—"best friends." Daily relationships with age mates in the school setting provide important social interactions for school-age children. During the later school years, groups are composed predominantly of children of the same sex. Conforming to the rules of the peer group provides children with a sense of security and relieves them of the responsibility of making decisions.

The nurse has been visiting an adolescent with recently acquired tetraplegia. The teen's mother tells the nurse, "I'm sick of providing all the care while my husband does whatever he wants to, whenever he wants to do it." What reaction should be the nurse's initial response? a. Refer the mother for counseling. b. Listen and reflect the mother's feelings. c. Ask the father in private why he does not help. d. Suggest ways the mother can get her husband to help.

ANS: B It is appropriate for the nurse to reflect with the mother about her feelings, exploring solutions such as an additional home health aide to help care for the child and provide respite for the mother. It is inappropriate for the nurse to agree with the mother that her husband is not helping enough. This judgment is beyond the role of the nurse and can undermine the family relationship. Counseling, if indicated, would be necessary for both parents. A support group for caregivers may be indicated. The nurse should not ask the father in private why he does not help or suggest way the mother can get her husband to help. These interventions are based on the mother's perceptions; the father may have a full-time job and other commitments. The parents may need an unbiased third person to help them through the negotiation of their new parenting responsibilities.

Congenital adrenal hyperplasia (CAH) is suspected in a newborn because of ambiguous genitalia. The parents are appropriately upset and concerned about their child's gender. In teaching the parents about CAH, what should the nurse explain? a. Reconstructive surgery as a female is preferred. b. Sexual assignment should wait until genetic sex is determined. c. Prenatal masculinization will strongly influence the child's development. d. The child should be raised as a boy because of the presence of a penis and scrotum.

ANS: B It is preferable to raise the child according to genetic sex. With hormone replacement and surgical intervention if needed, genetically female children achieve satisfactory results in reversing virilism and achieving normal puberty and ability to conceive. Reconstructive surgery as a female is only preferred for infants who are genetically female. Infants who are genetically male should be given hormonal supplementation. Sex assignment and rearing depend on psychosocial influences, not on genetic sex hormone influences during fetal life. It is not advised to raise the child as a boy because of the presence of a penis and scrotum unless the child is genetically male. If a genetic female, the child will be sterile and may never be able to function satisfactorily in a heterosexual relationship.

What is the primary treatment for warts? a. Vaccination b. Local destruction c. Corticosteroids d. Specific antibiotic therapy

ANS: B Local destructive therapy is individualized according to location, type, and number; surgical removal, electrocautery, curettage, cryotherapy, caustic solutions, x-ray treatment, and laser therapies are used. Vaccination is prophylaxis for warts, not a treatment. Corticosteroids and specific antibiotic therapy are not effective in the treatment of warts.

A child has been diagnosed with giardiasis. Which prescribed medication should the nurse expect to administer? a. Acyclovir (Zovirax) b. Metronidazole (Flagyl) c. Erythromycin (Pediazole) d. Azithromycin (Zithromax)

ANS: B Metronidazole is an antibiotic effective against anaerobic bacteria and certain parasites. It is prescribed to treat giardiasis. Zithromax is an antibiotic frequently used to treat respiratory infections. Zovirax is an antiviral medication and Pediazole is an antibiotic used to treat respiratory and skin infections.

A preadolescent has maintained good glycemic control of his type 1 diabetes through the school year. During summer vacation, he has had repeated episodes of hypoglycemia. What additional teaching is needed? a. Carbohydrates in the diet need to be replaced with protein. b. Additional snacks are needed to compensate for increased activity. c. The child needs to decrease his activity level to minimize episodes of hypoglycemia. d. Insulin dosage should be increased to compensate for a change in activity level.

ANS: B Most children have a different schedule during summer vacation. The increased activity and exercise reduce insulin resistance and increase glucose utilization. Additional snacks should be eaten before physical activity to increase carbohydrates and protein and compensate for increased activity. Physical activity should always be encouraged if the child is capable. The benefits include improved glucose utilization and decreased insulin requirements. In consultation with the practitioner, insulin dosage may need to be decreased because of improved glucose utilization.

The nurse is collecting a 24-hour urine sample on a child with suspected diagnosis of neuroblastoma. What finding in the urine is expected with neuroblastomas? a. Ketones b. Catecholamines c. Red blood cells d. Excessive white blood cells

ANS: B Neuroblastomas, particularly those arising on the adrenal glands or from a sympathetic chain, excrete the catecholamines epinephrine and norepinephrine. Urinary excretion of catecholamines is detected in approximately 95% of children with adrenal or sympathetic tumors.

The nurse is caring for a child receiving chemotherapy for leukemia. The child's granulocyte count is 600/mm3 and platelet count is 45,000/mm3. What oral care should the nurse recommend for this child? a. Rinsing mouth with water b. Daily toothbrushing and flossing c. Lemon glycerin swabs for cleansing d. Wiping teeth with moistened gauze or Toothettes

ANS: B Oral care is essential for children receiving chemotherapy to prevent infections and other complications. When the child's granulocyte count is above 500/mm3 and platelet count is above 40,000/mm3, daily brushing and flossing are recommended. Rinsing the mouth with water is not effective for oral hygiene. Lemon glycerin swabs are avoided because they have a drying effect on the mucous membranes, and the lemon may irritate eroded tissue and decay the child's teeth. Wiping teeth with moistened gauze or Toothettes is recommended when the child's granulocyte count is below 500/mm3 and platelet count is below 40,000/mm3.

A 5-year-old child will be starting kindergarten next month. She has cerebral palsy, and it has been determined that she needs to be in a special education classroom. Her parents are tearful when telling the nurse about this and state that they did not realize her disability was so severe. What is the best interpretation of this situation? a. This is a sign the parents are in denial. b. This is a normal anticipated time of parental stress. c. The parents need to learn more about cerebral palsy. d. The parents' expectations are too high.

ANS: B Parenting a child with a chronic illness can be stressful. At certain anticipated times, parental stress increases. One of these identified times is when the child begins school. Nurses can help parents recognize and plan interventions to work through these stressful periods. The parents are not in denial; rather, they are responding to the child's placement in school. The parents are not exhibiting signs of a remembering deficit; this is their first interaction with the school system with this child.

After the introduction of the Back to Sleep campaign in 1992, an increased incidence has been noted of which pediatric issues? a. Sudden infant death syndrome (SIDS) b. Plagiocephaly c. Failure to thrive d. Apnea of infancy

ANS: B Plagiocephaly is a misshapen head caused by the prolonged pressure on one side of the skull. If that side becomes misshapen, facial asymmetry may result. SIDS has decreased by more than 40% with the introduction of the Back to Sleep campaign. Apnea of infancy and failure to thrive are unrelated to the Back to Sleep campaign.

A feeling of guilt that the child "caused" the disability or illness is especially common in which age group? a. Toddler b. Preschooler c. School-age child d. Adolescent

ANS: B Preschoolers are most likely to be affected by feelings of guilt that they caused the illness or disability or are being punished for wrongdoings. Toddlers are focused on establishing their autonomy. The illness fosters dependency. School-age children have limited opportunities for achievement and may not be able to understand limitations. Adolescents face the task of incorporating their disabilities into their changing self-concept.

The nurse has been assigned as a home health nurse for a child who is technology dependent. The nurse recognizes that the family's background differs widely from the nurse's own. The nurse believes some of their lifestyle choices are less than ideal. What nursing intervention is most appropriate to institute? a. Change the family. b. Respect the differences. c. Assess why the family is different. d. Determine whether the family is dysfunctional.

ANS: B Respect for varied family structures and for racial, ethnic, cultural, and socioeconomic diversity among families is essential in home care. The nurse must assess and respect the family's background and lifestyle choices. It is not appropriate to attempt to change the family. The nurse is a guest in the home and care of the child. The family and the values held by the cultural group prevail. The nurse may assess why the family is different to help the nurse and other health professionals understand the differences. It is not appropriate to determine whether the family is dysfunctional.

What childhood cancer may demonstrate patterns of inheritance that suggest a familial basis? a. Leukemia b. Retinoblastoma c. Rhabdomyosarcoma d. Osteogenic sarcoma

ANS: B Retinoblastoma is an example of a pediatric cancer that demonstrates inheritance. The absence of the retinoblastoma gene allows for abnormal cell growth and the development of retinoblastoma. Chromosome abnormalities are present in many malignancies. They do not indicate a familial pattern of inheritance. The Philadelphia chromosome is observed in almost all individuals with chronic myelogenous leukemia. There is no evidence of a familial pattern of inheritance for rhabdomyosarcoma or osteogenic sarcoma cancers.

A 16-year-old boy with a chronic illness has recently become rebellious and is taking risks such as missing doses of his medication. What should the nurse explain to his parents? a. That he needs more discipline b. That this is a normal part of adolescence c. That he needs more socialization with peers d. That this is how he is asking for more parental control

ANS: B Risk taking, rebelliousness, and lack of cooperation are normal parts of adolescence, during which young adults are establishing independence. If the parents increase the amount of discipline, he will most likely be more rebellious. More socialization with peers does not address the problem of risk-taking behavior.

What is a significant secondary prevention nursing activity for lead poisoning? a. Chelation therapy b. Screening children for blood lead levels c. Removing lead-based paint from older homes d. Questioning parents about ethnic remedies containing lead

ANS: B Screening children for lead poisoning is an important secondary prevention activity. Screening does not prevent the initial exposure of the child to lead. It can lead to identification and treatment of children who are exposed. Chelation therapy is treatment, not prevention. Removing lead-based paints from older homes before children are affected is primary prevention. Questioning parents about ethnic remedies containing lead is part of the assessment to determine the potential source of lead.

A child has been admitted to the hospital with a blood lead level of 72 mcg/dL. What treatment should the nurse anticipate? a. Referral to social services b. Initiation of chelation therapy c. Follow-up testing within 1 month d. Aggressive environmental intervention

ANS: B Severe lead toxicity (lead level ?5=70 mcg/dL) requires immediate inpatient chelation treatment. Referral to social service and follow-up in 1 month are prescribed for lead levels of 15 to 19 mcg/dL. Aggressive environmental intervention would be initiated after chelation treatments.

A 3-month-old infant dies shortly after arrival to the emergency department. The infant has subdural and retinal hemorrhages but no external signs of trauma. What should the nurse suspect? a. Unintentional injury b. Shaken baby syndrome c. Congenital neurologic problem d. Sudden infant death syndrome (SIDS)

ANS: B Shaken baby syndrome causes internal bleeding but may have no external signs. Unintentional injury would not cause these injuries. With unintentional injuries, external signs are usually present. Congenital neurologic problems would usually have signs of abnormal neurologic anatomy. SIDS does not usually have identifiable injuries.

Homeostasis in the body is maintained by what is collectively known as the neuroendocrine system. What is the name of the nervous system that is involved? a. Central b. Skeletal c. Peripheral d. Autonomic

ANS: D The autonomic nervous system (composed of the sympathetic and parasympathetic systems) controls involuntary functions. In combination with the endocrine system, it maintains homeostasis. The central, skeletal, and peripheral subdivisions of the nervous system are not part of the neuroendocrine system.

A school-age child has begun to sleepwalk. What does the nurse advise the parents to perform? a. Wake the child and help determine what is wrong. b. Leave the child alone unless he or she is in danger of harming him- or herself or others. c. Arrange for psychologic evaluation to identify the cause of stress. d. Keep the child awake later in the evening to ensure sufficient tiredness for a full night of sleep.

ANS: B Sleepwalking is usually self-limiting and requires no treatment. The child usually moves about restlessly and then returns to bed. Usually the actions are repetitive and clumsy. The child should not be awakened unless in danger. If there is a need to awaken the child, it should be done by calling the child's name to gradually bring to a state of alertness. Some children, who are usually well behaved and tend to repress feelings, may sleepwalk because of strong emotions. These children usually respond to relaxation techniques before bedtime. If a child is overly fatigued, sleepwalking can increase.

The American Academy of Pediatrics (AAP) recommends that children younger than the age of 16 years be prohibited from participating in what? a. Skateboarding b. Snowmobiling c. Trampoline use d. Horseback riding

ANS: B The AAP views the use of snowmobiles and all-terrain vehicles as major health hazards for children. This group opposes the use of these vehicles by children younger than 16 years of age. The AAP recommends that children younger than the age of 10 years not use skateboards without parental supervision. Protective gear is always suggested. Trampoline use has increased along with injuries. Adults should supervise use. Horseback riding injuries are also a source of concern. Parents should determine the instructor's safety record with students.

A 5-year-old child is being prepared for surgery to remove a brain tumor. Preparation for surgery should be based on which information? a. Removal of the tumor will stop the various signs and symptoms. b. Usually the postoperative dressing covers the entire scalp. c. He is not old enough to be concerned about his head being shaved. d. He is not old enough to understand the significance of the brain.

ANS: B The child should be told what he will look and feel like after surgery. This includes the anticipated size of the dressing. The nurse can demonstrate on a doll the expected size and shape of the dressing. Some of the symptoms may be alleviated by removal of the tumor, but postsurgical headaches and cerebellar symptoms such as ataxia may be aggravated. Children should be prepared for the loss of their hair, and it should be removed in a sensitive, positive manner if the child is awake. Children at this age have poorly defined body boundaries and little knowledge of internal organs. Intrusive experiences are frightening, especially those that disrupt the integrity of the skin.

Which muscle is contraindicated for the administration of immunizations in infants and young children? a. Deltoid b. Dorsogluteal c. Ventrogluteal d. Anterolateral thigh

ANS: B The dorsogluteal site is avoided in children because of the location of nerves and veins. The deltoid is recommended for 12 months and older. The ventrogluteal and anterolateral thigh sites can safely be used for the administration of vaccines to infants.

Glucocorticoids, mineralocorticoids, and sex steroids are secreted by which gland? a. Thyroid gland b. Adrenal cortex c. Anterior pituitary d. Parathyroid glands

ANS: B The glucocorticoids, mineralocorticoids, and sex steroids are secreted by the adrenal cortex. The thyroid gland produces thyroid hormone and thyrocalcitonin. The anterior pituitary produces hormones such as growth hormone, thyroid-stimulating hormone, adrenocorticotropic hormone, gonadotropin, prolactin, and melanocyte-stimulating hormone. The parathyroid glands produce parathyroid hormone.

What is a primary goal in caring for a child with cognitive impairment? a. Developing vocational skills b. Promoting optimum development c. Finding appropriate out-of-home care d. Helping child and family adjust to future care

ANS: B The goal for children with cognitive impairment is the promotion of optimum social, physical, cognitive, and adaptive development as individuals within a family and community. Vocational skills are only one part of that goal. The focus must also be on the family and other aspects of development. Out-of-home care is considered part of the child's development. Optimum development includes adjustment for both the family and child.

When giving instructions to a parent whose child has scabies, what should the nurse include? a. Treat all family members if symptoms develop. b. Be prepared for symptoms to last 2 to 3 weeks. c. Carefully treat only areas where there is a rash. d. Notify practitioner so an antibiotic can be prescribed.

ANS: B The mite responsible for the scabies will most likely be killed with the administration of medications. It will take 2 to 3 weeks for the stratum corneum to heal. That is when the symptoms will abate. Initiation of therapy does not wait for clinical symptom development. All individuals in close contact with the affected child need to be treated. Permethrin, a scabicide, is the preferred treatment and is applied to all skin surfaces.

A school-age child with leukemia experienced severe nausea and vomiting when receiving chemotherapy for the first time. What is the most appropriate nursing action to prevent or minimize these reactions with subsequent treatments? a. Administer the chemotherapy between meals. b. Give an antiemetic before chemotherapy begins. c. Have the child bring favorite foods for snacks. d. Keep the child NPO (nothing by mouth) until nausea and vomiting subside.

ANS: B The most beneficial regimen to minimize nausea and vomiting associated with chemotherapy is to administer a 5-hydroxytryptamine-3 receptor antagonist (e.g., ondansetron) before the chemotherapy is begun. The goal is to prevent anticipatory signs and symptoms. The child will experience nausea with chemotherapy whether or not food is present in the stomach. Because some children develop aversions to foods eaten during chemotherapy, refraining from offering favorite foods is advised. Keeping the child NPO until nausea and vomiting subside will help with this episode, but the child will have discomfort and be at risk for dehydration.

What is an important consideration for the school nurse who is planning a class on bicycle safety? a. Most bicycle injuries involve collision with an automobile. b. Head injuries are the major causes of bicycle-related fatalities. c. Children should wear a bicycle helmet if they ride on paved streets. d. Children should not ride double unless the bicycle has an extra large seat.

ANS: B The most important aspect of bicycle safety is to encourage the rider to use a protective helmet. Head injuries are the major cause of bicycle-related fatalities. Although motor vehicle collisions do cause injuries to bicyclists, most injuries result from falls. The child should always wear a properly fitted helmet approved by the U.S. Consumer Product Safety Commission. Children should not ride double unless it is a tandem bike (built for two).

The nurse is assessing the coping behaviors of the parents of a child recently diagnosed with a chronic illness. What behavior should the nurse consider an "approach behavior" that results in movement toward adjustment? a. Being unable to adjust to a progression of the disease or condition b. Anticipating future problems and seeking guidance and answers c. Looking for new cures without a perspective toward possible benefit d. Failing to recognize the seriousness of the child's condition despite physical evidence

ANS: B The parents who anticipate future problems and seek guidance and answers are demonstrating approach behaviors. These are positive actions in caring for their child. Being unable to adjust, looking for new cures, and failing to recognize the seriousness of the child's condition are avoidance behaviors. The parents are moving away from adjustment or exhibiting maladaptation to the crisis of a child with chronic illness or disability.

The nurse is talking with a 10-year-old boy who wears bilateral hearing aids. The left hearing aid is making an annoying whistling sound that the child cannot hear. What intervention is the most appropriate nursing action? a. Ignore the sound. b. Suggest he reinsert the hearing aid. c. Ask him to reverse the hearing aids in his ears. d. Suggest he raise the volume of the hearing aid

ANS: B The whistling sound is acoustic feedback. The nurse should have the child remove the hearing aid and reinsert it, making sure no hair is caught between the ear mold and the ear canal. Ignoring the sound or suggesting he raise the volume of the hearing aid would be annoying to others. The hearing aids are molded specifically for each ear.

A parent tells the nurse that 80% of children with the same type of leukemia as his son's have a 5-year survival. He believes that because another child on the same protocol as his son has just died, his son now has a better chance of success. What is the best response by the nurse? a. "It is sad for the other family but good news for your child." b. "Each child has an 80% likelihood of 5-year survival." c. "The data suggest that 20% of the children in the clinic will die. There are still many hurdles for your son." d. "You should avoid the grieving family because you will be benefiting from their loss."

ANS: B This is a common misconception for parents. The success data are based on numerous factors, including the effectiveness of the protocol and the child's response. These are aggregate data that apply to each child and do not depend on the success or failure in other children. The failure of one child in a protocol does not improve the success rate for other children. Although the son does face more hurdles, these are aggregate data, not specific to the clinic. It may be difficult for this family to be supportive given their concerns about their child. Families usually form support groups in pediatric oncology settings, and support during bereavement is common.

The nurse is teaching the parent of a preschool child how to administer the child's insulin injection. The child will be receiving 2 units of regular insulin and 12 units of NPH insulin every morning. What should the parent be taught? a. Draw the insulin in separate syringes. b. Draw the regular insulin first and then the NPH into the same syringe. c. Draw the NPH insulin first and then the regular into the same syringe. d. Check blood sugar first, and if below 120, hold the regular insulin and give the NPH.

ANS: B To obtain maximum benefit from mixing insulins, the recommended practice is to (1) inject the measured amount of air (equivalent to the dosage) into the long-acting insulin; (2) inject the measured amount of air into the rapid-acting (clear) insulin and, without removing the needle; (3) withdraw the clear insulin; and (4) insert the needle (already containing the clear insulin) into the long-acting (cloudy) insulin and then withdraw the desired amount. The blood sugar may be checked before giving the insulin, but the prescribed dose should not be withheld if the blood sugar is 120.

A father calls the clinic because he found his young daughter squirting Visine eyedrops into her mouth. What is the most appropriate nursing action? a. Reassure the father that Visine is harmless. b. Direct him to seek immediate medical treatment. c. Recommend inducing vomiting with ipecac. d. Advise him to dilute Visine by giving his daughter several glasses of water to drink.

ANS: B Visine is a sympathomimetic and if ingested may cause serious consequences. Medical treatment is necessary. Inducing vomiting is no longer recommended for ingestions. Dilution will not decrease risk.

The school nurse practitioner is consulted by a fifth-grade teacher about a student who has become increasingly inattentive and hyperactive in the classroom. The nurse notes that the child's weight has changed from the 50th percentile to the 30th percentile. The nurse is concerned about possible hyperthyroidism. What additional sign or symptom should the nurse anticipate? a. Skin that is cool and dry b. Blurred vision and loss of acuity c. Running and being active during recess d. Decreased appetite and food intake

ANS: B Visual disturbances such as loss of visual acuity and blurred vision are associated with hyperthyroidism. They may occur before the actual onset of other symptoms. The child's skin is usually warm, flushed, and moist. Although the signs of hyperthyroidism include excessive motion, irritability, hyperactivity, short attention span, and emotional lability, these children are easily fatigued and require frequent rest periods. Children with hyperthyroidism have increased food intake. Even with voracious appetites, weight loss occurs.

A child with hypoparathyroidism is receiving vitamin D therapy. The parents should be advised to watch for which signs or symptoms of vitamin D toxicity? a. Headache and seizures b. Weakness and lassitude c. Anorexia and insomnia d. Physical restlessness, voracious appetite without weight gain

ANS: B Vitamin D toxicity can be a serious consequence of therapy. Parents are advised to watch for weakness, fatigue, lassitude, headache, nausea, vomiting, and diarrhea. Renal impairment is manifested through polyuria, polydipsia, and nocturia. Headaches may be a sign of vitamin D toxicity, but seizures are not. Anorexia and insomnia are not characteristic of vitamin D toxicity. Physical restlessness and a voracious appetite with weight loss are manifestations of hyperthyroidism.

The nurse is preparing to admit a 9-year-old child with syndrome of inappropriate antidiuretic hormone (SIADH). What interventions should the nurse include in the child's care plan? (Select all that apply.) a. Provide a low-sodium, low-fat diet. b. Initiate seizure precautions. c. Weigh daily at the same time each day. d. Encourage intake of 1 l of fluid per day. e. Measure intake and output hourly.

ANS: B, C, E Nursing care of the child with SIADH includes placing the child on seizure precautions, obtaining a daily weight at the same time each day, and accurately measuring the child's intake and output. The nurse does not need to provide a low-sodium, low-fat diet because there are no diet restrictions. The child would be on fluid precautions to avoid fluid overload, so 1 l of fluid would not be encouraged.

When teaching injury prevention during the school-age years, what should the nurse include? a. Teach children about the need to fear strangers. b. Teach basic rules of water safety. c. Avoid letting children cook in microwave ovens. d. Caution children against engaging in competitive sports.

ANS: B Water safety instruction is an important component of injury prevention at this age. The child should be taught to swim, select safe and supervised places to swim, swim with a companion, check sufficient water depth for diving, and use an approved flotation device. Teach stranger safety, not fear of strangers. This includes telling the child not to go with strangers, not to wear personalized clothing in public places, to tell parents if anyone makes child feel uncomfortable, and to say no in uncomfortable situations. Teach the child safe cooking. Caution against engaging in dangerous sports such as jumping on trampolines.

The nurse is administering an intravenous chemotherapeutic agent to a child with leukemia. The child suddenly begins to wheeze and have severe urticaria. What nursing action is most appropriate to initiate? a. Recheck the rate of drug infusion. b. Stop the drug infusion immediately. c. Observe the child closely for next 10 minutes. d. Explain to the child that this is an expected side effect.

ANS: B When an allergic reaction is suspected, the drug is immediately discontinued. Any drug in the line should be withdrawn, and a normal saline infusion begun to keep the line open. The intravenous infusion is stopped to minimize the amount of drug that infuses. The infusion rate can be confirmed at a later time. Observation of the child for 10 minutes is essential, but it is done after the infusion is stopped. These signs are indicative of an allergic reaction, not an expected response.

The mother of an infant tells the nurse that sometimes there is a whitish "glow" in the pupil of his eye. The nurse should suspect which condition? a. Brain tumor b. Retinoblastoma c. Neuroblastoma d. Rhabdomyosarcoma

ANS: B When the nurse examines the eye, the light will reflect off of the tumor, giving the eye a whitish appearance. This is called a cat's eye reflex. Brain tumors are not usually visible. Neuroblastoma usually arises from the adrenal medulla and sympathetic nervous system. The most common presentation sites are in the abdomen, head, neck, or pelvis. Supraorbital ecchymosis may be present with distant metastasis. Rhabdomyosarcoma is a soft tissue tumor that derives from skeletal muscle undifferentiated cells.

The school nurse recognizes that children respond to stress by using which tactics? (Select all that apply.) a. Passivity b. Delinquency c. Daydreaming d. Delaying tactics e. Becoming outgoing

ANS: B, C, D Children respond to stress by using coping mechanisms that include internalizing symptoms such as withdrawal, delaying tactics, and daydreaming, along with externalizing symptoms such as aggression and delinquency.

What are favorable prognostic criteria for acute lymphoblastic leukemia? (Select all that apply.) a. Male gender b. CALLA positive c. Early pre-B cell d. 2 to 10 years of age e. Leukocyte count ?7?50,000/mm3

ANS: B, C, D Favorable prognostic criteria for acute lymphoblastic leukemia include CALLA positive, early pre-B cell, and age 2 to 10 years. Leukocyte count less, not greater, than 50,000/mm3 and female, not male, gender are favorable prognostic criteria.

The nurse is planning to use an interpreter with a non-English-speaking family. What should the nurse plan with regard to the use of an interpreter? (Select all that apply.) a. Use a family member. b. The nurse should speak slowly. c. Use an interpreter familiar with the family's culture. d. The nurse should speak only a few sentences at a time. e. The nurse should speak to the interpreter during interactions.

ANS: B, C, D When parents who do not speak English are informed of their child's chronic illness, interpreters familiar with both their culture and language should be used. The nurse should speak slowly and only use a few sentences at a time. Children, family members, and friends of the family should not be used as translators because their presence may prevent parents from openly discussing the issues. The nurse should speak to the family, not the interpreter.

What guidelines should the nurse follow when handling chemotherapeutic agents? (Select all that apply.) a. Use clean technique. b. Prepare medications in a safety cabinet. c. Wear gloves designed for handling chemotherapy. d. Wear face and eye protection when splashing is possible. e. Discard gloves and protective clothing in a special container.

ANS: B, C, D, E Safe handling of chemotherapeutic agents includes preparing medications in a safety cabinet, wearing gloves designed for handling chemotherapy, wearing face and eye protection when splashing is possible, and discarding gloves and protective clothing in a special container. Aseptic, not clean, technique should be used.

The nurse is teaching parents the signs of a hearing impairment in a child. What should the nurse include as signs? (Select all that apply.) a. Outgoing behavior b. Yelling to express pleasure c. Asking to have statements repeated d. Foot stamping for vibratory sensation e. Failure to develop intelligible speech by age 24 months

ANS: B, C, D, E Signs of a hearing impairment in a child include yelling to express pleasure, asking to have statements repeated, foot stamping for vibratory sensation, and failure to develop intelligible speech by age 24 months. The child's behavior is shy, not outgoing.

The clinic nurse is assessing a child with a heavy ascariasis lumbricoides (common roundworm) infection. Which assessment findings should the nurse expect? (Select all that apply.) a. Anemia b. Anorexia c. Irritability d. Intestinal colic e. Enlarged abdomen

ANS: B, C, D, E The assessment findings in a heavy ascariasis lumbricoides infection include anorexia, irritability, intestinal colic, and an enlarged abdomen. Anemia is seen in hookworm infections but not ascariasis.

The clinic nurse is assessing a child with bacterial conjunctivitis (pink eye). Which assessment findings should the nurse expect? (Select all that apply.) a. Itching b. Swollen eyelids c. Inflamed conjunctiva d. Purulent eye drainage e. Crusting of eyelids in the morning

ANS: B, C, D, E The assessment findings in bacterial conjunctivitis include swollen eyelids, inflamed conjunctiva, purulent eye drainage, and crusting of eyelids in the morning. Itching is seen with allergic conjunctivitis but not with bacterial conjunctivitis.

An infant was brought to the emergency department (ED) after falling from a high chair, sustaining a basilar skull fracture. Which concerning assessment findings does the nurse expect? Clear drainage from the ear Pupils are unequal and sluggish Bleeding from the fractured area Intermittent confusion and lethargy

Clear drainage from the ear A basilar skull fracture is characterized with a patient who exhibits raccoon eyes, otorrhea, and hemotympanum.

What risk factors can cause a sensorineural hearing impairment in an infant? (Select all that apply.) a. Cat scratch disease b. Bacterial meningitis c. Childhood case of measles d. Childhood case of chicken pox e. Administration of aminoglycosides for more than 5 days

ANS: B, C, E Risk criteria for sensorineural hearing impairment in infants include bacterial meningitis; a case of measles; and administration of ototoxic medications (e.g., gentamicin, tobramycin, kanamycin, streptomycin), including but not limited to the aminoglycosides, for more than 5 days. Cat scratch disease and a childhood case of chicken pox are not risk factors that can cause a sensorineural hearing impairment.

The parent of a child with a chronic illness tells the nurse, "I feel so hopeless in this situation." The nurse should take which actions to foster hopefulness for the family? (Select all that apply.) a. Avoid topics that are lighthearted. b. Convey a personal interest in the child. c. Be honest when reporting on the child's condition. d. Do not initiate any playful interaction with the child. e. Demonstrate competence and gentleness when delivering care.

ANS: B, C, E To foster hopefulness, the nurse should convey a personal interest in the child, be honest when reporting on a child's condition, and demonstrate competence and gentleness when delivering care. The nurse should introduce conversations on neutral, non-disease-related, or less sensitive topics (discuss the child's favorite sports, tell stories). The nurse should be lighthearted and initiate or respond to teasing or other playful interactions with the child.

The nurse is caring for a child with retinoblastoma that was treated with an enucleation. What interventions should the nurse plan for care of an eye socket after enucleation? (Select all that apply.) a. Clean the prosthesis. b. Change the eye pad daily. c. Keep the opposite eye covered initially. d. Irrigate the socket daily with a prescribed solution. e. Apply a prescribed antibiotic ointment after irrigation.

ANS: B, D, E Care of the socket is minimal and easily accomplished. The wound itself is clean and has little or no drainage. If an antibiotic ointment is prescribed, it is applied in a thin line on the surface of the tissues of the socket. To cleanse the site, an irrigating solution may be ordered and is instilled daily or more frequently if necessary before application of the antibiotic ointment. The dressing consists of an eye pad changed daily. The prosthesis is not placed until the socket has healed. The opposite eye is not covered.

Characteristics of bullies include what? (Select all that apply.) a. Female b. Depressed c. Good peer relationships d. Poor academic performance e. Exposed to domestic violence

ANS: B, D, E Children who are bullies are likely to be male, depressed, have poor academic performance, be exposed to domestic violence, have poor peer relationships, and have poor communication with their parents.

The nurse is planning to admit a 10-year-old child with syndrome of inappropriate antidiuretic hormone (SIADH). What clinical manifestations should the nurse expect to observe in this child? (Select all that apply.) a. Polyuria b. Anorexia c. Polydipsia d. Irritability e. Stomach cramps

ANS: B, D, E Clinical signs of SIADH are directly related to fluid retention and hyponatremia. When cells in the brain are exposed to too much water as opposed to sodium, swelling occurs. When serum sodium levels are diminished to 120?9?mEq/l, affected children may display anorexia, nausea, vomiting, stomach cramps, irritability, and personality changes. Polyuria and polydipsia are manifestations of diabetes insipidus.

The nurse is teaching the family of a child with type 1 diabetes about insulin. What should the nurse include in the teaching session? (Select all that apply.) a. Unopened vials are good for 60 days. b. Diabetic supplies should not be left in a hot environment. c. Insulin can be placed in the freezer if not used every day. d. After it has been opened, insulin is good for up to 28 to 30 days. e. Insulin bottles that have been opened should be stored at room temperature or refrigerated.

ANS: B, D, E Insulin bottles that have been "opened" (i.e., the stopper has been punctured) should be stored at room temperature or refrigerated for up to 28 to 30 days. After 1 month, these vials should be discarded. Unopened vials should be refrigerated and are good until the expiration date on the label. Diabetic supplies should not be left in a hot environment. Insulin need not be refrigerated but should be maintained at a temperature between 15° and 29.5° C (59° and 85° F). Freezing renders insulin inactive.

The nurse is preparing to admit a 1-year-old child with pertussis (whooping cough). Which clinical manifestations of pertussis should the nurse expect to observe? (Select all that apply.) a. Earache b. Coryza c. Conjunctivitis d. Low-grade fever e. Dry hacking cough

ANS: B, D, E The clinical manifestations of pertussis include coryza, a low-grade fever, and a dry hacking cough. The child does not have an earache or conjunctivitis.

The nurse is preparing to admit a 5-year-old child who developed lesions of varicella (chickenpox) 3 days ago. Which clinical manifestations of varicella should the nurse expect to observe? (Select all that apply.) a. Nonpruritic rash b. Elevated temperature c. Discrete rose pink rash d. Vesicles surrounded by an erythematous base e. Centripetal rash in all three stages (papule, vesicle, and crust)

ANS: B, D, E The clinical manifestations of varicella include elevated temperature, vesicles surrounded by an erythematous base, and a centripetal rash in all three stages (papule, vesicle, and crust). The rash is pruritic, and a discrete pink rash is seen with exanthema subitum, not varicella.

The nurse is teaching parents the signs of a hearing impairment in infants. What should the nurse include as signs? (Select all that apply.) a. Lack of a fencing reflex b. Lack of a startle reflex to a loud sound c. Awakened by loud environmental noises d. Failure to localize a sound by 6 months of age e. Response to loud noises as opposed to the voice

ANS: B, D, E The fencing reflex is elicited when the infant is placed on his or her back; it does not indicate a hearing impairment. Awakening by a loud environmental noise is a normal response.

A child is receiving propylthiouracil for the treatment of hyperthyroidism (Graves disease). The parents and child should be taught to recognize and report which sign or symptom immediately? a. Fatigue b. Weight loss c. Fever, sore throat d. Upper respiratory tract infection

ANS: C Children being treated with propylthiouracil must be carefully monitored for the side effects of the drug. Parents must be alerted that sore throat and fever accompany the grave complication of leukopenia. These symptoms should be immediately reported. Fatigue and weight loss are manifestations of hyperthyroidism. Their presence may indicate that the drug is not effective but does not require immediate evaluation. Upper respiratory tract infections are most likely viral in origin and not a sign of leukopenia.

The nurse is discussing sexuality with the parents of an adolescent girl who has a moderate cognitive impairment. What factor should the nurse consider when dealing with this issue? a. Sterilization is recommended for any adolescent with cognitive impairment. b. Sexual drive and interest are very limited in individuals with cognitive impairment. c. Individuals with cognitive impairment need a well-defined, concrete code of sexual conduct. d. Sexual intercourse rarely occurs unless the individual with cognitive impairment is sexually abused.

ANS: C Adolescents with moderate cognitive impairment may be easily persuaded and lack judgment. A well-defined, concrete code of conduct with specific instructions for handling certain situations should be defined for the adolescent. Permanent contraception by sterilization presents moral and ethical issues and may have psychologic effects on the adolescent. It may be prohibited in some states. The adolescent needs to have practical sexual information regarding physical development and contraception. Cognitively impaired individuals may desire to marry and have families. The adolescent needs to be protected from individuals who may make intimate advances.

What type of chemotherapeutic agent alters the function of cells by replacing a hydrogen atom of a molecule? a. Plant alkaloids b. Antimetabolites c. Alkylating agents d. Antitumor antibiotics

ANS: C Alkylating agents replace a hydrogen atom with an alkyl group. The irreversible combination of alkyl groups with nucleotide chains, particularly deoxyribonucleic acid (DNA), causes unbalanced growth of unaffected cell constituents so that the cell eventually dies. Plant alkaloids arrest the cell in metaphase by binding to proteins needed for spindle formation. Antimetabolites resemble essential metabolic elements needed for growth but are different enough to block further DNA synthesis. Antitumor antibiotics are natural substances that interfere with cell division by reacting with DNA in such a way as to prevent further replication of DNA and transcription of ribonucleic acid (RNA).

The nurse is preparing a child for possible alopecia from chemotherapy. What information should the nurse include? a. Wearing hats or scarves is preferable to a wig. b. Expose head to sunlight to stimulate hair regrowth. c. Hair may have a slightly different color or texture when it regrows. d. Regrowth of hair usually begins 12 months after chemotherapy ends.

ANS: C Alopecia is a side effect of certain chemotherapeutic agents and cranial irradiation. When the hair regrows, it may be of a different color or texture. Children should choose the head covering they prefer. A wig should be selected similar to the child's own hairstyle and color before the hair loss. The head should be protected from sunlight to avoid sunburn. The hair usually grows back within 3 to 6 months after the cessation of treatment.

What condition is defined as reduced visual acuity in one eye despite appropriate optical correction? a. Myopia b. Hyperopia c. Amblyopia d. Astigmatism

ANS: C Amblyopia, or lazy eye, is reduced visual acuity in one eye. Amblyopia is usually caused by one eye not receiving sufficient stimulation. The resulting poor vision in the affected eye can be avoided with the treatment of the primary visual defect such as strabismus. Myopia, or nearsightedness, refers to the ability to see objects clearly at close range but not a distance. Hyperopia, or farsightedness, is the ability to see objects at a distance but not at close range. Astigmatism is unequal curvatures in refractive apparatus.

A child with cyanide poisoning has been admitted to the emergency department. What antidote does the nurse anticipate being prescribed for the child? a. Atropine b. Glucagon c. Amyl nitrate d. Naloxone (Narcan)

ANS: C Amyl nitrate is the antidote for cyanide poisoning. Atropine is an antidote for organophosphate poisoning, glucagon is an antidote for a beta-blocker poisoning, and naloxone (Narcan) is an antidote for an opioid poisoning.

What is a major premise of family-centered care? a. The child is the focus of all interventions. b. Nurses are the authorities in the child's care. c. Parents are the experts in caring for their child. d. Decisions are made for the family to reduce stress.

ANS: C As parents become increasingly responsible for their children, they are the experts. It is essential that the health care team recognize the family's expertise. In family-centered care, consistent attention is given to the effects of the child's chronic illness on all family members, not just the child. Nurses are adjuncts in the child's care. The nurse builds alliances with parents. Family members are involved in decision making about the child's physical care.

A female school-age child asks the school nurse, "How many pounds should I expect to gain in a year?" The nurse should give which response? a. "You will gain about 2.4 to 4.6 lb per year" b. "You will gain about 3.4 to 5.6 lb per year." c. "You will gain about 4.4 to 6.6 lb per year." d. "You will gain about 5.5 to 7.6 lb per year."

ANS: C Between the ages of 6 and 12 years, children will almost double in weight, increasing 2 to 3 kg (4.4 to 6.6 lb) per year.

What statement accurately describes physical development during the school-age years? a. The child's weight almost triples. b. Muscles become functionally mature. c. Boys and girls double strength and physical capabilities. d. Fat gradually increases, which contributes to children's heavier appearance.

ANS: C Boys and girls double both strength and physical capabilities. Their consistent refinement in coordination increases their poise and skill. In middle childhood, growth in height and weight occurs at a slower pace. Between the ages of 6 and 12 years, children grow 5 cm/yr and gain 3 kg/yr. Their weight will almost double. Although the strength increases, muscles are still functionally immature when compared with those of adolescents. This age group is more easily injured by overuse. Children take on a slimmer look with longer legs in middle childhood.

Chemotherapeutic agents are classified according to what feature? a. Side effects b. Effectiveness c. Mechanism of action d. Route of administration

ANS: C Chemotherapeutic agents are classified according to mechanism of action. For example, antimetabolites resemble essential metabolic elements needed for growth but are different enough to block further deoxyribonucleic acid (DNA) synthesis. Although the side effect profiles may be similar for drugs within a classification, they are not the basis for classification. Most chemotherapeutic regimens contain combinations of drugs. The effectiveness of any one drug is relative to the cancer type, combination therapy, and protocol for administration. The route of administration is determined by the pharmacodynamics and pharmacokinetics of each drug.

The parents of 9-year-old twin children tell the nurse, "They have filled up their bedroom with collections of rocks, shells, stamps, and bird nests." The nurse should recognize that this is which? a. Indicative of giftedness b. Indicative of typical twin behavior c. Characteristic of cognitive development at this age d. Characteristic of psychosocial development at this age

ANS: C Classification skills involve the ability to group objects according to the attributes they have in common. School-age children can place things in a sensible and logical order, group and sort, and hold a concept in their mind while they make decisions based on that concept. Individuals who are not twins engage in classification at this age. Psychosocial behavior at this age is described according to Erikson's stage of industry versus inferiority.

The nurse asks the mother of a child with a chronic illness many questions as part of the assessment. The mother answers several questions, then stops and says, "I don't know why you ask me all this. Who gets to know this information?" The nurse should respond in what manner? a. Determine why the mother is so suspicious. b. Determine what the mother does not want to tell. c. Explain who will have access to the information. d. Explain that everything is confidential and that no one else will know what is said.

ANS: C Communication with the family should not be invasive. The nurse needs to explain the importance of collecting the information, its applicability to the child's care, and who will have access to the information. The mother is not being suspicious and is not necessarily withholding important information. She has a right to understand how the information she provides will be used. The nurse will need to share, through both oral and written communication, clinically relevant information with other involved health professionals.

Calculate the absolute neutrophil count (ANC) for the following: WBC count of 5000 mm3; neutrophils (segs) of 10%; and nonsegmented neutrophils (bands) of 12%. a. 110/mm3 b. 500/mm3 c. 1100/mm3 d. 5000/mm3

ANS: C Determine the total percentage of neutrophils ("polys," or "segs," and "bands"). Multiply white blood cell (WBC) count by percentage of neutrophils. WBC = 1000/mm3, neutrophils = 7%, and nonsegmented neutrophils (bands) = 7% Step 1: 10% + 12% = 22% Step 2: 0.22 ´ 5000 = 1100/mm3 ANC

What is a common clinical manifestation of juvenile hypothyroidism? a. Insomnia b. Diarrhea c. Dry skin d. Rapid growth

ANS: C Dry skin, mental decline, and myxedematous skin changes are associated with juvenile hypothyroidism. Children with hypothyroidism often have sleepiness, constipation, and decelerated growth.

What statement best describes fear in school-age children? a. Increasing concerns about bodily safety overwhelm them. b. They should be encouraged to hide their fears to prevent ridicule by peers. c. Most of the new fears that trouble them are related to school and family. d. Children with numerous fears need continuous protective behavior by parents to eliminate these fears.

ANS: C During the school-age years, children experience a wide variety of fears, but new fears related predominantly to school and family bother children during this time. Parents and other persons involved with children should discuss children's fear with them individually or as a group activity. Sometimes school-age children hide their fears to avoid being teased. Hiding the fears does not end them and may lead to phobias.

Although a 14-month-old girl received a shock from an electrical outlet recently, her parent finds her about to place a paper clip in another outlet. Which is the best interpretation of this behavior? a. Her cognitive development is delayed. b. This is typical behavior because toddlers are not very developed. c. This is typical behavior because of toddlers' inability to transfer remembering to new situations. d. This is not typical behavior because toddlers should know better than to repeat an act that caused pain.

ANS: C During the tertiary circular reactions stage, children have only a rudimentary sense of the classification of objects. The appearance of an object denotes its function for these children. The slot of an outlet is for putting things into. This is typical behavior for a toddler, who is only somewhat aware of a causal relation between events. Her cognitive development is appropriate for her age.

Which serious reaction should the nurse be alert for when administering vaccines? a. Fever b. Skin irritation c. Allergic reaction d. Pain at injection site

ANS: C Each vaccine administration carries the risk of an allergic reaction. The nurse must be prepared to intervene if the child demonstrates signs of a severe reaction. Mild febrile reactions do occur after administration. The nurse includes management of fever in the parent teaching. Local skin irritation may occur at the injection site after administration. Parents are informed that this is expected. The injection can be painful. The nurse can minimize the discomfort with topical analgesics and nonpharmacologic measures.

The parents of a child who has just been diagnosed with type 1 diabetes ask about exercise. What effect does exercise have on a type 1 diabetic? a. Exercise increases blood glucose. b. Extra insulin is required during exercise. c. Additional snacks are needed before exercise. d. Excessive physical activity should be restricted.

ANS: C Exercise lowers blood glucose levels, decreasing the need for insulin. Extra snacks are provided to maintain the blood glucose levels. Exercise is encouraged and not restricted unless indicated by other health conditions.

Nursing care of the child with myelosuppression from leukemia or chemotherapeutic agents should include which therapeutic intervention? a. Restrict oral fluids. b. Institute strict isolation. c. Use good hand-washing technique. d. Give immunizations appropriate for age.

ANS: C Good hand washing minimizes the exposure to infectious organisms and decreases the chance of infection spread. Oral fluids are encouraged if the child is able to drink. If possible, the intravenous route is not used because of the increased risk of infection from parenteral fluid administration. Strict isolation is not indicated. When the child is immunocompromised, the vaccines are not effective. If necessary, the appropriate immunoglobulin is administered.

What nursing care should be included for a child diagnosed with syndrome of inappropriate antidiuretic hormone (SIADH)? a. Maintain the child NPO (nothing by mouth). b. Turn the child frequently. c. Restrict fluids. d. Encourage fluids.

ANS: C Increased secretion of ADH causes the kidney to reabsorb water, which increases fluid volume and decreases serum osmolarity with a progressive reduction in sodium concentration. The immediate management of the child is to restrict fluids but not food. Frequently turning the child is not necessary unless the child is unresponsive. Encouraging fluids will worsen the child's condition.

A 2-week-old infant with Down syndrome is being seen in the clinic. His mother tells the nurse that he is difficult to hold, that "he's like a rag doll. He doesn't cuddle up to me like my other babies did." What is the nurse's best interpretation of this lack of clinging or molding? a. Sign of detachment and rejection b. Indicative of maternal deprivation c. A physical characteristic of Down syndrome d. Suggestive of autism associated with Down syndrome

ANS: C Infants with Down syndrome have hypotonicity of muscles and hyperextensibility of joints, which complicate positioning. The limp, flaccid extremities resemble the posture of a rag doll. Holding the infant is difficult and cumbersome, and parents may feel that they are inadequate. A lack of clinging or molding is characteristic of Down syndrome, not detachment. There is no evidence of maternal deprivation. Autism is not associated with Down syndrome, and it would not be evident at 2 weeks of age.

What description identifies the pathophysiology of leukemia? a. Increased blood viscosity b. Abnormal stimulation of the first stage of coagulation process c. Unrestricted proliferation of immature white blood cells (WBCs) d. Thrombocytopenia from an excessive destruction of platelets

ANS: C Leukemia is a group of malignant disorders of the bone marrow and lymphatic system. It is defined as an unrestricted proliferation of immature WBCs in the blood-forming tissues of the body. Increased blood viscosity may result secondary to the increased number of WBCs. The coagulation process is unaffected by leukemia. Thrombocytopenia may occur secondary to the overproduction of WBCs in the bone marrow.

The nurse should know what about Lyme disease? a. Very difficult to prevent b. Easily treated with oral antibiotics in stages 1, 2, and 3 c. Caused by a spirochete that enters the skin through a tick bite d. Common in geographic areas where the soil contains the mycotic spores that cause the disease

ANS: C Lyme disease is caused by Borrelia burgdorferi, a spirochete spread by ticks. The early characteristic rash is erythema migrans. Tick bites should be avoided by entering tick-infested areas with caution. Light-colored clothing should be worn to identify ticks easily. Long-sleeve shirts and long pants tucked into socks should be the attire. Early treatment of erythema migrans (stage 1) can prevent the development of Lyme disease. Lyme disease is caused by a spirochete, not mycotic spores.

The school reviewed the pediculosis capitis (head lice) policy and removed the "no nit" requirement. The nurse explains that now, when a child is found to have nits, the parents must do which before the child can return to school? a. No treatment is necessary with the policy change. b. Shampoo and then trim the child's hair to prevent reinfestation. c. The child can remain in school with treatment done at home. d. Treat the child with a shampoo to treat lice and comb with a fine-tooth comb every day until nits are eliminated.

ANS: C Many children have missed significant amounts of school time with "no nit" policies. The child should be appropriately treated with a pediculicide and a fine-tooth comb. The environment needs to be treated to prevent reinfestation. The treatment with the pediculicide will kill the lice and leave nit casings. Cutting the child's hair is not recommended; lice infest short hair as well as long. With a "no nit" policy, treating the child with a shampoo to treat lice and combing the hair with a fine-tooth comb every day until nits are eliminated is the correct treatment. The policy change recognizes that most nits do not become lice.

What statement related to clinical trials developed for pediatric cancers is most accurate? a. Are accessible only in major pediatric centers b. Do not require consent for standard therapy c. Provide the best available therapy compared with an expected improvement d. Are standardized to provide the same treatment to all children with the disease

ANS: C Most clinical trials have a control group in which the patients receive the best available therapy currently known. The experimental group(s) receives treatment that is thought to be even better. The protocol outlines the therapy plan. Protocols are developed for many pediatric cancers. They can be accessed by pediatric oncologists throughout the United States. Consent is always required in treatment of children, especially for research protocols. The protocol is designed to optimize therapy for children based on disease type and stage.

What is characteristic of dishonest behavior in children ages 8 to 10 years? a. Cheating during games is now more common. b. Stealing can occur because their sense of property rights is limited. c. Lying is used to meet expectations set by others that they have been unable to attain. d. Dishonesty results from the inability to distinguish between fact and fantasy.

ANS: C Older school-age children may lie to meet expectations set by others to which they have been unable to measure up. Cheating usually becomes less frequent as the child matures. Young children may lack a sense of property rights; older children may steal to supplement an inadequate allowance, or it may be an indication of serious problems. In this age group, children are able to distinguish between fact and fantasy.

Autism is a complex developmental disorder. The diagnostic criteria for autism include delayed or abnormal functioning in which area with onset before age 3 years? a. Parallel play b. Gross motor development c. Ability to maintain eye contact d. Growth below the fifth percentile

ANS: C One hallmark of autism spectrum disorders is the child's inability to maintain eye contact with another person. Parallel play is play typical of toddlers and is usually not affected. Social, not gross motor, development is affected by autism. Physical growth and development are not usually affected.

Treatment for herpes simplex virus (type 1 or 2) includes which? a. Corticosteroids b. Oral griseofulvin c. Oral antiviral agent d. Topical or systemic antibiotic

ANS: C Oral antiviral agents are effective for viral infections such as herpes simplex. Corticosteroids, antibiotics, and griseofulvin (an antifungal agent) are not effective for viral infections.

Peripheral precocious puberty (PPP) differs from central precocious puberty (CPP) in which manner? a. PPP results from a central nervous system (CNS) insult. b. PPP occurs more frequently in girls. c. PPP may be viewed as a variation in sexual development. d. PPP results from hormonal stimulation of the hypothalamic gonadotropin-releasing hormone (Gn-RH).

ANS: C PPP may be viewed as a variation in sexual development. PPP results from hormone stimulation other than the hypothalamic Gn-RH. Isolated manifestations of secondary sexual development occur. PPP can be missed if these changes are viewed as variations in pubertal onset. CPP results from CNS insult, occurs more frequently in girls, and results from hormonal stimulation of the hypothalamic Gn-RH.

The nurse is teaching feeding strategies to a parent of a 12-month-old infant with Down syndrome. What statement made by the parent indicates a need for further teaching? a. "If the food is thrust out, I will reefed it." b. "I will use a small, long, straight-handled spoon." c. "I will place the food on the top of the tongue." d. "I know the tongue thrust doesn't indicate a refusal of the food."

ANS: C Parents of a child with Down syndrome need to know that the tongue thrust does not indicate refusal to feed but is a physiologic response. Parents are advised to use a small but long, straight-handled spoon to push the food toward the back and side of the mouth. If food is thrust out, it should be refed. If the parent indicates placing the food on the tongue, further teaching is needed.

What intervention is most appropriate for fostering the development of a school-age child with disabilities associated with cerebral palsy? a. Provide sensory experiences. b. Help develop abstract thinking. c. Encourage socialization with peers. d. Give choices to allow for feeling of control

ANS: C Peer interaction is especially important in relation to cognitive development, social development, and maturation. Cognitive development is facilitated by interaction with peers, parents, and teachers. The identification with those outside the family helps the child fulfill the striving for independence. Sensory experiences are beneficial, especially for younger children. School-age children are too young for abstract thinking. Giving school-age children choices is always an important intervention. Providing structured choices allows for a feeling of control.

A parent asks about whether a 7-year-old child is able to care for a dog. Based on the child's age, what does the nurse suggest? a. Caring for an animal requires more maturity than the average 7-year-old possesses. b. This will help the parent identify the child's weaknesses. c. A dog can help the child develop confidence and emotional health. d. Cats are better pets for school-age children.

ANS: C Pets have been observed to influence a child's self-esteem. They can have a positive effect on physical and emotional health and can teach children the importance of nurturing and nonverbal communication. Most 7-year-old children are capable of caring for a pet with supervision. Caring for a pet should be a positive experience. It should not be used to identify weaknesses. The pet chosen does not matter as much as the child's being responsible for a pet.

What is descriptive of the play of school-age children? a. They like to invent games, making up the rules as they go. b. Individuality in play is better tolerated than at earlier ages. c. Knowing the rules of a game gives an important sense of belonging. d. Team play helps children learn the universal importance of competition and winning.

ANS: C Play involves increased physical skill, intellectual ability, and fantasy. Children form groups and cliques and develop a sense of belonging to a team or club. At this age, children begin to see the need for rules. Conformity and ritual permeate their play. Their games have fixed and unvarying rules, which may be bizarre and extraordinarily rigid. With team play, children learn about competition and the importance of winning, an attribute highly valued in the United States but not in all cultures.

What behavior seen in children should be addressed by the nurse who is providing care to a child with a chronic illness? a. An infant who is uncooperative b. A toddler who expresses loneliness c. A preschooler who refuses to participate in self-care d. An adolescent who is showing independence

ANS: C Preschoolers thrive on being independent and are in the phase of gaining autonomy, so they want to perform as many self-care tasks as possible. If a preschooler is refusing to participate in self-care activities, then the home health nurse should address this. Infants are uncooperative by nature, and toddlers do not understand the concept of loneliness, so these are not observations that would need to be addressed. Adolescents are always striving for independence, so this is a normal observation; if the adolescent were becoming more dependent on family, it might require intervention.

A 5-year-old child has bilateral eye patches in place after surgery yesterday morning. Today he can be out of bed. What nursing intervention is most important at this time? a. Speak to him when entering the room. b. Allow him to assist in feeding himself. c. Orient him to his immediate surroundings. d. Reassure him and allow his parents to stay with him.

ANS: C Safety is the priority concern. Because he can now be out of bed, it is imperative that he knows about his physical surroundings. Speaking to the child is a component of nursing care that is expected with all clients unless contraindicated. Unless additional impairments are present, his meal tray should be set up, and he should be able to feed himself. Reassuring him and allowing his parents to stay with him are essential parts of nursing care for all children.

What does the nurse understand about caloric needs for school-age children? a. The caloric needs for the school-age children are the same as for other age groups. b. The caloric needs for school-age children are more than they were in the preschool years. c. The caloric needs for school-age children are lower than they were in the preschool years. d. The caloric needs for school-age children are greater than they will be in the adolescent years.

ANS: C School-age children do not need to be fed as carefully, as promptly, or as frequently as before. Caloric needs are lower than they were in the preschool years and lower than they will be during the coming adolescent growth spurt.

What statement best describes the relationship school-age children have with their families? a. Ready to reject parental controls b. Desire to spend equal time with family and peers c. Need and want restrictions placed on their behavior by the family d. Peer group replaces the family as the primary influence in setting standards of behavior and rules

ANS: C School-age children need and want restrictions placed on their behavior, and they are not prepared to cope with all the problems of their expanding environment. Although increased independence is the goal of middle childhood, they feel more secure knowing that an authority figure can implement controls and restriction. In the middle school years, children prefer peer group activities to family activities and want to spend more time in the company of peers. Family values usually take precedence over peer value systems.

What has the most significant impact on the socialization of school-age children? Parents Teachers Classmates The child's self-concept

Classmates Rationale Classmates have a significant impact on the socialization of children. Teachers and parents are important, but classmates have the most significant impact on the socialization of school-age children; peer relationships become increasingly important as the child grows older. The school-age child's self-concept is important but does not have the most significant impact on socialization.

The nurse is teaching a class on nutrition to a group of parents of 10- and 11-year-old children. What statement by one of the parents indicates a correct understanding of the teaching? a. "My child does not need to eat a variety of foods, just his favorite food groups." b. "My child can add salt and sugar to foods to make them taste better." c. "I will serve foods that are low in saturated fat and cholesterol." d. "I will continue to serve red meat three times per week for extra iron."

ANS: C School-age children should be eating foods that are low in saturated fat and cholesterol to prevent long-term consequences. The child's diet should include a variety of foods, include moderate amounts of extra salt and sugar, emphasize consumption of lean protein (chicken and pork), and limit red meat.

A child with corrosive poisoning is being admitted to the emergency department. What clinical manifestation does the nurse expect to assess on this child? a. Nausea and vomiting b. Alterations in sensorium, such as lethargy c. Severe burning pain in the mouth, throat, and stomach d. Respiratory symptoms of acute pulmonary involvement

ANS: C Severe burning pain in the mouth, throat, and stomach is a clinical manifestation of corrosive poisoning. Nausea and vomiting; alterations in sensorium, such as lethargy; and respiratory symptoms of acute pulmonary involvement are clinical manifestations of hydrocarbon poisoning.

The American Association on Intellectual and Developmental Disabilities (AAIDD), formerly the American Association on Cognitive Impairment, classifies cognitive impairment based on what parameter? a. Age of onset b. Subaverage intelligence c. Adaptive skill domains d. Causative factors for cognitive impairment

ANS: C The AAIDD has categorized cognitive impairment into adaptive skill domains. The child must demonstrate functional impairment in at least two of the following adaptive skill domains: communication, self-care, home living, social skills, use of community resources, self-direction, health and safety, functional academics, leisure, and work. Age of onset before 18 years is part of the former criteria. Low intelligence quotient (IQ) alone is not the sole criterion for cognitive impairment. Etiology is not part of the classification.

What intervention is most appropriate to facilitate social development of a child with a cognitive impairment? a. Provide age-appropriate toys and play activities. b. Avoid exposure to strangers who may not understand cognitive development. c. Provide peer experiences, such as infant stimulation and preschool programs. d. Emphasize mastery of physical skills because they are delayed more often than verbal skills.

ANS: C The acquisition of social skills is a complex task. Initially, an infant stimulation program should be used. Children of all ages need peer relationships. Parents should enroll the child in preschool. When older, they should have peer experiences similar to those of other children such as group outings, Boy and Girl Scouts, and Special Olympics. Providing age-appropriate toys and play activities is important, but peer interactions facilitate social development. Parents should expose the child to individuals who do not know the child. This enables the child to practice social skills. Verbal skills are delayed more often than physical skills.

A child with diazepam (Valium) poisoning has been admitted to the emergency department. What antidote does the nurse anticipate being prescribed? a. Succimer (Chemet) b. EDTA (Versenate) c. Flumazenil (Romazicon) d. Octreotide acetate (Sandostatin)

ANS: C The antidote for diazepam (Valium) poisoning is flumazenil (Romazicon). Succimer (Chemet) and EDTA (Versenate) are antidotes for heavy metal poisoning. Octreotide acetate (Sandostatin) is an antidote for sulfonylurea poisoning.

The school nurse is teaching male school-age children about the average age of puberty. What is the average age of puberty for boys? a. 12 years b. 13 years c. 14 years d. 15 years

ANS: C The average age of puberty is 14 years in boys. Boys experience little sexual maturation during preadolescence.

What action should the school nurse take for a child who has a hematoma (black eye) with no hemorrhage into the anterior chamber? a. Apply a warm moist pack. b. Have the child keep the eyes open. c. Apply ice for the first 24 hours. d. Refer to an ophthalmologist immediately.

ANS: C The care for a hematoma eye injury with no hemorrhage into the anterior chamber is to apply ice for the first 24 hours. A warm moist pack should not be applied, and the child should keep the eyes closed. Referral to an ophthalmologist is recommended if hyphema (hemorrhage into the anterior chamber) is present.

The nurse is counseling a pregnant 35-year-old woman about estimated risk of Down syndrome. What is the estimated risk for a woman who is 35 years of age? a. One in 1200 b. One in 900 c. One in 350 d. One in 100

ANS: C The estimated risk of Down syndrome for a 35-year-old woman is one in 350. One in 1200 is the risk for a 25-year-old woman, one in 900 is the risk for a 30-year-old woman, and one in 100 is the risk for a 40-year-old woman.

An adolescent with long-term, complex health care needs will soon be discharged from the hospital. The nurse case manager has been assigned to the teen and family. The adolescent's care involves physical therapy, occupational therapy, and speech therapy in addition to medical and nursing care. Who should be the decision maker in the adolescent's care? a. Adolescent b. Nurse case manager c. Adolescent and family d. Multidisciplinary health care team

ANS: C The extent to which children are involved in their own care and decision making depends on many factors, including the child's developmental age, level of interest, physical ability, and parental support. If the adolescent is developmentally age appropriate, then decision making should be the responsibility of child and family. Family needs to be involved because they will be caring for the adolescent in the home. Health care providers have necessary input into the care of the child, but ultimate decision making rests with the adolescent and family.

A father calls the emergency department nurse saying that his daughter's eyes burn after getting some dishwasher detergent in them. The nurse recommends that the child be seen in the emergency department or by an ophthalmologist. The nurse also should recommend which action before the child is transported? a. Keep the eyes closed. b. Apply cold compresses. c. Irrigate the eyes copiously with tap water for 20 minutes. d. Prepare a normal saline solution (salt and water) and irrigate the eyes for 20 minutes.

ANS: C The first action is to flush the eyes with clean tap water. This will rinse the detergent from the eyes. Keeping the eyes closed and applying cold compresses may allow the detergent to do further harm to the eyes during transport. Normal saline is not necessary. The delay can allow the detergent to cause continued injury to the eyes.

A 20-kg (44-lb) child in ketoacidosis is admitted to the pediatric intensive care unit. What order should the nurse not implement until clarified with the physician? a. Weigh on admission and daily. b. Replace fluid volume deficit over 48 hours. c. Begin intravenous line with D5 0.45% normal saline with 20 mEq of potassium chloride. d. Give intravenous regular insulin 2 units/kg/hr after initial rehydration bolus.

ANS: C The initial hydrating solution is 0.9% normal saline. Potassium is not given until the child is voiding 25 ml/hr, demonstrating adequate renal function. After initial rehydration and insulin administration, then potassium is given. Dextrose is not given until blood glucose levels are between 250 and 300 mg/dl. An accurate, current weight is essential for determination of the amount of fluid loss and as a basis for medication dosage. Replacing fluid volume deficit over 48 hours is the current recommendation in diabetic ketoacidosis in children. Cerebral edema is a risk of more rapid administration. Intravenous regular insulin 2 units/kg/hr after initial rehydration bolus is the recommended insulin administration for a child of this weight. Only regular insulin can be given intravenously, and it is given after initial fluid volume expansion

What is an important nursing consideration when a child is hospitalized for chelation therapy to treat lead poisoning? a. Maintain bed rest. b. Maintain isolation precautions. c. Keep an accurate record of intake and output. d. Institute measures to prevent skeletal fracture.

ANS: C The iron chelates are excreted though the kidneys. Adequate hydration is essential. Periodic measurement of renal function is done. Bed rest is not necessary. Often the chelation therapy is done on an outpatient basis. Chelation therapy is not infectious or dangerous. Isolation is not indicated. Skeletal weakness does not result from high levels of lead.

When communicating with other professionals about a child with a chronic illness, what is important for nurses to do? a. Ask others what they want to know. b. Share everything known about the family. c. Restrict communication to clinically relevant information. d. Recognize that confidentiality is not possible in home care.

ANS: C The nurse needs to share, through both oral and written communication, clinically relevant information with other involved health professionals. Asking others what they want to know and sharing everything known about the family are inappropriate measures. Patients have a right to confidentiality. Confidentiality permits the disclosure of information to other health professionals on a need-to-know basis.

The nurse observes that a seriously ill child passively accepts all painful procedures. The nurse should recognize that this is most likely an indication that the child is experiencing what emotional response? a. Hopefulness b. Chronic sorrow c. Belief that procedures are a deserved punishment d. Understanding that procedures indicate impending death

ANS: C The nurse should be particularly alert to a child who withdraws and passively accepts all painful procedures. This child may believe that such acts are inflicted as deserved punishment for being less worthy. A child who is hopeful is mobilized into goal-directed actions. This child would actively participate in care. Chronic sorrow is the feeling of sorrow and loss that recurs in waves over time. It is usually evident in the parents, not in the child. The seriously ill child would actively participate in care. Nursing interventions should be used to minimize the pain.

An adolescent diabetic is admitted to the emergency department for treatment of hyperglycemia and pneumonia. What are characteristics of diabetic hyperglycemia? a. Cold, clammy skin and lethargy b. Hunger and hypertension c. Thirst, being flushed, and fruity breath d. Disorientation and pallor

ANS: C The signs of hyperglycemia are thirst, being flushed, and fruity breath. The skin is not cold or clammy, and there is not hunger and hypertension. Disorientation and pallor are signs of hypoglycemia.

The nurse is explaining about the developmental sequence in children's capacity to conserve matter to a group of parents. What type of matter is last in the sequence for a child to develop? a. Mass b. Length c. Volume d. Numbers

ANS: C There is a developmental sequence in children's capacity to conserve matter. Children usually grasp conservation of numbers (ages 5 to 6 years) before conservation of substance. Conservation of liquids, mass, and length usually is accomplished at about ages 6 to 7 years, conservation of weight sometime later (ages 9 to 10 years), and conservation of volume or displacement last (ages 9 to 12 years).

A newborn assessment shows a separated sagittal suture, oblique palpebral fissures, a depressed nasal bridge, a protruding tongue, and transverse palmar creases. These findings are most suggestive of which condition? a. Microcephaly b. Cerebral palsy c. Down syndrome d. Fragile X syndrome

ANS: C These are characteristics associated with Down syndrome. An infant with microcephaly has a small head. Cerebral palsy is a diagnosis not usually made at birth; no characteristic physical signs are present. The infant with fragile X syndrome has increased head circumference; long, wide, or protruding ears; a long, narrow face with a prominent jaw; hypotonia; and a high-arched palate.

The nurse is teaching parents of a child with cataracts about the upcoming treatment. The nurse should give the parents what information about the treatment of cataracts? a. "The treatment may require more than one surgery." b. "It is corrected with biconcave lenses that focus rays on the retina." c. "Cataracts require surgery to remove the cloudy lens and replace it." d. "Treatment is with a corrective lenses; no surgery is necessary."

ANS: C Treatment for cataracts requires surgery to remove the cloudy lens and replace it (with an intraocular lens implant, removable contact lens, or prescription glasses). Treatment for glaucoma may require more than one surgery. Anisometropia is treated with corrective lenses. Myopia is corrected with biconcave lenses that focus rays on the retina.

The school nurse is presenting sexual information to a group of school-age girls. What approach should the nurse take when presenting the information? a. Put off answering questions. b. Give technical terms when giving the presentation. c. Treat sex as a normal part of growth and development. d. Plan to give the presentation with boys and girls together.

ANS: C When nurses present sexual information to children, they should treat sex as a normal part of growth and development. Nurses should answer questions honestly, matter-of-factly, and at the children's level of understanding. School-age children may be more comfortable when boys and girls are segregated for discussions.

The school nurse has been asked to begin teaching sex education in the fifth grade. What should the nurse recognize? a. Questions need to be discouraged in this setting. b. Most children in the fifth grade are too young for sex education. c. Sexuality is presented as a normal part of growth and development. d. Correct terminology should be reserved for children who are older.

ANS: C When sexual information is presented to school-age children, sex should be treated as a normal part of growth and development. They should be encouraged to ask questions. At 10 to 11 years old, fifth graders are not too young to speak about physiologic changes in their bodies. Preadolescents need precise and concrete information.

What is an important priority in dealing with the child suspected of having Wilms tumor? a. Intervening to minimize bleeding b. Monitoring temperature for infection c. Ensuring the abdomen is protected from palpation d. Teaching parents how to manage the parenteral nutrition

ANS: C Wilms tumor, or nephroblastoma, is the most common malignant renal and intraabdominal tumor of childhood. The abdomen is protected, and palpation is avoided. Careful handling and bathing are essential to prevent trauma to the tumor site. Before chemotherapy, the child is not myelosuppressed. Bleeding is not usually a risk. Infection is a concern after surgery and during chemotherapy, not before surgery. Parenteral therapy is not indicated before surgery.

The nurse should plan which actions to assist the stuttering child? (Select all that apply.) a. Ask the child to stop and start over. b. Promise a reward for proper speech. c. Set a good example by speaking clearly. d. Give the child plenty of time to finish sentences. e. Look directly at the child while he or she is speaking.

ANS: C, D, E Actions to be encouraged to help the stuttering child include setting a good example by speaking clearly, giving the child plenty of time to finish sentences, and looking directly at the child while he or she is speaking. Asking the child to stop and start over and promising a reward for proper speech are actions to be avoided with stuttering children.

The nurse is providing anticipatory guidance to parents of a 6-month-old on preventing an accidental poisoning injury. Which should the nurse include in the teaching? (Select all that apply.) a. Place plants on the floor. b. Place medications in a cupboard. c. Discard used containers of poisonous substances. d. Keep cosmetic and personal products out of the child's reach. e. Make sure that paint for furniture or toys does not contain lead.

ANS: C, D, E Anticipatory guidance for a 7-month-old infant to prevent a suffocation injury takes into account that the infant will become more active and eventually crawl, cruise, and walk. Used containers of poisonous substances should be discarded, cosmetic and personal products should be kept out of the child's reach, and paint for furniture or toys should be lead free. Plants should be hung out of reach or placed on a high shelf. Medications should be locked, not just placed in a cupboard.

The nurse is assessing a child with Down syndrome. The nurse recognizes that which are possible comorbidities that can occur with Down syndrome? (Select all that apply.) a. Diabetes mellitus b. Hodgkin's disease c. Congenital heart defects d. Respiratory tract infections e. Acute megakaryoblastic leukemia

ANS: C, D, E Children with Down syndrome often have multiple comorbidities, contributing to numerous other conditions. Respiratory tract infections are prevalent; when combined with cardiac anomalies, they are the chief cause of death, particularly during the first year. The incidence of leukemia is several times more frequent than expected in the general population, and in about half of the cases, the type is acute megakaryoblastic leukemia.

The nurse is developing a teaching pamphlet for parents of school-age children. What anticipatory guidelines should the nurse include in the pamphlet? a. At age 6 years, parents should be certain that the child is reading independently with books provided by school. b. At age 8 years, parents should expect a decrease in involvement with peers and outside activities. c. At age 10 years, parents should expect a decrease in admiration of the parents with little interest in parent-child activities. d. At age 12 years, parents should be certain that the child's sex education is adequate with accurate information.

ANS: D A 12-year-old child should have been introduced to sex education, and parents should be certain that the information is adequate and accurate and that the child is not embarrassed to talk about sexual feelings or other aspects of sex education. At age 6 years, a child does not need to be reading independently and usually still needs help with reading and enjoys being read to. At 8 years of age, parents should expect their child to show increased involvement with peers and outside activities and should encourage this behavior. A 10-year-old child exhibits increased feelings of admiration of parents, especially fathers, and parent-child activities should be encouraged.

An infant with respiratory syncytial virus (RSV) is being admitted to the hospital. The nurse should plan to place the infant on which precaution? a. Enteric b. Airborne c. Droplet d. Contact

ANS: D A patient with RSV is placed on contact precautions. The transmission of RSV is by contact of secretions, not by droplets or airborne. Enteric precautions are not required for RSV.

Which is described as an elevated, circumscribed skin lesion that is less than 1 cm in diameter and filled with serous fluid? a. Cyst b. Papule c. Pustule d. Vesicle

ANS: D A vesicle is elevated, circumscribed, superficial, smaller than 1 cm in diameter, and filled with serous fluid. A cyst is elevated, circumscribed, palpable, encapsulated, and filled with liquid or semisolid material. A papule is elevated; palpable; firm; circumscribed; smaller than 1 cm in diameter; and brown, red, pink, tan, or bluish red. A pustule is elevated, superficial, and similar to a vesicle but filled with purulent fluid.

The nurse is teaching an adolescent about giving insulin injections. The adolescent asks if the disposable needles and syringes can be used more than once. The nurse's response should be based on which knowledge? a. It is unsafe. b. It is acceptable for up to 24 hours. c. It is acceptable for families with very limited resources. d. It is suitable for up to 3 days if stored in the refrigerator.

ANS: D Bacterial counts are unaffected if insulin syringes are handled in an aseptic manner and stored in the refrigerator between use. The syringes can be used up to 3 days and result in a considerable cost savings. Bacterial counts remain low for up to 72 hours with proper technique. The family's resources are not an issue; if a practice is unsafe, the family should not be encouraged to endanger the child by reusing equipment.

A child has been diagnosed with scabies. Which statement by the parent indicates understanding of the nurse's teaching about scabies? a. "The itching will stop after the cream is applied." b. "We will complete extensive aggressive housecleaning." c. "We will apply the cream to only the affected areas as directed." d. "Everyone who has been in close contact with my child will need to be treated."

ANS: D Because of the length of time between infestation and physical symptoms (30 to 60 days), all persons who were in close contact with the affected child need treatment. Families need to know that although the mite will be killed, the rash and the itch will not be eliminated until the stratum corneum is replaced, which takes approximately 2 to 3 weeks. Aggressive housecleaning is not necessary, but surface vacuuming of heavily used rooms by a person with crusted scabies is recommended. The prescribed cream should be thoroughly and gently massaged into all skin surfaces (not just the areas that have a rash) from the head to the soles of the feet.

What pain management approach is most effective for a child who is having a bone marrow test? a. Relaxation techniques b. Administration of an opioid c. EMLA cream applied over site d. Conscious or unconscious sedation

ANS: D Children need explanations before each procedure that is being done to them. Effective pharmacologic and nonpharmacologic measures should be used to minimize pain associated with procedures. For bone marrow aspiration, conscious or unconscious sedation should be used. Relaxation, opioids, and EMLA can be used to augment the sedation.

What finding by the nurse is most characteristic of chronic sorrow? a. Lack of acceptance of child's limitation b. Lack of available support to prevent sorrow c. Periods of intensified sorrow when experiencing anger and guilt d. Periods of intensified sorrow at certain landmarks of the child's development

ANS: D Chronic sorrow is manifested by feelings of sorrow and loss that recur in waves over time. The sorrow is a response to the recognition of the child's limitations. The family should be assessed in an ongoing manner to provide appropriate support as their needs change. The sorrow is not preventable. The chronic sorrow occurs during the reintegration and acknowledgment stage.

The nurse is performing a physical assessment on a 3-year-old child. The parents state that the child excessively rubs the eyes and often tilts the head to one side. What visual impairment should the nurse suspect? a. Strabismus b. Astigmatism c. Hyperopia, or farsightedness d. Myopia, or nearsightedness

ANS: D Clinical manifestations of myopia include excessive eye rubbing, head tilting, difficulty reading, headaches, and dizziness. Strabismus, astigmatism, and hyperopia have other clinical manifestations.

What is probably the most important criterion on which to base the decision to report suspected child abuse? a. Inappropriate response of child b. Inappropriate parental concern for the degree of injury c. Absence of parents for questioning about child's injuries d. Incompatibility between the history and injury observed

ANS: D Conflicting stories about the "accident" are the most indicative red flags of abuse. The child or caregiver may have an inappropriate response, but this is subjective. Parents should be questioned at some point during the investigation.

What side effect commonly occurs with corticosteroid (prednisone) therapy? a. Alopecia b. Anorexia c. Nausea and vomiting d. Susceptibility to infection

ANS: D Corticosteroids have immunosuppressive effects. Children who are taking prednisone are susceptible to infections. Hair loss is not a side effect of corticosteroid therapy. Children taking corticosteroids have increased appetites. Gastric irritation, not nausea and vomiting, is a potential side effect. The medicine should be given with food.

Daily toothbrushing and flossing can be encouraged for the child on chemotherapy when the platelet count is above which? a. 10,000/mm3 b. 20,000/mm3 c. 30,000/mm3 d. 40,000/mm3

ANS: D Daily toothbrushing and flossing are encouraged in children with platelet counts above 40,000/mm3.

What chemotherapeutic agent is classified as an antitumor antibiotic? a. Cisplatin (Platinol AQ) b. Vincristine (Oncovin) c. Methotrexate (Texall) d. Daunorubicin (Cerubidine)

ANS: D Daunorubicin is an antitumor antibiotic. Cisplatin is classified as an alkylating agent. Vincristine is a plant alkaloid. Methotrexate is an antimetabolite.

What blood glucose measurement is most likely associated with diabetic ketoacidosis? a. 185 mg/dl b. 220 mg/dl c. 280 mg/dl d. 330 mg/dl

ANS: D Diabetic ketoacidosis is a state of relative insulin insufficiency and may include the presence of hyperglycemia, a blood glucose level greater than or equal to 330 mg/dl; 185, 220, and 280 mg/dl are values that are too low for the definition of ketoacidosis.

What statement applies to the current focus of the dietary management of children with diabetes? a. Measurement of all servings of food is vital for control. b. Daily calculate specific amounts of carbohydrates, fats, and proteins. c. The number of calories for carbohydrates remains constant on a daily basis; protein and fat calories are liberal. d. The intake ensures day-to-day consistency in total calories, protein, carbohydrates, and moderate fat while allowing for a wide variety of foods.

ANS: D Essentially the nutritional needs of children with diabetes are no different from those of healthy children. Children with diabetes need no special foods or supplements. They need sufficient calories to balance daily expenditure for energy and to satisfy the requirement for growth and development.

What is a condition that can result if hypersecretion of growth hormone (GH) occurs after epiphyseal closure? a. Cretinism b. Dwarfism c. Gigantism d. Acromegaly

ANS: D Excess GH after closure of the epiphyseal plates results in acromegaly. Cretinism is associated with hypothyroidism. Dwarfism is the condition of being abnormally small. Gigantism occurs when there is hypersecretion of GH before the closure of the epiphyseal plates.

Postoperative positioning for a child who has had a medulloblastoma brain tumor (infratentorial) removed should be which? a. Trendelenburg b. Head of bed elevated above heart level c. Flat on operative side with pillows behind the head d. Flat, on either side with pillows behind the back

ANS: D The child with an infratentorial procedure is usually positioned flat and on either side. Pillows should be placed against the child's back, not head, to maintain the desired position. The Trendelenburg position is contraindicated in both infratentorial and supratentorial surgeries because it increases intracranial pressure and the risk of hemorrhage.

One of the techniques that has been especially useful for learners having cognitive impairment is called fading. What description best explains this technique? a. Positive reinforcement when tasks or behaviors are mastered b. Repeated verbal explanations until tasks are faded into the child's development c. Negative reinforcement for specific tasks or behaviors that need to be faded out d. Gradually reduces the assistance given to the child so the child becomes more independent

ANS: D Fading is physically taking the child through each sequence of the desired activity and gradually fading out the physical assistance so the child becomes more independent. Positive reinforcement when tasks or behaviors are mastered is part of behavior modification. An essential component is ignoring undesirable behaviors. Verbal explanations are not as effective as demonstration and physical guidance. Consistent negative reinforcement is helpful, but positive reinforcement that focuses on skill attainment should be incorporated.

A child has an absolute neutrophil count (ANC) of 500/mm3. The nurse should expect to be administering which prescribed treatment? a. Platelets b. Packed red blood cells c. Zofran (ondansetron) d. G-CSF (Neupogen) daily

ANS: D G-CSF (filgrastim [Neupogen], pegfilgrastim [Neulasta]) directs granulocyte development and can decrease the duration of neutropenia following immunosuppressive therapy. G-CSF is discontinued when the ANC surpasses 10,000/mm3.

What nursing intervention is especially helpful in assessing feelings of parental guilt when a disability or chronic illness is diagnosed? a. Ask the parents if they feel guilty. b. Observe for signs of overprotectiveness. c. Talk about guilt only after the parents mention it. d. Discuss the meaning of the parents' religious and cultural background.

ANS: D Guilt may be associated with cultural or religious beliefs. Some parents are convinced that they are being punished for some previous misdeed. Others may see the disorder as a trial sent by God to test their religious beliefs. The nurse can help the parents explore their religious beliefs. On direct questioning, the parents may not be able to identify the feelings of guilt. It would be appropriate for the nurse to explore their adjustment responses. Overprotectiveness is a parental response during the adjustment phase. The parents fear letting the child achieve any new skill and avoid all discipline.

The nurse is administering the first hepatitis A vaccine to an 18-month-old child. When should the child return to the clinic for the second dose of hepatitis A vaccination? a. After 2 months b. After 3 months c. After 4 months d. After 6 months

ANS: D Hepatitis A vaccine is now recommended for all children beginning at age 1 year (i.e., 12 months to 23 months). The second dose in the two-dose series may be administered no sooner than 6 months after the first dose.

What suggestion by the nurse for parents regarding stuttering in children is most helpful? a. Offer rewards for proper speech. b. Encourage the child to take it easy and go slow when stuttering. c. Help the child by supplying words when he or she is experiencing a block. d. Give the child plenty of time and the impression that you are not in a hurry.

ANS: D Hesitancy and dysfluency should be considered a normal part of speech development. An important approach is to allow the child plenty of time to speak. Promising rewards for proper speech places additional pressure on the child. Encouraging the child to take it easy and go slow when stuttering draws attention to the dysfluency. The child needs to complete a sentence and thought without being interrupted.

The clinic nurse is assessing a child with hypopituitarism. Hypopituitarism can lead to which disorder? a. Gigantism b. Hyperthyroidism c. Cushing syndrome d. Growth hormone deficiency

ANS: D Hypopituitarism can lead to a growth hormone deficiency. An overproduction of the anterior pituitary hormones can result in gigantism (caused by excess growth hormone production during childhood), hyperthyroidism, or hypercortisolism (Cushing syndrome).

What should the nurse determine to be the priority intervention for a family with an infant who has a disability? a. Focus on the child's disabilities to understand care needs. b. Institute age-appropriate discipline and limit setting. c. Enforce visiting hours to allow parents to have respite care. d. Foster feelings of competency by helping parents learn the special care needs of the infant.

ANS: D It is important that the parents learn how to care for their infant so they feel competent. The nurse facilitates this by teaching special holding techniques, supporting breastfeeding, and encouraging frequent visiting and rooming in. The focus should be on the infant's capabilities and positive features. Infants do not usually require discipline. As the child gets older, this is necessary, but it is not a priority intervention at this time. The nursing staff negotiates with the family about the need for respite care.

A young boy is found squirting lighter fluid into his mouth. His father calls the emergency department. The nurse taking the call should know that the primary danger is what? a. Hepatic dysfunction b. Dehydration secondary to vomiting c. Esophageal stricture and shock d. Bronchitis and chemical pneumonia

ANS: D Lighter fluid is a hydrocarbon. The immediate danger is aspiration. Acetaminophen overdose, not hydrocarbons, causes hepatic dysfunction. Dehydration is not the primary danger. Esophageal stricture is a late or chronic consequence of hydrocarbon ingestion.

What statement characterizes moral development in the older school-age child? a. Rule violations are viewed in an isolated context. b. Judgments and rules become more absolute and authoritarian. c. The child remembers the rules but cannot understand the reasons behind them. d. The child is able to judge an act by the intentions that prompted it rather than just by the consequences.

ANS: D Older school-age children are able to judge an act by the intentions that prompted the behavior rather than just by the consequences. Rule violation is likely to be viewed in relation to the total context in which it appears. Rules and judgments become less absolute and authoritarian. The situation and the morality of the rule itself influence reactions.

What should the nurse explain about ringworm? a. It is not contagious. b. It is a sign of uncleanliness. c. It is expected to resolve spontaneously. d. It is spread by both direct and indirect contact.

ANS: D Ringworm is spread by both direct and indirect contact. Infected children should wear protective caps at night to avoid transfer of ringworm to bedding. Ringworm is infectious. Because ringworm is easily transmitted, it is not a sign of uncleanliness. It can be transmitted by seats with head rests, gym mats, and animal-to-human transmission. The drug griseofulvin is indicated for a prolonged course, possibly several months.

What is the most common type of burn in the toddler age group? a. Electric burn from electrical outlets b. Flame burn from playing with matches c. Hot object burn from cigarettes or irons d. Scald burn from high-temperature tap water

ANS: D Scald burns are the most common type of thermal injury in children, especially 1- and 2-year-old children. Temperature should be reduced on the hot water in the house and hot liquids placed out of the child's reach. Electric burns from electrical outlets and hot object burns from cigarettes or irons are both significant causes of burn injury. The child should be protected by reducing the temperature on the hot water heater in the home, keeping objects such as cigarettes and irons away from children, and placing protective guards over electrical outlets when not in use. Flame burns from matches and lighters represent one of the most fatal types of burns in the toddler age group but not one of the most common types of burn.

Parents ask for help for their other children to cope with the changes in the family resulting from the special needs of their sibling. What strategy does the nurse recommend? a. Explain to the siblings that embarrassment is unhealthy. b. Encourage the parents not to expect siblings to help them care for the child with special needs. c. Provide information to the siblings about the child's condition only as requested. d. Invite the siblings to attend meetings to develop plans for the child with special needs.

ANS: D Siblings should be invited to attend meeting to be part of the care team for the child. They can learn about an individualized education plan and help design strategies that will work at home. Embarrassment may be associated with having a sibling with a chronic illness or disability. Parents must be able to respond in an appropriate manner without punishing the sibling. The parents may need assistance with the care of the child. Most siblings are positive about the extra responsibilities. Parents need to inform the siblings about the child's condition before a nonfamily member does so. The parents do not want the siblings to fantasize about what is wrong with the child.

The parents of a 5-year-old child ask the nurse, "How many hours of sleep a night does our child need?" The nurse should give which response? a. "A 5-year-old child requires 8 hours of sleep." b. "A 5-year-old child requires 9.5 hours of sleep." c. "A 5-year-old child requires 10 hours of sleep." d. "A 5-year-old child requires 11.5 hours of sleep."

ANS: D Sleep requirements decrease during school-age years; 5-year-old children generally require 11.5 hours of sleep.

The nurse is teaching a nursing student about standard precautions. Which statement made by the student indicates a need for further teaching? a. "I will use precautions when I give an infant oral care." b. "I will use precautions when I change an infant's diaper." c. "I will use precautions when I come in contact with blood and body fluids." d. "I will use precautions when administering oral medications to a school-age child."

ANS: D Standard precautions involve the use of barrier protection (personal protective equipment [PPE]), such as gloves, goggles, a gown, or a mask, to prevent contamination from (1) blood; (2) all body fluids, secretions, and excretions except sweat, regardless of whether they contain visible blood; (3) nonintact skin; and (4) mucous membranes. Precautions should be taken when giving oral care, when changing diapers, and when coming in contact with blood and body fluids. Further teaching is needed if the student indicates the need to use precautions when administering an oral medication to a school-age child.

What specific gravity of the urine is desired so that hemorrhagic cystitis is prevented? a. 1.035 b. 1.030 c. 1.025 d. 1.005

ANS: D Sterile hemorrhagic cystitis is a side effect of chemical irritation to the bladder from chemotherapy or radiotherapy. It can be prevented by a liberal oral or parenteral fluid intake (at least one and a half times the recommended daily fluid requirement). The urine should be dilute so 1.005 is the expected specific gravity.

What often causes cellulitis? a. Herpes zoster b. Candida albicans c. Human papillomavirus d. Streptococci or staphylococci

ANS: D Streptococci, staphylococci, and Haemophilus influenzae are the organisms usually responsible for cellulitis. Herpes zoster is the virus associated with varicella and shingles. C. albicans is associated with candidiasis, or thrush. Human papillomavirus is associated with various types of human warts.

A child eats some sugar cubes after experiencing symptoms of hypoglycemia. This rapid-releasing sugar should be followed by which dietary intervention? a. Sports drink and fruit b. Glucose tabs and protein c. Glass of water and crackers d. Milk and peanut butter on bread

ANS: D Symptoms of hypoglycemia are treated with a rapid-releasing sugar source followed by a complex carbohydrate and protein. Milk supplies lactose and a more prolonged action from the protein. The bread is a complex carbohydrate, which with the peanut butter provides a sustained action. The sports drink contains primarily simple carbohydrates. The fruit contains additional carbohydrates. A protein source is needed for sustained action. The glucose tabs are simple carbohydrates. Complex carbohydrates are needed with the protein. Crackers are a complex carbohydrate, but protein is needed to stabilize the blood sugar.

The nurse is preparing to administer a measles, mumps, rubella, and varicella (MMRV) vaccine. Which is a contraindication associated with administering this vaccine? a. The child has recently been exposed to an infectious disease. b. The child has symptoms of a cold but no fever. c. The child is having intermittent episodes of diarrhea. d. The child has a disorder that causes a deficient immune system.

ANS: D The MMRV (measles, mumps, rubella, and varicella) vaccine is an attenuated live virus vaccine. Children with deficient immune systems should not receive the MMRV vaccine because of a lack of evidence of its safety in this population. Exposure to an infectious disease, symptoms of a cold, or intermittent episodes of diarrhea are not contraindications to receiving a live vaccine.

A child with acetaminophen (Tylenol) poisoning has been admitted to the emergency department. What antidote does the nurse anticipate being prescribed? a. Carnitine (Carnitor) b. Fomepizole (Antizol) c. Deferoxamine (Desferal) d. N-acetylcysteine (Mucomyst)

ANS: D The antidote for acetaminophen (Tylenol) poisoning is N-acetylcysteine (Mucomyst). Carnitine (Carnitor) is an antidote for valproic acid (Depakote), fomepizole (Antizol) is the antidote for methanol poisoning, and deferoxamine (Desferal) is the antidote for iron poisoning.

The parents of a child with cancer tell the nurse that a bone marrow transplant (BMT) may be necessary. What information should the nurse recognize as important when discussing this with the family? a. BMT should be done at the time of diagnosis. b. Parents and siblings of the child have a 25% chance of being a suitable donor. c. If BMT fails, chemotherapy or radiotherapy will need to be continued. d. Finding a suitable donor involves matching antigens from the human leukocyte antigen (HLA) system.

ANS: D The most successful BMTs come from suitable HLA-matched donors. The timing of a BMT depends on the disease process involved. It usually follows intensive high-dose chemotherapy or radiotherapy. Usually, parents only share approximately 50% of the genetic material with their children. A one in four chance exists that two siblings will have two identical haplotypes and will be identically matched at the HLA loci. The decision to continue chemotherapy or radiotherapy if BMT fails is not appropriate to discuss with the parents when planning the BMT. That decision will be made later.

The school nurse is caring for a child with a penetrating eye injury. Emergency treatment includes what intervention? a. Place a cool compress on eye during transport to the emergency department. b. Irrigate the eye copiously with a sterile saline solution. c. Remove the object with a lightly moistened gauze pad. d. Apply a Fox shield to the affected eye and any type of patch to the other eye.

ANS: D The nurse's role in a penetrating eye injury is to prevent further injury to the eye. A Fox shield (if available) should be applied to the injured eye and a regular eye patch to the other eye to prevent bilateral movement. Placing cool compress on the eye during transport to emergency department, irrigating eye copiously with a sterile saline solution, or removing object with a lightly moistened gauze pad may cause more damage to the eye.

What do nursing interventions to promote health during middle childhood include? a. Stress the need for increased calorie intake to meet increased demands. b. Instruct parents to defer questions about sex until the child reaches adolescence. c. Advise parents that the child will need increasing amounts of rest toward the end of this period. d. Educate parents about the need for good dental hygiene because these are the years in which permanent teeth erupt.

ANS: D The permanent teeth erupt during the school-age years. Good dental hygiene and regular attention to dental caries are vital parts of health supervision during this period. Caloric needs are decreased in relation to body size for this age group. Balanced nutrition is essential to promote growth. Questions about sex should be addressed honestly as the child asks questions. The child usually no longer needs a nap, but most require approximately 11 hours of sleep each night at age 5 years and 9 hours at age 12 years.

A child with a serious chronic illness will soon go home. The case manager requests that the family provide total care for the child for a couple of days while the child is still hospitalized. How should the request be viewed? a. Improper because of legal issues b. Supportive because families are usually eager to get involved c. Unacceptable because the family will have to assume the care soon enough d. Important because it can be beneficial to the transition from hospital to home

ANS: D This type of groundwork is essential for the family. Adequate family training and preparation will assist in the child's transition home. The nursing staff in the hospital is responsible for the child's care. The family will provide the care with assistance as needed. Although parents are eager to be involved, the purpose of this intervention is the development of family competency and confidence that they are capable. Arrangements for respite care are important for the family both during hospitalizations and while the child is at home.

The nurse is preparing to administer a prescribed dose of desmopressin acetate (DDAVP) intramuscularly (IM) to a child with diabetes insipidus. What action should the nurse take before drawing the medication into a syringe? a. Mix the medication with sterile water. b. Mix the medication with sterile normal saline. c. Have another nurse double-check the medication dose. d. Hold the medication under warm water for 10 to 15 minutes and then shake vigorously.

ANS: D To be effective, vasopressin must be thoroughly mixed in the oil by being held under warm running water for 10 to 15 minutes and shaken vigorously before being drawn into the syringe. If this is not done, the oil may be injected minus the antidiuretic hormone. Small brown particles, which indicate drug dispersion, must be seen in the suspension.

A mother tells the nurse that she does not want her infant immunized because of the discomfort associated with injections. What should the nurse explain? a. This cannot be prevented. b. Infants do not feel pain as adults do. c. This is not a good reason for refusing immunizations. d. A topical anesthetic can be applied before injections are given.

ANS: D To minimize the discomfort associated with intramuscular injections, a topical anesthetic agent can be used on the injection site. These include EMLA (eutectic mixture of local anesthetic) and vapor coolant sprays. Pain associated with many procedures can be prevented or minimized by using the principles of atraumatic care. Infants have neural pathways that will indicate pain. Numerous research studies have indicated that infants perceive and react to pain in the same manner as do children and adults. The mother should be allowed to discuss her concerns and the alternatives available. This is part of the informed consent process.

The school nurse is discussing after-school sports participation with parents of children age 10 years. The nurse's presentation includes which important consideration? a. Teams should be gender specific. b. Organized sports are not appropriate at this age. c. Competition is detrimental to the establishment of a positive self-image. d. Sports participation is encouraged if the type of sport is appropriate to the child's abilities.

ANS: D Virtually every child is suited for some type of sport. The child should be matched to the type of sport appropriate to his or her abilities and physical and emotional makeup. At this age, girls and boys have the same basic structure and similar responses to exercise and training. After puberty, teams should be gender specific because of the increased muscle mass in boys. Organized sports help children learn teamwork and skill acquisition. The emphasis should be on playing and learning. Children do enjoy appropriate levels of competition.

The nurse is providing palliative care for a child who cries when touched and moved. Which is the most appropriate intervention? Avoid massaging the child Making sure the child is awake Avoid changing the child's position Administering analgesia to the child

Administering analgesia to the child Rationale The most important intervention is administering analgesia because the child is in pain. The child should be provided with sleep enhancement techniques to help minimize the pain the child is experiencing. Massaging and repositioning the child would help to comfort the child and minimize pain.

Prolonged steroid therapy has caused a child to have Cushing syndrome. To lessen the cushingoid effects, the steroid should be administered at which time? a. In the PM b. After lunch c. QD in the AM d. QOD in the AM

ANS: D When cushingoid features are caused by steroid therapy, the effects may be lessened with administration of the drug early in the morning and on an alternate-day basis. Giving the drug early in the day maintains the normal diurnal pattern of cortisol secretion. If given during the evening, it is more likely to produce symptoms because endogenous cortisol levels are normally low and the additional supply exerts more pronounced effects. An alternate-day schedule allows the anterior pituitary an opportunity to maintain more normal hypothalamic-pituitary-adrenal control mechanisms.

A chronically ill infant has died. Which is the best nursing intervention? Accept the family's grief reactions. Leave the room so the family can be alone. Offer rationalizations for the infant's death. Console the members of the family, at least artificially.

Accept the family's grief reactions. Rationale The nurse should accept the family's grieving reactions, such as anger or crying. The nurse should stay with the family and stay quiet if they prefer not to talk. The nurse should not provide rationalizations about the child's death because it might anger or irritate the parents. The nurse should not artificially console the family and should stay quiet if the family members do not wish to talk.

A chronically ill infant has died. Which is the best nursing intervention? Accept the family's grief reactions. Leave the room so the family can be alone. Offer rationalizations for the infant's death. Console the members of the family, at least artificially.

Accept the family's grief reactions. Rationale The nurse should accept the family's grieving reactions, such as anger or crying. The nurse should stay with the family and stay quiet if they prefer not to talk. The nurse should not provide rationalizations about the child's death because it might anger or irritate the parents. The nurse should not artificially console the family and should stay quiet if the family members do not wish to talk.

A family wants to begin oral feeding of their 4-year-old son, who is ventilator dependent and currently tube fed. They ask the home health nurse to feed him the baby food orally. The nurse recognizes a high risk of aspiration and an already compromised respiratory status. What is the most appropriate nursing action? Refuse to feed the child orally because the risk is too high. Explain the risks involved and then let the family decide what should be done. Feed the child orally because the family has the right to make this decision for him. Acknowledge their request, explain the risks, and explore with the family the available options.

Acknowledge their request, explain the risks, and explore with the family the available options. Correct Parents want to be included in the decision making for their child's care. The nurse should discuss the request with the family to ensure this is the issue of concern, and then potential options can be explored. Refusing to feed the child orally does not determine why the parents want oral feedings to begin and does not involve them in problem-solving. The decision to begin oral feedings or not to change the feedings should be a collaborative one, made in consultation with the family, the nurse, and an appropriate member of the health care team

What pediatric oncologic emergencies are caused by the rapid release of intracellular metabolites during the initial treatment of some cancers? Hyperleukocytosis Overwhelming infection Acute tumor lysis syndrome Superior vena cava syndrome

Acute tumor lysis syndrome Correct Acute tumor lysis syndrome is caused by the rapid release of intracellular metabolites during the initial treatment of malignancies. Hyperleukocytosis, a white blood cell count greater than 100,000/mm3, can be present at diagnosis. It is not a result of the treatment. Infection may occur from bone marrow suppression that results from many chemotherapeutic agents. Superior vena cava syndrome can occur from compression of the mediastinal structures by Hodgkin disease and non-Hodgkin lymphoma

Which medicine would the nurse expect in the treatment plan of the patient being treated with immunosuppressive therapy who has a varicella infection? Acyclovir Hydroxyzine Acetaminophen Diphenhydramine

Acyclovir Rationale Acyclovir is an antiviral agent. It is used to treat varicella infections in susceptible immunocompromised people. The patient's immunity is compromised by the immunosuppressive therapy. Acyclovir is effective in decreasing the number of lesions, shortening the duration of fever, and decreasing itching, lethargy, and anorexia. Hydroxyzine is an antipruritic medication used to treat severe itching. Acetaminophen is an antipyretic agent and helpful in reducing elevated temperature. Diphenhydramine is an antipruritic agent used for the treatment of severe itching.

Home care is being considered for a young child who is ventilator dependent. What factor is most important in deciding whether home care is appropriate? Level of education of parents Adequate family training Presence of two parents in the home Family's ability to assume all health care costs

Adequate family training Correct One of the essential elements is the family's preparation and ability to care for the child. The family must be able to demonstrate all aspects of care for the child because in many areas nursing care may not be available on a continual basis. The amount of formal education reached by the parents is not the important issue. The determinant is the family's ability to care adequately for the child in the home. At least two family members should learn and demonstrate all aspects of the child's care in the hospital, but it does not have to be two parents. Few families can assume all health care costs. Creative financial planning, including negotiating arrangements with the insurance company or public programs, may be required.

The nurse cares for a child with rapidly progressing paralysis due to Guillain-Barré syndrome (GBS). After supportive care fails, the nurse anticipates what treatment as the next step in this patient's care? Consult palliative care Administer corticosteroids Teach about plasmapheresis Administer IV immunoglobulin

Administer IV immunoglobulin A child with rapidly progressing paralysis due to Guillain-Barré syndrome should be given high-dose IVIG. The IV immunoglobulin interferes with autoantibodies causing GBS and helps restore function in the patient.

A 12-year-old patient was recently diagnosed with sickle cell disease. The nurse notes that the patient is up to date with regular vaccines including completion of the four-dose series of PCV13 by age 15 months. The patient received one dose of PPSV23 and one dose of meningococcal vaccine nine weeks ago. Which vaccine does the nurse expect to administer today? Administer a dose of PPSV23. Administer a dose of Hepatitis A vaccine. Administer a dose of meningococcal vaccine. Administer a dose of inactivated poliovirus vaccine.

Administer a dose of meningococcal vaccine. Children with certain health conditions including sickle cell disease who have not yet received a complete series of meningococcal vaccine are recommended to receive two doses in a series, separated by at least eight weeks. This 10-year-old patient was recently diagnosed, so the patient would not have received the meningococcal vaccine series previously (the first dose is given at age 11 years). This dose should now be administered.

A child with cancer is scheduled to receive chemotherapy. Some of the regular childhood vaccinations of the child are due over the next few months. What is the best step for immunizing the child? Administer inactivated polio vaccine (IPV) during the chemotherapy. Administer live measles, mumps, and rubella vaccine during the chemotherapy. Administer live attenuated oral polio vaccine (OPV) 2 weeks before chemotherapy. Administer inactivated hepatitis A virus vaccine 7 months after the chemotherapy is instituted.

Administer inactivated hepatitis A virus vaccine 7 months after the chemotherapy is instituted. Rationale The child can receive inactivated vaccines 6 or more months after chemotherapy is stopped. Thus, inactivated hepatitis A virus can be safely administered 7 months after chemotherapy. Although inactivated vaccines can be given to immunosuppressed children, the immune response may be suboptimum. Therefore, it is better to delay the administration of IPV until after chemotherapy. A child receiving chemotherapy for cancer should not receive a live measles, mumps, and rubella vaccine because viral replication can cause serious disease in the immunocompromised child. If a child has been vaccinated during or 2 weeks before chemotherapy, the child would be considered unvaccinated and should be revaccinated 6 months after treatment. Thus, OPV should not be administered 2 weeks before chemotherapy.

The nurse is caring for a four-year-old child with leukemia who is receiving chemotherapy. The parent reports that the child has abdominal pain, nausea, and vomiting. Upon exam, the nurse notes poor skin turgor, pallor, dry mucus membranes, and poor appetite. Which action should the nurse take first? Administer pain medicine Assess hourly urine output Administer oral antiemetic Administer normal saline IV bolus

Administer normal saline IV bolus A child with leukemia is at risk for dehydration and fluid and nutrition imbalance. Thus, administering IV bolus of normal saline is a priority nursing intervention to treat signs of dehydration.

Which nursing intervention is appropriate for the child showing signs of shock, along with white, swollen mucous membranes and edema of the lips, tongue, and pharynx after ingesting mildew remover? Select all that apply. Administer oral fluids. Neutralize the caustic agent. Administer activated charcoal. Conduct gastric emptying by emesis. Ensure that a patent airway is maintained

Administer oral fluids. Ensure that a patent airway is maintained. Rationale The child has ingested mildew remover, which is a toxic agent that has resulted in shock and respiratory obstruction. If the child is able to tolerate them, the nurse should administer oral fluids, because the water or milk will dilute the ingested corrosive. The clinical manifestations indicate an airway obstruction that requires intubation so that a patent airway is maintained. The nurse should never neutralize the caustic agent. This can result in an exothermic reaction, where the heat emitted will further aggravate the existing symptoms and lead to a thermal burn in addition to the already existing chemical burn. Activated charcoal is contraindicated in such patients, because it may infiltrate tissue burned by the toxicity of the caustic agent. The process of gastric emptying by emesis or lavage is contraindicated as it reexposes the upper gastrointestinal tract to the caustic agent.

An 11-year-old patient is seen in the clinic. The nurse notes that the patient received all recommended vaccines up until the age of 6. After the child's parents give consent, what action does the nurse take? Administer HPV vaccine Administer the Tdap vaccine Administer the Hep B vaccine Administer the DTaP vaccine Administer the meningococcal B vaccine

Administer the Tdap vaccine The nurse should administer tetanus, diphtheria, & acellular pertussis (Tdap). This vaccine is recommended for patients aged seven years and older who are not fully immunized with DTaP. If additional doses are needed, the Td vaccine is used.

Which nursing intervention is appropriate for the child that ingested several 325-mg tablets of aspirin and hour ago? Select all that apply. Conduct lavage. Administer vitamin K. Conduct peritoneal dialysis. Administer anticonvulsants. Ensure a warm environment.

Administer vitamin K. Administer anticonvulsants. Rationale The ingestion of high doses of aspirin is considered an acute ingestion. The clinical manifestations include bleeding tendencies and seizures, along with renal and respiratory failure. Therefore the effective course of treatment will involve administering vitamin K to control bleeding and administering anticonvulsants if the child has seizures. Lavage is not beneficial, because it does not remove concretions of aspirin. For severe aspirin ingestion, hemodialysis, not peritoneal dialysis, is administered. This child needs to be provided with external cooling for hyperpyrexia.

A terminally ill child reports pain due to constipation. Which is the nurse's best action? Use absorbent pads Relaxation therapy Administration of stool softeners Avoidance of pressure on painful areas

Administration of stool softeners Rationale The nurse should focus on pain management in terminally ill patients, assessing the cause of pain and choosing appropriate interventions to reduce or eliminate the pain and preventing adverse effects secondary to pain management. Stool softeners can reduce constipation-related problems, making the patient more comfortable. Absorbent pads are used to prevent skin-related problems such as skin breakdown but will not reduce pain from constipation. Relaxation therapy may not be directly helpful in reducing pain from constipation. Avoiding stress on painful areas helps to increase the comfort level of the patient but is not specifically applicable to reducing the patient's constipation-related pain.

Which drugs are administered to the child experiencing bedwetting, perianal itching, restlessness, poor sleep, who has also developed perianal dermatitis and excoriation? Select all that apply. Tinidazole Albendazole Nitazoxanide Metronidazole Pyrantel pamoate

Albendazole Pyrantel pamoate Rationale The child has developed enterobiasis ( pinworms). Bedwetting, perianal itching, restlessness, poor sleep, perianal dermatitis, and excoriation are clinical manifestations of enterobiasis (pinworms). The drugs albendazole and pyrantel pamoate are administered to the child to treat the condition. Tinidazole, nitazoxanide, and metronidazole are administered to children to treat giardiasis.

The nurse notes that an adolescent that has been in the hospital for over a week has recently stopped talking to the staff when they enter the room. Which response by the nurse is most appropriate? Have the adolescent call a friend to talk about the illness and increase communication Allow the adolescent to make choices regarding how and when certain parts of care are completed Encourage the adolescent to have some peers come and visit during the hospitalization to decrease loneliness Tell the adolescent that interacting with staff and communicating needs must be done if treatment is going to be effective

Allow the adolescent to make choices regarding how and when certain parts of care are completed A lack of communication or change in communication can be an indication of a decrease in self-esteem and independence related to dealing with an illness. Allowing the adolescent to make some decisions regarding care can increase independence and self-esteem.

The parents of a three-month-old infant with neuroblastoma voice concern that the child is experiencing discomfort. The nurse notes the child is crying and appears to be uncomfortable, but the parents seem distant. Which action should the nurse take first? Ensure patent airway Notify the health care provider Obtain a history of bowel habits Allow the parent to hold the child

Allow the parent to hold the child The parents should be allowed to hold the child to provide comfort when the child is in moderate pain. This helps to maintain the child's normal psychosocial development and is an acceptable non-pharmacological method of pain management in an infant.

The nurse senses sadness in a parent with a child who has terminal cancer. Which action is the most appropriate for the nurse take in this situation? Place a referral to the hospital's chaplain. Allow the parent to talk and express emotions. Place a referral to a psychotherapist for grief counseling. Provide the parent with a list of support groups in the area.

Allow the parent to talk and express emotions. Allowing the parent to talk and express emotions provides the opportunity for the parent to grieve and express grief in a comfortable manner, and therefore this is the most appropriate action for the nurse to take.

A child with cancer is pronounced dead by the primary health care provider. The parents are upset and request some private time with the child. What does the nurse do in this situation? Give photographs of the child to the parents. Allow the parents to spend 15 minutes in private with the child. Allow the parents to spend as much time as they want in private with the child. Perform postmortem bathing and legacy-making prior to letting the parents view the child.

Allow the parents to spend as much time as they want in private with the child. Rationale When parents request to spend some time with the dead child in private, the nurse should allow the parents to spend as much time as they want with their child. The nurse can accompany the parents but should remain inconspicuous when the parents want to spend some time with their dead child. Providing the parents with photographs of the child is not the best intervention in this situation. The nurse should involve the parents in the postmortem care and legacy-making if they wish after they've spent time with the child.

Hodgkin's lymphoma

Although Hodgkin disease is extremely rare before 5 years of age, there is a striking increase in children ages 15 to 19 years, when it occurs with almost the same frequency as leukemia. Hodgkin disease is characterized by enlarged, firm, nontender, movable nodes in the supraclavicular or cervical area. In children, the sentinel node located near the left clavicle may be the first enlarged node. Enlargement of axillary and inguinal lymph nodes is less frequent. The primary therapies for Hodgkin disease are chemotherapy and radiation. Each may be used alone or in combination based on the clinical staging. The goal of treatment is obviously a cure; however, aggressive therapy increases the chances of complications in the disease-free state and can seriously compromise the quality of life.

Which statement by the nursing student indicates the need for further explanation about the management of poisoning in children? Activated charcoal is administered in aspirin poisoning. Inducing emesis is contraindicated in corrosives poisoning. Amyl nitrate is used as an antidote for acetaminophen poisoning. Chelation therapy with deferoxamine is used to treat iron poisoning.

Amyl nitrate is used as an antidote for acetaminophen poisoning. Rationale Amyl nitrate is used as an antidote for cyanide poisoning, whereas N-acetylcysteine is used as an antidote for acetaminophen poisoning. Activated charcoal is administered in aspirin poisoning to prevent further absorption. Inducing emesis is contraindicated in corrosives poisoning, because it redamages the mucosa. Chelation therapy with deferoxamine is used to treat iron poisoning to avoid further absorption.

The nurse is educating new mothers of infants when they return for a well-baby check. What infant is at high risk of developing sudden infant death syndrome (SIDS) when discussing with the mothers? An infant who sleeps in a crib An infant whose mother smokes An infant who sleeps on firm bedding An infant who sleeps in a supine position

An infant whose mother smokes Rationale SIDS is characterized by the unexplained sudden death of the infant. Maternal smoking is a major risk factor of SIDS. Studies suggest that high nicotine level in the infant's lung tissue may lead to SIDS. Co-sleeping is another risk factor. Infants who share a bed with an elder sibling or any other adult may have high risk of developing SIDS. Infants who sleep in their cribs tend to have decreased risk of SIDS. Infants who sleep in prone position may obstruct their airways. They also tend to inhale carbon dioxide, thereby increasing the risk of SIDS. Soft bedding may accidentally obstruct the airway and increase the risk of SIDS.

The mother of a 5-year-old patient reports unexplained weight loss in the child over the past month, frequent epistaxis, and persistent diarrhea. Which other findings support a possible diagnosis of cancer? Select all that apply. Anemia Frequent sneezing Poor concentration Ulcers of the mouth Mass in the child's neck

Anemia Anemia is a sign of malignancy and would support a diagnosis of cancer in a child with epistaxis, weight loss, and diarrhea. Mass in the child's neck Lymphadenopathy, or swollen lymph nodes, may manifest as a mass in the child's neck and is a sign of malignancy. This would support a diagnosis of cancer in a child with epistaxis, weight loss, and diarrhea.

The home health nurse is caring for a child who requires complex care. The family expresses frustration related to obtaining accurate information about their child's illness and its management. What is the best action for the nurse? Determine why the family is easily frustrated. Refer the family to the child's primary care practitioner. Provide the information the family requests, being confident that every question can be answered by the nurse. Answer questions in a straightforward manner and get professional assistance when an answer is unknown.

Answer questions in a straightforward manner and get professional assistance when an answer is unknown. Correct The philosophic basis for family-centered practice is the recognition that the family is the constant in the child's life. It is essential that the family have complete and accurate information about their child's illness and management. The nurse may first have to clarify what information the family believes has not been communicated. The family's frustration arises from their perception that they are not receiving information pertinent to their child's care. It does not help the family to refer the family to the child's primary care practitioner. The home health nurse should have access to the necessary information. Questions about what they need and want to know concerning their child's care should be addressed, but the nurse may not be able to answer every question

what is associated with stomatitis

Aphthous ulcers Canker sore- benign, cause unknown Persist for 4-12 days what are the s/s with aphthous ulcers Small, painful, white ulcerations with red border disease known as canker sore; benign but painful condition associated w/ mild traumatic injury (biting cheek, hitting mucosa w/ toothbrush, mouth appliance rubbing on mucosa), allergy, or emotional stress Herpes simplex virus what are the s/s of herpes simplex virus Fever, edematous, erythematous pharynx, painful vesicles Cervical lymphadenitis, foul mouth odo causes - stress, trauma, immunosuppression, exposure to extreme sunlight lasts 5-14 days disease that begins w/ fever, pharynx becomes edematous & erythematous, vesicles erupt on mucosa causing severe pain; breath has distinctly foul odor; vesicles appear on lips (singly or in groups) lasting 5-14 days w/ varying severity; caused by HSV type 1 Hand, foot, and mouth disease (HFMD Scattered vesicles on buccal mucosa what is nutritional education for pt with stomatitis bland foods, nothing spicy or acidic

Which complication is likely to be found in a toddler with sickle-cell disease? Stomatitis Conjunctivitis Aplastic anemia Gingivostomatitis

Aplastic anemia Rationale Sickle-cell disease is a hemolytic disease. The patient who has sickle-cell disease is at a higher risk of developing aplastic anemia from erythema infectiosum. Human parvovirus B19 infects and causes lysis of red blood cell (RBC) precursors, which interrupts the production of RBCs. Therefore, the virus precipitates a severe aplastic crisis. The patient may need increased RBC production to maintain normal RBC volumes. Stomatitis, conjunctivitis, and gingivostomatitis are not sickle-cell disease-associated complications. Stomatitis is inflammation of the oral mucosa; it may be caused by local or systemic factors. Conjunctivitis is an inflammation of the conjunctiva and occurs from a variety of causes, which are typically age related. Gingivostomatitis is the inflammation of the mouth and gums.

The nurse is treating a child who is approaching death within hours. Which respiratory responses are likely findings in a child who is approaching death? Select all that apply. Respiratory rate of 28 breaths per minute Relaxed appearance to the face of the child Apneic episodes of 40 seconds between respirations Retraction of muscles under the sternum and ribcage Loud sighing at the end of each respiration with a loud rattle noise

Apneic episodes of 40 seconds between respirations Periods of apnea increase as death approaches, and therefore this can be expected. Correct Retraction of muscles under the sternum and ribcage Increased work of breathing is evident by retractions and is associated with impending death. Correct Loud sighing at the end of each respiration with a loud rattle noise The death rattle is associated with secretions in the upper airway that cannot be cleared by the child.

At the end of every shift a nurse feels drained and begins crying, grieving the loss of patients. What can the nurse do to increase coping mechanisms? Select all that apply. Start taking a multivitamin supplement prior to each shift. Ask a more experienced pediatric nurse to mentor the nurse. Obtain at least 8 hours of uninterrupted sleep every night. Attend monthly social hour events with other nurses from the pediatric floor. Meet monthly with other pediatric nurses to discuss the past month's events.

Ask a more experienced pediatric nurse to mentor the nurse. Mentoring provides support to the nurse who is struggling with the grief associated with losing patients. Correct Obtain at least 8 hours of uninterrupted sleep every night. Obtaining the proper number of hours of uninterrupted sleep allows the nurse to take care of own physical health, which assists with increasing coping mechanisms. Meet monthly with other pediatric nurses to discuss the past month's events. Expressing feelings with other nurses provides support to the nurse who is struggling with the grief associated with losing patients.

A 12-year-old in the clinic reports "severe headaches." The nurse should include which focused assessments for this patient? Select all that apply. Ask about medication history. Auscultate for a bruit in the head. Assess mental status and level of consciousness. Evaluate gait, coordination, and muscle strength. Anticipate health care provider examination of optic disks for papilledema.

Ask about medication history. The nurse should ask about medication history of this child as part of evaluating this patient. Daily use of analgesics may cause rebound headaches. Correct Auscultate for a bruit in the head. The nurse should auscultate for a bruit as an indication of an aneurysm as part of evaluating patient. Correct Assess mental status and level of consciousness. The nurse should assess mental status of child as part of evaluating patient. Anticipate health care provider examination of optic disks for papilledema. The nurse should anticipate health care provider examination of optic disks for papilledema in child as part of evaluating patient.

The nurse is caring for a school-aged child with a chronic illness that requires the child to wear a mask when leaving the hospital room. The child does not want to attend social activities on the unit. Which action by the nurse can help encourage the child to participate? Ask the child why the hesitation to wear the mask when leaving the hospital room Bargain with the child to attend at least one activity a week to help increase the child's socialization on the unit Encourage the child to attend the social activities on the unit without the mask so that the child does not feel singled out Allow the child to decline attendance and participation in the social activities on the unit to decrease emotional distress and embarrassment

Ask the child why the hesitation to wear the mask when leaving the hospital room The nurse should question the child about the reason for not wanting to wear the mask and should allow the child to verbalize feelings.

A husband and wife are speaking to the nurse about their infant who has cerebral palsy. The father seems anxious when interacting with the infant. What is the nurse's best response to this situation? Have both parents work together to care for the infant so that neither parent feels singled out Ask the father to assist the nurse with bathing the infant and encourage the father to ask questions Ensure that the father is not left alone with the child because the father seems uncomfortable and unsafe Address the education primarily to the mother because she seems to feel more comfortable caring for the infant

Ask the father to assist the nurse with bathing the infant and encourage the father to ask questions Specifically involving the father in the care of the child and encouraging questions is important to increase the comfort level of the father.

A child with sickle cell disease expresses a desire to spend more time with a sibling who has not been seen because the family lives far away from the hospital. Which actions by the nurse should be most appropriate? Ask the health provider for a day pass so the ill child can visit the sibling. Ask the parents for a picture of the sibling to keep at the ill child's bedside. Ask the parents to bring the sibling to visit in the next few days when possible. Provide the child with books and movies to distract the ill child from thinking about the sibling.

Ask the parents to bring the sibling to visit in the next few days when possible. It is important to attempt to maintain the sibling relationships. Bringing the sibling to visit allows them to reconnect and maintain their sibling relationship.

A 16-year-old patient with cerebral palsy is having a birthday party and receiving a lot of attention from family and the home health care staff. A younger sibling is at the birthday party but looking unhappy. Which actions by the home health nurse are appropriate in this situation? Select all that apply. Ask the sibling about a special personal birthday event. Talk with the parents about reading a book to the sibling every night. Talk with the parents about taking the sibling to a movie later that day. Encourage the sibling to help cut the birthday cake and scoop ice cream. Encourage the sibling to avoid being negative and to be a positive support for the sibling

Ask the sibling about a special personal birthday event. This action provides sibling-centered interactions that foster communication and sense of importance in the sibling because it acknowledges important events valued by the sibling. Correct Talk with the parents about reading a book to the sibling every night. Reading a book together daily provides a consistent time for interaction with the parents. This fosters positive relationships between the parents and the sibling. Correct Talk with the parents about taking the sibling to a movie later that day. Encouraging the parents to spend some time every day with the sibling is essential for the family to maintain positive relationships. Correct Encourage the sibling to help cut the birthday cake and scoop ice cream. Becoming involved with activities allows the sibling to interact with the parents and ill child. It is a source of attention for the sibling.

Which assessment finding is appropriate to consider for an immediate danger in the preschooler with symptoms of a burning throat, choking, and coughing along with nausea and vomiting after ingesting turpentine? Allergic reaction Aspiration and pneumonia Burned tissue and localized damage Time taken to peak serum salicylate levels

Aspiration and pneumonia Rationale Ingestion of hydrocarbons such as turpentine results in respiratory symptoms that point toward acute pulmonary involvement. Therefore the nurse directs the assessment toward symptoms of aspiration and chemical pneumonia. Allergic reactions can be anticipated in the child when a toxic plant or plant part is ingested. The ingestion of a solid corrosive agent, such as batteries or clinitest tablets, burns tissues, because solid products tend to stick, thereby causing localized damage. The time taken to peak serum salicylate levels may occur with aspirin ingestion, and the time taken can vary with enteric aspirin or the presence of bezoars.

aspirin toxicity manifestations

Aspirin toxicity early symptoms n/v, tinnitus, hyperventilation, aspirin toxicity late symptoms hyperactivity, confusion, seizures, renal failure, resp failure chronic aspirin toxicity symptoms subtle onset and nonspecific often mistaken for viral illness, bleeding tendencies toxic dose of aspirin 300-500 mg/kg

A 6-year-old child comes to the emergency department and presents with respiratory distress from gasoline skin exposure. Which action should the nurse take if the child becomes unconscious? Administer naloxone Administer activated charcoal Assess and support CNS function Assess and support cardiorespiratory function

Assess and support cardiorespiratory function If the child loses consciousness, assessment of the cardiorespiratory functions is necessary. If deficits are noted, provide proper support.

The nurse is caring for a child who has sustained an acceleration-deceleration head injury. Which actions should the nurse take in assessing this patient? Select all that apply. Check child's gag reflex. Assess child for retinal injury. Check child for burns and bruising. Assess for associated extremity sprain. Contact health care provider because child needs head computed tomography (CT).

Assess child for retinal injury. An acceleration-deceleration head injury occurs with "shaken baby" syndrome, which is associated with retinal tears and hemorrhaging. This child will need to be assessed for retinal damage. Correct Check child for burns and bruising. Acceleration-deceleration injuries are associated with child abuse, and so checking for other signs of abuse will be necessary. Correct Assess for associated extremity sprain. Extremity sprains are not an associated condition occurring with acceleration-deceleration head injuries. They may however exist separately if child has been abused. Correct Contact health care provider because child needs head computed tomography (CT). The child has suffered from an acceleration-deceleration head injury and may have an epidural hematoma that will need to be diagnosed by a head CT scan.

The nurse assesses a pediatric patient and finds deficits in speech. What additional assessment does the nurse perform to gather more data about the patient's speech deficit? Select all that apply. Assess for drowsiness and jitteriness. Evaluate for hearing loss and deafness. Determine if oral-motor weakness is present. Observe interactions between child and parents. Inquire about potential nonaccidental injury since deficits in speech usually indicate history of abuse.

Assess for drowsiness and jitteriness. As part of Glasgow coma scale, the nurse will first assess if child's cognition is intact before assessing other reasons for speech deficit. If GCS is low, patient may require immediate intervention. Correct Evaluate for hearing loss and deafness. Once GCS is determined to be adequate, nurse can assess for other issues such as hearing impairment. Correct Determine if oral-motor weakness is present. Occasionally, there will be a muscular weakness or a nerve not functioning properly that can cause a speech deficit. Correct Observe interactions between child and parents. A complete assessment of speech dysfunction will include listening to and observing interactions. This may be a clue to many issues that could cause speech deficit, including autism.

A newborn infant presents to the emergency department with papilledema. Which assessment does the nurse perform first? Examine gross motor skills. Assess for patent fontanels. Measure head circumference. Evaluate the primitive reflexes.

Assess for patent fontanels. Infants typically do not experience papilledema because the patent fontanels allow for increased intracranial pressure; whereas, an older child or adult has no fontanels and can exhibit papilledema in reaction to increased intracranial pressure (ICP). Papilledema in infants can be a sign of immature closure of the fontanelles.

A 12-year-old patient presents with ataxia and poor coordination. A head CT reveals a tumor in the posterior fossa. Which action is priority for this patient? Assess serum electrolytes Assess for signs of high ICP Administer pain medication Prepare for lumbar puncture

Assess for signs of high ICP Tumors of the posterior fossa are often accompanied by increased ICP. This can lead to serious complications and should be evaluated immediately.

The nurse is caring for a child immediately after surgical resection of a brain tumor. The nurse notes respirations 14 per minute, BP 108/88, HR 76. Which immediate action should the nurse take? Discuss rehabilitation options Administer pain medication Assess level of consciousness Locate academic support for the patient

Assess level of consciousness All patients should be monitored for signs and symptoms of increased intracranial pressure after surgery. The patient's mental status should be assessed once hemodynamic stability is determined. Both vital sign and mental status measurements should be recorded frequently.

A two-year-old child diagnosed with Hodgkin Lymphoma presents with enlarged neck lymph nodes, and an x-ray reveals widening of the mediastinum. Which action should the nurse take first? Prepare for surgical excision Auscultate for bowel sounds Assess the child's airway for patency Educate parents or caregiver on chemotherapy

Assess the child's airway for patency The nurse should assess the airway patency of a child with cervical lymphadenopathy and mediastinal disease from Hodgkin lymphoma, because swelling of cervical lymph nodes and mediastinal disease can cause airway obstruction.

What is the initial action in the emergency treatment of poisoning in a child? Locate the poison. Assess the child. Prevent absorption of the poison. Terminate exposure to the toxic substance.

Assess the child. Correct The initial step in treating a poisoning is to assess the child. Then treat immediate life-threatening conditions and initiate cardiopulmonary resuscitation if indicated. Locating the poison, preventing absorption of the poison, and terminating exposure to the toxic substance are important but none of these is the first step

A teenager is having an acute exacerbation of cystic fibrosis. The parents of the teen state, "We were doing so well coping before this episode. We don't even know what to do." What should the nurse do to help the parents? Discuss the need to remain consistent in their activities to maintain normalcy in the family Help the parents to determine why they are not able to cope with the teen's illness like they have in the past Assist the parents in identifying what usually helps the family to cope to see if that can be used during the exacerbation of the chronic illness Explain the illness trajectory of cystic fibrosis so that the parents know that they need to be prepared for the exacerbations to continue to happen

Assist the parents in identifying what usually helps the family to cope to see if that can be used during the exacerbation of the chronic illness Under times of acute stress, some families may simply need gentle reminders and support to maintain their resiliency.

The nurse is assigned to an 8-year-old child who has been hospitalized with a chronic illness. The child has a tracheostomy and a parent is rooming-in. The parent insists on providing nearly all of the child's care and tells the nurses how to care for the child. When planning the child's care, the primary nurse should recognize that the parent is exhibiting which behavior? Controlling or demanding Confidence in assuming the nurse's role Assuming the role of expert in care of the child Fear of allowing the nurses to function independently

Assuming the role of expert in care of the child Rationale The nurse recognizes that the philosophy of family-centered care states that the parents are the experts in the care of their child. Because this parent cares for the child with complex health needs at home, the parent is most familiar with the care requirements and routines that work best for the child. The parent is in no way being controlling or demanding. The nurse's role includes assessment and evaluation, not just the implementation phase; therefore, the parent is not assuming the nurse's role. In fact, the parent is participating in the care of the child in the role as parent. In family-centered care, it is critical that the nurse works collaboratively with the family in caring for the child. No evidence indicates that the parent is afraid to allow the nurses to function in their role.

Bullying can be common during the school-age years. What is a priority concept for the nurse to teach when educating parents of school-agers on bullying? Can have a lasting effect on children. Is not a significant threat to self-concept. Is rarely based on anything that is concrete. Is usually ignored by the child who is being teased.

Can have a lasting effect on children. Correct Bullying in this age-group is common and can have a long-lasting effect. Increasing awareness of differences, especially when accompanied by unkind comments and taunts from others, may make a child feel inferior and undesirable. Physical impairments such as hearing or visual defects, ears that "stick out," or birth marks assume great importance

The nurse is assigned to an 8-year-old child who has been hospitalized with a chronic illness. The child has a tracheostomy and a parent is rooming-in. The parent insists on providing nearly all of the child's care and tells the nurses how to care for the child. When planning the child's care, the primary nurse should recognize that the parent is exhibiting which behavior? Controlling or demanding Confidence in assuming the nurse's role Assuming the role of expert in care of the child Fear of allowing the nurses to function independently

Assuming the role of expert in care of the child Rationale The nurse recognizes that the philosophy of family-centered care states that the parents are the experts in the care of their child. Because this parent cares for the child with complex health needs at home, the parent is most familiar with the care requirements and routines that work best for the child. The parent is in no way being controlling or demanding. The nurse's role includes assessment and evaluation, not just the implementation phase; therefore, the parent is not assuming the nurse's role. In fact, the parent is participating in the care of the child in the role as parent. In family-centered care, it is critical that the nurse works collaboratively with the family in caring for the child. No evidence indicates that the parent is afraid to allow the nurses to function in their role.

A nurse is caring for a four-year-old child who must have surgery. The child will require an IV and an indwelling urinary catheter. How can the nurse best prepare the child for this experience? Select all that apply. Assure the child that a nurse or parent will be caring for him or her at all times. Let the child know that everything will be fine and that worrying is not necessary. Ensure that the child is told about the IV and urinary catheter right before going to surgery. Show the child a doll that has an IV and a urinary catheter in place while explaining what each is for. Give simple explanations about the surgery and what the child should expect a few days before the surgery.

Assure the child that a nurse or parent will be caring for him or her at all times. Preschool-aged children often fear being left alone, so accurately assuring the child that someone will be with him or her would be an important nursing intervention. Show the child a doll that has an IV and a urinary catheter in place while explaining what each is for. Preschool-aged children need more than just a verbal explanation. They need visual aids such as dolls or pictures to understand information. Give simple explanations about the surgery and what the child should expect a few days before the surgery. Simple explanations are important and should be started days before a major procedure to allow the preschool-aged child to process the information.Show the child a doll that has an IV and a urinary catheter in place while explaining what each is for. .

The nurse is educating the parents of a child with leukemia about necessary homecare. During the teaching session, the mother states, "We plan to let him return to baseball practice next week." Which information is most important for the nurse to include in the response? Avoid physically exhausting activities. Apply ample amounts of sunscreen on child's skin while outside. Assess the child's mucus membranes for signs of dehydration frequently while playing. Notify the health care provider immediately if the child has shortness of breath while playing.

Avoid physically exhausting activities. Providing information about activities to be avoided, such as baseball, is the most important teaching that the nurse should provide the mother because patients with leukemia are often thrombocytopenic and neutropenic, and engaging in physically exhausting activities can trigger bleeding, injury, and infection.

Which nursing interventions should be done if a child receiving radiotherapy for cancer has dry desquamation? Select all that apply. Avoid referring to skin changes as a "burn." Advise the child and family to avoid exposure to the sun. Advise the child and family to apply lotions to moisten the skin. Advise the child and family to use soap daily to wash the skin well. Advise the child and family to keep the skin markings for radiation intact.

Avoid referring to skin changes as a "burn." Advise the child and family to avoid exposure to the sun. Advise the child and family to keep the skin markings for radiation intact. Rationale Radiotherapy may cause dry or moist desquamation as a side effect. The nurse should not refer the skin changes as a "burn" because the term may give the family a bad impression. The nurse should advise the child and family to avoid sun exposure because it may aggravate the skin condition. The nurse should advise the child and family to keep the skin markings for radiation fields intact. Lotions or creams should not be used on the skin changes because those products may cause a negative reaction to the affected area. The skin should be washed daily, but soap must be used sparingly.

what is the newborn etiology of conjunctivitis

Chlamydia trachomatis Neisseria gonorrheae Chemical (24 hours after birth) Blocked tear duct bacterial which type of conjunctivitis is treated w/ Bacitracin (Polysporin), sodium sulfaceteamide (Sulamyd) or trimethoprim and polymyxin (Polytrim)

The nurse is caring for a child with a Wilms tumor. Which is the most important nursing intervention before surgery? Avoiding abdominal palpation Closely monitoring the arterial blood gases Preparing the child and family for long-term dialysis Preparing the child and family for renal transplantation

Avoiding abdominal palpation Rationale Wilms tumor is encapsulated. It is extremely important to avoid any palpation of the mass to minimize the risk of dissemination of cancer cells to adjacent tissues and other sites. A sign should be placed over the bed indicating that no abdominal palpation should be conducted. Preoperative monitoring of arterial blood gases is not indicated for this abdominal surgery. Long-term dialysis is not indicated unless both kidneys must be removed. This option is considered a last resort. If both kidneys are involved, preoperative irradiation, chemotherapy, or both are used to minimize the tumor size. Renal transplantation is a last resort if both kidneys need to be removed and a compatible living donor exists.

Which nursing intervention is appropriate when treating a child that has consumed dishwasher detergent? Select all that apply. Inducing emesis Offering oral intake Avoiding neutralizing Administering analgesics Maintaining patient airway

Avoiding neutralizing Administering analgesics Maintaining patient airway Rationale Management of poisoning requires emergency treatment. Neutralizing should be avoided because it may cause an exothermic reaction; that is, it will produce heat, which increases symptoms or produces a thermal burn in addition to the chemical burn. A patent airway should be maintained because severe respiratory depression may produce hazardous effects. Analgesics should be administered to relieve pain. The nurse should avoid inducing emesis, because it can redamage the mucosa. Oral intake should be avoided, because it may worsen the condition.

What is an important nursing responsibility when dealing with a family experiencing the loss of an infant from sudden infant death syndrome (SIDS)? a. Discourage the parents from making a last visit with the infant. b. Make a follow-up home visit to the parents as soon as possible after the child's death. c. Explain how SIDS could have been predicted and prevented. d. Interview the parents in depth concerning the circumstances surrounding the child's death.

B A competent, qualified professional should visit the family at home as soon as possible after the death. Printed information about SIDS should be provided to the family. Parents should be allowed and encouraged to make a last visit with their child. SIDS cannot always be prevented or predicted, but parents can take steps to reduce the risk (e.g., supine sleeping, removing blankets and pillows from the crib, and not smoking). Discussions about the cause only increase parental guilt. The parents should be asked only factual questions to determine the cause of death.

The nurse is interviewing the father of a 10-month-old girl. The child is playing on the floor when she notices an electrical outlet and reaches up to touch it. Her father says "no" firmly and moves her away from the outlet. The nurse should use this opportunity to teach the father what? a. That the child should be given a time-out b. That the child is old enough to understand the word "no" c. That the child will learn safety issues better if she is spanked d. That the child should already know that electrical outlets are dangerous

B By age 10 months, children are able to associate meaning with words. The father is using both verbal and physical cues to alert the child to dangerous situations. A time-out is not appropriate. The child is just learning about the environment. Physical discipline should be avoided. The 10-month-old child is too young to understand the purpose of an electrical outlet.

What identified characteristics occur more frequently in parents who abuse their children? (Select all that apply.) a. Older parents b. Socially isolated c. Middle class parents d. Single-parent families e. Few supportive relationships

B, D, E Abusive families are often socially isolated and have few supportive relationships. Single-parent families are at higher risk for abuse. Younger parents more often are abusers of their children. Abusive parents have stressors such as low-income circumstances, with little education, and are not middle class parents.

A patient is admitted with Neisseria meningitidis. What should the nurse monitor for that indicates infection progression? Select all that apply. Fatigue that is not relieved by rest Blood glucose levels every 6 hours Purpura over the trunk of the patient Blood pressure and heart rate changes Vomiting, diarrhea, and abdominal cramps

Blood glucose levels every 6 hours Adrenal insufficiency is a condition that can develop with Neisseria meningitidis infection, and therefore the nurse must monitor for hypoglycemia and symptoms of it such as sweating and nausea. Correct Purpura over the trunk of the patient Purpura and petechiae are manifestations of Neisseria meningitidis that occur at the time of presentation or shortly thereafter. Correct Blood pressure and heart rate changes Septic shock indicates the infection is progressing rapidly. In addition to common symptoms of fever, hypotension and tachycardia are indicative of septicemia.

A father is concerned that his school-age child has started to lie on occasion. What information would the nurse provide to the parent in this situation? Be particular about remaining truthful in your relationship with your child. Do not worry because your child cannot yet distinguish between fantasy and reality. Resort to corporal punishment so that the dishonest behavior is rooted out at an early stage. Confront your child about the offensive behavior because it is normal in young children.

Be particular about remaining truthful in your relationship with your child. Rationale The nurse should help the father understand the importance of his own behavior as a role model and of being truthful in his relationship with his child. While parents can be reassured that all children lie sometimes, they can discuss the issue with the children directly to impress on them how much of their own security and respect is lost when they are not believed. Preschool children often have difficulty distinguishing between fact and fantasy; they often lie, even though they do not have the cognitive capacity to deliberately mislead. School-age children still continue this behavior, but they can distinguish between what is real and what is make-believe. Corporal punishment should not be used, because it is of limited value and is associated with increasingly disruptive behavior in children.

What time frame has the Advisory Committee on Immunization Practices (ACIP) of the Centers for Disease Control and Prevention (CDC) and American College of Obstetricians and Gynecologists recommended that pregnant adolescents and women who are not protected against pertussis receive the tetanus, diphtheria, and pertussis (Tdap) vaccine? Between 27 and 36 weeks of gestation or postpartum before discharge from the hospital During the first prenatal visit when pregnancy is confirmed The vaccine should be administered 24 hr prior to delivery This vaccine is only recommended during the first trimester

Between 27 and 36 weeks of gestation or postpartum before discharge from the hospital Correct The ACIP of the CDC and American College of Obstetricians and Gynecologists has recommended that pregnant adolescents and women who are not protected against pertussis receive the Tdap vaccine optimally between 27 and 36 weeks of gestation or postpartum before discharge from the hospital. The vaccine is not recommended during the first trimester. The vaccine is not recommended between 27 and 36 weeks to allot for antibody formation that will protect the mother and passive immunity to the infant. The vaccine is not recommended during the first trimester

The mother of a child is hepatitis B surface antigen (HBsAg) negative, when would receive his or her first dose of the hepatitis B virus (HBV) vaccine? 2 months of age, at the first well-child visit. Birth before discharge from the hospital. 6 months of age, at the third well-child visit. No time (this vaccine is not currently recommended).

Birth before discharge from the hospital. Correct It is recommended that newborns receive the hepatitis B vaccine before hospital discharge if the mother is HBsAg negative. The second dose of the vaccine is given at the first well-child visit. The third dose of the vaccine is given at the third well-child visit. The Advisory Committee on Immunization Practices of the Centers for Disease Control and Prevention and the Committee on Infectious Diseases of the American Academy of Pediatrics govern the recommendations for immunization, which include the hepatitis B virus vaccine

The nurse is caring for a child with a rapid breathing, headache, and the smell of wintergreen on the skin and clothes. Which additional signs and symptoms would the nurse assess for? Select all that apply. Bleeding Vomiting Confusion Diaphoresis Hyperglycemia Peripheral edema

Bleeding Wintergreen is a salicylate and is often used as a safe alternative to aspirin. The nurse would assess for bleeding and bruising related to the inhibition of prothrombin, decreased platelets levels, and capillary fragility. Correct Vomiting Wintergreen is a salicylate and is often used as a safe alternative to aspirin. Oral poisoning typically manifests nausea and vomiting related to GI irritation. Correct Confusion Wintergreen is a salicylate and is often used as a safe alternative to aspirin. Confusion, seizures, and coma are all related to the CNS effects of salicylate poisoning. Correct Diaphoresis Wintergreen is a salicylate and is often used as a safe alternative to aspirin. Dehydration, sweating, and decreased urine production are typical in salicylate poisoning.

Which drug is appropriate to administer to a child with a cough for 7 weeks, gagging, and emesis? Acyclovir Valacyclovir Clarithromycin Varicella-zoster immune globulin

Clarithromycin Rationale The child is suffering from pertussis. The drug clarithromycin, which is an antibiotic to treat pertussis, should be administered to the child. Varicella-zoster immune globulin, acyclovir, and valacyclovir are administered to immunocompromised children to treat varicella infection.

When caring for a fourteen-year-old child with traumatic brain injury, which complications must be addressed immediately? Select all that apply. Blood pressure 80/40 mm Hg Decreased muscle tone bilaterally Weak deep tendon reflex responses Oxygen saturation of 88% on room air Respiratory rate of 8 breaths per minute

Blood pressure 80/40 mm Hg The care for a child with traumatic brain injury includes aggressively managing hypotension of the patient because the injury could affect centers of the brain that control basic body functions such as vascular tension. Weak deep tendon reflex responses Weak deep tendon reflexes are concerning for syndrome of inappropriate secretion of antidiuretic hormone (SIADH) and for progressive increased intracranial pressure (ICP). Oxygen saturation of 88% on room air The care for a child with traumatic brain injury includes monitoring for and correcting hypoxemia of the patient that could develop because centers of the brain that control basic body functions such as breathing are altered. Respiratory rate of 8 breaths per minute The care plan for a child with traumatic brain injury includes monitoring for hypoventilation of the patient because the injury could affect the centers of the brain that control basic body functions such as breathing.

preadolescence 2

Body image is what children think about their bodies. School-age children are knowledgeable about the human body, and social development during this period focuses to a large extent on the body and its capabilities. During middle childhood, children may engage in what is considered to be antisocial behavior. Previously well-behaved children may engage in lying, stealing, and cheating. Such behaviors are disturbing and challenging to parents. Likes and dislikes established at an early age continue in middle childhood, although preferences for single foods subside, and children develop a taste for a variety of foods. The availability of junk foods, irregular family meals, and schedules of working parents often hamper optimal nutrition. The development of physical fitness is a goal for all children. Throughout middle childhood, children's increasing capabilities and adaptability permit greater speed and effort in physical activities. Larger, stronger muscles with greater efficiency and skill permit longer and increasingly strenuous play without exhaustion. Consequently, parents should expect and encourage a larger amount of physical activity. Children spend a significant amount of time each day involved in media-related activities. Because of the long periods of exposure, the media have more time to develop children's attitudes than do parents and teachers. Recommendations for media use include limit media time, monitor content, and increase access to educational games and information. Because the permanent teeth erupt during the school-age years, dental hygiene and regular attention to dental caries are important parts of health supervision during this period. Potential dental problems include caries, periodontal disease, malocclusion, and dental injury. Many children experience some form of sex play during or before preadolescence as a response to normal curiosity, not as a result of love or sexual urges. Increased socialization and media exposure make the school years an ideal time for sex education. School health ideally offers programs that include health appraisal, emergency care, safety education, communicable disease control, counseling, guidance, and health education with adjustment to individual student needs. The most common cause of severe injury and death in school-age children is motor vehicle accidents, either as a pedestrian or passenger. School-age children should be instructed on traffic safety and traffic signs. When in the car, school-age children should always be buckled properly in a weight-, height-, and age-appropriate seat and sit in the rear seat of any vehicle. Injury prevention should be directed toward safety education, provision of safe play areas and equipment, and good supervision of sports activities.

A child is suspected of having leukemia. Which is the most definitive method for the diagnosis of leukemia? Lumbar puncture Physical assessments Peripheral blood smear Bone marrow aspirate or biopsy

Bone marrow aspirate or biopsy Rationale A bone marrow aspirate or biopsy shows a monotonous infiltration of blast cells, which is a definitive diagnosis of leukemia. A lumbar puncture is done to collect samples to determine whether there is any central nervous system involvement. Physical manifestations are generally not severe and in most instances, physical signs and symptoms are few. A peripheral blood smear is done initially to check for immature forms of leukocytes and blood counts but is not a definitive method.

A young girl complains of pain in her right leg that is relieved when she places the leg in a flexed position. The girl's mother reports that her daughter frequently limps and is unable to hold heavy objects for even a short amount of time. On assessment the nurse notes a small palpable mass on the girl's right lower leg. Which diagnosis do these clinical manifestations suggest? Arthritis Bone tumor Osteomyelitis Idiopathic scoliosis

Bone tumor Rationale Clinical manifestations of bone tumors include pain at the localized site that is relieved by flexion, limping, palpable mass, and inability to hold heavy objects. These symptoms do not suggest arthritis, osteomyelitis, or idiopathic scoliosis

Which descriptors are common to both neuroblastoma and retinoblastoma? Select all that apply. Both are solid tumors. Both are abdominal tumors. Both can regress spontaneously. Both are commonly seen in adolescents. Both are commonly diagnosed after metastasis

Both are solid tumors. Both can regress spontaneously. Rationale Both neuroblastoma and retinoblastoma are solid tumors that may regress spontaneously. Neuroblastoma is an abdominal tumor; retinoblastoma is an intraocular tumor. Both are commonly seen in infants and young children rather than adolescents. Neuroblastoma is a silent tumor commonly diagnosed after metastasis has occurred. Retinoblastoma is rarely a metastatic disease at the time of diagnosis.

Which nursing recommendation is appropriate for the parent of a child that has ingested a toxic dose of iron, vomited, and complained of gastric pain an hour ago? Administer activated charcoal. Observe the child closely for 2 more hours. Bring the child to the hospital immediately. Administer ipecac to induce vomiting if the child does not vomit again within 1 hour.

Bring the child to the hospital immediately. Rationale The child should be transported to the hospital immediately for assessment and possible gastric lavage. The period of concern for complications of iron toxicity is 30 minutes to 6 hours after ingestion. Activated charcoal does not bind iron and therefore is not a treatment option for this child. Ipecac is not recommended for poisonings.

A 7-year-old child has ingested a toxic dose of iron. The parent reports that the child vomited and had gastric pain an hour ago but "feels fine" now. The parent is not sure when the child ingested the iron tablets. What is the appropriate recommendation by the nurse? Administer activated charcoal. Observe the child closely for 2 more hr. Bring the child to the hospital immediately. Administer ipecac to induce vomiting if the child does not vomit again within 1 hr.

Bring the child to the hospital immediately. Correct The critical period for observation after the ingestion of iron is 30 minutes to 6 hr. The child has had gastric pain, which may be symptomatic of toxicity, and needs to be monitored and possibly receive medical intervention. Although activated charcoal may be necessary, evaluation is indicated first. The child needs to be evaluated immediately. Ipecac is not recommended after the ingestion of toxic substances

What is the most effective means of preventing dental caries in school-age children? Flossing the teeth at least three times per week Unsupervised brushing of teeth before bedtime Regular consumption of fluorinated drinking water Brushing teeth after meals, after snacks, and at bedtime

Brushing teeth after meals, after snacks, and at bedtime Rationale Brushing teeth after meals and snacks and at bedtime is the most effective means of preventing dental caries in school-age children. Teeth need to be flossed, after brushing, two or three times a day, not three times per week. Unsupervised brushing of teeth before bedtime is not effective in preventing dental caries in school-age children. Regular consumption of fluorinated drinking water is not the most effective means of preventing dental caries in school-age children.

The school nurse is asked to speak with the parents of a 10-year-old boy who has been bullying other children. What is the knowledge basis the nurse would use for this interaction? Bullying at this age is considered normal. Children who bully others usually join gangs. Bullying often manifests itself in children who witness violence or abuse at home. Bullying is a short-term problem that is generally outgrown by the end of the school-age years.

Bullying often manifests itself in children who witness violence or abuse at home. Rationale Bullying often manifests in children who may witness violence or abuse at home. Children who bully may also come from homes with low parental involvement. Bullying is a maladaptive response to poor relationships with peers and lack of group identification; therefore, it is not considered normal behavior. Children who chronically bully tend to be impulsive, easily frustrated, and at increased risk for dropping out of school, but there is no direct correlation between bullying and joining a gang. Children who bully may be at risk for long-term psychological disturbances and psychiatric symptoms. Future problems for bullies may include violence, substance abuse, and criminal convictions, which often occur in adulthood.

A nurse is talking with the parent of a child with a newly diagnosed terminal illness. What is the most appropriate way for the nurse to assess how the parent is coping with the diagnosis? By remaining silent By changing the subject By asking an open-ended question By observing how the parent interacts with the child

By asking an open-ended question Rationale Asking the parent an open-ended question such as, "What do you do when something is worrying you about your child's diagnosis?" will elicit information about the parent's coping skills, support system, and ability to communicate. This will allow the nurse to identify appropriate resources for the family. Remaining silent in this situation will not help the nurse assess how the parent is coping with the child's diagnosis; nor will changing the subject. Although observing how the parent interacts with the child will give the nurse valuable information about the parent/child relationship, it is not the most appropriate way for the nurse to assess how the parent is coping with the diagnosis.

What is an appropriate action when an infant becomes apneic? a. Shake vigorously. b. Roll the infant's head to the side. c. Gently stimulate the trunk by patting or rubbing. d. Hold the infant by the feet upside down with the head supported.

C If an infant is apneic, the infant's trunk should be gently stimulated by patting or rubbing. If the infant is prone, turn onto the back. Vigorous shaking, rolling of the head, and hanging the child upside down can cause injury and should not be done.

The nurse is teaching parents about safety for their "latchkey" children. What should the nurse include in the teaching session? (Select all that apply.) a. Teach the child first-aid procedures. b. Keep the key in an easy place to find. c. Teach the child weather-related safety. d. Teach the child to open the door for delivery people. e. Emphasize fire safety rules and conduct practice fire drills.

C, E Safety for "latchkey" children includes teaching the child first-aid procedures, teaching the child weather-related safety, and emphasizing fire safety rules and conducting practice fire drills. Teach the child not to display keys and to always lock doors. The child should be taught to not open the door to anyone, even delivery people.

Which nursing intervention is appropriate when finding a child in stable condition has consumed dishwashing liquid on a home visit? Assess the airway, breathing, and circulation of the child. Induce vomiting to clear the stomach of suspected poison. Call the poison control center before starting any intervention. Place the child in the supine position on the floor or a flat surface.

Call the poison control center before starting any intervention. Rationale If the child is stable, the nurse's first step should be to call the poison control center. Any child who has ingested poison should be placed in a side-lying, sitting, or kneeling position to prevent the risk of aspiration. Because the child is conscious and coherent, the nurse does not need to assess the airway, breathing, or circulation. Inducing vomiting is generally contraindicated, because it can cause aspiration pneumonia or instant death in some cases.

What is included in the postoperative care of a preschool child who has had a brain tumor removed? Colorless drainage is to be expected. Analgesics are contraindicated because of altered consciousness. Positioning is on the operative side in the Trendelenburg position. Carefully monitor fluids because of cerebral edema.

Carefully monitor fluids because of cerebral edema. Correct Because of cerebral edema and the danger of increased intracranial pressure postoperatively, fluids are carefully monitored. Colorless drainage may be leakage of cerebrospinal fluid from the incision site. This needs to be reported as soon as possible. Analgesics can be used for postoperative pain. The child should not be positioned in the Trendelenburg position postoperatively

What assessment findings would be anticipated is a child with depression? Increased range of affective response Preoccupation with the need to perform well in school Change in appetite, resulting in weight loss or gain Tendency to prefer play to schoolwork

Change in appetite, resulting in weight loss or gain Correct Physiologic characteristics of children with depression include a change in appetite resulting in weight loss or gain, nonspecific complaints of not feeling well, alterations in sleeping pattern (insomnia or hypersomnia), and constipation. Children who are depressed have sad facial expressions with an absence or diminished range of affective response. Children who are depressed lack interest in doing homework or achieving in school, resulting in lower grades. These children withdraw from previously enjoyed activities and engage in solitary play or work. Schoolwork is not replaced by pla

Which sign does the nurse recognize as the most distressing for parents to observe in the final hours of their child's life? Confusion Loss of sensation Changes in heart rate Changes in respiratory pattern

Changes in respiratory pattern Rationale Parents report that changes in the respiratory pattern are the most distressing for them to observe in the final hours of their child's life. This is because the dying patient's respirations become labored, with deep breaths and long periods of apnea. Although changes in confusion, loss of sensation, and changes in heart rate are difficult for many parents to watch, the most distressing change is in the child's respiratory pattern.

A group of boys ages 9 and 10 have formed a "boys only" club that is open to neighborhood and school friends who have skateboards. What is this considered in terms of social development? Behavior that encourages bullying and sexism Behavior that reinforces poor peer relationships Characteristic of social development of this age Characteristic of children who are later at risk for membership in gangs

Characteristic of social development of this age Correct One of the outstanding characteristics of middle childhood is the creation of formalized groups or clubs. Peer-group identification and association are essential to a child's socialization. Poor relationships with peers and a lack of group identification can contribute to bullying. Forming a boys-only club at this age does not have a direct correlation with later gang activity

A 3-year-old child is scheduled for surgery to remove a Wilms tumor from one kidney. The parents ask the nurse what treatments, if any, will be necessary after recovery from surgery. The nurse's explanation is based on which knowledge? Chemotherapy is usually not necessary. No additional treatments are usually necessary. Kidney transplant will be indicated within the year. Chemotherapy with or without radiotherapy is indicated.

Chemotherapy with or without radiotherapy is indicated. Rationale The choice of chemotherapy, radiotherapy, or both as treatment modalities will be based on the histologic pattern of the tumor. Chemotherapy with or without radiotherapy is usually indicated. Additional therapy of some type is indicated after the tumor is removed. Chemotherapy or radiotherapy, or both, may be indicated as a postsurgical intervention. Most children with Wilms tumor do not require renal transplant.

A 3-year-old child is scheduled for surgery to remove a Wilms tumor from one kidney. The parents ask the nurse what treatments, if any, will be necessary after recovery from surgery. The nurse's explanation is based on which knowledge? Chemotherapy is usually not necessary. No additional treatments are usually necessary. Kidney transplant will be indicated within the year. Chemotherapy with or without radiotherapy is indicated.

Chemotherapy with or without radiotherapy is indicated. Rationale The choice of chemotherapy, radiotherapy, or both as treatment modalities will be based on the histologic pattern of the tumor. Chemotherapy with or without radiotherapy is usually indicated. Additional therapy of some type is indicated after the tumor is removed. Chemotherapy or radiotherapy, or both, may be indicated as a postsurgical intervention. Most children with Wilms tumor do not require renal transplant.

The nurse is caring for a child with a terminal illness who is approaching death. What physical signs can the nurse expect the child to demonstrate? Loss of hearing before sight Cheyne-Stokes respiration Increased appetite and thirst Increased pulse and blood pressure

Cheyne-Stokes respiration Rationale Cheyne-Stokes respiration, in which there is a waxing and waning of depth of breathing punctuated by regular periods of apnea, are common as death approaches. Other physical signs of approaching death include loss of sensation, loss of senses (with sight lost before hearing), confusion, muscle weakness, loss of bowel and bladder control, decreased appetite and thirst, difficulty swallowing, and weak, slow pulse and decreased blood pressure.

The nurse would implement contact and airborne isolation precautions for a child who is hospitalized with which illness? Mumps Chickenpox Exanthema subitum (roseola) Erythema infectiosum (fifth disease)

Chickenpox Rationale Chickenpox is communicable through direct contact, droplet spread, and contaminated objects. The Center for Disease Control recommends contact and airborne isolation for these children until lesions have crusted over. Mumps is transmitted by way of direct contact with saliva of an infected person and is most communicable before the onset of swelling. The transmission and cause of exanthema subitum (roseola) are unknown. Erythema infectiosum (fifth disease) is communicable before the onset of symptoms.

Which condition has a child with a fever, uneasiness, and vesicles on the face and head likely developed? Diphtheria Chickenpox Exanthem subitum Erythema infectiosum

Chickenpox Rationale The child has developed chickenpox. Children with chickenpox will experience slight fever and malaise. Development of vesicles on the upper extremities first instead of the lower extremities is an indication of chickenpox. Symptoms of diphtheria vary according to the anatomic location of the pseudomembrane. Symptoms of nasal diphtheria resemble the symptoms of the common cold. Exanthem subitum is associated with a high-grade fever above 103 o F. Development of rashes, abdominal pain, and vomiting are the symptoms of erythema infectiosum.

What disease would require strict isolation of the patient? Mumps Chickenpox Exanthema subitum (roseola) Erythema infectiosum (fifth disease)

Chickenpox Correct Chickenpox is communicable through direct contact, droplet spread, and contaminated objects. Mumps is transmitted from direct contact with the saliva of the infected person and is most communicable before the onset of swelling. The transmission and source of the viral infection exanthema subitum (roseola) is unknown. Erythema infectiosum is communicable before the onset of symptoms.

The nurse cares for a ten-year-old recovering from meningitis. The nurse is most concerned about which assessment finding? Child reports a persistent headache. Child fails to respond when called by name. Parents report the child is not engaging in activities. Parents insist the child complete missed schoolwork.

Child fails to respond when called by name. The child may be experiencing hearing loss secondary to neurologic damage sustained during the meningitis episode. This will not improve with time and needs to be addressed.

The nurse assesses a two-year-old child with papilledema related to hydrocephalus. Which finding causes the nurse the most concern? Child is holding head and crying Child is lethargic, responding to voice Child projectile vomits when sitting up Child has an increased head circumference

Child has an increased head circumference Sutures and fontanels close by 18 months of age. If sutures and fontanels are closed, child should not have an enlarged head circumference. This finding indicates there is an additional problem that would be most concerning to the nurse.

Place the events of host defense in order beginning with pathogen invasion?

Child inhales as sibling next to her, with influenza, coughs Replication of pathogen Host recognizes the invasion Immune response is activated The first stage of infection begins with colonization of the host by the pathogen. Microorganisms invade either by adhering to tissues or by invading cells. Initially, replication of the pathogen does not cause tissue damage, and colonization can occur without development of a clinical infection. As the host"recognizes" the invasion, the defense system—the immune response—is activated. The two components of the immune response are the innate, nonspecific immune response and the adaptive, specific immune response: cell mediated and humoral.

A 5-year-old child is admitted with complications related to an Arnold-Chiari malformation and myelomeningocele. What assessment findings cause the nurse to be concerned? Child is underweight for age. Child reports neck pain and stiffness. Child reports burning with urination. Child is experiencing severe headache.

Child is experiencing severe headache. Arnold-Chiari can result in hydrocephalus. A headache may indicate increasing ICP. If the child has a shunt, this may indicate a malfunction.

Which statement is correct regarding the administration of varicella and measles, mumps, and rubella vaccines? Immunocompromised children may receive them. Children can be vaccinated when they have the common cold. They are allowed for children with recently acquired passive immunity. Injection sites can be close to each other with simultaneous immunizations.

Children can be vaccinated when they have the common cold. Rationale Immunization of children with varicella and measles, mumps, and rubella vaccines who have the common cold will not produce any contraindicating reactions. Immunization should not be provided to immunocompromised children. Immunization with varicella and measles, mumps, and rubella vaccines should not be carried out in children who have recently received acquired immunity through transfusion. Immunization should be postponed in these cases for a minimum of 3 months. When two immunizations are given simultaneously to children, injection sites should be far away from each other.

Which statement by the nurse is appropriate when educating parents about the differences between nightmares and sleep terrors? Select all that apply. Children do not remember sleep terrors. Nightmares occur in rapid eye movement sleep. Nightmares occur in the first few hours of sleep. Children return to sleep from sleep terrors rapidly. Nightmares are not remembered by older children.

Children do not remember sleep terrors. Nightmares occur in rapid eye movement sleep. Children return to sleep from sleep terrors rapidly. Rationale Nightmares and sleep terrors are common in children. Children do not remember sleep terrors. A child who has sleep terrors usually returns back to sleep quickly. Nightmares occur during rapid eye movement (REM) sleep, which makes it difficult for the child to go back to sleep. Nightmares are usually remembered. Nightmares occur during the latter half of the sleep cycle.

What is the primary reason for universal screening of young children for lead poisoning? Children with lead poisoning rarely have symptoms. Most children are exposed to lead through herbal products. Water and food in the United States are usually contaminated with lead. Most children in the United States are exposed to toxic amounts of lead.

Children with lead poisoning rarely have symptoms. Rationale In the early stages of lead poisoning, children are asymptomatic. Water and food in the United States are not highly contaminated with lead. Risk is high in homes painted before 1978, when paints still contained high levels of lead. Universal screening will identify children who may receive lead in herbal supplements or environmental exposure, if applicable.

A nurse is discussing anticipatory socialization in children ready to go to school for the first time with parents of preschool-age children. What information will the nurse include to educate the parents? Children going to school for the first time do not have a realistic concept of what school involves. Clinging behavior by parents negatively affects the child's ability to adjust to the school environment. Early childhood programming that stresses social aspects is particularly effective in facilitating later academic achievement. Middle-class children have to make a lot of adjustment, because the values reflected in school are far from what they are used to at home.

Clinging behavior by parents negatively affects the child's ability to adjust to the school environment. Rationale Some parents may express their unconscious attempts to delay their child's maturity by clinging behavior, particularly with their youngest child. This behavior in the parents negatively affects the child's ability to adjust in the school environment. Most children have a fairly realistic concept of what school involves by the time they enter school. This is facilitated by the information they receive regarding the role of the pupil from the parents, playmates, and the media, as well as their experience in daycare or preschool and kindergarten. Early childhood programming that stresses cognitive more than social aspects appears to be more effective in facilitating later academic achievement. Schools tend to reflect dominant middle-class customs and values; therefore, middle-class children have fewer adjustments to make and less to learn about expected behavior.

A child with non-Hodgkin lymphoma is undergoing chemotherapy with cyclophosphamide. What are the appropriate nursing actions to prevent sterile hemorrhagic cystitis in this child? Select all that apply. Close monitoring for presence of hematuria Administration of mesna to inhibit urotoxicity Frequent voiding immediately after the urge is felt Administration of cyclophosphamide at nighttime before bed Limited fluid intake up to the recommended daily fluid requirement of 2L/m 2/day

Close monitoring for presence of hematuria Administration of mesna to inhibit urotoxicity Frequent voiding immediately after the urge is felt Rationale Use of cyclophosphamide may result in sterile hemorrhagic cystitis, which is characterized by microscopic hematuria or gross hemorrhage. The nurse should constantly evaluate the child's urine for hematuria. The drug mesna inhibits the urotoxicity of cyclophosphamide. The bladder works as a reservoir for the metabolites produced from cyclophosphamide. To reduce the contact time, the child should void immediately after feeling the urge. Cyclophosphamide should be administered in the early morning so that the child can have sufficient fluids and frequent voiding. The child should take at least one and a half times more fluids than the recommended daily requirement of 2L/m 2/day to wash out the metabolites from the body.

The postoperative care of a preschool child who has had a brain tumor removed should include which action? No administration of analgesics Recording of colorless drainage as normal on the nurses' notes Placement of the child on the right side in the Trendelenburg position Close supervision of the child while the child is regaining consciousness

Close supervision of the child while the child is regaining consciousness Rationale The child must be observed closely, with careful and frequent assessment of the vital signs and monitoring for signs of increasing intracranial pressure. Any changes should be reported immediately to the practitioner. Colorless drainage may represent cerebrospinal fluid leaking from the incision site. This needs to be reported to the practitioner immediately. The child should not be positioned in the Trendelenburg position after surgery. Analgesics may be used for postoperative pain as needed.

The postoperative care of a preschool child who has had a brain tumor removed should include which action? No administration of analgesics Recording of colorless drainage as normal on the nurses' notes Placement of the child on the right side in the Trendelenburg position Close supervision of the child while the child is regaining consciousness

Close supervision of the child while the child is regaining consciousness Rationale The child must be observed closely, with careful and frequent assessment of the vital signs and monitoring for signs of increasing intracranial pressure. Any changes should be reported immediately to the practitioner. Colorless drainage may represent cerebrospinal fluid leaking from the incision site. This needs to be reported to the practitioner immediately. The child should not be positioned in the Trendelenburg position after surgery. Analgesics may be used for postoperative pain as needed.

The parents of 9-year-old twins tell the nurse, "They've filled up their bedroom with collections of rocks, shells, stamps, and cars." The nurse would understand this behavior demonstrates what for this child? Giftedness Typical "twin" behavior Cognitive development at this age Psychosocial development at this age

Cognitive development at this age Rationale Classification skills are developed during the school-age years. This age group enjoys sorting objects according to shared characteristics. Giftedness is not measured simply by a school-age child's ability to classify objects, which is an expected cognitive skill for this age group. Giftedness signs include specific academic aptitudes, advanced memory skills, creative thinking, ability in the visual or performing arts, and psychomotor ability, either individually or in combination. The development of classification skills is characteristic of the school-age child and is not related to the behavior of twins. Psychosocial development of the school-age child is focused on accomplishment or industry, not the cognitive skills of classification that are described.

Which nursing action is appropriate when instructing parents about the tape test for enterobiasis? Collect the sample before the child goes to bed. Collect the sample in the morning when the child awakens. Place the sticky side of the tape at the center of a tongue depressor. Wrap the tongue blade in a napkin when bringing it in for examination.

Collect the sample in the morning when the child awakens. Rationale The tape test is used to identify the infective agent. In this test, the sample is collected in the morning, as soon as the child awakens, before the child has a bowel movement or bathes. From this sample, it is easy to identify the infective agent. The tongue blade should be placed in a glass jar, rather than a paper napkin, to protect it while it is brought in for microscopic examination. A sample collected before the child goes to bed may not yield appropriate test results. The sticky side of the tape is placed around the end of a tongue depressor, not at the center of a tongue depressor.

Parents of a school-age child tell the nurse that their child is being consistently bullied at school. What is appropriate information for the nurse to provide the parents to help the child? Call the police. Instruct the child in how to handle the situation. Discipline the child for being the victim of a bully. Communicate concerns to the teacher and principal.

Communicate concerns to the teacher and principal. Rationale When a child is being bullied, it is essential that the parents communicate their concerns with the teacher and principal at the school in order to come up with a solution to the problem. Calling the police may not be necessary in all cases, but school officials need to be informed in every case. Instructing the child in how to handle the situation is a good intervention, but consistent bullying at school needs to be reported to the teacher and principal. Disciplining the child is not an appropriate method of addressing bullying.

Parents are asking for information on ways to assist their 10-year-old child with school. What information would the nurse include when giving parents guidelines about helping their children in the school-age years? Punish children who fail to perform adequately. Help children as much as possible with their homework. Accept responsibility for children's successes and failures. Communicate with teachers if there appears to be a problem.

Communicate with teachers if there appears to be a problem. Rationale Parents should communicate with teachers if there is a problem and not wait for a scheduled conference. Parental involvement is one factor in a child's success in school. Children need to do their own homework; this cultivates responsibility. Discipline should be used to help children control behaviors that might be affecting school performance, but failure to perform adequately should not be punished itself. Communicating with the child is a better solution to getting to the root of the school performance problem. School-age children need to develop responsibility; keeping promises and meeting deadlines lays a successful foundation for adulthood and adult responsibilities.

A nurse is providing end-of-life care for an adolescent with a chronic illness. Which nursing interventions does the nurse employ to reduce the patient's fear of dying? Select all that apply. Communicating in simple language Reassuring that the adolescent will not be left alone Providing a sense of closeness among family members Discouraging the parents' presence while providing care Conveying information about the illness in an indirect manner

Communicating in simple language Reassuring that the adolescent will not be left alone Providing a sense of closeness among family members Rationale Explaining the treatment in simple and understandable language, providing a sense of closeness, and reassuring that the adolescent will not be left alone are all recommended ways of reducing fear. The nurse should encourage the parents to participate in the end-of-life care. The nurse should convey information about the illness in a frank and candid manner.

A student nurse is comparing the sources of stress for 6-year-old children and 12-year-old children. What source of stress is prevalent in both of these age groups? Health Aggression Competition Peer pressure

Competition Rationale Competition is a source of stress for both 6-year-old and 12-year-old children. The 6-year-old wants to be "first" or best, while the 12-year-old still continues to be highly competitive and looks to peer group for prestige. Health is a source of concern for the 12-year-old, who may become a hypochondriac during this period of development. This is not a point of concern for 6-year-old children. Aggression is a particular source of stress for 6-year-olds; temper tantrums peak at this age, and the child may become hostile or aggressive. Aggression is not a particular problem for 12-year-olds. Peer pressure is a powerful motivating force and a major source of stress for the 12-year-old child; however, this is not a concern felt by 6-year-olds.

A three-year-old child presents to the ER after a sudden onset of nosebleed, small red spots on the skin, stomachache, and poor appetite. Which evaluation should the nurse prepare this child? Flow cytometry Lumbar puncture Bone marrow aspiration Complete blood count (CBC)

Complete blood count (CBC) The nurse prepares the child with a sudden nosebleed, small red spots on the skin, stomachache, and poor appetite for a CBC with a differential count. This lab test is used as an initial evaluation for a suspected case of leukemia. The test would likely reveal a low hemoglobin count, decreased platelets, low white blood cell count, and increased lymphocytes.

An eleven-year-old patient with a history of allergic rhinitis was brought to the emergency department with headache and nuchal rigidity. In addition to a lumbar puncture, for what other testing does the nurse prepare the patient? Lateral chest x-ray Urine culture for Escherichia coli Rectal swab for group B streptococci (GBS) Computed tomography (CT) scan of sinus cavities

Computed tomography (CT) scan of sinus cavities Sinusitis is often a precipitating infection in meningitis in children because olfactory nerves provide unimpeded access into central nervous system (CNS).

What is one cognitive task that a school-age child is working to master? Conservation Intuitive reasoning Object permanence Transductive reasoning

Conservation Rationale Piaget describes the cognitive developmental task of children between 7 and 11 years as mastering the concept of conservation. Intuitive reasoning occurs in the preoperational stage, from 2 to 7 years of age. Object permanence occurs in the sensorimotor stage, between birth and 2 years. Transductive reasoning occurs in the preoperational stage, between 2 and 7 years.

A child receiving chemotherapy for cancer develops an aversion to food. What step can the nurse take to provide adequate nutrition to the child? Consider adding tofu to the diet of the child. Offer the favorite food of the child during chemotherapy. Avoid serving full-fat yogurt. Ensure the child has full meals rather than snacks in between.

Consider adding tofu to the diet of the child. Rationale The nurse should consider adding tofu to the child's diet. The child needs a high-protein, high-calorie diet and tofu is high in protein. The nurse should refrain from offering the child's favorite foods while the child is receiving chemotherapy so that the child does not develop aversion to those meals. Full-fat yogurt is high in calories and should be given to the child in place of low-fat or nonfat dairy products. High-calorie snacks, such as peanut butter and dried fruits, can increase the child's calorie intake and, the child should be allowed to snack in between meals if desired.

An infant is brought to the emergency department with retinal hemorrhages, increased irritability, and a burn mark on the arm. Once stabilized, what is the nurse's priority intervention for this patient? Consult with child protective services. Ask the case manager to arrange home health care. Provide stress management teaching to the parents. Gather a timeline of events based on the parents' reports.

Consult with child protective services. Abusive head trauma or "shaken baby" syndrome is the most likely condition in an infant with retinal hemorrhages. The child should not go home with the parents until the cause of the traumatic brain injury is determined. If parents are not charged with abuse, someone must still make sure safeties are in place so consultation with child protective services is a priority intervention.

What are the cardiovascular signs that death of a child is approaching? Select all that apply. Cool hands and feet Decreasing heart rate Weak peripheral pulses Increased blood pressure Cyanosis in the hands and feet

Cool hands and feet Cardiovascular changes affect the extremities first and are related to the changes in cardiac functioning. Cooler extremities will be an expected finding in patients who are about to die. Weak peripheral pulses Pulses and blood pressure may become difficult or impossible to palpate when the child's death is approaching. Cyanosis in the hands and feet Cardiovascular changes affect the extremities first and are related to the changes in cardiac functioning. This can lead to a decrease in oxygen supply to the extremities.

A 14-year-old patient presents with a new limp and swelling in the right lower leg. The nurse notes shiny, taut skin and dilated blood vessels. A CT scan reveals a mass in the right tibia. Which action should the nurse take first? Crutch teaching Assess patient vision Provide teaching on phantom pain Perform range of motion exercises

Crutch teaching A new limp, leg swelling, and dilated blood vessels indicate the presence of osteosarcoma. The child must be non-weight bearing once tumor is identified.

Which medication is appropriate for a child with stomatitis who cannot swish and swallow due to lesions in the mouth? Kank-A Orabase Anbesol Diphenhydramine solution

Diphenhydramine solution Rationale Stomatitis is inflammation of the oral mucosa. The patient may be unable to eat due to painful lesions in the mouth. The nurse would apply diphenhydramine and Maalox solution with a cotton-tipped applicator before the patient eats to help minimize pain. This also provides mild analgesia, reduces inflammation, and forms a protective coating on the lesions. Kank-A, Orabase, and Anbesol are also used as topical anesthetics, but they do not reduce inflammation. They do not form a protective coating on the lesions.

During a home visit, the nurse assesses the growth and development of a 7-year-old child. After the assessment, the nurse documents that the child's physical development is inadequate. What physical characteristic supports this finding and documentation? Decreased leg length in relation to height Decreased head circumference in relation to height Decreased waist circumference in relation to height Increased arm length in relation to other parts of body

Decreased leg length in relation to heigh Rationale During middle childhood, the child's physical maturity can be assessed by comparing the growth of body parts in relation to height. The child is considered to have inadequate physical development if the leg length is inadequate when compared to the increase in height. A healthy child with proper physical maturity can be identified by reduced head circumference when compared to height. Waist circumference is also decreased when compared to height. Healthy children have longer limbs due to skeletal lengthening.

The mother of a 4-year-old child with a heart defect is feeling overwhelmed with explaining an upcoming surgery to the child. What should the nurse do to help the parent's communication in this circumstance? Encourage the parent to show the child a picture of the heart Describe the operation in appropriate terms using dolls and a model of the heart Provide a pamphlet to the parents on congenital heart defects to review with the child Advise the parent to read a book about being in the hospital to the child and answer any questions

Describe the operation in appropriate terms using dolls and a model of the heart Describing the operation using age-appropriate terms and props provides the information necessary for the child to understand what is going to happen.

Droplet Precautions

Designed to reduce the risk for transmission of infectious agents that are spread when large particles generated during coughing, sneezing, or talking come into contact with the conjunctiva or the mucous membrane of the nose or mouth of a susceptible person; suctioning or bronchoscopy generates these particles

Contact Precautions

Designed to reduce the risk for transmission of microorganisms transmitted by direct or indirect contact

Airborne Precautions

Designed to reduce the transmission of infectious agents that remain suspended in air or by dust particles containing the infectious agent

What are the priorities when developing a nursing plan of care for a ten-year-old patient with Guillain-Barré syndrome (GBS)? Select all that apply. Assess daily for fever, stiff neck, or confusion. Determine bilateral lower extremity strength. Check vital signs and trends every four hours. Perform tests of lower and upper sensory function. Observe chewing and swallowing of solids and liquids. Evaluate respiratory rate and use of accessory muscles.

Determine bilateral lower extremity strength. Neuromuscular impairment can be a finding in patients affected by Guillain-Barré. Assessing the patient's lower extremity strength will therefore be a priority. Check vital signs and trends every four hours. Autonomic instability may cause dizziness or the inability to alter heart rate and is a priority when developing a nursing care plan for a patient with Guillain-Barré syndrome. Perform tests of lower and upper sensory function. Guillain-Barré syndrome often presents with limb paresthesia, which is defined as altered sensation from the patient's limbs. This will be a priority in developing a care plan for this patient. Observe chewing and swallowing of solids and liquids. The patient with Guillain-Barré syndrome may develop cranial nerve dysfunction that could inhibit the ability to swallow. This will therefore be a priority in developing a nursing care plan for the patient. Evaluate respiratory rate and use of accessory muscles. The phrenic nerve may be affected in a patient with Guillain-Barré syndrome resulting in respiratory failure of the patient. Evaluating respiratory status will be a priority in caring for this patient.

The nurse keeps which things in mind when communicating bad news to families? Select all that apply. Determine what the parent knows. Provide a setting conducive to communication. Recognize and make the family aware of their denial. Give all details of the situation to all families to build trust. Acknowledge all reactions and feelings, particularly using an emphatic response.

Determine what the parent knows. Provide a setting conducive to communication. Acknowledge all reactions and feelings, particularly using an emphatic response. Rationale When communicating bad news to families, the nurse should determine what the parent knows, provide a setting conducive to communication, and acknowledge all reactions and feelings, particularly using an emphatic response. The nurse should recognize denial but not acknowledge it at this stage. The nurse should determine what the parent wants to know regarding the information. The nurse should determine if the parents want details or the basic facts at this point.

Which clinical manifestations are likely to develop in a 3-year-old child after initial stabilization for bleach ingestion? Select all that apply. Development of metabolic acidosis Development of esophageal strictures Development of liver necrosis and jaundice Development of hypokalemia and dehydration Development of organ perforation and vascular complications

Development of esophageal strictures As the damaged esophagus begins to heal, the child may have continued difficulty swallowing due to the development of strictures. Development of organ perforation and vascular complications When a child has a severe burn, the damage can lead to eventual perforation of an organ. This can lead to vascular collapse and shock.

The nurse is caring for a child with potential non-Hodgkin Lymphoma (NHL). Which medical history information could increase the child's risk for developing the disease? Select all that apply. Diagnosis of AIDS Diagnosis of chronic pain Family history of diabetes History of kidney transplant Being born to a mother of advanced age

Diagnosis of AIDS AIDS compromises the immune system, which increases the patient's risk for developing NHL. History of kidney transplant Patients who have undergone organ transplantation are at increased risk for developing NHL, because these patients are receiving drugs that weaken their immune system to prevent it from attacking the new organs.

A nurse is teaching a group of parents about assessing the ABCDE's in children with toxic exposure. Which two assessment components should the nurse discuss in addition to the traditional ABC's of CPR? Select all that apply. Diuresis Disability Exposure Exudates Diaphoresis

Disability Seizure precautions should be implemented in poison exposures with neurological or metabolic side effects. The child's mental status should be assessed frequently. Exposure Treating toxic exposures and ingestions may include removal of dermal and ocular toxins, dilution of the toxin, administration of activated charcoal, and administration of an antidote. Gastric lavages are no longer recommended

The parents of a preschooler with a developmental delay come into the health care provider's office for help with recent tantrums. The parents want to know how they should discipline the child. How should the nurse respond? Select all that apply. The child should be left alone to calm down and gain control. Discipline might not be the answer because the child may be trying to communicate frustration. Punishment is important to decrease the child's use of tantrums to gain attention and get what is wanted. The tantrum might indicate that the child has a need that is not being met, and disciplining the child could make this situation worse. Ignoring the behavior and withdrawing attention from the child will help to decrease the tantrums and help the child learn to talk about needs. Instead of disciplining the child, help the child to better communicate needs by using a picture board that can be used to initiate getting the child's personal needs met.

Discipline might not be the answer because the child may be trying to communicate frustration. The child with a chronic illness may experience many frustrations with no known way to communicate them. Discipline may increase frustration because it does not address what is frustrating the child. The tantrum might indicate that the child has a need that is not being met, and disciplining the child could make this situation worse. The use of punishment in a developmentally delayed preschool-aged child could increase the frustration level in the child and make behaviors worse. Children this age may act out when they do not know how to get their needs met. The parents should determine if the child is trying to communicate a need first. Instead of disciplining the child, help the child to better communicate needs by using a picture board that can be used to initiate getting the child's personal needs met. If the tantrums are indicating frustration, making communication easier can decrease frustration. Let the child gain some independence by being able to initiate getting own personal needs met.

Which factors are important when selecting a daycare facility for a child? Select all that apply. Discipline policy Safety measures Adequate parking Caregiver-to-student ratio Infection-control procedures

Discipline policy Safety measures Caregiver-to-student ratio Infection-control procedures Rationale When guiding an infant's parents about the selection of a daycare facility for their child, it is most important for the nurse to instruct the parents to evaluate the facility's discipline policy, infection-control procedures, safety precautions, and caregiver-to-student ratio. Adequate parking is not an important area to be evaluated in the choice of a daycare facility.

The nurse is counseling the family of a dying child to cope with the situation. Which action of the nurse needs correction? Encouraging the parents to include siblings Providing details about the child's status to the parents Discouraging the parents from sharing their sorrow with the child Encouraging the parents to honestly answer the child's questions

Discouraging the parents from sharing their sorrow with the child Rationale The nurse should encourage the parents to share their feelings of sorrow with the dying child to promote the child's grief expression. The nurse should encourage the parents to include siblings to prevent them from feeling excluded. The nurse should provide the parents with information about the dying child's status. The nurse should encourage the parents to honestly answer the child's questions.

Rubella

Discrete pinkish red maculopapular rash appears on face and then spreads downward to neck, arms, trunk, and legs; greatest danger is teratogenic effect on fetus.

An adolescent with cancer who has undergone extensive irradiation is experiencing severe malaise. What does the nurse teach the child and the parents before discharge? Select all that apply. Discuss a feasible school schedule. Encourage strenuous physical education. Encourage the child to keep up with peers. Emphasize regular bed times and periodic rest times. Counsel the parents to monitor any abnormal signs and symptoms.

Discuss a feasible school schedule. Emphasize regular bed times and periodic rest times. Counsel the parents to monitor any abnormal signs and symptoms. Rationale Before discharge, the nurse should discuss a feasible school schedule with the parents of the adolescent to avoid having the patient feel overwhelmed or exhausted. The adolescent must have regular bed times and periodic rest times. The parents should monitor any abnormal signs and symptoms and come in for followup care. Elimination of strenuous physical education may be necessary for the adolescent because of low levels of energy. Due to enervation, the adolescent may not be able to keep up with peers. The nurse and the parents should advise the adolescent not to perform activities to the point of physical exhaustion.

A child with a chronic health condition is having trouble coping with the hospital setting and becomes upset when informed of a room change. Which action by the nurse is most appropriate? Do not change the room for several hours and introduce the change to the child slowly Encourage the child to do some deep breathing to cope with the stress of having to change rooms Discuss comfort measures with the child's parents that might help the child cope with the distress Change the room while the child is off the floor so that the child does not have to deal with the stress of the room change

Discuss comfort measures with the child's parents that might help the child cope with the distress It is important for the nurse to collaborate with parents to identify the child's preferences and actions that may help to pacify the child during times of stress.

A 5-year-old child has been placed in hospice care for terminal cancer. The child overhears the parents discussing a local hospice center for placement of the child. The child asks the nurse, "Why do I have to go away? I don't want to be away from my parents and pets." What action should the nurse take in response to the child's concern? Talk to the child about staying at the hospital to receive hospice care there. Discuss with the parents, the option of allowing the child's pets to visit in the hospital. Discuss the option of home hospice care with the parents, after sharing the concerns of the child with them. Tell the child that the hospice center will help the child to feel more comfortable and that the parents can spend the night, if they desire.

Discuss the option of home hospice care with the parents, after sharing the concerns of the child with them. Hospice at home allows the child to remain at home and have family, pets, and friends around the child. The home environment provides familiarity and comfort to the child.

Which nursing information is appropriate when teaching parents about poison prevention? Select all that apply. Only use mercury thermometers. Do not use plants for teas or medicine. Take medications out of sight of children. Post the contact number for the poison control center near the phone. Make babysitters aware of poison control numbers and home address details.

Do not use plants for teas or medicine. Take medications out of sight of children. Post the contact number for the poison control center near the phone. Make babysitters aware of poison control numbers and home address details. Rationale Teaching parents at a community health day regarding poison prevention should include teaching parents to not use plants for teas or medicine, advising parents to take medications out of sight of children, posting the contact number for the poison control center near the phone, and making babysitters aware of the poison control center number and home address details in case an emergency arises. Use of mercury thermometers is no longer recommended due to the risk of inhaled vapors from broken thermometers.

Which mode of transmission of pertussis (whooping cough) is common? Fecal Blood Droplet Mucous membranes

Droplet Rationale The typical mode of transmission of pertussis is direct contact or the droplet route. The fecal, blood, and mucous membrane routes are not typical modes of transmission of pertussis.

What is the reason of chemotherapeutic drugs to treat cancer are often given in combination? Drugs can be given by various routes. Drugs can be given at different times during the day. Patients cannot tolerate extremely high doses of single drugs. Drugs allow for optimum cell destruction with minimum toxic effect.

Drugs allow for optimum cell destruction with minimum toxic effect. Correct Combining drugs allows for synergistic effects. Optimum cell cycle destruction with minimum toxic effects and decreased resistance by the cancer cells to the agent are possible. Routes of administration depend on the pharmacologic attributes of the drug. The routes do not influence interactions. Combination therapy usually has specific intervals to maximize synergistic effect. High doses of drugs are indicated in different regimens

A nurse is planning to teach a class on overcoming barriers to immunization through patient and family education. Which examples of this should the nurse include? Select all that apply. Educate parents about the safety of vaccinations. Educate school-age children about the efficacy of vaccination. Educate parents about the most serious risks associated with vaccinations. Educate parents about the difference between mild and serious vaccine reactions. Educate parents about the individual and societal importance of vaccination.

Educate parents about the safety of vaccinations. Educating parents about the safety of vaccinations helps parents feel more comfortable and knowledgeable about the care their child is receiving. It can also help identify and resolve any misconceptions they may have. Educate parents about the difference between mild and serious vaccine reactions. Educating parents about the differences between mild and serious vaccine reactions can help them to feel less worried if their child experiences a common mild reaction after receiving a vaccine. Mild reactions are not dangerous and are not a reason to avoid future doses. Correct Educate parents about the individual and societal importance of vaccination. Parents should understand the benefit for their own child's safety and health as well as the role vaccination plays in protecting the health of the community.

A 17-year-old patient with a history of tuberous sclerosis presents with a new onset of balance issues while walking. Which initial diagnostic test would the nurse anticipate from the provider? Biopsy PET scan Head CT Abdominal MRI

Head CT The nurse would obtain an order to image the head for a patient with tuberous sclerosis and a history of poor coordination in order to determine the presence of a brain tumor. Although an MRI is the most common imaging modality used, a CT scan may be ordered instead.

The nurse is teaching parents and their child about health promotion. What interventions to promote health during middle childhood would be included? Instructing parents to defer questions about sex until the child reaches adolescence Stressing the need for increased calorie intake to meet the increased demands on the body Advising parents that the child will need decreasing amounts of rest toward the end of this period Educating the child and parents about the need for effective dental hygiene because these are the years in which permanent teeth erupt

Educating the child and parents about the need for effective dental hygiene because these are the years in which permanent teeth erupt Rationale Because the permanent teeth are present, it is important for the child to learn how to care for them. Parents should approach sex education with a life span approach and answer questions in a manner appropriate for the child's age. Caloric needs are diminished in relation to body size during the middle years; however, a balanced diet is important to prepare for the adolescent growth spurt. School-age children often need to be reminded to go to sleep.

The nurse is caring for a patient with suspected meningitis. Which interventions should need to be performed in caring for this patient? Select all that apply. Elevate head of bed to 30 degrees. Begin intravenous (IV) normal saline infusion. Administer broad-spectrum antibiotic as prescribed. Draw blood sample for a white blood cell (WBC) count. Discuss with the patient's family the importance of vaccination.

Elevate head of bed to 30 degrees. Although simple, this can be an important part of caring for a patient with suspected meningitis. It can help to control an increased intracranial pressure (ICP). Correct Begin intravenous (IV) normal saline infusion. This can be started after antibiotic infuses to prevent dehydration and ensure tissue perfusion. Correct Administer broad-spectrum antibiotic as prescribed. Early antibiotic therapy reduces morbidity and mortality. Correct Draw blood sample for a white blood cell (WBC) count. This is important information and therefore the blood sample can be obtained after antibiotic is begun.

The nurse is assessing a child whose parents report malaise, abdominal pain, knee, and leg pain. The nurse notes significant hepatosplenomegaly. Which other objective findings would indicate a possible leukemia diagnosis? Select all that apply. Elevated WBC count Temperature 99.4 F Depressed platelet count Elevated hemoglobin level Masses in the groin and axillae

Elevated WBC count Malaise, abdominal pain, and hepatosplenomegaly are signs of malignancy. Abnormal WBC would support a possible leukemia diagnosis because of the lack of fully developed leukocytes. Depressed platelet count Malaise, abdominal pain, and hepatosplenomegaly are signs of malignancy. A low platelet count would support a possible leukemia diagnosis because leukemia can be accompanied by thrombocytopenia. Masses in the groin and axillae Malaise, abdominal pain, and hepatosplenomegaly are signs of malignancy. Masses in the groin and axillae would support a possible leukemia diagnosis if lymph nodes are involved.

A school nurse is speaking with a 10-year-old child who has been acting out in class. The child becomes tearful and says, "My dad's right. I can't do anything good." Which type of abuse might this child be experiencing? Neglect Physical abuse Emotional abuse Sexual exploitation

Emotional abuse Emotional abuse occurs when someone subjects a child to behavior that can cause psychologic trauma. Insulting the child is a form of emotional abuse.

Which type of maltreatment is appropriate when describing the child that appears to have had plenty of food and has been kept clean but has not been held or given any attention? Physical abuse Physical neglect Emotional abuse Emotional neglect

Emotional neglect Rationale Emotional neglect generally refers to failure to meet the child's needs for affection, attention, and emotional nurturance. Emotional abuse refers to the deliberate attempt to destroy or significantly impair a child's self-esteem or competence. Emotional abuse may take the form of rejecting, isolating, terrorizing, ignoring, corrupting, verbally assaulting, or overpressuring the child. Physical neglect involves the deprivation of necessities, such as food, clothing, shelter, supervision, medical care, and education. The deliberate infliction of physical injury on a child, usually by the child's caregiver, is termed physical abuse. Physical abuse can include anything from bruises and fractures to brain damage.

The school nurse is discussing dental health with a class of first grade children. What teaching would be included by the nurse? Teaching how to floss teeth properly Recommending nonfluoridated toothpaste Emphasizing the importance of brushing before bedtime Recommending a toothbrush with hard nylon bristles to get between the teeth

Emphasizing the importance of brushing before bedtime Rationale Children should be taught to brush their teeth after meals and snacks and before bedtime to prevent dental caries. The American Dental Association recommends fluoridated toothpaste for this age group. Parents should help with flossing until children develop the dexterity required, when they are in about the third grade. A toothbrush with soft nylon bristles is recommended to prevent damage to the gums.

What should the nurse do to help a child get adjusted with the body image changes caused by cancer and its treatment? Select all that apply. Encourage good scalp hygiene. Encourage the child to wear a wig, cap or scarf. Discourage visits from friends before hair regrows. Stress the need of head covering while going outside. Explain that the regrown hair will be thinner than the earlier hair.

Encourage good scalp hygiene. Encourage the child to wear a wig, cap or scarf. Stress the need of head covering while going outside. Rationale The nurse should advise the family to maintain good scalp hygiene for the child to stave off infections. The nurse should encourage the child to wear a wig, cap or scarf. The nurse should stress the need for head covering to protect the child's head from excessive cold or heat from outside weather. The nurse should encourage visits from friends so they get used to the body image changes before the child is discharged. The nurse should explain that the hair will grow back in three to six months thicker, darker and curlier than before.

Analysis of cerebrospinal fluid (CSF) obtained from lumbar puncture shows bacterial meningitis. The nurse performs what action for the nine-year-old pediatric patient? Ensure patient is receiving antibiotics. Place patient on airborne precautions. Administer the meningococcal vaccine. Provide family with home care instructions.

Ensure patient is receiving antibiotics. Patient should have been on antibiotics shortly after symptoms began, but the nurse must ensure that antibiotics are being administered as ordered based on the results of CSF findings. A delay in antibiotic administration can be fatal.

The home health nurse is planning care for a 3-year-old boy who has Down syndrome and is on continuous oxygen. He recently began walking around furniture. He is spoon fed by his parents and eats some finger foods. What is the most appropriate goal to promote normal development? Encourage mobility. Encourage assistance in self-care. Promote oral-motor development. Provide opportunities for socialization.

Encourage mobility. Correct A major principle for developmental support in children with complex medical issues is that it should be flexible and tailored to the individual child's abilities, interests, and needs. This child is exhibiting readiness for ambulation. It is an appropriate time to take steps that encourage mobility (e.g., providing longer oxygen tubing). The parents should provide decreasing amounts of assistance with self-care as the child is able to develop these skills. He is receiving oral foods and is eating finger foods; therefore, he has already acquired oral-motor skills. Mobility is a new developmental task. Opportunities for socialization should be ongoing

A 5-year-old child is having trouble coping in the hospital and behaviors have become increasingly problematic to the nurse and staff. What actions by the nurse should help to alleviate this problem? Select all that apply. Encourage the child to express feelings through drawings Give the child different ways to communicate needs such as a picture board Allow the child to help with some aspects of personal care and with keeping the room tidy Remind parents the child's behavior is a normal part of dealing with an illness in a school-aged child Keep the child in the hospital room as a time-out until the child is able to stop having disruptive behaviors

Encourage the child to express feelings through drawings Drawing provides a different outlet for frustration that can decrease problematic behaviors, and therefore this should be encouraged by the nurse. Correct Give the child different ways to communicate needs such as a picture board The nurse should be providing different ways to communicate needs that can decrease frustration. Correct Allow the child to help with some aspects of personal care and with keeping the room tidy Giving responsibilities and some chores is helpful to school-age children to maintain control and normalcy.

A girl with osteosarcoma needs an amputation of the affected femur and a prosthetic limb is to be fitted. Which is the most appropriate postoperative nursing care management for the girl? Discourage the girl from wearing jeans. Encourage the girl to use sheer stockings. Administer calcitonin to alleviate phantom limb pain. Explain that phantom limb pain is a form of imagination.

Encourage the girl to use sheer stockings. Rationale The nurse should encourage the girl to wear sheer stockings. Well-fitted prostheses are so natural looking that sheer stockings will not reveal the device and the girl can wear feminine apparel. Even during the postoperative period, the nurse may encourage the girl to wear jeans to draw attention away from the deformity. Various medications including calcitonin have been used for phantom pain relief; however, further studies are needed to prove its effectiveness in relieving phantom limb pain. The nurse should explain to the child and her parents about the nature of phantom limb pain and should emphasize that it is a real pain.

An oncology nurse experiences exhaustion at the end of every shift and is looking for a way to cope. Which is an example of the best way the nurse's needs can be met? Limit the nurse's patient assignment to 3-4 patients Utilize team nursing to complete all patient care needs Encourage the nurse to talk with other nurses about what they are experiencing Talk with the nurse manager about how to improve time management and organization skills

Encourage the nurse to talk with other nurses about what they are experiencing Discussing feelings with coworkers demonstrates concern, respect and support for the nurse. Mutual support of each other is important for nurses who work with terminally ill children.

The infant with congestive heart failure needs to have an IV inserted. To increase the comfort of the baby, what should the nurse do? Encourage the parents to stay with the child as long as they are comfortable. Do not allow the anxious parents to leave the room because this can cause the infant to have separation anxiety. Ensure that the infant has the IV started in the crib because that is where the child is most comfortable and feels safest. The nurse should give a detailed explanation to the parents to decrease anxiety of what steps are involved in inserting the IV.

Encourage the parents to stay with the child as long as they are comfortable. The parents should stay if they are comfortable to decrease the infant's anxiety. If they are not comfortable, then they can transmit their anxiety to the infant.

A parent tells the nurse that her 8-year-old child throws a fit each night, refusing to go to bed. What interventions can the nurse suggest to the mother to help promote sleep in the school-age child? Forcing the child to take a nap after school Encouraging a quiet activity before bedtime Having the child watch a movie before bedtime Moving up the bedtime to encourage more sleep

Encouraging a quiet activity before bedtime Rationale A quiet activity such as coloring, reading, or listening to music before bed can help facilitate the process of going to bed. Many school-age children do not feel tired and therefore resist bedtime. Forcing the child to take a nap after school is not appropriate because children at this age no longer require naps. Having the child watch a movie before bedtime is not an effective way to promote sleep; watching TV or being in front of a screen has been shown to interfere with sleep. Setting back the bedtime, rather than moving it up, as the child gets older is a better way to promote sleep in the school-age child.

A nurse is discussing disease management options with a family who is coping with chronic illness. The nurse notices a sibling who is withdrawn. What should the nurse do to help the sibling feel more involved? Select all that apply. Engage with and ask if the sibling has any questions. Enquire about how much time the sibling spends with the parents. Ask whether the sibling would like to help change the child's bandage. Encourage the sibling to discuss any negative feelings with the child who is ill. Express interest and ask about the extracurricular activities the sibling is involved in.

Engage with and ask if the sibling has any questions. Engaging with the sibling by asking if there are any questions demonstrates an interest in the sibling and helps to develop a good rapport and helps the sibling to feel more involved. Correct Enquire about how much time the sibling spends with the parents. Assessing how much time the sibling spends with the parents provides information as to whether the sibling's needs are being met. Correct Ask whether the sibling would like to help change the child's bandage. By asking the sibling to assist with providing treatment to the ill child, the sibling can become more involved. Express interest and ask about the extracurricular activities the sibling is involved in. Asking about sibling activities demonstrates an interest and gives the sibling a chance to talk about personal accomplishment

The nurse is caring for a child with Wilms tumor. The health care provider has ordered an abdominal ultrasound, IV fluid rehydration, and oral pain medication. Which action should be priority for the nurse? Insert peripheral IV Administer IV pain medication Order IV fluids from pharmacy Ensure patient safety during diagnostic testing

Ensure patient safety during diagnostic testing The nurse's priority should be patient safety. Manipulation of the abdomen may cause rupture of the tumor and, therefore, special preparation for the US will need to be done.

A 14-year-old female patient with a newly diagnosed chronic illness presents to the nurse with feelings of worthlessness and showing signs of decreased self-esteem. What action by the nurse should help to strengthen this adolescent's feeling of autonomy as the patient learns to cope with a chronic illness? Select all that apply. Ensure that the teen is involved in care decisions and meetings with the health care team Give the adolescent opportunities to discuss needs and fears related to coping with the illness Encourage the patient to do as many of the usual activities with friends as the teen's condition allows Allow the adolescent to ask questions and give honest answers regarding the illness to increase understanding of the condition Stress that adherence is important and that making decisions is a privilege that can be lost if the treatment regimen is not followed

Ensure that the teen is involved in care decisions and meetings with the health care team Being involved in care decisions and care planning can increase the patient's sense of control and self-esteem. Correct Give the adolescent opportunities to discuss needs and fears related to coping with the illness Some adolescents, especially younger teens, might have feelings of guilt related to the illness. Expression of these feelings can decrease sadness and help the teen gain back some control in life. Correct Encourage the patient to do as many of the usual activities with friends as the teen's condition allows Assisting the teen to maintain contact with peer group and maintain usual activities can increase coping and age-appropriate development. Correct Allow the adolescent to ask questions and give honest answers regarding the illness to increase understanding of the condition The adolescent should be an integral decision maker, which cannot be possible without an adequate understanding of the illness

Which findings would support the suspicion of emotional abuse and neglect? Select all that apply. Enuresis Eating disorder Suicide attempts Unusual fearfulness Bruises on face, back, or buttock

Enuresis Eating disorder Suicide attempts Unusual fearfulness Rationale Findings that would support the concern and suspicion of emotional abuse and/or neglect include enuresis, eating disorders, suicide attempts, and unusual fearfulness. Bruises on the face, back, and buttock suggest physical abuse.

Which is the appropriate dressing care process instituted by a nurse after craniotomy in a child? Removing the dressing if it is soiled Changing the dressing if there is a foul odor Estimating approximate amount of drainage and recording it Reinforcing with dry sterile gauze in presence of colorless drainage

Estimating approximate amount of drainage and recording it Rationale The nurse should estimate the approximate amount of drainage and record it. Drainage may be due to leakage of cerebrospinal fluid or continuous hemorrhage. The nurse should not remove the dressing if soiled, but rather should reinforce it with dry sterile gauze. A foul odor may be an indication of infection. This finding should be reported to the health care provider and a culture should be taken, but the dressing should not be changed without further advisement. The presence of colorless drainage is an indication of leakage of cerebrospinal fluid. It needs immediate medical intervention rather than mere reinforcement of the dressing.

The nurse is caring for a child with Ewing sarcoma immediately following surgery. Which assessments should the nurse use to ensure patient outcomes are being met? Select all that apply. Assess patient's vision Evaluate the surgical site Prepare patient for a repeat biopsy Evaluate the patient's range of motion Assess patient feelings and concerns

Evaluate the surgical site An expected outcome of the patient with a bone or muscle tumor is that the patient will remain free from infection, as evidenced by a lack of redness or purulent drainage from the surgical site. Evaluate the patient's range of motion An expected outcome of the patient with a bone or muscle tumor is that the patient will regain maximal mobility, which can be assessed by evaluating range of motion. Assess patient feelings and concerns An expected outcome of the patient with a bone or muscle tumor is that the patient will have a positive body image, which can be assessed by determining the patient's state of mind.

A young child has Ewing sarcoma. What steps should the nurse follow when counseling the parents of the child? Select all that apply. Restrict the activity of the child to the minimum. Advise the use of ice packs on the irradiated area. Explain the reasons for doing various diagnostic tests. Advise parents to make the child wear loose-fitting clothes. Counsel thoroughly on damaging effects of radiotherapy.

Explain the reasons for doing various diagnostic tests. Advise parents to make the child wear loose-fitting clothes. Counsel thoroughly on damaging effects of radiotherapy. Rationale A series of diagnostic tests are required to be performed for this type of cancer. The family should be given adequate information about the reasons for performing these tests. Because hyperpigmentation commonly happens after high-dose radiotherapy, the child should wear loose-fitting clothes to minimize skin irritation. The parents should be counseled thoroughly on the damaging effects of radiotherapy because there are many various side effects. The child's level of activity should be tailored to the recovery. Sometimes, an active exercise program is planned to preserve maximum function. Ice packs are not recommended for use on the irradiated area.

The nurse is caring for a five-year-old child who is scheduled for surgery to remove a brain tumor. The child's caregivers indicate that they are concerned that the child will have a difficult time adjusting post-surgery. What is the best action by the nurse to address the caregiver's concerns? a. Explain to the child that everything will be ok b. Explain the surgery in detail to the child and caregiver c. Refer the family to a support group for brain tumor patients d. Explain to the child in simple terms what to expect before and after surgery

Explain to the child in simple terms what to expect before and after surgery Preoperative teaching at the child's developmental level prepares the child and caregivers for the surgery and what to expect afterward. This can help alleviate child anxiety and caregiver concern.

Which nursing action is appropriate during an emergent case of poisoning in a child? Admonish the family for negligence. Ask the family why the home is not childproofed. Question the family about the lack of supervision. Explore the circumstances of the injury with family members.

Explore the circumstances of the injury with family members. Rationale When there is an emergency that involves poisoning in a child, the emergency room nurse would note that the situation may lead to an emotional crisis in the parents, because they start feeling guilty. Therefore without accusing anyone, the emergency room nurse would explore the circumstances that led to the injury. This enables them to discuss ways of preventing the recurrence of such incidents in future. It is improper to admonish the family for negligence, question them about whether the house is childproofed, or ask about the lack of supervision; there is already a sense of insecurity in terms of parental responsibilities. Instead, the emergency room nurse would calm and support the parents and the child.

Which situation identifies an environment in which a child may be at increased risk of developing cancer? Residing in low-income housing Making frequent trips to a foreign country Visiting at private school near a clothing factory Exposure to radiation from repeated computed tomography (CT) scans

Exposure to radiation from repeated computed tomography (CT) scans Exposure to radiation can increase a child's risk of developing cancer.

A child presents with symptoms of nightmares, depression, and anxiety. While speaking with the child, the nurse finds that the child has a fatalistic orientation to the future. What is the most common reason for this child's assessment findings? Conflict with friends Perceived lack of love Exposure to repeated violence in the family Inability to distinguish between fact and fantasy

Exposure to repeated violence in the family Rationale The symptoms of nightmares, depression, and anxiety, along with a fatalistic orientation toward the future, can indicate that the child is suffering from posttraumatic stress disorder. Children exposed to repeated violence can display hyperarousal symptoms leading to posttraumatic stress disorder. Conflict with friends can be a source of stress for the child, but this is not likely to lead to posttraumatic stress disorder. Perceived lack of love can lead to antisocial behaviors in school-age children, and children can steal to make up for this perceived lack. However, symptoms of nightmares, depression, and anxiety are not specifically related to perceived lack of love. Inability to distinguish between fact and fantasy is common in preschoolers and leads children to lie. This is not related to posttraumatic stress disorder.

Which characteristics of the rash would the nurse expect to find in a child with roseola infantum? Select all that apply. Itchy Fades on pressure Rose-pink in color Appears on the face first Appears on the trunk first

Fades on pressure Rose-pink in color Appears on the trunk first Rationale The nurse should be aware of how different exanthematous diseases look. In cases of roseola infantum, the rash is seen as discrete rose-pink macules on the trunk first, which later spread to the neck, face, and extremities. The rash also fades when pressure is applied. The rash is usually nonpruritic (does not cause itching).

The nurse informs the family about which symptoms they might see as their child passes? Select all that apply. Fatigue Seizures Tardive dyskinesia Sleep disturbances Restlessness, agitation

Fatigue Seizures Sleep disturbances Restlessness, agitation Rationale Common central nervous symptoms experienced by dying children include fatigue, seizures, sleep disturbances, restlessness, and agitation. Tardive dyskinesia is a side effect associated with certain medication use but not directly a symptom experienced by dying children.

Why might some caregivers withhold pain control from a terminally ill child? Select all that apply. Fear of addiction Easier dying process Side effects of opioids Improved quality of life Decreased level of cognition

Fear of addiction Side effects of opioids Decreased level of cognition Rationale Rationales in withholding pain control from a terminally ill child include fear of addiction, decreased level of cognition associated with pain medication, and the side effects of opioids. Rationales for providing pain control for a terminally ill child include easier dying process and improved quality of life.

What is a common characteristic of the psychosocial development of school-age children? A developing sense of initiative is important. Peer approval has not yet become a motivating factor. Motivation comes from extrinsic rather than intrinsic sources. Feelings of inferiority or lack of worth can be derived from children themselves or from the environment.

Feelings of inferiority or lack of worth can be derived from children themselves or from the environmen Rationale The school-age child is eager to develop skills and participate in activities. All children are not able to do all tasks well, and the child must be prepared to accept some feelings of inferiority, as highlighted in Erikson's stage for this age group of industry versus inferiority. Initiative versus guilt is the stage characteristic of preschoolers. Peer group formation is one of the major characteristics of school-age children. School-age children gain satisfaction from independent behaviors that are internally driven and accomplished.

What is characteristic of the psychosocial development of school-age children? Peer approval is not yet a motivating power. A developing sense of initiative is very important. Motivation comes from extrinsic rather than intrinsic sources. Feelings of inferiority or lack of worth can be derived from children themselves or from the environment.

Feelings of inferiority or lack of worth can be derived from children themselves or from the environment. Correct A sense of industry, or accomplishment, is achieved between the ages of 6 years and adolescence. Erikson labels this stage as industry versus inferiority. Interaction with peers is a source of satisfaction for school-age children. Initiative is the developmental task of the preschooler. Intrinsic motivation is associated with increased competence in mastering skills

The nurse should advise parents about which mild reaction that may occur one to two weeks after receiving the live, attenuated influenza vaccine [LAIV)? Hives Fever Wheezing Swelling of the face

Fever Live, attenuated influenza vaccine (LAIV) is a live vaccine. Live vaccines can cause a fever as the immune system reacts during the first one to two weeks after the vaccination. This is considered a mild reaction and is not a contraindication to future doses.

Which are the clinical manifestations of Wilms tumor? Select all that apply. Fever Fatigue Hematuria Weight gain Abdominal swelling or mass

Fever Fatigue Hematuria Abdominal swelling or mass Rationale Clinical manifestations of Wilms tumor include abdominal swelling or a mass, hematuria, fatigue, and fever. Weight loss, rather than weight gain, is a clinical manifestation of Wilms tumor.

Assessment of neuroblastoma

Firm, nontender, irregular mass in the abdomen that crosses the midline Urinary frequency or retention Lymphadenopathy (cervical and supraclavicular) Bone pain Pallor, weakness, irritability, anorexia, weight loss Respiratory impairment Neurological impairment Paralysis from compression of the spinal cord

HIB

First Dose: 2 months of age. Second Dose: 4 months of age. Third Dose: 6 months of age (if needed, depending on brand of vaccine) IM

Which nursing intervention is appropriate when educating the community about the measles vaccination and Vitamin A injection? The minimum age for the measles vaccine is 12 months. Two vitamin A injections can prevent measles infections. Vitamin A injections are not useful if already infected with measles. First dose of measles vaccine is given at 6 months, with a second one after 12 months of age.

First dose of measles vaccine is given at 6 months, with a second one after 12 months of age. Rationale When there is an outbreak of measles (rubeola), the first dose of measles vaccine can be given any time after 6 months of age, followed by a second inoculation after 12 months. When there is no measles outbreak, the recommended age for vaccination is 12 to 15 months. Vitamin A supplementation can decrease the morbidity and mortality associated with measles, but it cannot prevent the spread of infection. Two doses of vitamin A (200,000 IU) are given on consecutive days to children under the age of 2 years.

Which drug would the nurse administer to a child with purulent drainage from the eyes, inflamed conjunctiva, and swollen lids? Acyclovir Vitamin A Corticosteroids Fluoroquinolones

Fluoroquinolones Rationale The child has developed bacterial conjunctivitis. Fluoroquinolones such as moxifloxacin, gatifloxacin, and besifloxacin are administered to children for treating bacterial conjunctivitis. Acyclovir is given to a child with viral herpetic gingivostomatitis. Vitamin A is administered to children with measles. Corticosteroids should be avoided in children with conjunctivitis because they reduce ocular resistance to bacteria.

An infant is at the clinic for a Nine-month well visit. The nurse reviews the chart and notes the infant did not receive the third DTaP vaccine at the six month visit. Which action by the nurse is most appropriate? Give third dose Start series over Give a double dose Schedule the fourth dose the following month

Give third dose The nurse should give the third dose at the nine month well visit since this will allow the infant to catch up.

A child falls from a two-story building and presents to the emergency department appearing drowsy. Which assessment should be performed by the nurse first? Glasgow coma scale Test deep tendon reflexes Assess gait, balance, and coordination Assess overall muscle strength and tone

Glasgow coma scale The Glasgow coma scale should be used to assess a child's level of consciousness and indicate next steps for evaluation and treatment.

Which focused assessment should the nurse use to evaluate a child's motor coordination? Select all that apply. Gait analysis Vibration sense Manual dexterity Muscle development Deep tendon reflexes

Gait analysis The analysis of a child's gait should be used to evaluate motor skills because the ability to walk is a complex skill and involves the motor system. Manual dexterity Assessing manual dexterity of a child such as ability to use a pencil can be used to evaluate motor skills because it evaluates fine motor skills and is a measure of myelinization. Muscle development The muscular development of a child is part of the focused assessment used to evaluate motor skills because poor muscle development will impede child's motor skill development.

Which clinical finding would the nurse interpret about the management of care for a child with foul-smelling, watery, pale, and greasy stools prescribed tinidazole? Giardiasis spreads from one person to another. A single dose of tinidazole has a low cure rate. Giardiasis is a common intestinal bacterial infection. Tinidazole is an antifungal drug similar to fluconazole (Diflucan).

Giardiasis spreads from one person to another. Rationale The child's signs and symptoms indicate giardiasis, which spreads from one person to the other through fecal-oral transmission. Giardia is the most common intestinal parasitic pathogen in the United States. It lives in the upper part of the intestines. It can be seen under a microscope in stool samples, but sometimes more than one stool sample is required. This is because the protozoan is not shed in every stool sample. It is not a bacteria, fungus, or virus. It is a protozoan. Fluconazole (Diflucan) is an antifungal drug, and tinidazole (Tindamax) is an antiparasitic drug. A single dose of tinidazole (Tindamax) is reported to have an 80% to 100% cure rate.

A child who is receiving palliative care has loss of sensation in the lower extremities, which is progressing toward the upper body. Which other symptoms, if present in the patient, indicate impending death? Select all that apply. Decreased sleeping Kussmaul breathing Grayish-blue skin Cheyne-Stokes respirations Increased urine concentration

Grayish-blue skin Cheyne-Stokes respirations Increased urine concentration Rationale Physical changes, such as mottling of skin, occur more often in children dying of prolonged illness. The child's skin might become grayish-blue in color. Cheyne-Stokes respiration (a waxing and waning of deep breathing with regular periods of apnea) would also be observed in the child. Generally, as a child approaches death, urine output would reduce and urine concentration would increase. As a child approaches death, the child would sleep more than usual. Kussmaul breathing is a deep and labored breathing associated with severe metabolic acidosis such as diabetic ketoacidosis.

A twelve-year-old child has begun experiencing difficulty in playing at recess and states, "I just can't run as fast or throw and catch the ball like I could before, so I just don't play now." On what should the nurse focus the neurologic assessment? Primitive reflex presence Gross and fine motor skills Cognitive and learning skills Language and communication skills

Gross and fine motor skills Movement and coordination must be assessed. The nurse should suspect that the child's gross and fine motor skills are underdeveloped or regressing due to lack of myelin. A decrease in myelinization slows the action potentials of the central nervous system (CNS) and therefore prevents coordination of the movement of motor units. In adults, myelin naturally regresses in the elderly. In children, myelin should increase along with motor skills.

What vaccinations are included in health promotion during infancy? (Select all that apply.) Haemophilus influenzae type b (Hib) Hepatitis C virus (HCV) Diphtheria, tetanus, and pertussis (DTaP) Poliovirus Hepatitis B virus (HBV)

Haemophilus influenzae type b (Hib) Correct Diphtheria, tetanus, and pertussis (DTaP) Correct Poliovirus Correct Hepatitis B virus (HBV) Correct The Advisory Committee on Immunization Practices of the Centers for Disease Control and Prevention and the Committee on Infectious Diseases of the American Academy of Pediatrics govern the recommendations for immunization, which include diphtheria, tetanus, and pertussis (DTaP using acellular pertussis); poliovirus; measles, mumps, and rubella (MMR); Hib; HBV; hepatitis A virus (HAV); meningococcal; pneumococcal conjugate vaccine (PCV); and influenza (and H1N1) during infancy. There is no current vaccination to prevent the transmission of hepatitis C virus.

Which is a common side effect of several chemotherapeutic drugs? Hair loss Glossomegaly Increased energy Increased appetite

Hair loss Rationale Hair loss is a common side effect of several chemotherapeutic drugs. Glossomegaly is not a common side effect of chemotherapeutic drugs, although stomatitis is. Decreased energy, rather than increased energy, is a common side effect of chemotherapeutic drugs. Reduced appetite, rather than increased appetite, is a common side effect of chemotherapeutic drugs.

Which control measures would the nurse include when teaching parents how to prevent the spread of infection at home? Select all that apply. Hand washing Immunizations Contact precautions Not sharing eating utensils Covering the face when sneezing

Hand washing Not sharing eating utensils Covering the face when sneezing Rationale Infection-control measures include hand washing, not sharing eating utensils, and covering the face when sneezing. Immunizations are a primary prevention method. Contact precautions are an isolation method used in the hospital to help prevent the spread of disease.

The nurse is evaluating a child who was admitted for persistent cough, dyspnea, swelling, and discomfort in the neck and axilla. Lymph node biopsy confirmed that the child has Non-Hodgkin Lymphoma. Which question would best determine the effectiveness of nursing care to this patient? Is the child's core body temperature stable? Is the child's urine output sufficient for age? Has the child's respiratory status remained stable? Are the parents knowledgeable about the disease process?

Has the child's respiratory status remained stable? Evaluating the child's respiratory status for rate, rhythm, and breath sounds, and ensuring that they are stable indicates that the nursing care rendered to the child is effective.

What describes the cognitive abilities of school-age children? Have developed the ability to reason abstractly. Become capable of scientific reasoning and formal logic. Progress from making judgments based on what they reason to making judgments based on what they see. Have the ability to place things in a logical order, to group and sort, and to hold a concept in their minds while making decisions based on that concept.

Have the ability to place things in a logical order, to group and sort, and to hold a concept in their minds while making decisions based on that concept. Correct In Piaget's stage of concrete operations, children have the ability to group and sort and make conceptual decisions. Children cannot reason abstractly until late adolescence. Scientific reasoning and formal logic are skills of adolescents. Progressing from making judgments based on what school-age children reason to making judgments based on what they see is not a developmental skill.

What intervention lowers the risk of sudden infant death syndrome (SIDS)? Smoking near the infant Having the infant sleep in the supine position Having the infant sleep with the parents instead of alone in a crib Making certain the infant is kept very warm while sleeping

Having the infant sleep in the supine position Correct The supine position is recommended for all infants while sleeping. Smoking in the infant's environment increases the infant's risk for SIDS by 1.9%. There is a positive association between bed sharing with parents and SIDS. Thermal stress or overheating is associated with an increased risk of SIDS

The nurse is caring for a child with a history of recurrent diarrhea and no identified physical cause for the diarrhea. The nurse believes the child to have diarrhea secondary to stress. What signs and symptoms of stress does the nurse anticipate assessing in this child? Select all that apply. Pyrexia Headache Bedwetting Chronic illness Disturbed sleep patterns

Headache Bedwetting Disturbed sleep patterns Rationale While caring for children, a nurse should look for signs and symptoms of excessive stress. The nurse should further explore the feelings of the patient if there are symptoms such as headache, bedwetting, and disturbed sleep patterns. Chronic illness is not a symptom of stress. Pyrexia is not a symptom of stress but can be seen in other conditions such as diarrhea.

How is the clinical manifestation of infratentorial brain tumors different from that of supratentorial brain tumors? Visual disturbances Emesis after feeding Development of diabetes insipidus Headache, especially on awakening

Headache, especially on awakening Rationale In cases of infratentorial tumors, it is common to have headaches, especially on awakening. Supratentorial tumors most commonly involve visual disturbances. In infratentorial tumors, there are occurrences of vomiting, but it is not related to feeding. Supratentorial tumors involve structures of the midbrain, such as the hypothalamus and pituitary gland tumors, and may cause endocrinopathies such as diabetes insipidus. Diabetes insipidus is not a common manifestation in infratentorial tumors.

The nurse can expect a grieving family to experience certain behaviors. Which common grief reaction might the nurse prepare the parents to experience? Feeling especially close to others Hearing the deceased child's voice Feeling content with the life the child had Feeling thankful for the time they spent with their child

Hearing the deceased child's voice Rationale The nurse can provide the family with anticipatory guidance on what to expect during the period of grieving. Family members who are grieving commonly report hearing the deceased person's voice, feeling distant from others, or seeking reassurance that they did everything possible to help save their child. Most parents can expect to feel distant from others during this time of grieving. Feeling content with the life the child had and feeling thankful for the time they spent with their child are not common grief reactions among parents.

The nurse can expect a grieving family to experience certain behaviors. Which common grief reaction might the nurse prepare the parents to experience? Feeling especially close to others Hearing the deceased child's voice Feeling content with the life the child had Feeling thankful for the time they spent with their child

Hearing the deceased child's voice Rationale The nurse can provide the family with anticipatory guidance on what to expect during the period of grieving. Family members who are grieving commonly report hearing the deceased person's voice, feeling distant from others, or seeking reassurance that they did everything possible to help save their child. Most parents can expect to feel distant from others during this time of grieving. Feeling content with the life the child had and feeling thankful for the time they spent with their child are not common grief reactions among parents.

A patient is learning to cope with a newly diagnosed chronic illness. To help the patient adjust, which actions should the nurse take? Select all that apply. Initiate a standardized plan of care for the child Help the child with incorporating the illness into each stage of life Provide nursing interventions during acute exacerbations of the illness Provide support as the child accepts the illness as a part of each stage of life Educate the child in a way that there is an understanding that illness will be present long-term

Help the child with incorporating the illness into each stage of life Helping the child incorporate the illness appropriately into each stage of growth and development will be a necessary part of nursing care. Provide support as the child accepts the illness as a part of each stage of life Assisting the child and family to accept the illness appropriately into each stage of growth and development, regardless of the child's age at diagnosis, is important for the nurse to address. Educate the child in a way that there is an understanding that illness will be present long-term Nursing care includes assisting the child and family to accept, understand, and incorporate the illness appropriately into each stage of growth and development, regardless of the child's age at diagnosis.

How are the symptoms of supratentorial brain tumor different from those of infratentorial brain tumor? Select all that apply. Ataxia is more common in supratentorial brain tumor. A headache is more common in supratentorial brain tumor. Nystagmus is more common in supratentorial brain tumor. Hemiparesis is more common in supratentorial brain tumor. Precocious puberty is more common in supratentorial brain tumor

Hemiparesis is more common in supratentorial brain tumor. Precocious puberty is more common in supratentorial brain tumor. Rationale Supratentorial tumors are located over the tentorium cerebelli or lie within the midbrain structures. Supratentorial symptoms include hemiparesis. Involvement of the pituitary causes precocious puberty. Ataxia is a symptom of cerebellar tumors, which are a type of infratentorial tumors. Infratentorial tumors often block the flow of cerebrospinal fluid, which leads to increased intracranial pressure and headaches. Nystagmus is also a symptom of cerebellar tumors.

A nine-year-old patient comes to the clinic before a trip to China. The nurse reviews the record and notes that the patient has received the following vaccine doses: 3 Hep B, 2 RV1, 5 DTaP, 4 HIB, 4 PCV13, 2 MMR, and 2 Varicella. Which vaccination(s) will the nurse recommend? Hep A MMR Hep B PPSV23

Hep A The nurse should recommend Hep A vaccination at this time because of the upcoming trip to China. The CDC recommends that individuals who will be traveling to countries that have high or intermediate rate of infection should receive Hep A before traveling.

During a routine physical exam of a child, the nurse notes an abdominal mass, elevated blood pressure and hematuria. The parents report abdominal pain, fatigue and recent fevers. Which lab value would prompt immediate further investigation and intervention? Hyperkalemia Hypoglycemia Low hemoglobin levels Elevated creatinine levels

Hyperkalemia Hyperkalemia is a concerning condition for a patient exhibiting signs of Wilms tumor because it may lead to cardiac dysrhythmias. This finding would prompt immediate further investigation and intervention.

The nurse is reviewing the immunization record for a healthy six-month-old patient at a well visit. The record indicates that the patient received all the recommended vaccines up to this point, including two doses each of RV1, PCV13, and DTaP. Which vaccine(s) should the nurse prepare for this visit? Select all that apply. Rotavirus (RV1) Meningococcal Hepatitis B (Hep B) Inactivated poliovirus (IPV) Pneumococcal conjugate (PCV13) Diphtheria, tetanus, & acellular pertussis (DTaP)

Hepatitis B (Hep B) The third dose of Hep B is usually administered at the six month visit. The first doses of Hep B are administered at birth and again between one to two months of age. The third dose of Hep B is administered between 6 and 18 months of age. Inactivated poliovirus (IPV) The nurse should prepare inactivated poliovirus (IPV) for this visit. This vaccine may be administered any time between 6 and 18 months of age. The final dose in the IPV series can be administered on or after the fourth birthday, at least six months after the last dose. Pneumococcal conjugate (PCV13) The nurse should prepare pneumococcal conjugate (PCV13) for this visit. This will be the third dose in the four-dose series. The final dose of PVC13 will be administered between 12 to 18 months of age. Diphtheria, tetanus, & acellular pertussis (DTaP) The nurse should prepare the third dose of diphtheria, tetanus, & acellular pertussis (DTaP) to administer at the visit. The first two doses are administered at two and four months of age, the third at six months, the fourth between 15 and 18 months, and a booster is given between four and six years of age.

The nurse is caring for a four-year-old patient receiving chemotherapy for non-Hodgkin's lymphoma. The patient is up to date on regularly scheduled vaccines. Which additional vaccine dose does the nurse recommend for this patient? Hib RV1 Tdap Hep A

Hib Haemophilus influenzae type b vaccine is indicated for patients aged 12 to 59 months (just under five years) who are in certain high-risk groups, including those who are receiving chemotherapy.

An adolescent has an enlarged, firm, nontender, movable node in the cervical area. Reed-Sternberg cells are present on histologic evaluation. What is the most probable diagnosis? Wilms tumor Hodgkin lymphoma Infectious mononucleosis Non-Hodgkin lymphoma

Hodgkin lymphoma Rationale In Hodgkin lymphoma, the malignant cells are Reed-Sternberg cells. The disease is characterized by painless enlargement of lymph nodes. In Wilms tumor, an abdominal mass confined to one side of the abdomen is typical. Although Reed-Sternberg cells are common in infectious mononucleosis, the disease affects the respiratory tract and not the cervical area. Non-Hodgkin lymphoma (NHL) is usually diffuse rather than nodular as in Hodgkin lymphoma.

A child is undergoing a bone marrow transplant. Which is the most important step in preventing graft-versus-host disease in the child? Immune system suppression Strict precautions for infections Human leukocyte antigen (HLA) matching Radiotherapy combined with chemotherapy

Human leukocyte antigen (HLA) matching Rationale Human leukocyte antigen (HLA) matching is the most crucial stage to prevent the serious complication of graft-versus-host disease (GVHD). The closer the donor's HLA system matches that of the recipient's, the smaller the chances are of developing GVHD. While the child's immune system is suppressed to prevent rejection of the transplanted bone marrow, a donor's marrow may still contain antigens that do not match the recipient's antigen, which increases the risk of GVHD. Taking all precautions to prevent infections does not have any effect on the chances of developing graft-versus-host disease. Radiotherapy combined with chemotherapy helps to fight cancer rather than prevent GVHD.

An 11-year-old patient is in the clinic for a well visit. The nurse notes that the patient has previously received three doses of Hep B, two doses of Hep A, four doses of Hib, four doses of PCV, four doses of IPV, two doses of MMR, and one dose of VAR. Which vaccines will the nurse plan to administer today? Select all that apply. Varicella (VAR) Hepatitis A (Hep A) Human papillomavirus (HPV) Meningococcal conjugate vaccine Tetanus and diphtheria toxoids and acellular pertussis (Tdap)

Human papillomavirus (HPV) The nurse will plan to administer the first of the three-dose HPV series today. The recommendation is to administer the three doses on a schedule of zero, one to two, and six months to all adolescents aged 11 to 12 years. Correct Meningococcal conjugate vaccine The nurse will plan to administer the first dose of meningococcal vaccine today. A single dose of Menactra or Menveo should be given at age 11 to 12 years, with a booster dose at age 16 years. Correct Tetanus and diphtheria toxoids and acellular pertussis (Tdap) The nurse will plan to administer the first and only dose of Tdap, which is recommended at age 11 to 12 years. Tdap may be administered regardless of when the last tetanus and diphtheria toxoid-containing vaccine was given. Varicella (VAR) The nurse will plan to administer the second and final dose of varicella (VAR) vaccine. This vaccine series is recommended to be administered at age 12 to 15 months and age four to six years, but since the patient has only received one dose to date, the nurse will plan to administer the second catch up dose today.

Which immune defense is enhanced by previous exposure to the pathogen? Passive immunity Humoral immunity Nonspecific immunity Cell-mediated immunity

Humoral immunity Humoral immunity is enhanced by previous exposure to the pathogen. The system "recognizes" and responds to pathogens by destroying them.

A child with lymphoma is receiving extensive radiotherapy. What is the most common side effect of this treatment? Malaise Seizures Neuropathy Lymphadenopathy

Malaise Correct Malaise is the most common side effect of radiotherapy. For children, the fatigue may be especially distressing because it means they cannot keep up with their peers. Seizures are unlikely because irradiation would not usually involve the cranial area for treatment of lymphoma. Neuropathy is a side effect of certain chemotherapeutic agents. Lymphadenopathy is one of the findings of lymphoma

A mother calls the school nurse saying that her daughter has developed school phobia. She has been out of school for 3 days. What would the nurse include in recommendations to the mother? Immediately return the child to school. Determine the cause of the phobia before returning the child to school. Explain to the child that this is the last day she can stay home. Seek professional counseling before forcing the child to return to school.

Immediately return the child to school. Correct The primary goal is to return the child to school. Parents must be convinced gently, but firmly, that immediate return is essential and that it is their responsibility to insist on school attendance. The longer the child is permitted to stay out of school, the more difficult it will be for the child to re-enter. This will only delay the return to school and inhibit the child's ability to cope. Professional counseling is recommended if the problem persists, but the child's return to school should not wait for the counseling

Which intervention does the nurse recognize as having the greatest impact on the significant decline in incidence of communicable diseases of childhood? Immunizations New medical treatments Proper hand washing skills Secondary prevention measures

Immunizations Rationale Immunizations have resulted in a significant decline in the incidence of communicable diseases of childhood. New medical treatments help treat disease but do not reduce the incidence of communicable diseases of childhood. Proper hand washing skills help reduce the spread of infection, but immunizations have the greatest impact in the decline in communicable diseases of childhood. Secondary prevention measures have not had the greatest impact in the significant decline in communicable diseases of childhood.

A mother of a dying 14-year-old is trying to decide whether to order a do not attempt resuscitation (DNAR) order on her child. Which are the next best actions for the nurse to take? Select all that apply. Inform the mother that the DNAR order can be stopped at any time. Try to prevent the DNAR order since it is a time-consuming and difficult process. Intervene immediately since DNAR orders prevent new treatments from being initiated. Discuss how the DNAR order withholds treatments to the child, which facilitates the dying process. Inform the mother that DNAR order prevents interventions from being performed after the child stops breathing.

Inform the mother that the DNAR order can be stopped at any time. A DNAR order can be revoked at any time, and therefore the nurse should inform the mother. Inform the mother that DNAR order prevents interventions from being performed after the child stops breathing. A DNAR order does prevent chest compressions, ventilations, and advance life support treatments from being initiated when the patient's breathing or heartbeat ceases.

An adolescent female is refusing to have the Papanicolaou (Pap) smear exam during her yearly physical because she was told the exam is only for adults. Based on clinical judgment, what is the nursing assessment finding? Fever and chills related to ineffective temperature regulation Fatigue related to discomfort associated with the infectious disease Inability to meet routine health care needs related to insufficient knowledge about the exam Risk for infection because of inadequate knowledge of how to avoid the spread of infectious disease

Inability to meet routine health care needs related to insufficient knowledge about the exam Inability to meet routine health care needs related to insufficient knowledge is the appropriate nursing assessment in an adolescent female who is refusing a Pap smear. Pap smear tests for cervical cancer in both adults and sexually active adolescents. The nurse should provide education to the adolescent regarding the need for the exam.

An unimmunized five-month-old infant is brought to the clinic. The parents decide to vaccinate the infant. Based on the catch up schedule, what vaccines will the infant need to receive today? Select all that apply. Meningococcal Inactivated poliovirus (IPV) Pneumococcal conjugate (PCV13) Haemophilus influenzae type b (Hib) Diphtheria, tetanus, & acellular pertussis (DTaP)

Inactivated poliovirus (IPV) The infant will receive IPV #1 today. The infant may receive IPV #2 in four weeks (per catch up schedule). Correct Pneumococcal conjugate (PCV13) The infant will receive PCV13 #1 today. The infant may receive PCV13 #2 in four weeks (per catch up schedule). Correct Haemophilus influenzae type b (Hib) The infant will receive Hib #1 today. The infant may receive Hib #2 in four weeks (per catch up schedule). Correct Diphtheria, tetanus, & acellular pertussis (DTaP) The infant will receive DTaP #1 today. The infant may receive DTaP #2 in four weeks (per catch up schedule).

Which types of maltreatment are considered sexual? Select all that apply. Incest Neglect Pedophilia Molestation Exhibitionism Child pornography

Incest Pedophilia Molestation Exhibitionism Child pornography Rationale Types of sexual maltreatment include incest, pedophilia, molestation, exhibitionism, and child pornography. Neglect is not considered sexual maltreatment, but can be physical or emotional maltreatment or both.

The thirteen-month-old child had prenatal microsurgery for a myelomeningocele. Which assessment finding indicates the surgery was not completely successful? Increased head circumference Child is unable to walk or stand up A decreased ability to pick up objects Child is only saying sounds like "bah bah"

Increased head circumference Increased head circumference can indicate hydrocephalus which results from the myelomeningocele complications, meaning the surgery was not completely successful in repairing the neural tube defect.

What behavior is characteristic of the concrete operations stage of cognitive development? Inability to put oneself in another's place Progression from reflex to imitative behavior Increasingly logical and coherent thought processes Ability to think in abstract terms and draw logical conclusions

Increasingly logical and coherent thought processes Rationale Increasingly logical and coherent thought processes are characteristic of concrete operations. Children in this stage are able to classify objects. Progression from reflex activity to imitative behavior is characteristic of the sensorimotor stage, which spans the period from birth to 2 years of age. Inability to put oneself in another's place is characteristic of the preoperational stage, ages 2 to 7 years. Adolescents, in the formal operations stage, have the ability to think in abstract terms and draw logical conclusions.

A 7-year-old child has recently died after being treated for leukemia for the past 5 years. The child was unresponsive for days prior to dying and the parents were at the bedside the entire time. The nurse overhears the parents say statements such as, "I feel like this is a dream. I don't know what to do now." "Thank God it is finally over; he's at peace." "Is it wrong to feel this way?" Which emotions or reactions are being expressed? Select all that apply. Indifference to activities of daily living Relief that the child is no longer suffering Numbness to any emotions when around others Guilt related to being relieved over the death of the child Hostility related to the pain experienced during the dying process by the child

Indifference to activities of daily living Emptiness and lack of interest in activities can be displayed as indifference to many aspects of daily living in someone who has experienced a great loss, such as the death of a child. Correct Relief that the child is no longer suffering Parents may be relieved when the child is no longer suffering physically and emotionally, and therefore it is normal for the parents to express these emotions. Correct Numbness to any emotions when around others Numbness may be displayed by parents whose child has just died. It is an expected emotion for parents who have lost a child. Correct Guilt related to being relieved over the death of the child Parents may feel guilt related to the relief experienced when the child died, and therefore this is an expected statement for the parents to make.

L-asparaginase

Induce depletion of asparagine to aspartic acid and ammonia, lowering glutamine and inhibiting protein synthesis for neoplastic cells USES: A.L.L. and A.M.L. in kids ADVERSE: hypersensitivity, hemorrhage, hyperglycemia, pancreatitis

The phases of chemotherapy

Induction-achieves a complete remission or disappearance of leukemic cells Intensification or consolidation therapy- decreases the tumor burden further Prophylactic therapy- prevents leukemic cells from invading the central nervous system Maintenance therapy- serves to maintain the remission phase

What infant is at a high risk for sudden infant death syndrome (SIDS)? Infant weighing 2.7 kg Infant with an Apgar score of 4 Infant with campylobacter infection Infant with gestational age of 42 weeks

Infant with an Apgar score of 4 Rationale The infant with an Apgar score of 4 is at a high risk of SIDS. The infant who weighs 2.7 kg at birth is considered to be of normal birth weight. Campylobacter infection is caused by bacteria and is not a risk factor for SIDS. A gestational age of 42 weeks is not associated with SIDS.

Which statement is correct regarding immunization? Infants should be immunized at birth or within 2 weeks of birth. Preterm infants should receive initial immunization after 6 months. Infants should not receive two vaccines for two different diseases simultaneously. Infants who miss an immunization dose should begin the whole course from the beginning.

Infants should be immunized at birth or within 2 weeks of birth. Rationale The recommended age of immunization in infants is at birth or 2 weeks from birth. Preterm infants should receive vaccines based on their chronologic age. Hepatitis B vaccines, poliovirus vaccines, and varicella can be administered simultaneously at separate injection sites. Infants who miss an immunization dose in between do not have to start the entire course from the beginning, and only that particular dose can be injected.

Manifestations of increased intracranial pressure in infants and children

Infants: Tense bulging fontanel Separated cranial cutures Macewen's sign (cracked-pot sound on percussion) Irritability High pitched cry Increased head circumference Distended scalp veins Poor feeding Crying when disturbed Children: Headache Nausea Forceful vomiting Diplopia; blurred vision Seizures

Which frequent complication of treatment for childhood cancer results from neutropenia? Alopecia Infection Mood changes Nausea and vomiting

Infection Rationale Overwhelming infection is a frequent complication of treatment for childhood cancer secondary to neutropenia. Nausea and vomiting are a result of the side effects of the chemotherapy agents. Alopecia, not neutropenia, is a side effect of the chemotherapy agents. Mood changes are the result of chemotherapy agents and living with the disease, not a result of neutropenia.

The school nurse is auditing the students' immunization records. The nurse reviews the record of a six-year-old who has received four doses of DTaP, three doses of IPV, two doses of MMR, and two doses of varicella since age four. The nurse makes a note to follow up with the student's parents about which vaccine(s)? Select all that apply. Varicella (VAR) Influenza (LAIV) Inactivated poliovirus (IPV) Measles, mumps, rubella (MMR) Diphtheria, tetanus, & acellular pertussis (DTaP)

Influenza (LAIV) The school immunization record does not indicate that the child has received any annual influenza vaccinations. This vaccine is recommended to all healthy individuals beginning at age six months. Inactivated poliovirus (IPV) The fourth and final dose of inactivated poliovirus (IPV) is given between four and six years of age. The record indicates that only three doses of IVP have been administered. To remain up to date, this child should see the primary care provider to receive this vaccine soon. Diphtheria, tetanus, & acellular pertussis (DTaP) The fifth dose of diphtheria, tetanus, & acellular pertussis (DTaP) is given between four to six years of age. This child's record indicates that only four doses have been administered to date. The child should receive another dose of DTaP to remain up to date.

A patient presents with Guillain-Barré syndrome (GBS). What does the nurse anticipate finding in the history and physical? Select all that apply. Tuberculosis skin test that was positive Toe infection positive for staphylococcus Influenza vaccine received one month ago Upper respiratory infection two weeks ago History of frequent urinary tract infections

Influenza vaccine received one month ago The flu shot has been demonstrated to have a link to the autoimmune process triggered in Guillain-Barré. Upper respiratory infection two weeks ago Upper respiratory infection has been associated with the subsequent development of Guillain-Barré syndrome.

A young boy dies in the emergency department after being accidentally shot by his brother with their father's pistol. Although the child's body is disfigured, the father asks to see the body. What is the most appropriate nursing action? Contacting a clergyperson to discuss the situation with the family Informing the father of what to expect and letting him see the child Asking the health care provider to explain the negative consequences of this request Encouraging the father to wait for viewing until the funeral home has prepared the body

Informing the father of what to expect and letting him see the child Rationale Preparing the family before the viewing by telling them what to expect, including the bodily changes from the accident, any tubes, and cold skin, is an appropriate nursing response to this family's request. Had the father requested a clergyperson, calling one would be an appropriate nursing action. Asking the health care provider to explain the negative consequences of the request is not an appropriate response by the nurse. Delaying or keeping family members from seeing the child is not appropriate. A staff member with bereavement training should accompany the family.

The nurse is assessing a child with non-Hodgkin's Lymphoma and notes elevated serum sodium, serum potassium, BUN, and uric acid levels. The child reports urinating only once in the past two days. Which action is most important for the nurse to take? Insert urine catheter. Obtain a urine culture. Initiate IV fluid resuscitation. Monitor serum electrolyte levels.

Initiate IV fluid resuscitation. Elevated serum sodium, serum potassium, BUN, and uric acid levels in a child with non-Hodgkin's Lymphoma who has urinated only once in the past two days indicates a need for intensive IV fluid administration to prevent kidney failure.

Natural immunity

Innate resistance to infection

Which are the nursing interventions for myelosuppression in a child receiving radiotherapy for cancer? Select all that apply. Institute bleeding precautions. Use rectal suppositories if needed. Administer antibiotics as prescribed. Initiate workup for sepsis as ordered. Monitor for high rectal temperatures.

Institute bleeding precautions. Administer antibiotics as prescribed. Initiate workup for sepsis as ordered. Rationale The nurse should institute precautions for bleeding because many of the drugs along with radiotherapy cause myelosuppression. These precautions can include telling the patient to avoid any activities that may cause injury or bleeding as well as telling the patient to avoid aspirin and aspirin-containing products. In myelosuppression, the body is defenseless against invading organisms, especially the body's normal bacterial flora. The nurse should administer antibiotics as prescribed and initiate a sepsis workup. Giving rectal suppositories and taking rectal temperatures should be avoided when myelosuppression is suspected. These processes can traumatize the body and may cause hemorrhage.

A child has undergone a surgery for the removal of a brain tumor. Which infection prevention measures should the nurse employ when caring for the child? Select all that apply. Check ocular signs twice a day. Institute deep breathing technique. Perform regular pulmonary assessments. Check level of consciousness and sleep patterns. Report if there is any foul odor from the dressing.

Institute deep breathing technique. Perform regular pulmonary assessments. Check level of consciousness and sleep patterns. Rationale Deep breathing techniques and regular pulmonary assessments are instituted in order to prevent respiratory tract infections. The nurse should also check the child's level of consciousness and sleep patterns. A lethargic or irritable state may indicate increasing intracranial pressure that could be due to meningitis infection, hemorrhage, or edema. A nurse should check and record ocular signs such as papillary changes at least every hour, because a change could indicate an increase in intracranial pressure following the surgery. A foul odor from the dressing may be an indication of infection that is already present. It should be reported and a culture should be taken. Such an action does not prevent infection but ensures early and adequate measures to tackle the infection.

what are the nursing prioritis in care of a child with communicable disease

Institute isolation precautions Provide comfort support the family itching - cool baths no soap, calamine lotion, light non irritating clothing, keep out of sun, keep nails short or use mittens for severe itchin antipruritic - diphehendramine, hydroxyzine antypyuretic for fever lozeges for sore throat and analgesics bland foods and increased fluids, bed rest beneficial distraction from discomfort

Which nursing intervention is appropriate in the care of a child with bacterial conjunctivitis? Oral antihistamines to minimize itching Continuous warm compresses to relieve discomfort Application of optic corticosteroids to reduce inflammation Intermittent warm, moist compresses to remove crusts on the eye area

Intermittent warm, moist compresses to remove crusts on the eye area Rationale The eye should be kept clean. Intermittent warm, moist compresses can soften the crusting for easier removal, maintaining the cleanliness of the eye. Antihistamines are not usually necessary for bacterial conjunctivitis. Continuous warm compresses would promote bacterial growth. Antibiotics are the treatment of choice for bacterial infections; optic corticosteroids are not warranted.

A mother shouts at her 7-year-old child for doing wrong. Two days after this incident, the child develops fever. The child believes that fever is a punishment by God. How does the nurse interpret the child's response? The child may require spiritual healing. The child has a psychologic disorder. The child is suffering from depression. It is a normal behavior seen at this age.

It is a normal behavior seen at this age. Rationale At this age, children are fascinated by the concepts of God, hell, and heaven. They show concrete thinking at this age, and abstract thoughts like those of spirituality and God are difficult for them to understand. Children believe that God will punish them for misbehavior. Fever as a disease manifestation is not associated with spiritual beliefs, psychologic disorders, or depression.

A child has optic glioma that requires operation. How does the nurse prepare the child mentally before a brain surgery? Select all that apply. Minimize the number of counseling sessions. Introduce the idea of wearing a wig postoperatively. Emphasize that various symptoms would be stopped. Provide information based on the child's level of understanding. Refrain from disclosing if there will be blindness in one eye postoperatively.

Introduce the idea of wearing a wig postoperatively. Provide information based on the child's level of understanding. Rationale Because the child's hair will be shaved before the surgery, the nurse should introduce the idea of wearing a wig. The child should be taught about the surgery in simple language and provided information as per the level of the child's understanding. There should be several counseling sessions starting from the planning of the surgery until before the surgery. Information should be delivered in small amounts to let the child pursue additional answers. The nurse should refrain from saying that various symptoms would be stopped. Postoperatively, the child may encounter various symptoms of discomfort and ill health (in the case of an optic glioma, this may include blindness in one eye). Being honest with the child helps the child cope better afterwards.

A child has passed away and it is now time to clean the body. Which should the nurse do next? Invite the parents and siblings to assist in preparing the body Quickly clean the body before the parents have time with the dead child Ask the parents to leave the room until death cares are completed Remove all elimination waste from the body and prepare the body for the morgue

Invite the parents and siblings to assist in preparing the body Including the family in the final act of care for the child may provide a means of closure for them, and therefore this is an appropriate action for the nurse to take.

Indirect contact transmission

Involves contact of a susceptible host with a contaminated intermediate object, usually an inanimate object in the patient's environment

Which treatment modes require further education when reviewing common poisonings in children? Select all that apply. Iron and activated charcoal Paint thinner and inducing emesis Drain cleaner and activated charcoal Acetaminophen and N-acetylcysteine Acetylsalicylic acid and activated charcoal

Iron and activated charcoal Paint thinner and inducing emesis Drain cleaner and activated charcoal Rationale Poisons and treatment modes that need further education include activated charcoal being used to treat iron and drain cleaner ingestion. Activated charcoal does not absorb iron and is contraindicated for drain cleaner because it may infiltrate burned tissue. Inducing emesis is generally contraindicated for substances such as paint thinner. Acetaminophen should be treated with its antidote, N-acetylcysteine. Acetylsalicylic acid is treated with activated charcoal.

The nurse is assessing a child with retinoblastoma. The medical history indicates recent vitreous seeding. Chemotherapy was used to decrease the size of the tumor. Which further treatment does the nurse expect to be done? Enucleation Irradiation Drug therapy Antibiotic therapy

Irradiation Rationale Irradiation is used when vitreous seeding has taken place, and chemotherapy has been done to decrease the size of the tumor. Enucleation is done when there is no chance of restoring vision. Drug therapy is used to treat metastatic disease. Antibiotic therapy is useful to treat severe eye infections.

What is descriptive of a parent who is an abuser? Usually has a good support system. Is usually well educated. Is likely a single parent or from a young parent family. Often has good knowledge of parenting skills.

Is likely a single parent or from a young parent family. Correct Younger parents and single parents are at higher risk to be abusers. Abusive families are often socially isolated and have few support systems. They often have additional stressors such as low-income circumstances and little education

A child comes to the clinic and the nurse finds out the patient has been at home for the past three weeks. Which factor needs to be addressed in this nurse's discussion with this child's parents? Fatigue Isolation from friends Risk for fever and chills Inability to meet routine health care needs

Isolation from friends Prolonged confinement at home predisposes one to isolation from friends. Being away from school and friends contributes to this assessment.

What is the purpose in allowing family members to interact with the body after the patient dies? It allows the family members to say their final good-byes. To assist the nurse with preparing the body for the funeral home. To make sure the wishes of the family are upheld until the body goes to the funeral home. To complete the legal paperwork associated with a child's death prior to the body going to the funeral home

It allows the family members to say their final good-byes. Allowing interaction with the body after the patient dies allows for emotional closure for the family members, which is important for grieving.

How is proton beam radiation different from traditional form of radiation? It requires higher doses of radiation. It deposits energy beyond the cancer site. It emits positively charged subatomic particles. It works on the principle of selective cell destruction.

It emits positively charged subatomic particles. Rationale Proton beam radiation emits protons or positively charged subatomic particles. These are used in destroying cancer cells. The traditional form of radiation emits x-ray particles that are not charged. Proton beam radiation does not require higher doses of radiation than x-ray radiation. Proton beam radiation does not affect the organs surrounding the target areas; x-ray radiation deposits energy beyond the cancer site and that has a long-term effect. Both proton beam radiation and x-rays work on the principle of selective cell destruction.

Laboratory Assessment/cancer

Majority of patients have a CBC, serum chemistries, LFT's, coagualtion studies, and a urinalysis done upon initial presentation; Frequent CBC's are necessary for monitoring effects of therapy and in some hematologic malignancies, response to therapy; Blood chemistry yields important information with regards to kidney, liver, bone function, and electrolyte balance; Blood chemistry tests are important to help detect the extent of disease and also to monitor for S/E during therapy; Regular blood chemistries and urinalysis are standard procedures through the course of the disease

A mother shouts at her 7-year-old child for doing wrong. Two days after this incident, the child develops fever. The child believes that fever is a punishment by God. How does the nurse interpret the child's response? The child may require spiritual healing. The child has a psychologic disorder. The child is suffering from depression. It is a normal behavior seen at this age.

It is a normal behavior seen at this age. Rationale At this age, children are fascinated by the concepts of God, hell, and heaven. They show concrete thinking at this age, and abstract thoughts like those of spirituality and God are difficult for them to understand. Children believe that God will punish them for misbehavior. Fever as a disease manifestation is not associated with spiritual beliefs, psychologic disorders, or depression.

The parents of a 12-year-old child complain to the nurse that the child asks them about masturbation after watching a contraceptive advertisement. The parents are anxious about how to respond to their child's questions. What response by the nurse helps relieve the parents' anxiety? It is an age-associated behavior. The child has a mental disorder. The child has a hormonal disorder. The child needs psychiatric treatment.

It is an age-associated behavior. Rationale The nurse tells the parents that masturbation is common in school-aged children due to hormonal changes in the body. The parents should have a good relationship with the child and provide sex education. Curiosity about sex does not indicate that the child has a mental, hormonal, or psychiatric disorder.

Several nurses tell their nursing supervisor that they want to attend the funeral of a child for whom they had cared. Which is the supervisor's best response? It is inappropriate because it worsens burnout on the nursing staff. It is appropriate because families expect this expression of concern. It is appropriate because it can assist in the resolution of personal grief. It is inappropriate because it is considered unprofessional behavior on the part of the nurses

It is appropriate because it can assist in the resolution of personal grief. Rationale Nurses should attend the funeral of a child if they were close to the family. This will help the nurses grieve and gain closure. Families may or may not expect this expression of concern. The behavior is appropriate if a relationship existed between the nurses and family. Attending the funeral of a child with whom they have developed a relationship may provide nurses a means of grieving and thus help prevent burnout.

Why would continuous infusion be preferred to intermittent infusion when administering the drug cytosine arabinoside? It is less toxic. It is less painful. It is more effective. It is easier to monitor.

It is less toxic. Rationale Continuous infusions are used over an extended period using syringe pumps. Certain drugs can be administered by continuous infusion in higher doses with less toxicity than when they are administered intermittently. Both continuous and intermittent intravenous administrations are done through venous access devices; thus, there is not a difference in the amount of pain. Both infusion systems are equally effective for patient chemotherapy. Accurate monitoring of dosage can be done in both infusion systems.

Why would continuous infusion be preferred to intermittent infusion when administering the drug cytosine arabinoside? It is less toxic. It is less painful. It is more effective. It is easier to monitor.

It is less toxic. Rationale Continuous infusions are used over an extended period using syringe pumps. Certain drugs can be administered by continuous infusion in higher doses with less toxicity than when they are administered intermittently. Both continuous and intermittent intravenous administrations are done through venous access devices; thus, there is not a difference in the amount of pain. Both infusion systems are equally effective for patient chemotherapy. Accurate monitoring of dosage can be done in both infusion systems.

Which features help the nurse to distinguish Hodgkin lymphoma (HL) from non-Hodgkin lymphoma (NHL)? Select all that apply. It primarily involves the lymph nodes. It commonly invades the meninges. It shows Reed-Sternberg cells in the biopsy. It metastasizes to the spleen, liver, and lungs. It usually occurs in children younger than 14 years.

It primarily involves the lymph nodes. It shows Reed-Sternberg cells in the biopsy. It metastasizes to the spleen, liver, and lungs. Rationale HL primarily involves the lymph nodes, whereas NHL is usually diffuse rather than nodular. It is characterized by the presence of Reed-Sternberg cells in a lymph node biopsy. These large cells are multilobed and nucleated with abundant cytoplasm. They have a halo-like clear zone around the nucleolus and are often described as having an "owl's eyes" appearance. HL metastasizes to nonnodal or extralymphatic sites, especially the spleen, liver, bone marrow, and lungs. Invasion of meninges is common in NHL. NHL usually occurs in children younger than 14 years. HL is prevalent in adolescence and in the young adult period, with a striking increase between ages 15 and 19.

A 3-year-old child develops oral ulcers as a side effect from chemotherapy. Which nursing interventions are performed to take proper care of the child's mouth? Select all that apply. Keep child on a bland, moist, soft diet. Rinse the mouth with viscous lidocaine. Use lemon glycerin swabs to clean the mouth. Clean the mouth with a soft sponge toothbrush. Rinse the mouth with chlorhexidine mouthwash.

Keep child on a bland, moist, soft diet. . Clean the mouth with a soft sponge toothbrush. Rinse the mouth with chlorhexidine mouthwash. Rationale Oral ulcers are very painful and often a child does not want to eat. A bland, moist, soft diet is helpful in feeding the child. A toothbrush may hurt the mouth so a soft, sponge toothbrush or Toothette is recommended to use on the child's mouth. Chlorhexidine mouthwash is helpful in maintaining proper oral hygiene. Viscous lidocaine should not be used in a young child because it may result in aspiration and possible seizures. Lemon glycerin swabs are not used because the lemon may irritate the eroded tissue and have a drying effect on the mucosa.

Where are Wilms tumors (nephroblastomas) located? Bone Brain Kidney Lymphatic system

Kidney Correct Wilms tumor, or nephroblastoma, is the most common intra-abdominal and kidney tumor of childhood. Wilms tumors are encapsulated and are located in the abdomen.

Which type of poisoning is appropriate when describing a Hispanic child with symptoms of hyperactivity and hearing impairment and a history of eating tamarindo jellied fruit candy? Iron Lead Arsenic Mercury

Lead Rationale Hyperactivity and hearing impairment along with impulsivity, distractibility, and mild intellectual deficits are observed when the patient is exposed to low-dose lead poisoning. Therefore the nurse anticipates the child to be suffering from lead poisoning caused by the fruit candy, which contains high levels of lead. Iron poisoning occurs due to an overdose of mineral supplements or minerals containing iron. It can lead to symptoms such as abdominal pain, vomiting, and diarrhea, which are not the symptoms observed in the child. Arsenic poisoning usually occurs through contaminated drinking water and does not cause hyperactivity and hearing impairment. Mercury poisoning occurs usually through consumption of certain predator fishes and broken thermometers or thermostats among various other means; it can lead to symptoms such as anorexia, delirium, and tremors.

Which statements are appropriate regarding the relationship between anemia and lead poisoning? Select all that apply. Lead can interfere with the binding of iron onto the heme molecule. Blood lead level (BLL) test is currently used for screening and diagnosis. Lead on the heme molecule can make it appear as those a child has anemia. Children who are iron deficient absorb lead more readily than those with sufficient iron stores. Erythrocyte protoporphyrin (EP) level is a good indicator of toxicity from lead and is mostly used as a screening test.

Lead can interfere with the binding of iron onto the heme molecule. Blood lead level (BLL) test is currently used for screening and diagnosis. Lead on the heme molecule can make it appear as those a child has anemia. Children who are iron deficient absorb lead more readily than those with sufficient iron stores. Rationale True statements regarding the relationship between anemia and lead poisoning are: (1) Lead can interfere with the binding of iron onto the heme molecule, (2) Blood lead level (BLL) test is currently used for screening and diagnosis, (3) Lead on the heme molecule can make it appear as though a child has anemia, and (4) Children who are iron deficient absorb lead more readily than those with sufficient iron stores. It is true that erythrocyte protoporphyrin (EP) level is a good indicator of toxicity from lead, but it is no longer used as a screening test.

Which source is a common cause of lead poisoning in children? Lead-based paint Cigarette butts and ashes Folk remedies that contain lead Pottery or dishes that contain lead

Lead-based paint Rationale Lead-based paint in houses built before 1978 is the most common source of lead poisoning in children. Cigarette butts and ashes do not contain lead. Folk remedies that contain lead, and pottery and dishes that contain lead are not the most common sources of lead poisoning in children.

What is the most frequent source of symptomatic lead poisoning in children? Folk remedies Lead-based paint Unglazed pottery Cigarette butts and ashes

Lead-based paint Correct Lead-based paint in houses built before 1978 is the most frequent source of lead poisoning. Some folk remedies and unglazed pottery may contain lead, but they are not the most frequent source. Cigarette butts and ashes do not contain lead.

Which statement is appropriate regarding abusive head trauma (AHT)? Leads to intracranial bleeding Always shows signs of external injuries Is caused by low water content in the brain Is the result of a small head-to-body ratio

Leads to intracranial bleeding Rationale Abusive head trauma often leads to intracranial bleeding such as subdural and subarachnoid hematomas. AHT involves the violent shaking of a baby, leading to tears in the blood vessels in the brain as the brain rotates inside the skull. It is not true that AHT will always show signs of external injuries. The high water content in an infant's brain furthers the damage, because when the brain rotates, it results in shearing forces that cause tears in the blood vessels and neurons. The large head-to-body ratio of infants is one of the main causes of AHT, because it leads to weak neck muscles.

A child is rushed to the emergency department following a collision on the school yard impacting the left side of the head. The nurse expects which physical finding associated with this injury? Raccoon eyes Hemotympanum Left arm weakness Right leg numbness

Left arm weakness In a contrecoup injury, the weakness will occur ipsilateral to the side of injury because of the rebound of the brain within the skull. Normally an injury to the left side of the head would impact function on the right side of the body.

The nurse is caring for a fourteen-year-old patient suffering from a migraine headache. Which findings cause the nurse the most concern? Select all that apply. Left arm weakness Projectile vomiting Nausea and dizziness Urinary incontinence Sound and light intolerance

Left arm weakness This is a sign of a stroke that can occur with a severe migraine because it is a vascular headache. Though rare, this can be a complication of migraine that warrants immediate attention. Weakness should not be confused with arm tingling, which can be part of the patient's aura. Urinary incontinence This could be a sign of a seizure or even a stroke. Though rare, this can be a complication of migraine that warrants immediate attention.

What is the most common form of childhood cancer? Sarcoma Epistaxis Leukemia Retinoblastoma

Leukemia Rationale Leukemia is the most common form of childhood cancer. Epistaxis is not cancer; it is the medical term for a nosebleed. Sarcoma is not the most common form of childhood cancer; nor is retinoblastoma.

leukemia contd

Leukemia and the bones: there's a weakening of the bones causing fractures Sx of leukemia: -fever -pallor -fatigue -anorexia -petechiae -bone and joint pain (displays few sx) With leukemia, any break in the skin is.. a potential site for infection Onset of leukemia: varies from acute to insidious When do parents start noticing children before being dx with leukemia? -weight loss -petechiae -bruising -complaints of pain Prognosis of leukemia relates to: -WBC count time of dx -Age -genetics -sex Therapeutic management of leukemia: IV and intrathecal chemo agents divided into phases: 1. induction 2.cns preventive therapy/consolidation 3 interim maintenance 4 delayed. intensification 5.maintenance or continuation of therapy 6.remission - less than 5% blast cells in bone marrow during remission - supportive care of transfusion and antibacterial and antifangal agents

Which nursing action is categorized as a passive measure in terms of preventing the recurrence of poisoning in a child at home? Keep drugs out of the reach of children. Limit the number of tablets in one container. Strategize and implement means for effective discipline. Discontinue drugs if there is any evidence of mild toxicity.

Limit the number of tablets in one container. Rationale Passive measures are those that do not require the active participation of either the child or the parent. When there are recurrent instances of child poisoning, it is essential to adopt measures that can prevent poisoning. Limiting the number of tablets in one container effectively serves this purpose. Active measures are those that call for active participation by the child or the parents. Advising the parent to keep drugs out of the reach of children will simply eliminate the chance of drug overdose leading to poisoning. Instructing the parents to formulate ways to ensure effective discipline is a method for dealing with the child's behavior of noncompliance. Instructing the parent to discontinue drugs if mild toxicity is observed is suggested while educating the parents on the correct administration of the drug.

What are the goals associated with hospice care? Select all that apply. Live life without medical support. Live life with choices and dignity. Live life to the fullest without pain. Live life to the fullest without cancer. Live life with the support of family. Live life in the familiar environment of home.

Live life with choices and dignity. Live life to the fullest without pain. Live life with the support of family. Live life in the familiar environment of home. Rationale The goal of hospice care is for children to live life to the fullest without pain, with choices and dignity, in the familiar environment of their homes, and with the support of their families. Medical support is used to support those goals. The goal of hospice is to support the patient. They are not able to remove cancer. If the person is cancer-free, then hospice care is not needed.

Assessment of osteosarcoma

Localized pain at the affected site Palpable mass Limping Limited range of motion Unable to hold heavy objects Pathological fractures at the tumor site Prosthetic fitting Phantom limb pain Problems with self image

Which nursing action is appropriate when providing emergency treatment for poisoning? Look for environmental clues. Perform a self-analysis to identify the poison. Inform the poison control center after stabilizing the patient. Avoid discussing potential poisoning with the patient's relatives to prevent panic.

Look for environmental clues. Rationale When providing emergency treatment to a patient who is a victim of poisoning, it is important to look for environmental clues. An empty container, a nearby spill, or an odor on the patient's breath can provide vital clues regarding the variant of poison ingested. It is not advisable to perform a self-analysis in identifying the poison; instead the nurse should question the victim and witnesses. It is important to immediately contact the poison control center for quick advice regarding treatment. It is necessary to discuss the potential poisoning and its symptoms with the patient's relatives in the absence of other evidence. This includes symptoms of ocular or dermal exposure, to ensure the patient's safety.

The nurse teaches the parents of an infant about which sign of a mild reaction that may occur after vaccination? Hives Seizure Wheezing Loss of appetite

Loss of appetite Loss of appetite is a common mild reaction, particularly in infants, along with fussiness and sleepiness. This is not a medical emergency, and it should be self-limiting.

The family of a dying child expresses concern to the nurse that their child is sleeping more and does not want to eat. The nurse informs the family of which changes that occur as part of the dying process? Select all that apply. Loss of bowel and bladder control Increased sleep and decreased appetite Increased urination and decreased thirst Mottling of the skin and muscle weakness Loss of sensation of the upper body progressing to the lower body

Loss of bowel and bladder control Increased sleep and decreased appetite Mottling of the skin and muscle weakness Rationale Physical signs of approaching death include loss of bowel and bladder control, increased sleep, decreased appetite and thirst, mottling of the skin, and muscle weakness. Decreased, not increased, urination occurs as death is approaching. Also, loss of sensation and movement in the lower extremities occurs, progressing toward the upper body.

Which environment would the nurse prepare for a child in the dying process? Select all that apply. Low lighting and soft music playing. Quiet space away from a lot of activity. Multiple opportunities to create cherishing moments. An energetic, social environment with friends and family. An environment conducive to allowing family members to stay with the child.

Low lighting and soft music playing. The goal of nursing care for the dying child is to create a calm and peaceful environment. Quiet space away from a lot of activity. The goal of nursing care include creating a quiet and secluded environment. Multiple opportunities to create cherishing moments. Creating memories allows the family to spend time together, which become a source of comfort after the child dies. An environment conducive to allowing family members to stay with the child. One of the goals of nursing care for the dying child is to allow family members to stay with the child

A child is suspected to have infratentorial brain tumor. Which is the most useful tool for diagnostic evaluation of such a tumor? Lumbar puncture (LP) Electroencephalography Computerized tomography (CT) Magnetic resonance imaging (MRI)

Magnetic resonance imaging (MRI) Rationale MRIs are the gold standard diagnostic procedures for detecting infratentorial tumors because they help in the early detection of brain tumors and assessment of tumor growth during or after treatment. Lumbar punctures (LP) are dangerous; if there is increased intracranial pressure, there is a possibility of brainstem herniation after the sudden release of pressure. Electroencephalography is another procedure that can be used in diagnostic evaluation, but it is not ideal. A computerized tomography is performed commonly in urgent cases when an MRI is not available.

A child presents to the emergency department with increased intracranial pressure. Which strategies should be included in the nurse's care plan to help manage the pressure? Select all that apply. Maintain a comfortable room temperature. Administer intravenous fluids (IV) as prescribed. Elevate the child's head to 30 degrees at all times. Administer anticonvulsant medications as needed. Administer insulin for elevated blood glucose levels as prescribed.

Maintain a comfortable room temperature. Normothermia should be maintained in children with increased intracranial pressure because hyperthermia and hypothermia cause stress in the brain as it tries to maintain normothermia. Administer intravenous fluids (IV) as prescribed. Normovolemia is necessary. The nurse ensures that there is not hypervolemia or hypovolemia because decreased or increased blood flow causes stress on the brain. Elevate the child's head to 30 degrees at all times. The child with increased intracranial pressure should have the head elevated to decrease pressure caused by positioning. Administer insulin for elevated blood glucose levels as prescribed. Normal blood glucose is essential to prevent increased metabolic demands placing stress on the brain.

A nurse is preparing discharge instructions for a school-aged child newly diagnosed with an autoimmune disease. What information should the nurse provide to both the child and the parents to prevent behavioral problems at home? Be lenient, which will be helpful, while your child learns to cope with the autoimmune disorder. Remain firm in the expectations of your child and refrain from any modifications of those expectations. Maintain consistent discipline, which is important to help your child continue to develop socially and emotionally. Minimize the expectation that the child performs chores and understand that the child might struggle in school so allowing a decrease in grades is acceptable.

Maintain consistent discipline, which is important to help your child continue to develop socially and emotionally. Consistency is important to the development of a school-aged child. The child should have similar expectations about behavior after a chronic illness diagnosis.

In a child with a complete spinal cord injury at T6, which interventions should the nurse implement to prevent complications? Select all that apply. Administer saline nose spray for nasal congestion. Maintain patient's bed position at 45-degree angle. Notify health care provider for BP of 162/89 mm Hg. Cover with blanket when goose bumps are observed. Hold steroid medications for heart rate less than 50 bpm.

Maintain patient's bed position at 45-degree angle. Angling the bed at 45 degrees promotes venous blood return, decreased intracranial pressure (ICP), and keeping child's head midline. Correct Notify health care provider for BP of 162/89 mm Hg. The onset of autonomic dysreflexia in child with a complete spinal cord injury at T6 includes sudden increase in systolic blood pressure. Urgent intervention will be needed.

The hospice nurse is planning care for a child with a terminal illness. Which nursing interventions does the nurse include in the plan of care? Select all that apply. Discontinue eye and mouth care Maintain skin with skin lotion as tolerated Maintain good hygiene by giving baths as tolerated Discontinue unnecessary medications and procedures Continue administering oral medications to manage symptoms

Maintain skin with skin lotion as tolerated Maintain good hygiene by giving baths as tolerated Discontinue unnecessary medications and procedures Rationale Physical care during the terminal phase of a patient's life include maintaining good hygiene by giving baths and skin care with lotion as tolerated. Unnecessary medications and procedures are discontinued. Necessary medications are administered intravenously or subcutaneously if at all possible (not orally, because often the level of consciousness is diminished). Eye and mouth care should be done frequently.

What are included in nursing considerations related to the administration of chemotherapeutic drugs? Anaphylaxis cannot occur because the drugs are considered toxic to normal cells. Infiltration will not occur unless superficial veins are used for the intravenous infusion. Many chemotherapeutic agents are vesicants that can cause severe cellular damage if drug infiltrates. Good hand washing is essential when handling chemotherapeutic drugs, but gloves are not necessary.

Many chemotherapeutic agents are vesicants that can cause severe cellular damage if drug infiltrates. Correct Chemotherapeutic agents can be extremely damaging to cells. Nurses experienced with the administration of vesicant drugs should be responsible for giving these drugs and be prepared to treat extravasations if necessary. Anaphylaxis is a possibility with some chemotherapeutic and immunologic agents. Infiltration and extravasations are always a risk, especially with peripheral veins. Gloves are worn to protect the nurse when handling the drugs, and the hands should be thoroughly washed afterward

Which nursing consideration is related to the administration of chemotherapeutic drugs? Anaphylaxis cannot occur because the drugs are considered toxic to normal cells. Infiltration will not occur unless superficial veins are used for the intravenous infusion. Good hand washing is essential when chemotherapeutic drugs are handled, but gloves are not necessary. Many chemotherapeutic agents are vesicants that can cause severe cellular damage if the drug infiltrates.

Many chemotherapeutic agents are vesicants that can cause severe cellular damage if the drug infiltrates. Rationale Chemotherapeutic agents can be extremely damaging to cells. Nurses experienced with the administration of vesicant drugs should be responsible for giving these drugs and prepared to treat extravasation if necessary. Gloves are worn to protect the nurse handling the drugs, and the hands should be thoroughly washed afterward. Infiltration and extravasations are always a risk, especially with peripheral veins. Anaphylaxis is a possibility with some chemotherapeutic and immunologic agents, including asparaginase (Elspar).

Which vaccine would the nurse instruct the patient to avoid because of risk of allergic reaction from neomycin? Tetanus Pertussis Inactivated polio Measles, mumps, and rubella (MMR)

Measles, mumps, and rubella (MMR) Rationale MMR vaccine contains minute amounts of neomycin and so it is contraindicated in patients who are allergic to neomycin. The tetanus vaccine, pertussis vaccine, and inactivated polio vaccine do not contain neomycin and are safely used in patients who are allergic to neomycin.

Which nursing action is appropriate when administering N-acetylcysteine? Monitor for addiction Medication may turn urine red to orange Medication should be diluted with fruit juice or soda Medication should be sipped slowly to prevent aspiration

Medication should be diluted with fruit juice or soda Rationale When administering N-acetylcysteine, the medication should be diluted with fruit juice or soda due to its offensive odor. N-acetylcysteine is not addictive and should be taken quickly because it has such an offensive odor. Chelation therapy may cause urine to turn red to orange.

Which side effect of metronidazole would the nurse recognize in a child with giardiasis? Constipation Inflammation Urine retention Metallic taste in mouth

Metallic taste in mouth Rationale Metronidazole (Flagyl), taken for giardiasis, can leave a metallic taste in the mouth. Inflammation, urine retention, and constipation are not side effects of metroindazole (Flagyl).

The clinic is loaning a federally approved car seat to a 4.5-kg (10-pounds) infant's family. Where does the nurse teach the family is the most appropriate location to place the infant car seat? Back seat facing forward. Middle of the back seat facing rearward. Front seat with air bags on passenger side. Front seat if there is no air bag on the passenger side.

Middle of the back seat facing rearward. Correct The rear-facing car seat provides the best protection for an infant's disproportionately heavy head and weak neck in the event of an accident. The middle of the back seat is the safest position for the child. Severe injuries and deaths in children have occurred from air bags deploying on impact in the front passenger seat.

Which occupations or hobbies involve lead and can contribute to lead poisoning? Select all that apply. Mining Sewing Radiator repair Furniture refinishing Stained-glass making

Mining Radiator repair Furniture refinishing Stained-glass making Rationale Occupations or hobbies that involve lead and can contribute to lead poisoning include mining, radiator repair, furniture refinishing, and stained-glass making. Sewing does not expose a person to lead.

Postoperative interventions of neuroblastoma

Monitor for postop complications Monitor for complications from radiation/chemo Provide support Refer parents to appropriate community services

Chemotherapy assessments

Monitor for severe bone marrow suppression Monitor for infection and bleeding Protect the child from life threatening infections Monitor for nausea, vomiting, and diarrhea Stool softeners as prescribed Provide rectal hygeine administer antiemetics before chemo Monitor for dehydration, hemorhage cystitis, peripheral neuropathy Assess oral mucous membranes for mucositis Hair loss Central venous access device Feelings and concerns of patient and family introduce to other families with cancer consult chaplains if necessary

The nurse is caring for a child with a spinal cord injury. Which intervention is a priority? Assess each shift for characteristics of stool. Administer an antacid medication twice daily. Bathe, dry, and provide skin care to the patient. Monitor the patient's temperature and skin changes.

Monitor the patient's temperature and skin changes. A spinal cord injury can disrupt the patient's sympathetic nervous system which controls dermal vasodilation. Hyper or hypothermia causes increased stress on the healing of the neurologic system, and monitoring temperature and skin changes will be a priority.

A 3-year-old patient is diagnosed with hydrocephalus. A ventricular shunt was placed to relieve the pressure. What is the nursing care priority for a patient who had a ventricular shunt put into place? Assessing for proper bowel movements Assessing neurologic status every two hours Monitoring for fluid leaking from the incision Monitoring for headaches when the patient sits up

Monitoring for fluid leaking from the incision The main priority in the nursing care of a patient who had a ventricular shunt placed is the prevention or early detection of shunt infection, malformation, and malfunction. A leaking incision presents a high risk of infection.

What is an important consideration in the prevention of injuries during middle childhood? Most injuries occur in or near school or home. Peer pressure is not strong enough to affect risk-taking behavior. Injuries from burns are the highest at this age because of fascination with fire. Lack of muscle coordination and control results in an increased incidence of injuries.

Most injuries occur in or near school or home. Rationale Most children in the middle years spend the majority of their time in and around school or home; therefore the risk for injuries is increased in and around these areas. Peer pressure as an impetus for risk-taking behavior begins in the school-age years but is more significant in adolescence. Burn injuries are higher in the toddler years, when children are curious and mobile. They may expose themselves to objects capable of burning them (e.g., hot pots of water in the kitchen). Automobile accidents, with children as pedestrians or passengers, account for the majority of severe accidents in the middle years. School-age children have more refined muscle development, which results in an overall decrease in the number of accidents. Lack of muscle coordination and control leading to injuries occurs in younger children.

Neuroblastoma common locations

Most tumors develop in the adrenal gland or the retroperitoneal sympathetic chain Other sites may be within the head, neck, chest, or pelvis

Which is appropriate mouth care for a toddler with mucosal ulceration related to chemotherapy? Mouthwashes with normal saline Lemon glycerin swabs for cleansing Mouthwashes with hydrogen peroxide Local anesthetic such as viscous lidocaine before meals

Mouthwashes with normal saline Rationale Normal saline mouthwashes are the preferred mouth care for this age group. The rinse will keep the mucosal surfaces clean without adverse effects on mucosa or problems if the child swallows the rinse. Lemon glycerin swabs can irritate eroded tissue and decay teeth. Hydrogen peroxide delays healing by breaking down protein. Viscous lidocaine is not recommended for toddlers because it depresses the gag reflex and could result in aspiration.

Which disease would cause a child to have symptoms of fever, headache, earache while chewing, neck swelling, and testicular pain? Rubella Mumps Rubeola Pertussis

Mumps Rationale The signs and symptoms in this child indicate mumps. Fever, headache, and earache while chewing food are present in the prodromal stage of mumps. The swelling in the neck is caused by parotitis, an inflammation of the parotid salivary gland. Epididymo-orchitis is inflammation of the testes and epididymis, and it is caused by mumps. It is one of the common complications seen in male children. These findings are not present in the other childhood diseases such as rubella, rubeola, and pertussis.

Which syndrome is appropriate when a parent fabricates a child's medical history and symptoms and insists on tests being run on the child? Psychosis Child abuse Abusive head trauma Munchausen syndrome by proxy

Munchausen syndrome by proxy Rationale Munchausen syndrome by proxy, also known as medical child abuse or factitious disorder by proxy, is a rare but serious form of child abuse in which caregivers deliberately exaggerate or fabricate histories and symptoms or induce symptoms. Because of the history of symptoms provided by the caregiver, the child endures painful and unnecessary medical testing and procedures. Common symptoms presented are seizures, nausea and vomiting, diarrhea, and altered mental status; usually only the perpetrator witnesses them. Abusive head trauma is a serious form of physical abuse caused by violent shaking of infants and young children. Psychosis is a mental health condition associated with disconnect from reality. This could be connected with this condition but does not best identify this situation.

acetominophen poisoning treatment

N-acetylcysteine diluted in fruit juice or soda, oral admin is given as loading dose followed by 17 doses, or continual infusion IV

How is childhood non-Hodgkin lymphoma (NHL) different from Hodgkin lymphoma? In NHL, there is a presence of Reed-Sternberg cells. In NHL, the B-cells are more abundant than T-cells. NHL is usually a nodular rather than diffuse lymphoma. NHL disseminates earlier, more often, and more rapidly.

NHL disseminates earlier, more often, and more rapidly. Rationale In NHL, the dissemination occurs more often and more rapidly than with Hodgkin disease. The presence of Reed-Sternberg cells is the characteristic feature of Hodgkin lymphoma, not NHL. Both B- and T-cells are evenly present in NHL. NHL is usually diffuse while Hodgkin lymphoma is mostly nodular, with painless enlargement of lymph nodes.

Which nursing action appropriately describes a nurse wearing gloves while assessing a child with herpetic gingivostomatitis? Unnecessary because the virus is not easily spread Unnecessary because the virus is sexually transmitted Necessary because the virus can easily enter breaks in the skin Necessary only if the nurse touches the nurse's own mouth after touching the child's mouth

Necessary because the virus can easily enter breaks in the skin Rationale Herpes simplex virus (HSV) easily enters breaks in the skin and can cause herpetic whitlow on the fingers. Herpetic gingivostomatitis is usually caused by HSV-1. HSV-2 is usually transmitted through sexual activity. Gloves are always necessary because the virus is easily spread.

During assessment of a 6-month-old infant, the nurse notes the child is in the 1st percentile for weight, has the psychosocial development of a 3-month-old infant and is wearing soiled clothing. Which type of child abuse would the nurse suspect? Sexual Neglect Physical Emotional

Neglect Neglect is characterized by failure to meet the child's basic needs, such as cleanliness and nutrition. This child's symptoms indicate neglect.

A 15-year-old patient is in the clinic for a well visit. The nurse notes that the patient received the first Hep B vaccination with Recombivax HB over eight months ago and the second dose four months ago. Which action is most appropriate? Schedule the third dose of Recombivax HB in four weeks. Administer the third and final dose of Recombivax HB today. No action is necessary because Recombivax HB is a dose-dose series. Inform the patient and parent that dose two of Recombivax HB should be repeated.

No action is necessary because Recombivax HB is a dose-dose series. Recombivax HB is an adult formulation licensed for use for individuals aged 11 through 15 years. It is a two-dose series, and the doses are recommended to be administered with an interval of four months.

The nurse assesses the patient's breathing patterns and laboratory values. Which findings in the pediatric patient with a head injury indicate a possible brainstem herniation? Select all that apply. Elevated arterial blood gas values. Decreased arterial blood gas values. Normal arterial blood gas (ABG) values. Breathing pattern with periods of increased rate and depth followed by periods of decreased rate and depth. Breathing pattern with long periods of decreased rate and shallow depth followed by short periods of apnea.

Normal arterial blood gas (ABG) values. A Cheyne-Stokes breathing pattern will be associated with normal arterial blood gas values which can help to differentiate this from other breathing patterns by patients experiencing respiratory distress. Breathing pattern with periods of increased rate and depth followed by periods of decreased rate and depth. Cheyne-Stokes is characterized by periods of increased rate and depth followed by periods of decreased rate and depth of breath, a pattern that will help differentiate from other breathing patterns and indicates imminent death as expected with a brainstem herniation.

The patient is diagnosed with Guillain-Barré syndrome. The nurse expects which findings in the cerebral spinal fluid (CSF) analysis? Select all that apply. Normal glucose level Decreased fluid pressure Clear cerebral spinal fluid Elevated protein concentration Increased red blood cell level (RBC) Normal white blood cell level (WBC)

Normal glucose level Glucose levels are decreased in many bacterial infections as the bacteria "eat" the glucose found. GB is unique in that there may be normal glucose levels found in the CSF. Clear cerebral spinal fluid Cloudy CSF is expected in the case of bacterial meningitis. Correct Elevated protein concentration Protein is elevated due to inflammation and will be seen in the CSF of a patient with Guillain-Barré syndrome. Normal white blood cell level (WBC) White blood cell levels are not affected by Guillain-Barré syndrome. This differs from other infections where elevated white blood cell levels can be seen.

Parents whose child recently died tell the nurse that they are able to hear the voice of their dead child. What is the nurse's best explanation for this behavior? Poor coping Mental breakdown Normal grief reaction Complicated grief reaction

Normal grief reaction Rationale Parents telling the nurse that they are able to hear the voice of their dead child indicates a normal grief reaction and does not indicate that the parents are coping poorly. A poor coping mechanism would lead to depressive disorders or anxiety and poor health. There is no evidence of mental breakdown, which is characterized by depression and anxiety. Having intense intrusive thoughts, unusual sleep disturbances, and loss of interest in personal activities are the symptoms of a complicated grief reaction.

A group of boys has formed a club that is open to other boys who like to play sports. What is this behavior considered? Bullying Promoting gang violence Promoting isolation and exclusion Normal social development of school-age children

Normal social development of school-age children Rationale One of the outstanding characteristics of middle childhood is the creation of formalized groups or clubs. Peer-group identification and association are essential to a child's socialization. Poor relationships with peers and a lack of group identification can contribute to bullying. Poor relationships with peers can promote gang violence and isolation or exclusion

Which nursing interventions are appropriate for maintaining adequate thermoregulation in a child with febrile illness? Select all that apply. Keep the child in a warm bed with heavy blankets. Note the development of tachycardia, irritability, lethargy, tachypnea, or seizure activity. Teach the parents to encourage cool fluids, popsicles, and monitor intake and output. Teach the parents measures to lower temperature through the administration of aspirin. Monitor axillary temperature at least every hour until temperature stabilizes below 99.5° F (37.5° C).

Note the development of tachycardia, irritability, lethargy, tachypnea, or seizure activity. Note the development of tachycardia, diaphoresis, lethargy, tachypnea, or seizure activity is an appropriate intervention for maintaining adequate thermoregulation in the child with a febrile illness. Teach the family the signs and symptoms of hyperthermia and the complications that should be promptly reported to their primary medical caregiver. Teach the parents to encourage cool fluids, popsicles, and monitor intake and output. Teaching the parents to encourage cool fluids, popsicles, and monitor intake and output is an appropriate intervention for maintaining adequate thermoregulation in the child with a febrile illness because the child is at risk for altered fluid balance. Monitor axillary temperature at least every hour until temperature stabilizes below 99.5° F (37.5° C). Teaching the family normal temperature parameters and temperature-monitoring techniques is an appropriate intervention for maintaining adequate thermoregulation in the child with a febrile illness. Many health care providers define fever as an oral temperature of 100.5° F or greater.

A child undergoes surgical removal of a brain tumor. During the postoperative period, the nurse is monitoring the child and notes that the child is restless, the pulse rate is elevated, and the blood pressure has decreased significantly from the baseline value. The nurse suspects that the child is in shock. Which is the most appropriate nursing action? 1. Place the child in a supine position. 2. Notify the health care provider (HCP). 3. Place the child in Trendelenburg's position. 4. Increase the flow rate of the intravenous fluids.

Notify the health care provider (HCP). In the event of shock, the HCP is notified immediately before the nurse changes the child's position or increases intravenous fluids. After craniotomy, a child is never placed in the supine or Trendelenburg's position because it increases intracranial pressure (ICP) and the risk of bleeding. The head of the bed should be elevated. Increasing intravenous fluids can cause an increase in ICP.

A child being treated for pain related to neuroblastoma reports abdominal pain and nausea. The nurse notes abdominal distention and diminished bowel sounds. The patient's last bowel movement was two days ago. What is the nurse's priority action? Monitor respiratory effort Educate about high fiber diet Notify the healthcare provider Obtain an order for a stool softener

Notify the healthcare provider Monitoring for bowel obstruction is an important nursing assessment for the patient with neuroblastoma. The nurse should notify the health care provider immediately if signs of obstruction are present. Obtaining a history of bowel habits will alert the nurse to the presence of dramatically altered bowel habits in the presence of other signs of obstruction.

The nurse is caring for a child with leukemia. The patient has severe pain associated with weight loss following a chemotherapy treatment. What intervention does the nurse perform for improving the health status of the patient? Antibiotic therapy Parenteral therapy Nutritional therapy Ventilation therapy

Nutritional therapy Rationale Children with chronic disorders require prolonged health care for reducing the side effects of the disease condition or the treatment. Disorders like leukemia require a prolonged chemotherapy regimen. Repeated chemotherapy may result in severe pain and is associated with conditions like weight loss. Interventions include nutritional therapy, preferably home-based nutritional therapy. Antibiotic therapy is used for patients with chronic microbial infections. Parenteral therapy includes administration of intravenous fluids and is used when enteral or oral nutrition is challenging. Ventilation therapy is provided for patients with respiratory disorders.

The nurse is working with a new graduate in developing a plan of care for a newborn infant with spina bifida (myelomeningocele) and hydrocephalus. The nurse reminds the graduate to monitor for increased intracranial pressure (ICP). Which assessment technique should be performed to detect the presence of an increase in ICP? Measure urine specific gravity Assess for increased muscle tone Observe anterior fontanel for bulging Monitor blood pressure for signs of hypotension

Observe anterior fontanel for bulging Excessive buildup of CSF in the newborn's brain will cause expansion and fullness. The nurse can observe this through bulging (secondary to pressure) in the fontanels.

In evaluating a child with a chronic illness, which additional actions by the nurse assist in identifying the child's expected developmental process? Select all that apply. Observe for the presence of developmental delays. Observe for age-appropriate responses by the ill child. Observe errors in the child's perception of the chronic illness. Monitor for miscommunication between family members regarding the child's illness. Evaluate the influence of the family's cultural beliefs on the child's cognitive milestone.

Observe for the presence of developmental delays. Evaluating for developmental delays provides information regarding the child's progress in growth and development. Correct Observe for age-appropriate responses by the ill child. Observing for age-appropriate responses by the child provides information regarding the child's progression in growth and development. Correct Observe errors in the child's perception of the chronic illness. Observing for errors in the child's perception of the illness identifies can be helpful for understand the child's progress in emotional growth and development. Correct Monitor for miscommunication between family members regarding the child's illness. Caring for the child will include knowing how well the family understands the illness. Any miscommunication or misunderstanding about the illness could have an impact on the child's growth and development.

For case management to be most effective, who is the most appropriate case manager? One nurse A panel of experts The insurance company A multidisciplinary team

One nurse Correct Nursing case managers are ideally suited to provide the care coordination necessary. Care coordination is most effective if a single person works with the family to accomplish the many tasks and responsibilities that are necessary. The family retains the role as primary decision maker. It is preferable that the family have only one individual to work with. Most likely, the insurance company will have a case manager focusing on the financial aspects of care. This does not include coordination of care to assist the family

A child presents with a firm protuberant abdomen, cough, and joint pain. Which additional assessments are most important for the nurse to obtain? Select all that apply. Observe gait Assess lung sounds Measure respiratory rate Measure cognitive level Measure strength of extremities

Observe gait A firm protuberant abdomen, cough, and joint pain indicate possible neuroblastoma. The nurse should observe the patient's gait because the tumor could invade the lower extremities and affect balance. Correct Assess lung sounds A firm protuberant abdomen, cough, and joint pain indicate possible neuroblastoma. The nurse should assess lung sounds because the tumor may interfere with the patient's ability to expand the lungs and affect respiration. Correct Measure respiratory rate A firm protuberant abdomen, cough and joint pain indicate possible neuroblastoma. The nurse should measure respiratory rate because the tumor may interfere with the patient's ability to expand the lungs and affect respiration.

A family has decided to gather in the room of a child who has impending death. A prayer has started and then all the family members begin praying and laying their hands on the patient. What action will the nurse take? Ask the chaplain to come and pray with the family. Lay hands on the patient and pray with the parents. Observe the child for restlessness, moaning, or increased muscle tension. Remind the parents that only 2-3 visitors are allowed in the room at a time.

Observe the child for restlessness, moaning, or increased muscle tension. Restlessness, moaning, and increased muscle tension demonstrate increased agitation and could negatively impact the child's comfort level.

The parents of a three-year old child with Wilms tumor report new onset of cough. The nurse notes tachypnea and diminished lung sounds on the left lower lobe. Which order would the nurse anticipate from the provider? Obtain a chest x-ray Prepare patient for biopsy Perform chest physiotherapy Administer breathing treatment

Obtain a chest x-ray Tachypnea, cough, and diminished lung sounds could indicate metastasis to the lungs. A chest x-ray will need to be ordered by the provider to determine if that is the case.

A six-year-old child with leukemia presents with altered level of consciousness (LOC) and sluggish pupils but no cardiac compromise after falling from a bike. The nurse notes a platelet count of 10,000, WBC of four, and hemoglobin of seven. After the patient's airway is stabilized, which priority order should the nurse anticipate? Obtain a head CT Assess serum glucose Administer IV fluid bolus Administer pain medication

Obtain a head CT The nurse should first obtain a head CT on a patient with thrombocytopenia and altered LOC after a head injury to evaluate for cerebral hemorrhage

The nurse cares for a child with Guillain-Barré syndrome. The nurse notes a frequent, weak cough and decreased bilateral hand grips. What actions should the nurse take? Select all that apply. Place a nasal cannula on the patient at 2 L/min. Obtain a pillow nurse call light for patient's use. Raise head of the bed to a semi-Fowler's position. Do not allow patient to have anything to eat or drink. Explain to patient what was assessed and the meaning.

Obtain a pillow nurse call light for patient's use. With inadequate grip strength, and Guillain-Barré, which is progressively decreasing strength from toes working toward neck, patient needs a way to call the nurse. A pillow call light allows patient to turn the head slightly to touch pad that calls the nurse. Correct Raise head of the bed to a semi-Fowler's position. The patient is having difficulty with oral and respiratory muscles. This will affect swallowing of saliva and diaphragmatic excursion. Placing the head of bed (HOB) up will decrease the patient's work and help prevent aspiration and hypoventilation. Correct Do not allow patient to have anything to eat or drink. Patient is at risk for aspiration and needs to be NPO until swallowing can be further evaluated. Correct Explain to patient what was assessed and the meaning. The patient should usually be told what is going on and what health care provider is doing. Patient's Guillain-Barré is progressing as evidenced by decreased strength in hands and inability to forcefully cough. Patient may soon require intubation and should be aware of that.

The nurse is caring for a 16-year-old child diagnosed with Hodgkin Lymphoma who reports shortness of breath, and chest pain. The nurse notes hoarseness of patient's voice when speaking. Which prescription would the nurse take first? Obtain sputum culture Administer pain medication Administer albuterol nebulizer Obtain an emergency chest radiograph

Obtain an emergency chest radiograph Respiratory symptoms and hoarseness suggest mediastinal involvement. Because this can lead to airway obstruction, this must be evaluated immediately. The nurse would obtain a STAT chest x-ray to determine whether a mediastinal tumor is present.

Parents of a two-year-old child report an abdominal mass felt while giving the child a bath. The nurse notes elevated blood pressure, fever, and abnormal hemoglobin. Which priority order should the nurse expect? Obtain sample for urinalysis Prepare patient for head CT Prepare patient for chest radiography Evaluate mass through palpation of abdomen

Obtain sample for urinalysis Abnormal hemoglobin suggests potential hematuria in a patient with an abdominal mass. The nurse would anticipate obtaining urinalysis to assess for hematuria.

The nurse is caring for a 17-year-old with single-node Hodgkin lymphoma who reports right upper quadrant pain, nausea and vomiting. The nurse notes icteric skin and sclera, and abdominal distention. Which prescription would the nurse anticipate? Administer IV pain medicine Assess serum potassium level Obtain serum AST and ALT levels Administer oral antiemetic medication

Obtain serum AST and ALT levels The patient's symptoms indicate potential hepatomegaly and liver involvement related to Hodgkin lymphoma. The nurse would obtain liver function tests to determine the extent of liver damage.

A 6-year-old child is on chemotherapy for acute lymphocytic leukemia. The child refuses to take the drugs because of severe episodes of delayed vomiting in previous chemotherapy cycles. Which drug/s can prevent this from occurring? Ondansetron given after chemotherapy Ondansetron given before chemotherapy Ondansetron and granisetron given after chemotherapy Ondansetron and dexamethasone given before chemotherapy

Ondansetron and dexamethasone given before chemotherapy Rationale Nausea and vomiting can occur shortly after administration of chemotherapeutic drugs or can occur as a delayed response. They can be very profound for the child. Serotonin-receptor antagonists like ondansetron and granisetron are used in the treatment of these symptoms. However, the treatment of choice is administration of ondansetron, granisetron, or palonosetron in combination with dexamethasone given before chemotherapy because this may prevent nausea and vomiting altogether.

An eight-year-old child is brought to the emergency department by his parents with signs of late hydrocephalus. The nurse manages what expected findings in this patient? Select all that apply. Setting-sun sign Ongoing seizure activity Restlessness and irritability Blood pressure 140/90 mm Hg Heart rate of 45 beats per minute

Ongoing seizure activity Seizures occur in a child with late hydrocephalus due to brain stress from excess fluid. The nurse administers benzodiazepines. Blood pressure 140/90 mm Hg An increased blood pressure will be exhibited in patient with late hydrocephalus. If parents desire more than comfort care, nurse may give medications to decrease blood pressure. Heart rate of 45 beats per minute A child with late hydrocephalus will exhibit decreased heart rate. If parents desire more than comfort care, nurse may give medications as ordered to increase heart rate.

The nurse classifies the eight-month-old patient as having a severe injury based on what findings? Exhibits hypoxia, tachycardia, and hypotension Pushes a toy rather than picking it up to move it Unable to sit with support, does not bear weight on legs Opens eyes to pain, motor extension, and moans to pain

Opens eyes to pain, motor extension, and moans to pain This exhibits a higher score on the modified Glasgow coma scale that is used by the nurse to determine severity of head injury.

An adolescent complains of frequent pain in the distal femoral region. The intermittent pain varies in intensity. Upon investigation, the x-ray revealed a Codman triangle and a "sunburst" appearance. Which is the most likely diagnosis? Wilms' tumor Osteosarcoma Ewing sarcoma Hodgkin lymphoma

Osteosarcoma Rationale A Codman triangle is a type of periosteal reaction commonly seen in a patient with osteosarcoma and a "sunburst" appearance of the bone on the x-ray plate is a characteristic of osteosarcoma. Wilms' tumor is a common kidney tumor. Ewing sarcoma is a type of cancer that involves bones. Although a Codman triangle is commonly seen in this type of cancer, a "sunburst" appearance is not seen. Hodgkin lymphoma originates in the lymphoid system and primarily involves the lymph nodes.

A 12-year-old with Hodgkin disease (a type of lymphoma or blood cancer) visits the clinic for a well child visit. The nurse reviews the immunization record and sees that the patient is up to date on all regularly scheduled vaccinations and also received 1 dose of PPSV23 at age 7. The patient later received 1 dose of meningococcal vaccine, 1 dose of Tdap, and 3 doses of HPV vaccine at age 11, and the annual influenza vaccine each year. Which vaccination does the nurse suggest that the patient receive at this visit? HPV vaccine Tdap vaccine Hep B vaccine PPSV23 vaccine

PPSV23 vaccine For many children in high-risk groups, including those with Hodgkin disease, a single booster dose of PPSV23 should be administered five years after the first dose. This patient received the first dose of PPSV23 at age seven years (after the PCV13 series was complete), so dose two can be administered now.

The nurse is caring for a child receiving chemotherapy and notes bruising on the arms and legs. Which test will help identify the cause for bruising? Biopsy Albumin level Platelet levels Head computed tomography (CT)

Platelet levels Chemotherapy causes damage to rapidly growing cells, including platelets, which can lead to bruising and bleeding. A platelet level will help identify the severity of the patient's thrombocytopenia.

Assessment of Hodgkin's disease

Painless enlargement of the lymph nodes Enlarged, firm, nontender, moveable nodes in the supraclavicular area Nonproductive cough Abdominal pain Advanced lymph node and extra lymphatic involvement Positive biopsy specimen of a lymph node Computed tomography scan of the liver, spleen, and bone marrow may be done to detect metastasis

The nurse understands which goals are appropriate goals of care in palliative care for a dying child? Select all that apply. Palliative care seeks to lengthen the life of the person with life-limiting conditions. Palliative care seeks to assist in complex decision making surrounding life-limiting conditions. Palliative care involves being hopeful for a cure and setting goals to encourage patients to live. Palliative care seeks to relieve the physical, emotional, social, and spiritual distress produced by life-limiting conditions. Palliative care seeks to limit interruptions by health providers to once monthly so the family can be with the patient as much as possible.

Palliative care seeks to assist in complex decision making surrounding life-limiting conditions. Palliative care seeks to relieve the physical, emotional, social, and spiritual distress produced by life-limiting conditions. Rationale Appropriate goals of care in palliative care for a dying child include seeking to assist in complex decision making surrounding life-limiting conditions. Palliative care seeks to relieve the physical, emotional, social and spiritual distress produced by life-limiting conditions. Palliative care's focus is to provide realistic goals for the patient, not to give false hope. Palliative care seeks to enhance (not limit) the quality of life of the person with life-limiting conditions. In addition, palliative care seeks to provide the family and patient with frequent, periodic, and timely discussions of the goals of the care.

The nurse is interviewing a mother and her 12-year-old child who report that the child has fallen for the fifth time in a month. Which observations would indicate to the nurse the possibility of child abuse? Select all that apply. Child refuses to speak to the nurse. Child clings to the mother during the interview. Parent answers all questions for the child during the interview. Parent refuses to leave the child's side during the examination. Child has deep purple bruises on the arms and graying bruises on the legs.

Parent answers all questions for the child during the interview. The nurse would suspect child abuse if the parent refuses to allow the child to speak during the interview and examination. Parent refuses to leave the child's side during the examination. The nurse would suspect child abuse if the parent refuses to allow a brief, private examination of the child. Child has deep purple bruises on the arms and graying bruises on the legs. Bruises in multiple stages of healing are signs of physical abuse. Child refuses to speak to the nurse. Withdrawal can be an indicator of child abuse.

children immunization

Parents must be given appropriate information regarding vaccine safety, benefits, and risks so they can make informed decisions regarding vaccinations for their children. Children who are immunocompromised should not be given live virus vaccine components such as varicella. Recommended immunizations for selected groups of children include rotavirus and the human papillomavirus vaccine. The influenza vaccine is recommended yearly for children to prevent serious respiratory infection and pneumonia; the 2014 influenza vaccines include protection against the H1N1 strain that resulted in many deaths in 2009. Combination vaccines combine two or more vaccines into one serum, thus decreasing the overall number of injections. Pediarix, MenHibrix, and Kinrix are examples of combination vaccines.

Which interventions are appropriate to assist children who delay going to bed? Select all that apply. Parents should avoid taking children to the parents' bed. Parents should keep a light on in the room and provide a favorite toy as a transitional object. Children should have surprises before bedtime to help them look forward to bedtime. Parents should leave a drink of water by the bed of the child in case the child desires a drink of water. Increased activity an hour before bedtime helps tire out children and makes them more ready to go to sleep.

Parents should avoid taking children to the parents' bed. Parents should keep a light on in the room and provide a favorite toy as a transitional object. Parents should leave a drink of water by the bed of the child in case the child desires a drink of water. Rationale Interventions recommended to assist children who delay going to bed include parents should avoid taking children to the parents' bed, parents should keep a light on in the room and provide a favorite toy as a transitional object, and they should leave a drink of water by the bed of the child in case the child desires a drink. Surprises and increased activity are stimulating for children and likely will not help transition the child to bedtime

A 9-month-old infant has been brought to the emergency department with a change in level of consciousness. The parent states, "My child has colic, which is why he won't wake up." Which form of abuse would the nurse suspect? Neglect Physical abuse Emotional abuse Munchausen syndrome by proxy

Physical abuse The child's signs and symptoms are indicative of physical abuse, specifically, abusive head trauma.

The nurse is caring for patients in the pediatric headache clinic. Which patients will the nurse assess urgently? Select all that apply. Patient reporting difficulty speaking clearly to the headache clinic receptionist. Patient diagnosed with tension headaches reporting a headache lasting three weeks. Patient diagnosed with migraine headaches reporting abdominal pain, nausea, and vomiting. Patient diagnosed with an arteriovenous malformation (AVM) reporting a brief vision loss this morning. Patient reporting pain in the neck muscles impairing movement of the head from side to side but no nuchal rigidity.

Patient reporting difficulty speaking clearly to the headache clinic receptionist. Difficulty speaking is an uncommon headache symptom and may indicate a stroke or seizure. This patient will need to be assessed urgently due to the severity of symptoms. Patient diagnosed with an arteriovenous malformation (AVM) reporting a brief vision loss this morning. An AVM causes vascular headaches and complications of the AVM include symptoms consistent with bleeding in the brain such as weakness, vision loss, difficulty speaking, and unsteadiness. This patient will need to be assessed urgently by the nurse.

The eleven-year-old patient is admitted with an incomplete spinal cord injury at C4. Which findings cause the nurse the most concern? Select all that apply. Patient reports difficulty taking a deep breath. Patient is unable to lift arms or grasp eating utensils. Patient's blood pressure increases to 150/92 mm Hg. Patient has "goose bumps," small raised bumps on the skin. Patient's whole blood glucose level increases to 190 mmol/L

Patient reports difficulty taking a deep breath. A cervical injury affects the ability to control abdominal muscles and other muscles in the thorax, such as the diaphragm which helps to control depth of purposeful inspiration. This could be a sign that the spinal cord injury is actually complete, has progressed, or that there is additional damage to the spinal cord. The patient may require intubation as progression continues. Patient's blood pressure increases to 150/92 mm Hg. Autonomic dysreflexia is characterized by a sudden rise in blood pressure and will need to be a priority in caring for this patient to prevent intracranial hemorrhage, seizures, and heart attack. Patient has "goose bumps," small raised bumps on the skin. Typically associated with being cold, these raised papillae could indicate that the patient is experiencing hypothermia or autonomic dysreflexia and intervention is needed quickly.

What measures should the nurse take to prevent hemorrhage in a child undergoing chemotherapy for cancer? Select all that apply. Perform regular mouth care. Use rectal suppositories as needed. Avoid skin punctures whenever possible. Take temperature with a rectal thermometer. Use pressure-reducing mattress under bony prominences.

Perform regular mouth care. Avoid skin punctures whenever possible. Use pressure-reducing mattress under bony prominences. Rationale Thorough mouth care should be performed regularly, because gingival bleeding with mucositis is a common problem in a child undergoing chemotherapy. Skin punctures can cause hemorrhage and are sites at risk for infections. To prevent pressure sores and decubital ulcers, a pressure-reducing mattress can be used. Rectal suppositories and rectal thermometers should not be used, because they may cause additional trauma and lead to hemorrhage.

Poliomyelitis

Permanent paralysis may occur

What are the most common signs and symptoms of leukemia related to bone marrow involvement? Petechiae, fever, and fatigue Headache, papilledema, and irritability Muscle wasting, weight loss, and fatigue Decreased intracranial pressure, psychosis, and confusion

Petechiae, fever, and fatigue Correct Signs of infiltration of the bone marrow are petechiae from lowered platelet count, fever related to infection from the depressed number of effective leukocytes, and fatigue from the anemia. Headache, papilledema, irritability, muscle wasting, weight loss, fatigue, decreased intracranial pressure, psychosis, and confusion are not signs of bone marrow involvement

Which clinical manifestations are associated with leukemia? Petechiae, fever, fatigue Headache, papilledema, irritability Muscle wasting, weight loss, fatigue Decreased intracranial pressure, psychosis, confusion

Petechiae, fever, fatigue Rationale Signs of leukemia include petechiae resulting from lowered platelet count, infection resulting from the depressed number of effective leukocytes, and fatigue resulting from the anemia. Headache, papilledema, irritability, muscle wasting, weight loss, fatigue, decreased intracranial pressure, psychosis, and confusion are not signs of bone marrow involvement seen with leukemia.

Which sources of lead in and around the home are appropriate when considering causes of lead poisoning? Select all that apply. Pewter Car seats Fishing sinkers Vinyl mini blinds Playground equipment

Pewter Fishing sinkers Vinyl mini blinds Playground equipment Rationale Sources of lead in and around the home that could contribute to lead poisoning include pewter, fishing sinkers, vinyl mini blinds, and playground equipment. Car seats are not known to have sources of lead.

The nurse notices burn marks on a child's arm. The child is also looking fearful and withdrawn. When the nurse asks him about the injuries, he hesitates and looks fearfully to his mother before answering. For which types of abuse should the nurse assess this patient? Select all that apply. Sexual Neglect Physical Emotional Munchausen syndrome by proxy

Physical Burns are a sign of physical abuse. Correct Emotional A fearful expression is a sign of emotional abuse.

Bleeding associated with Leukemia

Platelet transfusions are generally reserved for active bleeding episodes that do not respond to local treatment and that may occur during induction or relapse therapy Packed red blood cells may be prescribed for a child with severe blood loss

Which maltreatment is appropriate when an unsupervised child is found with pneumonia? Physical abuse Physical neglect Emotional abuse Emotional neglect

Physical neglect Rationale Physical neglect involves the deprivation of necessities, such as food, clothing, shelter, supervision, medical care, and education. Emotional neglect generally refers to failure to meet the child's needs for affection, attention, and emotional nurturance. Emotional abuse refers to the deliberate attempt to destroy or significantly impair a child's self-esteem or competence. Emotional abuse may take the form of rejecting, isolating, terrorizing, ignoring, corrupting, verbally assaulting, or overpressuring the child. The deliberate infliction of physical injury on a child, usually by the child's caregiver, is termed physical abuse. Physical abuse can include anything from bruises and fractures to brain damage.

Match the meningeal layers and structures with the appropriate description

Pia Vascular, transparent membrane Dura Composed of two meningeal layers Arachnoid Avascular, serous membrane Tentorium Separates Cerebrum from cerebellum

The hospitalized child with spina bifida has broken out in a rash. What actions should the nurse take? Select all that apply. Place a precautions sign on the door and in the room. Change out the gloves in the room and outside the door. Request that the health care provider prescribe a steroid. Check the patient's vital signs for a temperature elevation. Ask the hospital's rapid response team to evaluate the child.

Place a precautions sign on the door and in the room. There is likely a latex allergy. A sign indicating the patient is allergic to latex is needed on the door and above the bed to alert other health care workers. Additionally, at a minimum, it should be listed on the patient's armband. The next exposure could induce anaphylaxis. Correct Change out the gloves in the room and outside the door. Children with spina bifida are at high risk for developing latex allergies due to chronic illness resulting in increased exposure to latex-containing products over time. If latex-containing gloves are in the room, they must be removed until it can be determined definitely whether this rash is related to latex. Correct Request that the health care provider prescribe a steroid. A steroid and/or antihistamine will calm the immune reaction to the latex. Correct Check the patient's vital signs for a temperature elevation. This rash could be related to a virus instead of latex. The nurse should begin to rule that out by assessing the patient for a fever.

A child is diagnosed with early stage hydrocephalus. What actions should the nurse perform? Select all that apply. Place padding on all four of bed rails. Teach parents to expect high-pitched crying. Administer ondansetron (Zofran) for vomiting. Provide orientation to the room, call light, and personnel. Consult dietician for dietary supplement recommendations.

Place padding on all four of bed rails. Although seizures are a late sign, any time intracranial pressure (ICP) is increased enough to be symptomatic, seizure precautions should be instituted to protect patient. Administer ondansetron (Zofran) for vomiting. Vomiting occurs due to pressure within the brain on structures that control vomiting, however the action of vomiting increases ICP. The nurse should make every attempt to reduce actions that increase ICP. Correct Provide orientation to the room, call light, and personnel. The child may exhibit confusion, and providing orientation may reduce the severity of the confusion. Correct Consult dietician for dietary supplement recommendations. An infant with early stage hydrocephalus will present with poor feeding, and therefore the nurse may notice the child is not gaining weight.

Which nursing action is appropriate for an infant with a low-grade fever, sneezing, tears from the eyes, who exhibits a short and rapid cough that occurs mainly at night and is followed by a crowing sound? Select all that apply. Place the baby on droplet precautions. Place the child on the bed in supine position. Provide humidified oxygenation and suction. Administer mild sedatives to the child as necessary. Restrict oral fluids, because this may aggravate the cough.

Place the baby on droplet precautions. Provide humidified oxygenation and suction. Rationale The child's symptoms are indicative of pertussis (whooping cough). Therefore, the nurse would give humidified oxygen to the child and suction the baby as needed. The nurse would also use droplet precautions for the child, because pertussis can be transmitted through droplets. The child is at risk of aspiration during coughing fits. Therefore, the child would be placed on the side rather than in the supine position in the bed. The child needs oral fluids for adequate hydration. Mild sedatives are given to children with poliomyelitis, because this helps to relieve anxiety and promote rest.

A 3-month-old infant is brought to the well-child clinic for positional plagiocephaly. What teaching does the nurse provide to the parents to treat this condition? Select all that apply. Place the infant to sleep in an infant seat. Have the infant wear a soft helmet for 15 hours a day. Place the infant in prone position when the infant sleeps. Place the infant in prone position for 15 to 20 minutes while awake. Keep alternating the infant's head position while the infant is sleeping.

Place the infant in prone position for 15 to 20 minutes while awake. Keep alternating the infant's head position while the infant is sleeping. Rationale The infant should be placed in prone position for 15 to 20 minutes when the infant is awake to prevent plagiocephaly. This is also called "tummy time." The infant's head position should be altered when the infant is sleeping to prevent unilateral molding. Placing the infant to sleep in an infant seat or restraint seat increases the risk of developing plagiocephaly. Infants with plagiocephaly should wear a soft helmet for 23 hours a day for a prescribed period of time. The infant should not be placed in prone position when the infant sleeps because it increases the risk of sudden infant death syndrome (SIDS).

Which nursing advice is appropriate when explaining proper restraint of toddlers in motor vehicles to a group of parents? Fitting the seat belt snugly over the toddler's abdomen Placing the car seat in the back seat of the car, facing forward Using lap and shoulder belts when child is over 3 years of age Placing the car seat in the front passenger seat if there is an airbag

Placing the car seat in the back seat of the car, facing forward Rationale Car seats are required for toddlers to prevent injury in case of a motor vehicle accident. The car seat should be placed in the back seat, facing forward. A seat belt can cause injuries if it is placed over a toddler's abdomen. Car seats should be placed in the rear of the car because airbags can injure the toddler. Three-year-old children should be restrained in car seats.

The nurse is reviewing the immunization record of a 10-week-old infant in the clinic for a well visit. The infant received the second dose of Hepatitis B (Hep B) and the first dose of rotavirus (RV1) at eight weeks of age. Which vaccines should the nurse prepare to administer at this visit? Select all that apply. Inactivated poliovirus (IPV) Measles, mumps, rubella (MMR) Pneumococcal conjugate (PCV13) Haemophilus influenzae type b (Hib) Diphtheria, tetanus, & acellular pertussis (DTaP)

Pneumococcal conjugate (PCV13) The nurse should prepare to administer the first dose of PCV13, which should be given at two months of age. Correct Haemophilus influenzae type b (Hib) The nurse should prepare to administer the first dose of Hib, which should be given at two months of age. Correct Diphtheria, tetanus, & acellular pertussis (DTaP) The nurse should prepare to administer the first dose of DTaP, which should be given at two months of age. Inactivated poliovirus (IPV) The nurse should prepare to administer the first dose of IPV, which should be given at two months of age.

A 17-year-old patient has recently been diagnosed with Hodgkin disease (a type of lymphoma or blood cancer). The nurse reviews the immunization record and notes that the patient is up to date on all regularly scheduled vaccines. The nurse prepares to administer which additional vaccine for this high-risk patient? Varicella Hepatitis B Inactivated poliovirus vaccine Pneumococcal conjugate (PPSV23)

Pneumococcal conjugate (PPSV23) Even though the patient has received the four-dose series of pneumococcal conjugate (PCV13), the CDC recommends adose of PPSV23 for patients ages 6 to 18 who are in certain high-risk groups. one dose of PPSV23 should be administered at least eight weeks after the most recent dose of PCV13.

A two-year-old child is in the clinic for a well visit. The nurse notes that the patient's vaccinations are up to date. Which additional vaccination would the nurse suggest to the provider after noting a recent diagnosis of type one diabetes mellitus? Hepatitis B (Hep B) Meningococcal vaccine Human papillomavirus (HPV) Pneumococcal polysaccharide (PPSV23)

Pneumococcal polysaccharide (PPSV23) The nurse would suggest the pneumococcal polysaccharide (PPSV23) vaccine for this patient, due to the recent diagnosis of diabetes mellitus. The CDC recommends that providers administer PPSV23 at least eight weeks after the most recent dose of PCV13 for children with certain high-risk conditions.

The nurse is caring for an eight-year-old child undergoing a lumbar puncture. What actions should the nurse take? Select all that apply. Allow the parents to the hold the child. Position the child to control movement. Encourage the child to take deep breaths. Tell the child that it "won't last much longer." Explain to the child what to expect during the procedure.

Position the child to control movement. It is essential that the child remain in a controlled position to allow correct spinal needle insertion and minimize side effects and adverse events. Encourage the child to take deep breaths. Taking deep breaths is a well-known, easy to implement pain and anxiety-reduction strategy that also prevents the child from holding his or her breath while experiencing pain. Explain to the child what to expect during the procedure. Children cooperate better when they know what to expect and understand what is happening to them.

The nurse sees an 11-year-old female patient in the clinic who reports vaginal discharge. Which additional finding is the most likely indicator of sexual abuse? 3+ bacteria in urine Nausea and vomiting Positive test result for gonorrhea Frequent emergency department visits

Positive test result for gonorrhea A positive test result for a sexually transmitted infection is the most likely indicator of sexual abuse. This patient should be further evaluated for sexual abuse.

A four-year-old child comes to the Emergency Department with his mother, who states that the child is just not feeling himself. The child has been irritable and often seems out of breath. A urinalysis shows evidence of uric acid crystals. Which action should the nurse do first? Prepare child for a chest x-ray Obtain consent for lumbar puncture Discuss with the family about a pelvic CT scan Prepare patient for a bone marrow aspiration and biopsy

Prepare child for a chest x-ray After identifying the presence of hyperuricemia, a chest x-ray is obtained to look for mediastinal disease and tracheal deviation. Thus, the nurse's priority action is to prepare the patient for a chest x-ray, as the child displays signs of potential respiratory compromise.

A two-year-old child presents with fever, night sweats, splenomegaly, and blood studies indicating presence of Reed-Sternberg cells. Which action is a priority for the nurse to take? Administer IV fluids. Obtain a blood sample. Prepare child for gallium scan. Prepare child for bone marrow aspiration.

Prepare child for bone marrow aspiration. Splenomegaly, fevers, night sweats and the presence of Reed-Sternberg cells are signs of Hodgkin lymphoma. The nurse should prepare the child for bone marrow aspiration and biopsy for definitive diagnosis.

The nurse is caring for a child with Non-Hodgkin's Lymphoma who presents in a tripod position, is diaphoretic, and has shortness of breath. The nurse notes a left-sided tracheal deviation, diminished lung sounds on the right side, and SpO2 of 84 percent. Which action is most important for the nurse to take? Administer oxygen Prepare for intubation Administer albuterol nebulizer Obtain a STAT chest radiograph

Prepare for intubation The patient's respiratory compromise is life-threatening. Preparing for intubation is a priority intervention.

A 5-year-old child presents to the emergency department and begins to exhibit neurological side effects after ingesting an unknown poison at home. Which action should the nurse take after assessing that the airway is stable? Gastric lavage Administer naloxone Initiate IV fluid resuscitation Prepare for seizure precautions

Prepare for seizure precautions Patients with neurological or metabolic side effects are prone to seizures and precautions are necessary.

A child admitted for suspicion of retinoblastoma has an eye exam that reveals strabismus and an intact macula. Upon assessment, the child exhibits normal cognitive ability for age and does not appear to have an altered level of consciousness. Which provider order would the nurse question? Prepare patient for MRI Prepare patient for CT scan Prepare for ultrasound imaging Prepare patient for lumbar puncture

Prepare patient for lumbar puncture The nurse would question the lumbar puncture because it is only required if metastases to the brain are suspected.

A 24-hour urine collection done on a two-year old patient shows elevation of both homovanillic (HVA) and vanillylmandelic (HMA) acid. Which order from the provider would the nurse anticipate next? Prepare patient for a CT scan Prepare the patient for biopsy Obtain a history of bowel habits Provide patient teaching on neuroblastoma

Prepare the patient for biopsy Elevation of HVA and HMA is indicative of neuroblastoma. The nurse would anticipate preparing the patient for biopsy, which is necessary for a definitive diagnosis.

A nurse assesses the growth of a 1-month-old infant and prepares to measure the head circumference and assess the fontanels. The nurse knows that which aspect(s) of this infant's head anatomy should be an expected finding? Select all that apply. Closed sutures Closed foramina Presence of a posterior fontanel Presence of an anterior fontanel Anterior fossa containing the pituitary gland

Presence of a posterior fontanel The posterior fontanel should be present in a 1-month-old infant because it normally closes at 2-months of age. Correct Presence of an anterior fontanel The anterior fontanel should be present in a 1-month-old infant. This fontanel does not close until 18 months of age.

What is a common characteristic of those who sexually abuse children? Pressure victim into secrecy. Are usually unemployed and unmarried. Are unknown to victims and victims' families. Have many victims that are each abused only once.

Pressure victim into secrecy. Correct Sex offenders may pressure the victim into secrecy, referring to the activity as a "secret between us" that other people may take away if they find out. The offender may be anyone, from a family member to a stranger at any level of society. Sex offenders are usually trusted acquaintances of the victims and victims' families. Many victims are abused many times over a long period.

What are the clinical situations in which a nurse is aware of an increased risk for sudden infant death syndrome (SIDS)? Select all that apply. Firstborn child Preterm infant Postterm infant Bronchopulmonary dysplasia Neonates with low Apgar score

Preterm infant Bronchopulmonary dysplasia Neonates with low Apgar score Rationale Preterm infants, infants with bronchopulmonary dysplasia, and neonates with low Apgar scores have immature or underdeveloped lungs. Therefore they are at increased risk of developing SIDS. Subsequent siblings have a higher risk than firstborns. Postterm infants are not at a higher risk of developing SIDS.

Which period describes the interval between early manifestations of a disease and the overt clinical syndrome? Prodromal Incubation Desquamation Period of communicability

Prodromal Rationale The prodromal period is defined as the symptoms that occur between early manifestations of the disease and overt clinical symptoms. The incubation period is the time from exposure to the appearance of the first symptom. The desquamation period refers to the shedding of skin when applicable for a syndrome or disorder. The period of communicability describes the period when the child is infectious.

An important role of the nurse in ambulatory settings and schools is the identification of communicable diseases for treatment and the prevention of spread. What is an important component related to the first period of the contagiousness of disease? Source Causative agent Prodromal stage Constitutional symptoms

Prodromal stage Correct The prodromal period is the interval between the early manifestations of the disease and the time when the overt clinical syndrome is evident. Most communicable diseases are contagious during this time. Although the source and causative agent of the disease are important, recognizing the early signs can help the nurse prevent spread and refer the child for medical therapy when indicated. Constitutional symptoms occur during the active disease phase. The child has been contagious, and the time for early intervention may have passed.

The parents of a young child explain that they do not want the child vaccinated because they are afraid the child will suffer permanent injury as a result of adverse immunization effects. In which way(s) should the nurse address the concerns of these parents? Select all that apply. Encourage the parents to talk to friends who have similar concerns. Provide a printed vaccine information sheet describing the safety of vaccination. Verbally educate the parents about the common, mild adverse effects of vaccination. Explain the scientific theory of immunization to help the parent understand how they work. Teach the parents to research Pubmed since so many articles cover immunization implications.

Provide a printed vaccine information sheet describing the safety of vaccination. Providing written materials describing the safety of vaccinations will help relieve anxiety the parents may have. Correct Verbally educate the parents about the common, mild adverse effects of vaccination. Verbally educating the parents about the common, mild adverse effects to vaccinations can help relieve anxiety the parents may have about the adverse effects their child may experience.

The parents of an adolescent with a newly diagnosed chronic illness state, "She was a little disrespectful and defiant before, but now things have gotten completely out of control. We feel like we really need some help coping with this." What can the nurse do to enhance coping in these parents? Instruct the parents on ways to maintain age-appropriate expectations and limitations on negative behaviors Provide education regarding support groups for families of children with special health care needs as a way to get emotional needs met Teach the parents ways to enhance the coping of the adolescent so that she is able to find more productive ways to work through her frustrations Have the parents ignore the behavior because the change in behavior is a normal adolescent reaction to the frustration of dealing with a chronic illness

Provide education regarding support groups for families of children with special health care needs as a way to get emotional needs met Parents also need an outlet for frustration, and support groups that include other parents going through similar situations may be the best solution.

In which ways can healthcare reform improve immunization rates? Select all that apply. Offer more vaccine choices Provide free or low-cost vaccinations Increase the number of clinics providing vaccinations Increase the availability of vaccines without need for health care provider referral Allow vaccinations to be given in single doses rather than requiring two or more doses

Provide free or low-cost vaccinations Providing free or low-cost vaccinations is a way healthcare reform can overcome barriers to immunization. Increase the number of clinics providing vaccinations Increasing clinics providing vaccinations is a way healthcare reform can overcome barriers to immunization. Increase the availability of vaccines without need for health care provider referral Increased availability of vaccines without the need for health care provider referral is a way healthcare reform can overcome barriers to immunization.

Parents of a 3-year-old patient are expressing fear over treatments used for their child's illness. What action by the nurse should be most effective for reducing fear in the parents? Provide information on support groups related to the illness Initiate a referral to a psychotherapist for grief counseling services Assess the parent's spiritual beliefs and practices related to the illness Provide information related to the illness and current treatment modalities

Provide information related to the illness and current treatment modalities Education about the child's condition and treatment often reduces fear.

The nurse is preparing a 4-year-old for bone scintigraphy. Which action is appropriate for the nurse to take? Position the patient prone Inspect skin for best puncture site Place a pillow under the child's hips Provide mild sedative to the patient if ordered

Provide mild sedative to the patient if ordered A sedative may be provided for small children because bone scintigraphy requires them to lie still for 45-60 minutes during the procedure.

Parents of a child with asthma are feeling overwhelmed and worried about how to access care to meet their child's health care needs. What should the nurse do to help the parents in this situation? Select all that apply. Provide the parent with the clinic hours of operation. Provide the parent with information about after-hour clinic provider coverage. Gently remind the parents that excessive worry can have a negative effect on the child. Utilize nursing resources to provide information on asthma and when to seek medical attention. Provide the parent with the address and telephone number of the closest emergency department.

Provide the parent with the clinic hours of operation. Providing the parents with clinic hours of operation will decrease problems accessing the health care system and should decrease frustration. Provide the parent with information about after-hour clinic provider coverage. Providing the parents with after-hour clinic and provider coverage decreases problems accessing the health care system and should decrease frustration. Utilize nursing resources to provide information on asthma and when to seek medical attention. Utilizing nursing resources to provide information on asthma for the parents may decrease their frustration as they will know when medical care is necessary. Provide the parent with the address and telephone number of the closest emergency department. Providing information regarding emergency access to health care services assists in eliminating a barrier to health care services.

Which is the priority of nursing care for children in the terminal stages of an illness? Enhancing family relationships Providing adequate pain control Keeping the skin clean and preventing breakdown Providing medications aimed at curing the child's illness

Providing adequate pain control Rationale Adequate pain control is the highest priority of nursing care for children in the terminal stage of illness. Although enhancing family relationships and keeping the skin clean and preventing skin breakdown are important, pain control is the highest priority. Providing medications aimed at curing the illness is not a priority for the child who is in the terminal stage of an illness.

Which nursing intervention is appropriate for a child with itching related to chickenpox? Giving aspirin or acetaminophen Administering an equine antitoxin Applying a thick coat of pramoxine lotion over open lesions Providing an antipruritic medication such as diphenhydramine

Providing an antipruritic medication such as diphenhydramine Rationale Antipruritic medicines such as diphenhydramine are useful for severe itching, which interferes with sleep and may contribute to secondary infection. Aspirin should never be given to children with chickenpox, because there is an increased risk of Reye syndrome. Caladryl lotion, which contains diphenhydramine, should be applied sparingly, not in thick coats, over open lesions to minimize absorption. An equine antitoxin (usually intravenously) is an intervention for diptheria, not chickenpox.

Which nursing intervention would the nurse follow when injecting a hepatitis B vaccination into a 2-month-old infant? Using a 25-millimeter needle Injecting the vaccine in the deltoid muscle Avoiding use of topical anesthesia before immunization Providing oral sucrose solution (24%) 2 minutes before immunization

Providing oral sucrose solution (24%) 2 minutes before immunization Rationale The nurse should provide sucrose solution (24%) orally for the neonate. This will reduce the immunization pain. A 16-millimeter needle should be used by the nurse for immunization for the 2-month-old infant. A 25-millimeter needle is used for older children. Using topical anesthesia before immunization at the injection site will reduce the pain.

How is Ewing sarcoma different from osteosarcoma? Psychological trauma is less in Ewing sarcoma. Ewing sarcoma involves the metaphyseal region of bones. A "sunburst" appearance is seen on x-ray images of Ewing sarcoma. Surgery and chemotherapy are the treatment of choice for Ewing sarcoma.

Psychological trauma is less in Ewing sarcoma. Rationale Psychological trauma is less in Ewing sarcoma than osteosarcoma. Many families accept the diagnosis with a sense of relief, because it does not necessitate amputation. Ewing sarcoma mostly involves the diaphysis of bones whereas osteosarcoma involves the metaphyseal region. A "sunburst" appearance is a characteristic feature of osteosarcoma and is not seen in Ewing sarcoma. Optimum treatment of osteosarcoma includes surgery and chemotherapy whereas most cases of Ewing sarcoma require involved field radiotherapy and chemotherapy.

The nurse is caring for a patient diagnosed with periorbital Rhabdosarcoma. On assessment, which clinical signs would the nurse expect to observe in this patient? Select all that apply. Ptosis Icterus Conjunctivitis Decreased vision Decreased eye movement

Ptosis Ptosis is a clinical sign of periorbital rhabdosarcoma. Decreased vision Periorbital rhabdosarcoma may involve visual changes including decreased vision. Decreased eye movement Periorbital rhabdosarcoma may involve the cranial nerves, resulting in characteristic symptoms, including decreased ability to move the eyes from side to side.

While assisting with a lumbar puncture, the nurse places the highest priority on monitoring which physiologic parameters? Select all that apply. Pulse rate Temperature Pulse oximetry Blood pressure Respiratory rate

Pulse rate The pulse rate of the child needs to be monitored by the nurse during a lumbar puncture. A rapidly increasing or decreasing heart rate indicates a problem that needs to be addressed immediately. Pulse oximetry Part of continually assessing the patient's cardiorespiratory status is ensuring that the respiratory rate is providing adequate oxygenation while the child's torso is in a position that restricts expansion of the thorax. Respiratory rate The positioning required during a lumbar puncture constricts normal lung movement, air intake, and the ability of the nurse to visualize that the patient is breathing adequately so monitoring respiratory rate is a priority.

A 14-year-old patient with cancer asks the nurse about dying. The nurse knows that the parents don't want that information shared with the child. What should the nurse do? Pursue an ethics consult. Discuss the situation with the parents. Discuss the situation with the nurse manager. Be honest with the child and answer the question.

Pursue an ethics consult. When faced with conflict regarding upholding the desires of the parent and being honest with the patient, the nurse should consider consulting the ethics committee for assistance in developing an agreeable solution.

Which nursing action is appropriate for the toddler that has developed lead poisoning, has a peanut allergy, and has an order to be given British anti-Lewisite? Give flumazenil to the patient. Question the order of British anti-Lewisite. Administer British anti-Lewisite in a high dose. Avoid administration of British anti-Lewisite with EDTA

Question the order of British anti-Lewisite. Rationale British anti-Lewisite should not be administered in the patient who has a peanut allergy, because this may lead to complications. It should not be administered in high doses, because this can prove fatal. Flumazenil is not helpful in this condition. Flumazenil is an antidote for benzodiazepines. British anti-Lewisite administration is contraindicated in this patient. It is always administered in conjunction with EDTA.

What illnesses does respiratory hygiene and cough etiquette by the Centers for Disease Control and Prevention (CDC) prevent? HBV, Hib, and pertussis HSV, influenza, and HBV RSV, influenza, and adenovirus RSV, pertussis, and varicella

RSV, influenza, and adenovirus Correct The CDC (2007) recommends respiratory hygiene and etiquette to prevent the transmission of RSV, influenza, adenovirus, and other droplet-transmitted unknown viruses. HBV, HSV, and varicella are not transmitted via droplets.

How is rhabdomyosarcoma different from retinoblastoma? In rhabdomyosarcoma, metastases to distant organs are rare. Rhabdomyosarcoma may occur in both hereditary and nonhereditary forms. In orbital rhabdomyosarcoma, there are late manifestations of signs and symptoms. Radical procedures with high morbidity are avoided in management of rhabdomyosarcoma.

Radical procedures with high morbidity are avoided in management of rhabdomyosarcoma. Rationale In rhabdomyosarcoma, the tumor is chemosensitive, so radical procedures with high morbidity should be avoided. In the majority of cases, once biopsy is done, the tumor is treated with chemotherapy, irradiation or both. The therapeutic management of retinoblastoma is complex and in severe cases, enucleation is the option of treatment along with chemotherapy and irradiation. Rhabdomyosarcoma attacks the striated muscles of the body and has high potential for metastases, whereas metastasis in retinoblastoma is rare. Retinoblastoma is one of the few types of cancers that occurs in both hereditary and nonhereditary forms. In orbital rhabdomyosarcoma, the development of signs and symptoms are early in the course of the illness whereas in retinoblastoma, the signs and symptoms manifest late.

Rubeola

Rash appears 3 to 4 days after onset and maculopapular eruption on face with gradual spread downward; Koplik spots are present before the rash.

Roseola

Rash composed of rose-pink macules or maculopapules, appearing first on trunk, then spreading to neck, face, and extremities; rash is nonpruritic

A 5-year-old patient with a terminal illness is expressing sadness and confusion about dying. What can the nurse do to help the child cope and understand the dying process? Select all that apply. Give the child a hug and settle the child in for the night. Read the book "Fall of Freddy the Leaf" with the child. Do a Google search about pediatric terminal illnesses and coping mechanisms. Review the dying process using appropriate terms with the child when the child is awake. Access the Children's Hospice International website for information on hospice for the child.

Read the book "Fall of Freddy the Leaf" with the child. Reading a book about death and dying to the child may provide information on dying that decreases the patient's confusion. Review the dying process using appropriate terms with the child when the child is awake. Using appropriate terms when reviewing the dying process with the child may decrease the sadness and confusion experienced by the patient. Access the Children's Hospice International website for information on hospice for the child. The Children's Hospice International website is an excellent resource for the nurse to use to obtain information regarding pediatric hospice, palliative care, and end-of-life care. Information from this site may assist in decreasing the confusion experienced by the patient in relation to death and dying.

A child is brought to the emergency department following a motor vehicle accident and a Glasgow coma score (GCS) is obtained with the following results: Opens eyes spontaneously and obeys motor commands but seems confused. Which should be an appropriate step in nursing management of this patient? Administer intravenous (IV) fluids. Ask the parents to keep the child awake. Manage cerebrospinal fluid (CSF) pressure. Reassess neurologic status within thirty minutes.

Reassess neurologic status within thirty minutes. The nurse should continue to monitor the patient since the child is showing signs of a mild head injury.

Two healthy siblings of a child with chronic illness are talking with a nurse about the expected death of the child. One sibling begins to cry and the other sibling punches the crying sibling in the arm. The parent expresses concern over the differences in coping. How should the nurse respond? Select all that apply. Reassure the parents that differences in grieving are normal. Explain what resources are available for children experiencing grief. Explain that most families progress through the stages of grief together. Explain that grief presents differently according to the developmental stage of the child. Reassure the parents that their children will eventually progress through the stages of grief together.

Reassure the parents that differences in grieving are normal. A sibling's response to death and dying varies, and the child can experience one stage of grief many times or out of sequence. Explain what resources are available for children experiencing grief. Children who are grieving may benefit from attending support groups created just for children. Therefore this is an appropriate response for the nurse to make. Explain that grief presents differently according to the developmental stage of the child. A sibling's response to death and dying varies according to the sibling's age and developmental level.

A parent confides in the nurse and states, "I believe it's my fault that my child is sick." The parent is crying and also admits to not sleeping well at night. Which actions by the nurse are most appropriate? Select all that apply. Discuss the concept of "survivor guilt" with the parent. Educate the parent on over-the-counter sleep medicine. Recommend a referral to a psychologist specializing in grief therapy. Talk with the child's health care provider regarding the parent's statements. Provide the parent with the date and time of the next grief support meeting in the area.

Recommend a referral to a psychologist specializing in grief therapy. A referral to a psychologist is appropriate based on the statements of the parent, the crying, and lack of sleep expressed by the parent. Talk with the child's health care provider regarding the parent's statements. Talking with the health care provider will alert the provider to check on the parent's coping with the child's illness and provides the opportunity for addressing the parent's concerns through a variety of treatment modalities. Provide the parent with the date and time of the next grief support meeting in the area. Grief support groups provide a way for the parent to express grief. Expressing emotions is beneficial for most individuals.

A mother treated her 6-year-old child's fever with aspirin. The child was brought to the emergency department and diagnosed with a varicella infection. The nurse assesses that the patient has impaired consciousness. What are the next steps in nursing management? Select all that apply. Record intake and output every 1-2 hours. Administer a broad-spectrum antibiotic as prescribed. Continue to treat the fever with aspirin every 4-6 hours. Monitor ammonia levels, liver enzymes, and coagulation studies. Monitor the child's oxygen saturation and heart rate continuously.

Record intake and output every 1-2 hours. Maintenance of fluid and electrolyte balance is essential in this child with increased intracranial pressure (ICP) cause by Reye's syndrome. Monitor ammonia levels, liver enzymes, and coagulation studies. Reye syndrome can cause liver dysfunction, affecting ammonia levels, liver function, and coagulation abilities. The nurse monitors laboratory studies for intervention. Correct Monitor the child's oxygen saturation and heart rate continuously. Because of the child's impaired consciousness, the cardiorespiratory status needs to be monitored due to the risk of rapid deterioration.

Which nursing intervention is appropriate when the blood lead level (BLL) is within 5 and 14 mcg/dL in an adolescent client? Select all that apply. Refer to social services. Provide appropriate chelation therapy. Provide diagnostic testing within 1 to 2 days. Conduct followup testing at least once every month. Conduct followup testing within 1 month and then for every 3 to 4 months.

Refer to social services. Conduct followup testing within 1 month and then for every 3 to 4 months. Rationale When the BLL is within 5 and 14 mcg/dL, it is necessary to refer to social services. It is also recommended to conduct follow-up testing within 1 month and for every 3 to 4 months thereafter, apart from providing the family with lead education. It is recommended to provide treatment with appropriate chelation therapy when the BLL is 20 to 44 mcg/dL and above. It is necessary to conduct follow-up testing once every month along with diagnostic testing within 1 to 2 days when the BLL is 45-69 mcg/dL and above.

Which nursing intervention is appropriate when communicating with an adolescent patient that is describing an experience of abuse? Tell the child that it has to be sad to be a part of such a family. Refrain from using leading statements that can distort the report. Refrain from using terms that the child uses to refer to body parts. Tell the child that the child must talk freely in the presence of the parents.

Refrain from using leading statements that can distort the report. Rationale It is important to communicate clearly while talking to a child who is revealing an experience of abuse. The nurse would not use any leading statements that can alter the adolescent's report. Instead, the child should be allowed to speak at ease. It is never advisable to criticize the family of the child when communicating. It helps make the communication effective if the child feels free to talk in a manner that the child is used to, such as by using certain terms to refer to body parts. When the child is revealing experiences of abuse, the child should be allowed to talk in private and in an environment where the child feels comfortable.

A child undergoing chemotherapy for cancer develops rectal ulcers. What interventions are taken by the nurse to promote healing of the ulcers? Select all that apply. Regular administration of polyethylene glycol Use of warm sitz baths after each bowel movement Increase fiber in the diet to ease bowel movement Use of suppositories if the child is unable to defecate Application of an occlusive ointment to the ulcerated area

Regular administration of polyethylene glycol Use of warm sitz baths after each bowel movement Application of an occlusive ointment to the ulcerated area Rationale Polyethylene glycol is a laxative that is used to stimulate evacuation and to soften stools, thus making it easier for a child to defecate without discomfort. Warm sitz baths after each bowel movement help to heal the rectal ulcers. An occlusive ointment should be used regularly to promote healing of the ulcers. Dietary changes with increased fiber intake will increase the volume of stool and increase the discomfort. Rectal suppositories will irritate the ulcerated area and cause more damage.

toddler safety

Regular dental examinations, fluoride supplementation, removal of plaque, and provision of a low-cariogenic diet promote optimum dental health. Toddlers require parental supervision and assistance with toothbrushing and flossing. Because of increased locomotion, toddlers are at high risk for sustaining injuries. Fatal injuries are primarily a result of motor vehicle accidents, drownings, and burns. Many of the deaths are caused by injuries within the car when restraints have not been used or have been used improperly. Unrestrained children riding in the vehicle's front seat are at highest risk for injury. The National Highway Safety Administration recently changed the rule about the use of the LATCH system. If the combined weight of the child and the car seat is more than 65 pounds (29.5 kg), parents should use the shoulder-lap belt restraint to restrain the child in the car seat instead of relying on the LATCH system for maximum protection.

In order to terminate ocular exposure, which interventions should the nurse provide to a patient who has experienced exposure to a powdered poison? Select all that apply. Administer a chelating agent Remove contaminated clothing Irrigate the eyes with warm water or saline Induce vomiting to reduce absorbed poison levels Eliminate powder from skin and clothing; wash skin

Remove contaminated clothing Remove any contaminated clothes; residual powder could endanger the child and health care workers. Irrigate the eyes with warm water or saline Irrigation of the eyes with water or normal saline is crucial for terminating ocular exposure of any poison. Eliminate powder from skin and clothing; wash skin Brush off chemical powders from the skin, and wash the skin. Residual powdered poison is dangerous for both the child and health care workers.

Which statement explains how cerebral spinal fluid (CSF) maintains homeostasis? Drains into choroid plexus Produces white blood cells Flows through brain tissue Removes wastes from the brain

Removes wastes from the brain The CSF functions to maintain homeostasis by removing wastes from the brain because as it flows through the cranial vault and vertebral column it collects wastes and returns them to the venous system.

Which nursing action is appropriate when administering supplements to a toddler with measles that has ophthalmologic evidence of a vitamin A deficiency? Using a higher dose. Single oral dose of 200,000 IU. Single oral dose of 100,000 IU. Repeat dose the next day and after 4 weeks.

Repeat dose the next day and after 4 weeks. Rationale Vitamin A supplementation reduces both morbidity and mortality in measles. The patient has ophthalmologic evidence of vitamin A deficiency. Therefore, the dose of vitamin A should be repeated the next day and after 4 weeks. Administration of a higher dose should be avoided, because it may lead to vomiting and headache for a few hours. A single oral dose of 200,000 IU is recommended for children who are at least 1 year old. A single oral dose of 100,000 IU is recommended for children aged between 6 and 12 months.

The parents of a 4-year-old child have decided to try an herbal remedy to help relieve their child's suffering. After observing this, what should the nurse do next? Select all that apply. Research the herb to see if it is appropriate to use. Provide emotional support and comfort to the family members. Report the herbal supplement use to the primary care provider. Report the treatment to the FDA because it is not an approved herb. Remind the parents that only medications ordered by the health care provider are able to be administered to the patient.

Research the herb to see if it is appropriate to use. The nurse must ensure that there are no harmful effects of the herbal supplement. Provide emotional support and comfort to the family members. Use of alternative therapy may provide hope to the patient and family. Emotional support and comfort aids in providing hope to the family and patient. Report the herbal supplement use to the primary care provider. The health care provider should be aware of all of the treatments utilized by the patient. Interactions between drug and herbal supplements may occur, so the provider should know everything the patient is utilizing.

Consider the Chain of Infection. Match the transmission of influenza to the links of the chain.

Reservoir Patient's lungs and air passages Portal of Exit Respiratory tract (i.e., coughing) Susceptible Host One who has not received their annual vaccine Mode of transmission Hands/contaminated surfaces

A child presents to the emergency department with sudden bilateral ascending weakness and is diagnosed with Guillain-Barré syndrome. What should the nurse most closely monitor? Capillary refill Respiratory status Heart rate and rhythm Level of consciousness

Respiratory status Along with achieving optimal neurologic function, the nurse should prioritize monitoring the respiratory rate in a child with bilateral ascending weakness.

Which is the most common congenital malignant intraocular tumor of childhood? Leukokoria Glioblastoma Retinoblastoma Nephroblastoma

Retinoblastoma Rationale The most common congenital malignant intraocular tumor of childhood is a retinoblastoma. Glioblastoma is a type of brain cancer; nephroblastoma is a type of cancer that occurs in the kidneys. Leukokoria is a white reflex that suggests retinoblastoma.

Which is the most common soft-tissue sarcoma in children? Osteosarcoma Ewing sarcoma Rhabdomyosarcoma Skeletal tuberculosis

Rhabdomyosarcoma Rationale Rhabdomyosarcoma is the most common soft-tissue sarcoma in children. Osteosarcoma is the most common bone cancer in children. Ewing sarcoma is the second most common malignant bone tumor. Skeletal tuberculosis is an infection of the bones and joints.

Which health care issue can be directly addressed by proper hand hygiene? Fever and chills resulting from infection Fatigue and discomfort associated with the infectious disease Isolation from friends resulting from long hospital confinement Risk for infection because of inadequate knowledge about measures to avoid infectious disease

Risk for infection because of inadequate knowledge about measures to avoid infectious disease Risk for infection is supported by the intervention of proper hand hygiene, aimed at minimizing the risk of infection for both the patient and contacts.

The parents of an infant ask for information about the vaccines their child is scheduled to receive at the upcoming four-month well visit. The nurse provides patient education handouts about which vaccine(s) typically administered at four months of age? Select all that apply. Rotavirus (RV1) Inactivated poliovirus (IPV) Measles, mumps, rubella (MMR) Pneumococcal conjugate (PCV13) Haemophilus influenzae type b (Hib)

Rotavirus (RV1) Rotavirus is administered at the four month visit. If Rotarix (RV1) is used, this would be the second and final dose. If RotaTeq (RV5) is used for either the first or second dose, then a three-dose series is required at ages two, four, and six months. Inactivated poliovirus (IPV) The IPV vaccine is administered at the four month visit. This is the second shot in the series. The next will be due between 6 and 18 months of age. Pneumococcal conjugate (PCV13) Pneumococcal conjugate (PCV13) is administered at the four month visit. This four-dose series is administered at ages two, four, and six months and again between 12 and 15 months of age. Haemophilus influenzae type b (Hib) Haemophilus influenzae type b (Hib) is due at the four month visit. Depending on the specific vaccine used, this may be the final dose in a two-dose primary series or another dose may be required to complete the primary series. A booster dose is administered between 12 to 15 months of age to complete the full Hib series.

According to Freud, middle childhood is associated with an increase in relationships with which children? Older children Same-sex peers Younger children Opposite-sex peers

Same-sex peers Rationale According to Freud, middle childhood is associated with an increase in relationships with same-sex peers. The incidence of opposite-sex peer relationships increases during the adolescent years. Relationships with younger children are not common during middle childhood, nor are relationships with older children.

Which condition demonstrates symptoms of 103°F fever, pulse rate of 130 beats per minute, vomiting, and a red and swollen tongue? Mumps Measles Chickenpox Scarlet fever

Scarlet fever Rationale The child has developed scarlet fever. The abruptly high fever, increased pulse rate, and red swollen tongue are indications of scarlet fever. Children with mumps will develop a slight fever and earache that are aggravated by chewing. Children with measles will develop fever, malaise, cough, and Koplik spots. Children with chickenpox will develop slight fever, malaise, anorexia, and itchy skin.

A four-year-old child diagnosed with leukemia presents with a headache, nausea and vomiting, and slight confusion. Which action should the nurse take? Secure consent for lumbar puncture. Assess the child's level of consciousness. Administer acetaminophen as prescribed. Reassure parents that symptoms are expected

Secure consent for lumbar puncture. A child with leukemia who manifests with headache, nausea and vomiting, and slight confusion, indicates central nervous system involvement. The nurse secures an informed consent from the parents or legal guardian, as the child may undergo a lumbar puncture.

The nurse is teaching the parents of an infant about adverse vaccine reactions. The nurse explains that which of these is considered a medical emergency? Seizure Diarrhea Low-grade fever Localized pain and swelling

Seizure It is rare, but seizures may occur after vaccination. Seizures are a medical emergency because in some instances, they may lead to brain damage or coma. If this serious adverse effect is observed, parents should seek immediate medical attention.

The Hib conjugate vaccine protects an infant against what diseases? (Select all that apply.) Bacterial meningitis Epiglottitis Bacterial pneumonia Septic arthritis Sepsis

Sepsis Correct Hib conjugate vaccines protect against a number of serious infections caused by Haemophilus influenza type b, especially bacterial meningitis, epiglottitis, bacterial pneumonia, septic arthritis, and sepsis.

Which nursing action is appropriate for the child with acetylsalicylic acid poisoning refusing the activated charcoal treatment because of its black, muddy color? Administer amyl nitrate to treat the poisoning. Avoid administering activated charcoal slurry. Mix activated charcoal with cola instead of water. Serve the slurry in an opaque container with a cover.

Serve the slurry in an opaque container with a cover. Rationale The child refuses to take activated charcoal slurry because of its black, muddy color; therefore the nurse should serve the slurry in an opaque container with a cover. This would help the child drink it more easily. Amyl nitrate should not be administered to the child, because it is not an antidote for acetylsalicylic acid poisoning; it is an antidote for cyanide. Activated charcoal forms a complex and prevents the further absorption of acetylsalicylic acid. Therefore it should not be avoided. The child refuses to take activated charcoal slurry due to its black mud color, not because of its taste. Therefore mixing it with cola may not be helpful.

A child suffering from retinoblastoma is admitted to the hospital and scheduled for surgery. The child's parents are very apprehensive about the surgery and the appearance of the child after surgery. Which is the most appropriate action by the nurse? Tell the parents they are worrying unnecessarily and should focus on child care. Ask the parents to speak to the primary health care provider in order to explore some alternative. Show the parents a picture of another postsurgery child who has an artificial eye. Explain to the parents that surgery is more important than physical appearance.

Show the parents a picture of another postsurgery child who has an artificial eye. Rationale Patients suffering from retinoblastoma undergo a surgery called enucleation. It is common for parents to be apprehensive when this surgery is suggested for their child. The nurse can show them pictures of other children who underwent similar procedures and now have a normal appearance due to an artificial eye. It is extremely unprofessional for the nurse to ignore the parents' apprehension or to direct them to the primary health care provider for alternatives to surgery. Implying that physical appearance will be affected is unprofessional.

An infant with retinoblastoma is scheduled for an enucleation. How does the nurse prepare the family of the child? Show the parents pictures of another child with an artificial eye. Explain that following the enucleation procedure, the area will appear sunken. Explain that enucleation is bloody and involves ripping the eyeball from the socket. Inform the parents that the child's facial appearance will be normal 4 to 6 weeks after the prosthesis is fitted.

Show the parents pictures of another child with an artificial eye. Rationale The nurse should show the parents pictures of another child with an artificial eye. This may help them to adjust to the thought of disfigurement. The nurse should explain that the area does not appear sunken after enucleation and a surgically implanted sphere maintains the shape of the eyeball. The nurse should refrain from explaining enucleation as a bloody procedure that involves ripping the eyeball from the socket. It may disturb the parents. The surgery should be explained as a procedure that is similar to scooping a nut out of its shell. The nurse should inform the family that after the prosthesis is fitted, the child's facial appearance will become normal, usually within 3 weeks.

An infant is being evaluated for bacterial meningitis. The nurse holds the patient in which position for the sampling of cerebrospinal fluid (CSF)? Select all that apply. Prone Sitting Standing Squatting Side-lying

Sitting Having the child in a sitting position will provide adequate flexion of the lumbar spine for performing a lumbar puncture. Side-lying The side-lying position provides adequate flexion of the lumbar spine for performing a lumbar puncture.

Which cause is appropriate when describing the preschooler that has daytime tiredness, negative behavior, hyperactivity, difficulty in concentrating, impaired learning ability, and poor control of emotions and impulses? Dyslalia Aggression Sleep disturbance Mental abnormality

Sleep disturbance Rationale The preschooler has symptoms of sleep disturbance, which affects physical and mental health. Due to sleep disturbance, the preschooler may feel tired in the daytime and show behavioral changes. The preschooler may also have difficulty in concentrating, which affects learning ability. Poor emotion control may also be present. Dyslalia is a speech-related problem present in preschoolers with articulation difficulties. Aggression is related to behavior in which the preschooler may attempt to hurt a person or destroy property. Daytime tiredness is not associated with mental illness.

Which sleep finding is consistent with a child waking up at night, screaming, thrashing, sweating, is not aware of the parents' presence when checked on, and lies back down and sleeps without parental intervention? Nightmares Sleep apnea Sleep terrors Seizure activity

Sleep terrors Rationale In sleep terrors the child is only partially aroused; therefore the child does not remember the parents' presence. A nightmare is a frightening dream followed by full awakening; the child would realize that the parents are present. The description of the child's experience does not indicate the presence of seizure activity. Sleep apnea is a cessation of breathing during sleep.

The twelve-year-old patient with spina bifida exhibits learning delays. What other assessment findings does the nurse anticipate? Select all that apply. Slow to follow directions Difficulty swallowing foods Upper limb discoordination Frequent respiratory infections Bowel and bladder incontinence

Slow to follow directions If the child's spine is affected high enough to result in learning delays, additional processing delays, such as following directions, might be expected. Difficulty swallowing foods The child's cognitive delays are related to the height of the neural tube defect, however difficulty swallowing may occur and can also indicate the child also has Chiari II malformation. Upper limb discoordination The child with learning delays secondary to spina bifida will also have difficulty with gross and fine motor skills using the arms and hands. Bowel and bladder incontinence Even fairly low neural tube defects can result in incontinence. A defect as high as this one definitely leads the nurse to anticipate incontinence.

Which medicine does the nurse expect to be prescribed for a toddler with bacterial conjunctivitis? Acyclovir Hydroxyzine Mebendazole Sodium sulfacetamide

Sodium sulfacetamide Rationale Bacterial conjunctivitis is inflammation of the conjunctiva due to bacterial infection and is treated with topical antibacterial agents. Sodium sulfacetamide is a topical antibacterial agent. Acyclovir is an antiviral agent that is used to treat varicella infections. Hydroxyzine is an antipruritic medication used to treat severe itching. Mebendazole is used for the treatment of pinworms.

The parents of a 7-year-old boy tell the nurse that their son wants to join a baseball team. What guidelines about sports at this age would the nurse include when responding to the parents' statement? Organized sports, such as baseball, are not appropriate at this age. Competition is detrimental to the establishment of a positive self-image. Sports participation is encouraged if the sport is appropriate to the child's abilities. Boys should compete only against other boys because at this age they are larger than girls.

Sports participation is encouraged if the sport is appropriate to the child's abilities. Rationale Sports participation should be encouraged if the sport is appropriate to the child's abilities and physical and emotional development. Organized sports can be beneficial for school-age children. School-age children enjoy competition. With appropriate guidelines, children can be taught the proper techniques and safety measures to avoid injuries. Before puberty there is no essential difference in strength and size between boys and girls.

The parents of an 8-year-old girl tell the nurse that their daughter wants to join a soccer team. What is the appropriate recommendation, based in the nurse's knowledge of this age group? Organized sports, such as soccer, are not appropriate at this age. Competition is detrimental to the establishment of a positive self-image. Sports participation is encouraged if the sport is appropriate to the child's abilities. Girls should compete only against girls because at this age boys are larger and have more muscle mass.

Sports participation is encouraged if the sport is appropriate to the child's abilities. Rationale The parents should help the child select a sport that is suitable to her capabilities and interests. Team sports contribute to the school-age child's social, intellectual, and skill growth. Organized sports for school-age children can provide safe, appropriate activities with supportive parents and coaches. The desire to participate in competitive team sports develops from a need for peer interaction for the school-age child. A sport that meets the child's capabilities and interests should be selected. The physical changes in boys described take place during puberty, later in the school-age years; there is therefore no reason for boys and girls to compete separately at the age of 8 years.

The parents of an 8-year-old girl tell the nurse that their daughter wants to join a soccer team. What information does the nurse provide regarding participation in sports at this age? Organized sports such as soccer are not appropriate at this age. Competition is detrimental to the establishment of a positive self-image. Sports participation is encouraged if the type of sport is appropriate to the child's abilities. Girls should compete only against girls because at this age boys are larger and have more muscle mass.

Sports participation is encouraged if the type of sport is appropriate to the child's abilities. Correct Virtually every child is suited for some type of sport. Participation should be encouraged if the chosen sport and level are appropriate for the child's abilities and her physical and emotional make-up. Organized sports can be very beneficial for this age-group. School-age children enjoy competition. With appropriate guidance, children can be taught the proper techniques and safety measures to avoid injuries. Before puberty, there is no essential difference in strength and size between boys and girls.

Which precautions should the nurse implement for a child with an undiagnosed exanthema? Institute airborne and droplet precautions. Institute droplet and standard precautions. Institute standard and airborne precautions. Standard and all transmission-based precautions.

Standard and all transmission-based precautions. Rationale The nurse should be aware that both standard and transmission-based precautions (contact, airborne, and droplet) must be instituted when an exanthema is undiagnosed. Exanthema can be a symptom of measles, chickenpox, mumps, and scarlet fever, and these conditions may spread if all precautions are not taken.

Which information would the nurse include while educating mothers about the usage of walkers to prevent injuries in young children? Mobile walkers increase coordination and prevent falls in children. All walkers should be avoided to prevent accidental injuries in children. Stationary walkers are preferable to mobile walkers to prevent accidents. Children using mobile walkers tend to walk sooner than those using stationary ones.

Stationary walkers are preferable to mobile walkers to prevent accidents. Rationale Stationary walkers are preferred over mobile walkers, because they prevent accidental injuries due to slipping. It is not necessarily true that all walkers need to be avoided to prevent injuries, though parents should never use walkers near stairs. Mobile walkers do not increase coordination and there is no substantial evidence supporting the use of walkers in early establishment of independent walking in children.

A student nurse compares the sources of stress for both 6-year-olds and 10-year-olds. What source of stress is prevalent in only one of these age groups? Stature Teasing Shyness Competition

Stature Rationale Stature is a source of stress only in 10-year-olds. Both boys and girls at this stage may be upset by the fact that the girls are taller, and the extremely small or extremely large child may be concerned about personal size. Stature is not a source of stress for 6-year-olds, when there is not much difference in height between boys and girls. Teasing is a source of stress for both these age groups. A 6-year-old engages in teasing but becomes upset when on the receiving end. A 10-year-old also engages in teasing, scapegoating, or vicious attacks to temporarily boost self-image, even though there is a sense of guilt felt afterward. Shyness is a source of stress for both these age groups. A 6-year-old may initially be shy in a new situation but usually recovers quickly; an already existing shyness problem in a 10-year-old is likely to become more pronounced at this age. A 6-year-old wants to be the "first" or best; a 10-year-old continues to be highly competitive and looks to peer group for prestige.

How can the nurse explain the difference between bone marrow transplantation and stem cell transplantation? Stem cells transplants have fewer side effects than BMT. Stem cells transplants treat hematologic problems and BMT treats oncologic problems. Stem cell transplants replenish immune function; BMT replenishes endocrine function. Stem cells are infused into the blood stream. Bone marrow replaces the contents of long bones.

Stem cells are infused into the blood stream. Bone marrow replaces the contents of long bones. Autologous stem cell transplantation is performed by infusing stem cells into the patient's peripheral blood supply. BMT requires bone marrow to be placed directly into the bone.

What is the most common piece of medical equipment that can transmit harmful microorganisms among patients? Thermometer Stethoscope Injection needle Disposable gloves

Stethoscope Correct A stethoscope is commonly used between patients, and if not correctly disinfected, it can be a dangerous source of spreading microorganisms. Thermometers of all types have barriers to prevent this. Needles are discarded immediately after injections and never reused, so they are not a common source of transmission. Disposable gloves are not reused, so they are not a common source of transmission

The nurse on a pediatric unit finds an infant in the prone position who is not breathing. The nurse places the infant in the supine position. The nurse then gently tries to stimulate the trunk by rubbing the sternum and flicking the infant's feet. After a full minute goes by, and the nurse implements cardiopulmonary resuscitation (CPR). What nursing intervention was done in error? Rubbing the infant's back Flicking the feet of the infant Turning the infant on the back Stimulating the infant for 1 minute

Stimulating the infant for 1 minute Rationale The nurse should try to gently stimulate the back of an infant when the nurse finds that the infant is in an apneic state. This should be done by gently rubbing or patting the infant. If the baby is in a prone position, the nurse should put the baby on the back and flick the infant's feet. The nurse should try to stimulate the infant for no more than 10 to 15 seconds. If no response is seen, then CPR should be implemented.

A nurse is caring for a child receiving a chemotherapeutic agent intravenously (IV). The child begins to complain of stinging pain at the IV site. Which is the most appropriate nursing action in this situation? Stopping the infusion Cleaning the area around the IV site Waiting 30 minutes to see whether the pain disappears Adjusting the IV rate and making the IV more comfortable for the child

Stopping the infusion Rationale Stinging pain and redness at the cannulation site are signs of infiltration. Chemotherapy agents are vesicants that can cause severe cellular damage if the drug infiltrates, so the infusion must be stopped immediately. Cleaning the IV site or adjusting the IV is not appropriate or effective in this situation. Waiting 30 minutes to see whether the pain disappears increases the possibly of severe cellular damage.

A child is receiving chemotherapeutic drugs through a free-flowing intravenous line. The nurse notices slight redness at the site, and the child complains of a strange sensation around the cannula. What is the best nursing response to this situation? Observing the site for 20 minutes to see whether symptoms worsen Stopping the infusion and starting an intravenous line at another site Beginning a normal saline continuous infusion to clear the intravenous line Stopping the infusion immediately and monitoring the child's vital signs and responses

Stopping the infusion immediately and monitoring the child's vital signs and responses Rationale Any swelling, stinging, redness, or pain at the site may indicate that a reaction or infiltration is occurring. The best nursing response is to stop the infusion immediately and monitor the child's vital signs and responses. Never simply observe the site to see if the symptoms worsen. Stopping the infusion is good, but the child needs to be observed before chemotherapy is started in another line. In case of infiltration, a continuous infusion should not be started until the child and intravenous site have been evaluated.

Which nursing instruction is appropriate when teaching parents of toddlers about how to prevent poisoning? Keep ipecac in the home. Consistently use safety caps. Store poisonous substances out of reach. Store poisonous substances in a locked cabinet.

Store poisonous substances in a locked cabinet. Rationale Safe storage of poisonous substances is an appropriate way to prevent the curious toddler from getting into them. Not all poisonous substances have safety caps, and safety caps are not always foolproof. Ipecac does not prevent poisoning and is not recommended as a treatment for poisoning. Toddlers can climb and are curious; therefore storing substances out of reach only does not eliminate the potential for poisoning.

The nurse is interviewing the parents of a 4-month-old infant who has been brought to the emergency department. The infant has no vital signs on arrival and is not able to be resuscitated. The parents state that they found the baby in the crib with a blanket over the head, lying face down with blood-tinged fluid from the nose and mouth. The parents said they placed the infant in the crib after falling asleep and did not notice any problems at that time. What probable cause of death does the nurse suspect? Suffocation Child abuse Infantile apnea Sudden infant death syndrome (SIDS)

Sudden infant death syndrome (SIDS) Rationale This death is consistent with the appearance of sudden infant death syndrome (SIDS). The infant is usually found in a disheveled bed; with blankets over the head; huddled into a corner and clutching the sheets; with frothy, blood-tinged fluid in the mouth and nose; and lying face down. The diaper is also usually full of stool, indicating a cataclysmic type of death. Although the child was found under the blanket, the other findings are consistent with SIDS. The findings as reported are consistent with SIDS, not child abuse or infantile apnea.

Which is the most appropriate nursing response when the 7-year-old sibling of a child who recently died in the hospital expresses a desire to see the body at the funeral? The child is too young to attend the funeral. Encourage the child to attend the funeral but not see the body. Suggest that the child wait and visit the grave site after the funeral. Suggest having a supportive person near the child throughout the viewing and funeral.

Suggest having a supportive person near the child throughout the viewing and funeral. Rationale For children of this age and older, attendance at funerals is both useful and meaningful. It helps the child acknowledge the death, honor the deceased, and receive comfort and support from a parent or significant person. If an open coffin is used for the funeral, the child should be prepared for how the sibling will look. A 7-year-old child is not too young to attend the funeral of a sibling. Suggesting that the child wait and visit the grave site after the funeral is not helpful because research shows that children who participate in the funeral planning and attend services have higher self-esteem than those who are excluded.

Which is the most appropriate nursing response when the 7-year-old sibling of a child who recently died in the hospital expresses a desire to see the body at the funeral? The child is too young to attend the funeral. Encourage the child to attend the funeral but not see the body. Suggest that the child wait and visit the grave site after the funeral. Suggest having a supportive person near the child throughout the viewing and funeral.

Suggest having a supportive person near the child throughout the viewing and funeral. Rationale For children of this age and older, attendance at funerals is both useful and meaningful. It helps the child acknowledge the death, honor the deceased, and receive comfort and support from a parent or significant person. If an open coffin is used for the funeral, the child should be prepared for how the sibling will look. A 7-year-old child is not too young to attend the funeral of a sibling. Suggesting that the child wait and visit the grave site after the funeral is not helpful because research shows that children who participate in the funeral planning and attend services have higher self-esteem than those who are excluded.

An anxious child with symptoms of airway obstruction has cyanosis of the face, neck and upper chest; facial and upper extremity edema; and distended neck and chest veins. The child had a central venous access device placed for delivering a chemotherapeutic agent. Which complication does the nurse suspect? Acute asthma attack Foreign body ingestion Superior vena cava syndrome Acute laryngotracheobronchitis

Superior vena cava syndrome Rationale The symptoms suggest that the child is having superior vena cava syndrome (SVCS), in which a space-occupying lesion may cause airway compromise and then respiratory failure. Up to 40% of all SVCS cases are due to a thrombus formation or a fibrotic reaction from intravenous implantable devices. Asthma results in airflow limitation or obstruction and causes recurrent episodes of wheezing, breathlessness, chest tightness and coughing, especially at night or in the early morning. In foreign body ingestion, the child who is in distress becomes cyanotic, cannot speak, and collapses. Some organisms cause acute laryngotracheobronchitis (LTB). The initial symptom is rhinitis for several days and then the child develops a cough, hoarseness and stridor. Edema and vein distension are not commonly seen in LTB.

A family requests giving nutrition and hydration to their terminally ill child. Why would a hospice nurse hesitate to provide nutrition and hydration to a terminally ill child? Select all that apply. Supplemental feeding may decrease the tumor growth. Supplemental feeding may cause more nausea or vomiting. Supplemental feeding relieves the hunger or thirst the child is experiencing. Increasing fluids may contribute to congestive heart failure or increased respiratory secretions. Withholding supplemental feeding or hydration is believed to provide a more comfortable and natural death.

Supplemental feeding may cause more nausea or vomiting. Increasing fluids may contribute to congestive heart failure or increased respiratory secretions. Withholding supplemental feeding or hydration is believed to provide a more comfortable and natural death.

The nurse is caring for a 17-year-old who has been ill for many years. The child is told that she is expected to have 12 months left to live. What action by the nurse is most appropriate? Find an anger outlet for the patient. Support the child as they experience feelings of denial. Reassure the patient that the illness is not something they caused. Provide comfort as the child experiences overwhelming feelings of sadness.

Support the child as they experience feelings of denial. Denial is often the first reaction of the grieving process experienced by individuals going through the dying process. The nurse can expect that this will likely be the first response of the child that the nurse will provide care for.

An infant has torticollis and plagiocephaly. What nursing interventions are appropriate for this infant? Select all that apply. Use customized helmet 4 hours daily. Switch head position sides regularly. Refer to craniofacial surgeon by 2 years. Refer to pediatric neurosurgeon by 1 year. Promote exercises to loosen the tight muscles.

Switch head position sides regularly. Promote exercises to loosen the tight muscles. Rationale Torticollis is characterized by abnormal and asymmetric head or neck position. Plagiocephaly is asymmetric distortion of the skull. Management includes switching head position sides during feeding, carrying, and while asleep; this helps to promote development of a symmetric shape. Exercises should be encouraged to loosen tight muscles. After 4 to 8 weeks of physical therapy, a customized helmet should be worn for 23 hours a day and for the recommended duration. Referral to a craniofacial surgeon or a pediatric neurosurgeon has to be done by 4 to 6 months of age.

Which treatments are appropriate for the ingestion of a poisonous substance that requires gastric decontamination? Select all that apply. Gastric gavage Syrup of ipecac N-acetylcysteine Chelation therapy Activated charcoal

Syrup of ipecac Activated charcoal Rationale Types of gastric emptying and gastric decontamination include syrup of ipecac and activated charcoal. Gastric lavage may be performed to empty the stomach, but gastric gavage is not. N-acetylcysteine is an antidote but is not used for gastric emptying or gastric decontamination. Chelation therapy is a chemical compound that combines with metal poisons for rapid and safe excretion.

Which nursing recommendation for the parent is appropriate for the child experiencing violent hemoptysis, severe burning in the mouth, drooling, and extreme agitation after ingesting toilet-cleaning solution? Administer activated charcoal. Administer ipecac to induce vomiting. Observe the child closely and call back in 1 hour. Take the child to the nearest emergency department.

Take the child to the nearest emergency department. Rationale The child needs to be taken to the nearest emergency department for treatment. The parent should not observe the child closely and call back in 1 hour, because emergency treatment is indicated. The parent should not administer activated charcoal or ipecac, because induced vomiting will damage the mucosa.

Parents of a two-year-old child with type 1 diabetes are concerned about their child's diabetes and its impact over time. How should the nurse respond to their concerns? Select all that apply. Teach and help the parents adjust and cope with the child having type 1 diabetes. Formulate a plan of care based on the manifestations and pathophysiology of type 1 diabetes. Inform the parents how treatment for type 1 diabetes will be incorporated into their daily lives. Inform the parents that care needs and approaches will likely change as the child grows and develops. Help prepare the parents to care for the child by letting them know what future school accommodations to expect with type 1 diabetes.

Teach and help the parents adjust and cope with the child having type 1 diabetes. Teaching the parents provides information that will help them adjust and cope with the child's chronic illness. This creates the necessary psychosocial environment that will positively influence the child's growth and development. Inform the parents how treatment for type 1 diabetes will be incorporated into their daily lives. Informing the parents of the treatment regime for type I diabetes and how it fits into their lives allows for normalization of the family unit and maximized function throughout the illness. Inform the parents that care needs and approaches will likely change as the child grows and develops. Because type 1 diabetes is a chronic illness and will be present for the rest of the child's life, care must incorporate changes in the child's growth and development. Help prepare the parents to care for the child by letting them know what future school accommodations to expect with type 1 diabetes. Informing the parents regarding what to expect when the child enters school with type 1 diabetes allows them to develop a plan of care and aids in achieving and maintaining the highest level of health and social functioning for the child.

The parents of a 10-year-old child reveal to the nurse that after watching the news of a child-kidnapping case, their child is experiencing nightmares and bedwetting. The child has also stopped going out to play due to the fear of being kidnapped. What priority intervention does the nurse employ for the child? Teach relaxation techniques to the child. Encourage the child to play outdoor games. Explain the reason for nightmares to the child. Suggest parents wake the child up at night to use the restroom.

Teach relaxation techniques to the child. Rationale The increase in violence and rapid spread of information through media may cause stress in the child. Nightmares and bedwetting are indications of stress. The immediate nursing intervention is to teach relaxation techniques to the child. The child needs physical activity, so the nurse should encourage the child to play outdoor games under the parent's supervision. Explaining the reasons for nightmares and bedwetting should be postponed until the child is free from immediate stress. The nurse should not suggest waking the child at this time. Work on removing the fear and stress of kidnapping.

The health care provider examines a 7-year-old child, revealing increased deep tendon reflexes, hypertonia, flexion, and a scissors gait. Which intervention does the nurse include in this patient's plan of care? Give the parents teaching pamphlets about antiseizure medication. Ensure the child has a hospital bed and bedside commode for use at home. Provide the child with a diet plan that includes reduced-fat milk and cheeses. Teach the child and parents how to monitor for and address learning difficulties.

Teach the child and parents how to monitor for and address learning difficulties. Children with cerebral palsy tend to have learning disabilities and poor attention spans. Educating parents on how to seek help for these problems as they arise is an essential part of this child's care plan.

What would the nurse teach the parent of a child, who is home alone after school, regarding telephone use? Select all that apply. Teach the child the parent's name. Ensure that the child knows how to report emergencies. Teach the child the home address so that the child can inform anyone who calls. Teach the child to say that the parents are not at home and the caller should call back later. Instruct the child to tell the caller that the parents cannot come to the phone right now and will call back later.

Teach the child the parent's name. Ensure that the child knows how to report emergencies. Instruct the child to tell the caller that the parents cannot come to the phone right now and will call back later. Rationale A latchkey child is the term used to describe children in elementary school who are left to care for themselves before or after school without supervision of an adult. A nurse who is caring for such a child would instruct the parent to teach the child the home telephone number, address, and parent's name. This information would come in handy if the child ever has to report an emergency situation through a phone call. The parent should ensure that the child knows how to report emergencies and that there is a list of emergency numbers by the telephone. The parent should also instruct the child not to tell a caller that parents are not at home; instead, the child should be taught to say that the parents cannot come to the phone right now and will call back later. While the child needs to be taught the home address, this is to inform an emergency contact person if necessary, but not to inform casual callers. The parent needs to instruct the child never to tell casual callers that the parents are not home.

During middle childhood, some children tend to tell stories to their parents by exaggerating events at school. What suggestion does the nurse provide parents to manage this behavior? Inform the child's teacher immediately. Consult a child psychiatrist immediately. Check with the peers and playmates of the child. Teach the difference between reality and fantasy.

Teach the difference between reality and fantasy. Rationale During middle childhood, children tend to tell stories and often exaggerate the events that take place in school. They tell these stories to impress their friends and family. However, most children are able to distinguish between fantasy and reality, but some may not acknowledge the difference. The nurse should suggest that parents teach them the difference between reality and fantasy. Telling stories to the parents and peers in an exaggerated way is a normal behavior in school-age children. Therefore it does not indicate that the child has mental illness, and the child need not be referred to a psychiatrist. The parents should not encourage the child by listening to the child's stories. The child may get confused or feel rejected if parents complain to the teacher. Checking with the peers and playmates of the child may not stop the child from telling imaginary stories.

Passive immunity

Temporary immunity obtained by tranfusing immunoglobulin or antitoxins from a person or animal that has been actively immunized against the antigen

Match the cerebral palsy (CP) symptoms to the nursing intervention for the hospitalized pediatric patient.

Tense muscles Continue to monitor Visible tremors Provide guided imagery Loss of coordination Place bed in lowest position Involuntary movement Administer a benzodiazepine

An instructor is teaching a group of nursing students about ethical considerations for managing symptoms in terminally ill patients. Which information is appropriate to include in the teaching? Terminally ill patients need palliative care. Terminally ill patients can be offered euthanasia. Terminally ill patients should be withdrawn from treatment. Terminally ill patients can be given information regarding assisted suicide.

Terminally ill patients need palliative care. Rationale Palliative care is often provided to support patients and their families through a terminal illness, particularly when no curative treatment seems to work. Palliative care helps improve the quality of life by symptom management and may include medication that does not cure the disease but provides comfort and reduces pain. Euthanasia, also called mercy killing, involves ending the life a terminally ill patient, as through medication administration. At this time, it is not legal and as such is inappropriate to be offered to the patient. Assisted suicide refers to providing the patient with the means to end the patient's own life, though this practice is illegal in most of the United States. It is not ethical to withdraw treatment without the consent of the patient and/or family.

A 4-year-old child is expected to die within the week. The child sleeps all day and has not eaten in 2 days. What should the nurse communicate to the child? Select all that apply. That the child will not be left alone to die. That the hospital chaplain will visit and pray with the child. That the child is loved and supported by the nurses and family members. That the child will need to communicate more clearly with the nurses and family. That the illness and impending death are not the child's fault or a result of poor judgement.

That the child will not be left alone to die. For reassurance it should be consistently and frequently communicated to the child and family members that the child won't be left alone to die. That the child is loved and supported by the nurses and family members. Children who are dying need to experience complete love and acceptance. That the illness and impending death are not the child's fault or a result of poor judgement. The dying child should be consistently reassured that the illness and approaching death are not the result of any action or omission committed by the child.

The nurse cares for patients in a pediatric headache clinic. Which patient should the nurse assess first? The 4-year-old child with abdominal pain and nausea. The 10-year-old child vomiting in the examination room. The 12-year-old child who reports smelling smoke and fire. The 6-year-old who screams when someone touches the child's head.

The 10-year-old child vomiting in the examination room. This child with vomiting is the most serious of these patients, indicating either a migraine in need of medicating or increased intracranial pressure (ICP).

The nurse is performing annual assessments of height and weight in school children of ages 6 to 12 years. The nurse compares the current assessment findings of height and weight of the children with the previous year's assessment findings. What changes does the nurse anticipate in the healthy children? The 6-year-old children are 75 to 85 cm tall. The children lost 4.5 to 6.5 pounds on average. The weight of girls is more when compared to boys. The 12-year-old children are about 150 to 155 cm tall.

The 12-year-old children are about 150 to 155 cm tall. Rationale The height and weight of children between ages 6 and 12 increases due to the change in bone growth and density. The height of 12-year-old healthy children is 150 to 155 cm. The weight of healthy children increases 4.5 to 6.5 pounds or 2 to 3 kg per year. The height of 6-year-old healthy children is approximately 116 cm. The height and weight of boys are more compared to girls.

The 12-year-old sibling of an infant born with Down syndrome has been increasingly difficult and lashing out on family and friends. The parents are spending time with the 12-year-old child and do not understand why the child's behavior has changed so much. What information should the nurse provide? The family is likely spending too much time with the infant and not enough time with the 12-year-old. Regression is expected from a sibling when any new infant is brought into the family because the child feels jealous. The 12-year-old is likely experiencing anger as part of a grief response to the infant being born with a chronic illness. A 12-year-old does not understand that the infant has a chronic condition, so these behaviors are likely from a lack of discipline.

The 12-year-old is likely experiencing anger as part of a grief response to the infant being born with a chronic illness. Any family member can experience grief when a child has a chronic illness. A 12-year-old may not know how to express this grief and could have difficulties navigating the grieving process. Anger is part of the natural grieving process.

A patient is dying with a rare disorder and an autopsy will be required. At what stage of the dying process will the parents be notified that an autopsy will occur? The parents should be made aware immediately after the child has died. The parents should be made aware of the autopsy request prior to the death of the child. The parents should be made aware of the need for an autopsy after death cares are completed. The parents should be made aware of the need for an autopsy shortly after receiving the initial diagnosis.

The parents should be made aware of the autopsy request prior to the death of the child. Families should be provided information before the child's death because it affects their time and involvement with their child after death occurs.

Cause of Leukemia

The cause is unknown. It seems to involve the genetic damage of cells, leading to the transformation of cells from a normal state to a malignant state

Which finding is suspicious for child abuse? Select all that apply. Bruises are located only on the right arm and leg. There is a history of a broken arm from falling off a swing last year. Caregiver is anxious for the child to get immediate medical attention. The child has been brought to the emergency department by an unrelated adult. Red, green, and yellow bruises are present on more than one plane of the body.

The child has been brought to the emergency department by an unrelated adult. Red, green, and yellow bruises are present on more than one plane of the body. Rationale A child brought to a health care provider for a trauma or suspicious injury by an unrelated adult or who arrives when the primary care provider is totally unavailable is a sign of abuse. The finding of varying degrees of healing of bruises in more than one plane of the body is a warning of abuse. Falling down stairs can be an unintentional injury. An isolated documented injury is not a warning sign of abuse. Multiple fractures of differing ages are a warning sign of abuse. An anxious caregiver is a normal response in a case involving an injured child; a delay in seeking care would serve as a warning sign of abuse.

The nurse performs a developmental assessment of a nine-month-old infant. Which finding causes the nurse to be concerned that myelinization of the neurons is impaired? The child is not yet making babbling sounds such as "mama" or "baba." The child is unable to transfer an object from one hand to the other hand. The child does not look for a toy that the nurse has shown and then hidden. The child is not yet able pull to stand while holding onto a hand or furniture.

The child is unable to transfer an object from one hand to the other hand. The myelinization of the neurons allows for effective transmission of neural signals required for proper coordination of movements. The ability to coordinate movement of the hands to move an object is a 9-month developmental mileston

A nurse working with school-age children is aware of physical maturation. What information is accurate and knowledge is important for a nurse working with school-age children? Bladder capacity is greater in boys than in girls. The immune system becomes less competent in its ability to localize infections. Bones cease to ossify and yield to pressure, and muscle tissue pulls more readily. The heart grows more slowly during the middle years and is smaller in relation to the rest of the body.

The heart grows more slowly during the middle years and is smaller in relation to the rest of the body. Rationale The heart grows more slowly during the middle years and is smaller in relation to the rest of the body. Bladder capacity is greater in girls than in boys. The immune system is not less but more competent in its ability to localize infections. Bones continue to ossify throughout childhood but yield to pressure, and muscle pulls more readily than with mature bones.

A twelve-year-old child's spina bifida lesion affects the upper lumbar vertebrae. The nurse evaluates that the child is meeting therapeutic goals when the child demonstrates which behaviors? Select all that apply. The child participates in exercise activities daily. The child walks without using leg braces prescribed. The child has successful attempts at bladder emptying. The child chooses to play alone at school during recess. The child bathes, dresses, and puts on shoes without help.

The child participates in exercise activities daily. Increasing the child's mobility is a goal in the care of a child with spina bifida, and therefore the child's continued ability to participate in daily exercise indicates the goals of care are being met. The child has successful attempts at bladder emptying. A child with spina bifida must be placed in a bladder-emptying program, and therefore successful attempts at emptying the bladder is an indicator that goals are being achieved. The child bathes, dresses, and puts on shoes without help. This is an appropriate act of independence and demonstrates the child is maintaining mobility and is actively working toward therapeutic goals.

The family of a child with a terminal illness has been informed of the option of palliative care for the child. Which hallmark principles does the nurse draw on to further educate the family? Select all that apply. The child patient is the unit of care. The child patient and family are the unit of care. An interdisciplinary team of health care professionals are part of the child's care. Focus is on relief of suffering, but curative treatments will be used if appropriate. Focus is on curative treatment and disease-modifying treatments from the interdisciplinary team.

The child patient and family are the unit of care. An interdisciplinary team of health care professionals are part of the child's care. Focus is on relief of suffering, but curative treatments will be used if appropriate. Rationale Hallmark principles of palliative care include the focus of care being the child patient and the family, not just the child. The focus of palliative care is on relief of suffering, but if curative or disease-modifying treatments are appropriate, they are instituted. An interdisciplinary team of health care professionals are part of the child's care. Palliative care is different from primary care in that curative and disease-modifying treatments are not the focus unless it is appropriate.

What is an important consideration when the nurse is discussing enuresis with the parents of a young child? Enuresis is more common in girls than boys. Enuresis is neither inherited nor has a familial tendency. Psychogenic factors that cause enuresis persist into adulthood. The child should be encouraged to take charge of treatment interventions.

The child should be encouraged to take charge of treatment interventions. Correct Because any treatment involves and requires the child's active participation, the child is in charge of the interventions, and the parents should learn to support the child rather than intervene. Enuresis is more common in boys than in girls, and it has a strong family tendency. Psychogenic factors may influence enuresis, but it is doubtful that they are causative.

A 12-month-old child presents to the clinic for a well visit after missing several appointments. The child began her immunization schedule but has missed several follow-up appointments and doses of immunizations. What is the most appropriate nursing intervention? Administer initial immunizations from the beginning of schedule. The child cannot receive missed immunizations if the schedule is not followed and will not be vaccination. The child should only receive the missed doses of immunizations based on catch-up schedule. The child should receive double-strength immunizations at this well visit.

The child should only receive the missed doses of immunizations based on catch-up schedule. Correct Children who began primary immunization at the recommended age but fail to receive all the doses do not need to begin the series again but instead should receive only the missed doses. The child may receive missed vaccinations on a catch-up schedule per CDC guidelines.

The nurse wishes to educate parents on what to expect as the child is approaching death. Which response regarding the child's breathing pattern is most appropriate? The child breathes deeply and rapidly with a fruity odor on the breath. The child will have Cheyne-Stokes respirations, leading to respiratory arrest. The child breathes shallowly but with a normal rate until respiratory arrest occurs. The child may breathe more shallowly and faster than normal, which leads to respiratory arrest.

The child will have Cheyne-Stokes respirations, leading to respiratory arrest. Cheyne-Stokes respirations is a cyclic period of slowing respirations with apnea, followed by an increased respiratory rate to a peak, and then slowing into apnea again. The respirations are associated with impending respiratory arrest

While performing assessments of students at a grade school, the nurse learns that some are latchkey children. What characteristics occur in latchkey children? The children have many friends. The children are brave and free from illness. The children spend most of their time at home. The children are frequently supervised by adults.

The children spend most of their time at home. Rationale Children in elementary school who are left to care for themselves before or after school without supervision of an adult are referred to as latchkey children. Such children may lack proper care. They spend most of their time at home and do not spend time outside or doing physical activities. These children are fearful and have chronic illnesses due to lack of care and physical activities. These children have fewer friends, because they are not able to interact with peers. Such children have to spend time alone before and after school and lack supervision by adults and parents.

A 5-year-old child's sibling dies of sudden infant death syndrome. The parents are concerned because the child showed more outward grief when their cat died than at the sibling's death. The nurse draws on knowledge of development to explain what? The child is not old enough to have a concept of death. The child is not old enough to have formed a significant attachment to the sibling. This behavior suggests maladaptive coping, and a referral is needed for counseling. The death may be so painful that the child must deny it for now to protect the psyche.

The death may be so painful that the child must deny it for now to protect the psyche. Rationale A child at this age has limited defense mechanisms. Often the child will react with more overt grief to a less significant loss than to the loss of a significant person. The child's behavior is suggestive of limited defense mechanisms, not maladaptive coping. The child is beginning to understand the permanence of death. At age 5 years the child will have formed relationships, including one with a sibling.

Herd immunity

The majority of the population is vaccinated and the spread of disease is stopped without the rest of the population getting vaccinated.

A young nurse who does not deal well with death is looking for support. What is the best way a more experienced nurse can respond? Select all that apply. The experienced nurse offers to mentor the young nurse. Tell the young nurse to exercise every day to "blow it off". Explain the need to suppress emotions as this is part of the job. Inform the young nurse about availability and time to talk. The experienced nurse refers the young nurse to grief programs offered at the hospital.

The experienced nurse offers to mentor the young nurse. Mentoring the young nurse provides the support the nurse needs while learning how to deal with death and dying. Inform the young nurse about availability and time to talk. Meeting with a nurse and taking the time to listen shows support for what that nurse is experiencing. Correct The experienced nurse refers the young nurse to grief programs offered at the hospital. Providing a resource for dealing with grief demonstrates support and concern for the young nurse. Grief programs are important for health care professionals to use to repair, maintain, or enhance their emotional well-being.

A mother brings in her 4-year-old child to the health care provider, stating that the child has marked constipation and describing the child as "sluggish." Which patient's social history is most significant? The family has recently moved into a historic house. The child has recently attended an outdoor day camp. The family has just returned from a vacation to the ocean. The child has recently started attending preschool at a newly built facility.

The family has recently moved into a historic house. A historic home may have lead paint and leaded glass which can lead to toxicity causing constipation. Exposure to lead can cause the symptoms described in this scenario.

A mother discusses options about hospice care for an only child who has three months to live. The child has osteosarcoma (bone cancer) and has had frequent admissions to the hospital for pain control. The mother states they live on a farm 30 miles from the hospital. Which piece of information is most important in influencing the decision for hospital-based hospice care? The child has bone cancer. The child has 3 months to live. The family lives 30 miles from the hospital. The patient has been readmitted frequently for pain control.

The family lives 30 miles from the hospital. The family lives quite a distance away from the health care facility. This could make home hospice care difficult as resources are limited, and it is a great distance to obtain medical assistance.

Which scenario reflects the type of interaction allowed in the hospital moments after a child passes away? The child's body is brought to the morgue for an autopsy. The nurse asks the family to leave the room until the body has been cleaned. The family spends time with the child but then leaves for the nurse to clean the body and prepare the child for the funeral home. The family members remain in the room with the child immediately following the death of the child and the nurse provides privacy.

The family members remain in the room with the child immediately following the death of the child and the nurse provides privacy. Family members should have the opportunity to spend time with their child, even before the body is cleaned.

The nurse is working with a family that has two parents and three children. The oldest child was just diagnosed with diabetes mellitus. The parents want to know how they should schedule their health care provider's appointments. What information should the nurse provide? The child who is dealing with a chronic illness requires extra attention from the parents and siblings, so everyone should attend the appointments. The family should try to schedule health care appointments at times that do not interfere with any of the children's school or extracurricular activities. The child's medical needs are top priority. Cancelling other activities or engagements for the children is an acceptable adjustment for the oldest child's health. The family needs to maintain a balance, so some scheduled appointments may be missed to allow the parents and child not to miss any extracurricular or school activities.

The family should try to schedule health care appointments at times that do not interfere with any of the children's school or extracurricular activities. Normalcy is important to the family, and each child needs to feel like their activities are important. Whenever possible, appointments should be scheduled around activities.

A young nurse developed a friendship with a teenaged patient who died. What can this nurse expect during the grieving process? The grieving process for the nurse may take a lot longer to get over. The grieving process should be shorter since the nurse works with terminal patients. The nurse may become more involved with a new patient that has a similar diagnosis. The nurse may experience certain stages of grief repetitively when working through the grieving process.

The grieving process for the nurse may take a lot longer to get over. When a nurse becomes more involved with a patient, a more intense response or a delay in the resolution of grief may occur.

Nephroblastoma (Wilms Tumor)

The most common intraabdomincal and kidney tumor of childhood. It may manifest unilaterally and localized of bilaterally, sometimes with metastasis to other organs

Acute Lymphocytic Leukemia

The most frequent type of cancer in children

lead poisoning

The most important factor contributing to lead poisoning is its availability in the child's environment. Lead-based paint is the most toxic source of lead. A child does not need to eat loose paint chips to be exposed to the toxin; normal hand-to-mouth behavior, coupled with the presence of lead dust in the environment that has settled over the decades, is the usual method of poisoning. The neurologic system is of most concern when young children are exposed to lead. Mild and moderate lead poisoning can cause a number of cognitive and behavioral problems in young children, including aggression, hyperactivity, impulsivity, delinquency, disinterest, and withdrawal. Long-term neurocognitive signs of lead poisoning include developmental delays, lowered intelligence quotient (IQ), reading skill deficits, visual-spatial problems, visual-motor problems, learning disabilities, and lower academic success.

The nurse is treating a child and has determined that the child has been a victim of sexual abuse. Which person is most likely to be the abuser? A daycare worker Physical therapist The mother's boyfriend Neighborhood stranger

The mother's boyfriend A child abuser is most often a parent's significant other, and therefore the mother's boyfriend is most likely to be the abuser.

The nurse evaluates a three-year-old child for developmental delays. When the nurse notes that the child has difficulty maintaining balance while walking, what other assessments does the nurse perform? Select all that apply. The nurse assesses overall muscle tone and strength. The nurse assesses for speech impairments and delays. The nurse assesses deep tendon and primitive reflexes. The nurse assesses level of consciousness and orientation. The nurse assesses for developmental milestone variances.

The nurse assesses overall muscle tone and strength. Cerebral palsy (CP) is characterized by abnormal muscle tone and therefore the nurse will assess the overall tone and strength at this time. Correct The nurse assesses for speech impairments and delays. Impaired speech is a characteristic developmental impairment found in CP and therefore this would be assessed at this time. Correct The nurse assesses deep tendon and primitive reflexes. A child with CP may have persistent primitive reflexes, which should have disappeared in infancy. In assessing a suspected neurologic disorder, the nurse should assess deep tendon reflexes as well. The nurse assesses for developmental milestone variances. The child with cerebral palsy may not have reached other milestones at appropriate ages, and therefore should be assessed for these at this time.

A child accidentally aspirated lighter fluid after playing with a lighter. Which roles does the nurse have in managing this patient? Select all that apply. The nurse will administer IV fluids. The nurse will utilize measures to prevent emesis. The nurse will administer oxygen and support ventilation. The nurse will administer chelators and anti-coagulant medications. The nurse will monitor vital signs and observe for signs of CNS depression.

The nurse will administer IV fluids. Administration of IV fluids supports circulatory function and prevents dehydration. Correct The nurse will utilize measures to prevent emesis. Prevention of emesis will decrease the likelihood of additional aspiration of the low-density hydrocarbons. Correct The nurse will administer oxygen and support ventilation. Administration of oxygen and support of ventilation are essential due to potential damage to the lungs. The nurse will monitor vital signs and observe for signs of CNS depression. Vital signs and changes in CNS function are critical. This should be assessed regularly.

Which scenario accurately reflects a parent experiencing anticipatory grief? The parents cry when holding their sleeping child. The parents research treatment options available to cure the illness. The parents pray for a cure at the bedside of the child hourly while reading the Bible. The parents suggest a desire for treatment to continue for their child even though no benefits are provided.

The parents cry when holding their sleeping child. Crying when hold a sleeping child is an example of anticipatory grief, a response to the impending death of a loved one.

Which intervention is most appropriate regarding a child's attendance at a sibling's funeral service? The parents force the sibling to attend the funeral. The parents explain how the deceased child will look. The parents do not allow the sibling to stay for a long time. The parents bring the sibling to the funeral service after the relatives arrive.

The parents explain how the deceased child will look. Rationale Before attending the funeral, the parents should explain to the sibling what to expect at the funeral and how the deceased child will look in the coffin. A significant family member or friend can explain this if the parents cannot communicate due to grief. The sibling should not be forced to attend the funeral; the decision should be left to the sibling. The sibling should be brought to the funeral service before many visitors arrive (to allow for some private time with the deceased) and be allowed to stay as long as the sibling wishes.

The parents of a 13-year-old child often work until late at night and are not able to be home when the child returns from school. What action employed by the parent is appropriate? The parents set the water heater to 69.9°C for faster heating. The parents instruct the child to heat food in the microwave at low heat. The parents advise the child to play on the trampoline if the child is bored. The parents advise the child to take aspirin if the child has fever.

The parents instruct the child to heat food in the microwave at low heat. Rationale The parents should follow safety guidelines and instruct the child to follow them to prevent accidents. Children in middle childhood exhibit explorative behavior and often tend to do experiments. The parents should give repeated instructions and teach safety guidelines. The parents should teach the child to use the microwave at low heat to prevent burns. The parents should set the water heater to 48.9°C to avoid scald burns. Trampoline injuries can be very serious, and the child should be allowed to play on it only under adult supervision. Medicines, including aspirin, should be taken only with adult supervision.

The nurse is working with a parent of a child that needs to be confined due to a communicable disease. Which factors should the nurse consider, now that the child will remain in the home? Select all that apply. The period of confinement for this illness is generally three weeks. Once the child is feeling better, the child is no longer confined and can return to school. While the child is confined, having visitors is important to minimize the risk for social isolation. The parents will not be able to leave the home and visit with friends, with the child being confined. Even though the child is ill, providing toys for play will be important as aspect in their road to recovery.

The parents will not be able to leave the home and visit with friends, with the child being confined. The parents are not able to leave the home, with the child being confined. This statement indicates an understanding of the process. The parent can maintain contact by phone, email, or text while the child is isolated. Correct Even though the child is ill, providing toys for play will be important as aspect in their road to recovery. Providing toys for play demonstrates an understanding of the confinement process. The child should have age-appropriate toys and engage in age appropriate activities for normal growth and development.

The nurse is caring for patients on a pediatric neurology unit. Which patient should the nurse assess first? The patient diagnosed with a brain mass reporting a sudden loss of vision. The patient with a head injury after falling off a bicycle, reporting nausea and abdominal cramping. The patient admitted with meningitis, with a recently recorded temperature of 101.2 Fahrenheit. The patient diagnosed with a seizure disorder with the parent reporting the child has significant loss of appetite.

The patient diagnosed with a brain mass reporting a sudden loss of vision. A sudden loss of vision indicates rapidly increasing ICP, probable bleeding, and needs immediate evaluation.

A nurse is caring for a teenage girl and suspects that she may be experiencing abuse at home. Which part of the patient's history would suggest the she may be a victim of abuse? Select all that apply. The patient sneaks out at night to visit friends. The patient states that she feels isolated from her friends. The patient belongs to a family of seven and acts out in school. The patient reports that she is expected to help care for younger siblings on weekends. The patient lives in a single-parent home and is required to live by very strict rules.

The patient states that she feels isolated from her friends. Feelings of being isolated from friends can be a sign that the child is living in an abusive household. The patient belongs to a family of seven and acts out in school. Children from large families are more susceptible to neglect. They may struggle for needed attention and may not get the love, nurturing, and support required for adequate emotional growth and development. The patient lives in a single-parent home and is required to live by very strict rules. A child who lives in a home with unreasonable rules and strict family roles may be more susceptible to abuse.

Which statement is correct with regard to the safety of toddlers in car restraints? The front seat is the best place for toddlers. Restraints are used until the age of 7 years. The seat belt should be worn on the abdominal area. The shoulder belt is used if it does not cross child's neck or face.

The shoulder belt is used if it does not cross child's neck or face. Rationale Shoulder belts are used for toddlers only if they do not cross the child's neck or face, which may lead to choking. The backseat is the safest area in the car for children. Car restraints are used for children until they are 12 years of age. Seat belts should not be worn on the abdomen; they should be worn low on the hips and fit snugly.

The nurse tells a preschooler that the preschooler's ill sister will never come back because the preschooler's sister is dead. What does the preschooler assume in this situation? The sibling can breathe and eat. The sibling will not come back to life. The sibling's body decays in the ground. The sibling cannot see or hear anything.

The sibling can breathe and eat. Rationale Preschoolers generally see death as a form of sleep and they may think the dead person can still breathe and eat. They do not have any understanding about the universality of death. Preschoolers tend to not have a good understanding of the irreversibility of death and think that dead people may come back to life. A school-age child, not a preschooler, may be able to understand that a dead person's body decays in the ground. A preschooler often does not understand that a person who has died no longer has the abilities of someone who is alive, such as seeing and hearing.

A sibling is noticeably upset at the death of a sister. Which is an example of an activity that allows the sibling to assist in the immediate care of the body? The sibling gives the sibling a hug before the body leaves the room. The sibling brings the nurse the clothes the deceased child will wear to the funeral home. The sibling places the child's teddy bear with the body before it goes to the funeral home. The sibling washes the hands and face of the sibling alongside the nurse during death care.

The sibling washes the hands and face of the sibling alongside the nurse during death care. Care of the body immediately after death includes washing the hands and face of the deceased child.

toddler behavior

The solitary play of infancy progresses to parallel play—the toddler plays alongside, not with, other children. Parental concerns during the toddler years include toilet training, coping with sibling rivalry, limit setting and discipline, dealing with temper tantrums, negativism, and regression. Temper tantrums are nearly universal during toddlerhood as independence is established and more complex tasks are attempted that may overwhelm the child emotionally. Toddlers may assert their independence by violently objecting to discipline. Negativism is not an expression of being stubborn or insolent but a necessary assertion of self-control.

How is stereotactic surgery different from open craniotomy? Select all that apply. The stereotactic surgery is minimally invasive. The stereotactic surgery is computerized tomography guided. The stereotactic surgery is magnetic resonance imaging guided. The stereotactic surgery often involves a longer stay in the hospital. The stereotactic surgery may cause considerable neurologic damage. The stereotactic surgery reconstructs the tumor in all three dimensions.

The stereotactic surgery is minimally invasive. The stereotactic surgery is computerized tomography guided. The stereotactic surgery is magnetic resonance imaging guided. The stereotactic surgery reconstructs the tumor in all three dimensions. Rationale Stereotactic surgery is a minimally invasive procedure compared to open craniotomy. The procedure is computed tomography (CT) and magnetic resonance imaging (MRI) guided in conjunction with other special computer techniques. In open craniotomy, the CT and MRI are done prior to the procedure to know the exact location of the tumor. In stereotactic surgery, there is reconstruction of the tumor in three-dimensional imaging and the computer-assisted instruments are used in performing the surgery. Stereotactic procedures involve a shorter hospital stay because the recovery phase is shorter. Residual neurologic damage is minimal in the stereotactic procedure.

The nurse is completing physical assessments on school-age children. What is a physical change the nurse would anticipate noting on physical exam for these children? The cranium grows faster than the face in school-age children. There is no correlation between the physical indicators of maturity and success in school. The increase in head circumference with respect to standing height indicates increased maturity. The strength of school-age children is less than what the increase in their muscles would warrant.

The strength of school-age children is less than what the increase in their muscles would warrant. Rationale The school-age child has an increase in the percentage of body weight represented by muscle tissue. By the end of this age period, both boys and girls have doubled their strength and physical capabilities. This increased strength is often misleading, because the muscles are still functionally immature when compared with those of the adolescent, and they are more readily injured by overuse. Facial proportions change in the school-age child, because the face grows faster in relation to the remainder of the cranium; in fact, the skull and brain grow very slowly during this period. The physical indicators of maturity seem to correlate with success in school. The decrease in head circumference with respect to standing height in a school-age child is one of the most pronounced changes that seems to best indicate increasing maturity.

A teenaged patient with leukemia is frustrated by the amount of time spent in the hospital and expresses anger to the nurse. Which nursing intervention is most appropriate? The teen should be encouraged to calm down and decrease anger. The teen should be allowed to express anger and frustration through talking. The teen should be left alone to process feelings until frustration and anger is eliminated. The teen should be allowed to leave the hospital because hospitalization is the source of the frustration.

The teen should be allowed to express anger and frustration through talking. Encouraging discussion of difficult topics like those that trigger anger and frustration provide an appropriate outlet for feelings and decrease those feelings with support and subsequent intervention by the nurse.

sudden infant death syndrome (SIDS),

The third leading cause of infant death (birth to 12 months) in the United States is sudden infant death syndrome (SIDS), defined as the sudden death of an infant younger than 1 year of age that remains unexplained after a complete postmortem examination, including an investigation of the death scene and a review of the case history. SIDS is the leading cause of postneonatal deaths (1 month to 12 months). Factors that place the infant at high risk for SIDS include prone sleeping position, soft bedding, sleeping in a noninfant bed with an adult or older child, and maternal prenatal smoking. Protective factors for SIDS prevention include breastfeeding for the first 6 months of life, pacifier use at bedtime and naptime, and a supine sleep position. Maternal co-sleeping with the infant is not a protective factor for SIDS. Since the Back to Sleep campaign started in 1992 promoting supine sleeping, the incidence of SIDS in the United States has declined by as much as 53%.

Which nursing action is appropriate when considering a 12-year-old child who has not received the hepatitis B (HBV) vaccine? One dose is needed at age 14 years. The three-dose series would be started at this time. Only one dose will be needed sometime during adolescence. The three-dose series would be started at age 16 years or sooner if the adolescent becomes sexually active.

The three-dose series would be started at this time. Rationale Adolescents should be vaccinated against hepatitis B at this age if they have not been previously. Three doses of HBV are required to achieve immunity. It is recommended that the HBV vaccine series be started at birth. The American Academy of Pediatrics recommends that vaccinations be completed by the age of 13 years.

The nurse is speaking to a parent and 2-year-old child who is displaying sweating, fatigue, and rash. Which factor needs to be included in the nurse's instructions to the parent? The use of 81 mg aspirin to control these symptoms. The use of 160 mg of acetaminophen to control these symptoms. The use of heavy blankets and warm pajamas to control these symptoms. The use of 4 ounces of cool liquid twice a day to control these symptoms.

The use of 160 mg of acetaminophen to control these symptoms. The use of acetaminophen or ibuprofen as directed for fever control is a factor to include in the discussion with the parent. 160 mg is the appropriate dose and can be given every 4 to 6 hours as needed.

What is the appropriate site to administer an intramuscular (IM) vaccine to a newborn? The dorsal gluteal muscle The vastus lateralis muscle The ventral gluteal muscle The biceps muscle

The vastus lateralis muscle Correct If the vaccine is given intramuscularly, then it is given in the vastus lateralis in newborns or in the deltoid for older infants and children. Regardless of age, the dorsogluteal site should be avoided because it has been associated with low antibody seroconversion rates, indicating a reduced immune response, and it is no longer an acceptable evidence-based practice site for IM injections. The ventral gluteal muscle and the biceps muscle are not appropriate sites for IM injections

What are some of the ways in which play helps school-age children develop? They learn to not experience guilt when they misbehave. They learn that misfortunes are punishment for misdeeds. They acquire mastery over themselves, their environment, and others. They begin to understand the rules and behaviors of social interaction and the reasons behind them.

They acquire mastery over themselves, their environment, and others. Rationale Play helps school-age children develop by teaching them to acquire mastery over themselves, their environment, and others. Play does not teach children to not experience guilt when they misbehave. Play does not teach children that misfortunes are punishment for misdeeds. Although school-age children can understand the rules and behaviors of social interaction, play does not help them understand the reasons behind them.

What characteristic is often observed in latchkey children? They are brave. They have confidence. They bully other children. They feel isolated and lonely.

They feel isolated and lonely. Rationale Children who are left to care for themselves before or after school without adult supervision are called latchkey children. These children feel isolated and do not interact with their peers. They are not brave but fearful. These children do not prefer to socialize with peers. It is very unlikely that they bully others; instead they are bullied by other children. As they feel lonely, they are less likely to be confident.

What describes moral development in younger school-age children? The standards of behavior now come from within themselves. They do not yet experience a sense of guilt when they misbehave. They know the rules and behaviors expected of them but do not understand the reasons behind them. They no longer interpret accidents and misfortunes as punishment for misdeeds.

They know the rules and behaviors expected of them but do not understand the reasons behind them. Correct Children who are ages 6 and 7 years know the rules and behaviors expected of them but do not understand the reasons for these rules and behaviors. Young children do not believe that standards of behavior come from within themselves but that rules are established and set down by others. Younger school-age children learn standards for acceptable behavior, act according to these standards, and feel guilty when they violate them. Misfortunes and accidents are viewed as punishment for bad acts

Which statement is appropriate when young children report sexual abuse by a parent? In most cases, the child has fabricated the story. They may exhibit various behavioral manifestations. Their stories are not believed unless other evidence is apparent. They should be able to retell the story the same way to another person.

They may exhibit various behavioral manifestations. Rationale There is no diagnostic profile of the child who is being sexually abused. Many different behavioral manifestations may be exhibited, from outward sexual behaviors with others to withdrawal and introversion. It is never appropriate to assume that a child has fabricated the story of sexual abuse. Adults are reluctant to believe children, and sexual abuse often goes unreported. Physical examination is normal in approximately 80 percent of abused children. The child will usually try to protect the parents and may accept responsibility for the act.

Varicella

This condition has a rash that begins as macules, rapidly progressing to papules and then to vesicles, eventually breaking and forming crusts.

What is the purpose of palliative chemotherapy or palliative radiotherapy? Select all that apply. To cure the disease process of the patient To enhance the quality of life of the patient To change the outcome of cancer for the patient To increase comfort by slowing the progression of an incurable tumor To increase comfort by reducing swelling or pressure from a tumor that is causing pain

To enhance the quality of life of the patient To increase comfort by slowing the progression of an incurable tumor To increase comfort by reducing swelling or pressure from a tumor that is causing pain Rationale The purpose of palliative chemotherapy or palliative radiotherapy is to enhance the quality of life of the patient by increasing comfort by slowing the progression of an incurable tumor, or by reducing swelling or pressure from a tumor that is causing pain. The need for palliative care is not to cure the disease or change the outcome of the disease, but to give comfort when it is incurable.

Which rationale is appropriate for ensuring an intact and protected airway when using activated charcoal to treat a patient that ingested a poison? To administer antidotes To facilitate gastric lavage To facilitate the administration of ipecac To enable the effectiveness of activated charcoal after 1 hour

To facilitate gastric lavage Rationale Gastric lavage is performed to empty the stomach of a toxic agent and is likely to lead to aspiration. Therefore it is necessary to ensure that the patient has an intact and protected airway so that complications related to gastrointestinal perforation are better dealt with. Antidotes are administered in cases of toxic ingestion; however, the administration is not related to an intact or protected airway. Syrup of ipecac is an emetic that works by stimulating the vomiting center. Because it delays or inhibits the effectiveness of activated charcoal, it does not improve the patient outcome. It is absolutely necessary for activated charcoal to be administered within 1 hour after the poison has been ingested in order to ensure its effectiveness.

The nurse is providing end-of-life care for a patient. The child is receiving medication for pain management. What is the rationale behind the administration of laxatives in this patient? To prevent escalation of pain To make it easier for the child to breathe To reduce secretions and lessen death rattle To prevent untoward side effects related to pain treatment

To prevent untoward side effects related to pain treatment Rationale Laxatives, stool softeners, or diphenhydramine are administered to the child as required to prevent untoward side effects related to pain management. Pain medications are provided around the clock to prevent the recurrence or escalation of pain. Pillows or other supportive devices are used to support the child in a comfortable position and make it easier for the child to breathe. Anticholinergic drugs are administered to children during end-of-life care to reduce gastrointestinal and salivary secretions and lessen death rattle.

Attenuate

To reduce the virulence of a pathogenic microorganism by treating it or cultivating it on a certain medium

The nurse reviews the laboratory report of a child with leukemia who is receiving chemotherapy. The nurse notices that the platelet count is severely reduced. Based on the laboratory report, which intervention does the nurse include in the plan of care? To use a soft brush for dental care To encourage frequent fluid intake To monitor the child's temperature every 4 hours To administer a broad-spectrum antibiotic as prescribed

To use a soft brush for dental care Rationale Severe thrombocytopenia puts the child at an increased risk of bleeding. To reduce chances of gum bleeding, the nurse should use a soft brush for dental care. Frequent fluid intake should be encouraged to reduce the risk of cystitis but will not help with the platelet count. Temperature monitoring will help to detect fever but will not aid in restoring platelets. Broad-spectrum antibiotics may be administered if symptoms of infection are present, but this intervention will not affect the platelet count.

What are reasons for children to require very high doses of opioids to control pain in palliative care? Select all that apply. Addiction to opioids Tolerance of opioids Progression of the disease Psychological dependence Other physiologic causes of pain

Tolerance of opioids Progression of the disease Other physiologic causes of pain Rationale Reasons for children to require very high doses of opioids to control pain in palliative care include tolerance, progression of the disease, or other physiologic causes of pain. Addiction or psychological dependence is not a factor in managing terminal pain in a child. The child will not become addicted.

Diphtheria

Tonsillar pharyngeal areas are covered with white or gray membrane; complications include myocarditis and neuritis.

The nurse conducts an assessment on a child who presents with swelling and discomfort in the axilla, persistent cough, and dyspnea. Which assessment is a priority? Temperature Lung sounds Tracheal position Breathing pattern

Tracheal position The nurse would check the position of the trachea. Cough and respiratory distress indicate signs of mediastinal disease, which may also cause tracheal deviation.

Which principle method is appropriate when treating a child that has ingested a poison? Treat the child first, not the poison. Remove the poison, then assess the child. Establish what contributed to the poisoning. Administer syrup of ipecac to treat a poisoning that occurs in the home.

Treat the child first, not the poison. Rationale The first and most important principle in dealing with a poisoning is to treat the child first, not the poison. Removing the poison first and then assessing the child is not the method of treating a poisoning. Establishing what contributed to the poisoning is not the most important principle in treating a poisoning. Ipecac is not recommended for routine poison treatment in the home.

What is most descriptive of the therapeutic management of osteogenic sarcoma? Intensive irradiation is the primary treatment. Amputation of the affected extremity is rarely necessary. Treatment usually consists of surgery and chemotherapy. Bone marrow transplantation offers the best chance of long-term survival.

Treatment usually consists of surgery and chemotherapy. Correct Optimum treatment of osteosarcoma is surgery and chemotherapy. Surgical biopsy is followed by either limb salvage or amputation and chemotherapy. Radiation and bone marrow transplantation are not part of the therapy for osteosarcoma. Amputation is often required when limb salvage is not possible

Which toxic agent is an appropriate cause of dermatitis? Gasoline Tulip bulb Spider plant Mildew remover

Tulip bulb Rationale Tulip bulbs have toxic properties, and topical contact can result in dermatitis. Gasoline is a hydrocarbon that can cause nausea and respiratory distress among several other symptoms. Spider plants are nonpoisonous and therefore will not cause any skin reactions. Mildew remover is likely to cause respiratory obstruction, signs of shock, and severe burning pain in the mouth, throat, and stomach, but not dermatitis.

During assessment, the nurse observes that the child's new secondary teeth are appearing too large for the face. What is the stage of child development for this child? Inferiority stage Ugly duckling stage Prepubescence stage Accomplishment stage

Ugly duckling stage Rationale The child is in the ugly duckling stage. In this stage, the teeth appear too large for the face. Facial appearance also changes, making the child's face appear disproportionate. A sense of inferiority is related to the accomplishment stage of Erikson's psychosocial development theory. Prepubescence, which lasts approximately 2 years, begins at the end of middle childhood and ends with the 13th birthday. Accomplishment is one of the stages of Erikson's psychosocial development theory.

The pediatric patient with a moderate brain injury demonstrates decreased ability to walk normally. Which finding indicates to the nurse a severe and progressive change in intracranial pressure (ICP)? Patient often lies in the fetal position. Upper extremities are drawn in toward chest. Feet exhibit decreased sensation when touched. Patient is unable to eat without experiencing GI upset.

Upper extremities are drawn in toward chest. Flexion and decorticate posturing indicate increased ICP.

Which nursing intervention is appropriate for the child that has ingested two strips of a mineral supplement containing iron? Select all that apply. Conduct lavage. Provide ventilation. Use bowel irrigation. Administer chelation therapy. Administer activated charcoal.

Use bowel irrigation. Administer chelation therapy. Rationale A patient who has ingested iron supplements in large quantities will exhibit gastrointestinal symptoms. The appropriate nursing intervention will involve flushing out the iron from the body system; therefore using bowel irrigation will be beneficial if radiopaque tablets are visible on an abdominal x-ray. Also, chelation therapy with deferoxamine is recommended in severe intoxication, whereby the urine may turn red or orange. Lavage will not be beneficial, because emesis empties the stomach more effectively than lavage. A patient who has ingested iron supplements will not show signs of respiratory distress, so ventilation is not required. Activated charcoal is not effective in absorbing iron.

Which recommendation is appropriate to provide to the parents of toddlers about car restraints? Use car restraints even for short trips. Add extra head cushions for comfort. Discourage the toddler from holding a toy. Add padding between the child and the restraint strap

Use car restraints even for short trips. Rationale Car restraints should be used even for a short trip to provide safety to the toddler. Adding an extra head cushion for comfort should be avoided, because this creates spaces between the child and the restraint, and decreases support to the head. Encouraging the toddler to hold a favorite toy will help the toddler to play quietly. Adding any padding between the child and the restraint strap should be discouraged, because it will create space between the child and the restraint. This will result in decreased support to the back of the toddler.

Which instruction would the nurse give to the parents of a toddler with a skin infection to reduce discomfort? Select all that apply. Use nonirritating clothing. Give hot baths to the patient. Use a calamine lotion regularly. Avoid sun exposure to the patient. Avoid administering hydroxyzine.

Use nonirritating clothing. Use a calamine lotion regularly. Avoid sun exposure to the patient. Rationale The patient may feel discomfort due to the skin infection and may need special precautions. Calamine lotion can be used to reduce itching. Nonirritating clothing can also help to reduce irritation and itching. Hot baths should be avoided, because an increase in temperature increases itching. Sun exposure should be avoided, because exposure to the sun increases skin irritation and worsens the condition. If the patient has severe itching, hydroxyzine can be administered to reduce itching.

Nursing care of the child with myelosuppression from leukemia or chemotherapeutic agents must include which intervention? Restriction of oral fluids Institution of strict isolation Use of good hand washing technique Administration of immunizations appropriate for age

Use of good hand washing technique Rationale Good hand washing technique is the most effective means of preventing disease transmission in children with myelosuppression. There is no indication for reduction of fluids in children with myelosuppression. Strict isolation is not necessary in children with myelosuppression. The child should not receive any live vaccines, because the immune system is not capable of responding appropriately to them.

How should the nurse position the child during an infratentorial procedure versus a supratentorial procedure? Place the child in the supine position. Avoid placing the child on the operative site. Use pillows against the child's back. Place the child in Trendelenburg position.

Use pillows against the child's back. Rationale In an infratentorial procedure, the child may remain flat or on either side. Pillows should be placed against the child's back, rather than behind the head, to maintain the desired position depending on the area of surgery. The head and neck are kept midline with the body or slightly extended. However, in supratentorial procedure, the head is elevated above the heart. The child may be placed in the supine position after either procedure is done, depending on the operative site. The child should not be placed on the operative site in either case. The Trendelenburg position is always avoided because this position tends to increase intracranial pressure and increase the risk of hemorrhage.

Radiation nephroblastoma

Used commonly with later stage disease and provides palliation for metastatic lesions in bones, lungs, live, and brain

Chemotherapy nephroblastoma

Used for extensive local or disseminated disease

The nurse is conveying some bad news to parents about their child, who is chronically ill. Which action by the nurse needs correction? Providing information slowly Using medical terms to explain in-depth information Acknowledging the parents' reactions and feelings Stopping regularly to check if the parents understand.

Using medical terms to explain in-depth information Rationale The nurse should use easily understandable language and should avoid medical terms, which the parents would find difficult to understand. The nurse should be patient and slowly explain the process and condition of the illness to the parents. The nurse should acknowledge the parents' reactions and make sure that the parents understand the information provided. Checking in regularly with parents to see if they have any questions about the discussion is also recommended.

Which disease presents with fever, malaise, and a highly pruritic rash that started as a papule and became a vesicle? Varicella Diphtheria Roseola infantum Erythema infectiosum

Varicella Rationale A slight fever, malaise, and highly pruritic rash starting with papules and progressing to vesicles are highly suggestive of varicella, or chickenpox. Diphtheria is not characterized by a rash. Roseola infantum or erythema infectiosum rashes are not vesicular.

A healthy child is brought to the clinic for a 15-month well visit. The nurse notes that the toddler received haemophilus influenzae type b (Hib) #4; measles, mumps, rubella (MMR)#1; and pneumococcal conjugate (PCV13)#4 at the one-year visit. Which vaccination(s) will the nurse prepare for this visit? Select all that apply. Varicella (VAR) Meningococcal B Measles, mumps, rubella (MMR) Pneumococcal conjugate (PCV13) Haemophilus influenzae type b (Hib) Diphtheria, tetanus, & acellular pertussis (DTaP)

Varicella (VAR) The nurse will prepare the first dose of varicella (VAR), which is to be given at 12 to 15 months. The second dose will be given at four to six years. Diphtheria, tetanus, & acellular pertussis (DTaP) The nurse will prepare the fourth dose of diphtheria, tetanus, & acellular pertussis (DTaP) at this visit. DTaP is given at age 15 to 18 months unless it was given at 12 months.

Which causative agent results in chickenpox? Varicella zoster virus Human parvovirus B19 Human herpesvirus type 6 Group A ß-hemolytic streptococcus

Varicella zoster virus Rationale Varicella zoster virus is the causative agent for chickenpox. Human parvovirus B19 is the causative agent for erythema infectiosum (fifth disease). Human herpesvirus type 6 is the causative agent for exanthema subitum (roseola). Group A ß-hemolytic streptococcus is the causative agent for scarlet fever.

An arterial blood gas is drawn on a patient and it shows a decrease in the arterial partial pressure of carbon dioxide (PaCO2). The nurse should expect which response of the cerebral vasculature? Vasodilation, increased blood flow Vasodilation, decreased blood flow Vasoconstriction, increased blood flow Vasoconstriction, decreased blood flow

Vasoconstriction, decreased blood flow A decrease in the partial pressure of carbon dioxide will result in vasoconstriction and a decrease in cerebral flood flow.

Which drug administration is recommended to decrease the risk of mortality in a child with measles? Vitamin C Vitamin D Vitamin A Vitamin K

Vitamin A Rationale Measles with vitamin deficiency can be fatal to children. Two doses of vitamin A (200,000 IU) should be given on 2 successive days after the diagnosis of measles is made. This can reduce the mortality rate, pneumonia-specific mortality, and morbidity associated with measles. Vitamin A prevents any complications associated with measles, such as diarrhea and infections. A single dose of vitamin A may not be effective. Vitamins C, D, and K do not play any role in treating or preventing measles. Vitamin C deficiency causes scurvy, vitamin D deficiency causes rickets, and vitamin K deficiency can lead to blood clotting abnormalities.

The nurse is teaching a community health promotion class to parents and school-age children related to bicycle safety. What information would be included in this educational session? Walking bicycles through busy intersections Installing reflectors only on bicycles that are to be ridden at night Wearing a bicycle helmet only if the child is planning to ride in traffic Riding bicycles against, rather than with, traffic so the rider can see the cars

Walking bicycles through busy intersections Rationale Bicycles should be walked through busy intersections to allow the child to have full view of the traffic and be able to react accordingly, with safety the number one priority. Bicycle helmets should be worn at all times to prevent head injuries. Reflectors should be installed on all bicycles, whether they are ridden during the daytime or at night only. Bicycles should always be ridden with the traffic, not against the traffic. This will help prevent accidents.

Which measure is appropriate to prevent lead poisoning for people living in a community in which most of the houses were built in the year 1960? Select all that apply. Wash toys and pacifiers frequently. Run the water for 2 minutes before using it. Use only hot water for drinking or cooking. Use cold water for the preparation of formula. Vacuum hard-surfaced floors and windowsills daily.

Wash toys and pacifiers frequently. Run the water for 2 minutes before using it. Use cold water for the preparation of formula. Rationale People living in a community in which most of the houses were built in the year 1960 are at high risk of lead poisoning. Toddlers and infants tend to put toys and pacifiers in the mouth and may get poisoned if these have lead on them. Thus all parents should be advised to wash their children's toys and pacifiers frequently. The nurse should inform them that, when using water from pipes, only cold water should be used for drinking, cooking, and reconstituting powder infant formula. In water pipes, lead dissolves quickly into hot water; therefore only cold water is safe for use. Water that sits in the pipes for a time has a relatively high concentration of lead. Therefore the water should be allowed to run for about 2 minutes to flush any residual lead from the pipes prior to drinking. Lead dust can be spread by vacuuming. Hence, vacuuming should not be done on hard surfaces such as floors and windowsills. These floors should be wet mopped.

A parent has a 2-year-old child in the clinic for a well-child checkup. What statement by the parent would indicate to the nurse that the parent needs more instruction regarding accident prevention? "We locked all the medicines in the bathroom cabinet." "We turned the thermostat down on our hot water heater." "We placed gates at the top and bottom of the basement steps." "We stopped using the car seat now that our child is older."

We stopped using the car seat now that our child is older." Correct Convertible car seats are necessary until the child is at least 18 kg (40 pounds). Booster seats are required until the child is 36.2 kg (80 pounds). Locking medicines in the bathroom cabinet will help prevent the child from accidentally ingesting medicines. Decreasing the temperature on the water heater can help prevent burns. Gates are appropriate to keep the toddler from falling down the stairs.

Which precaution is appropriate for hookworm prevention? Washing hands before eating food Not consuming undercooked pork Wearing shoes when playing in soil Covering all food using fly covers

Wearing shoes when playing in soil Rationale Soil contaminated with hookworm eggs can lead to hookworm infestation. The worms can enter the body by penetrating through the skin. Therefore, wearing shoes is recommended as one of the preventive measures. Although washing hands in general is recommended before eating, hookworms are not transmitted through food or by eating undercooked pork or meat. Flies and mosquitoes do not transmit hookworms.

the most common clinical manifestation of wilms tumor is: a. painless, firm abdominal mass that does not cross the midline b. painful abdominal mass that does cross the midline c. nausea and vomiting d. anorexia

a. painless, firm abdominal mass that does not cross the midline

which of the following strategies would be best for the nurse to use to support the family's spiritual needs if their chid's death is imminent and clergy member is unavailable? a. pray appropriately with the family b. implement relaxation techniques c. make an appointment for the family to speak with an expert d. review the physical signs of death with the family

a. pray appropriately with the family

physical assessment for the child physical abuse a. should identify all the injuries b. should always begin with rapid assessment of airway, breathing, circulation, and neurologic systems c. should recognize that all forms of physical abuse have obvious signs d. should occur only after legal authorities have been notified

a. should identify all the injuries

Which is the main difference between neuroblastoma and Wilms tumor? Wilms tumor is confined to one side of the abdomen. Neuroblastoma is devoid of any kind of urinary symptoms. Neuroblastoma causes a painless swelling in the abdomen. Wilms tumor is often diagnosed in children less than 5 years of age.

Wilms tumor is confined to one side of the abdomen. Rationale Wilms tumor is usually confined to one side of the abdomen whereas neuroblastoma crosses the midline of the abdomen. Urinary symptoms are seen in both types of tumors. The swelling or abdominal mass is painless in both neuroblastoma and Wilms tumor. Both conditions are often diagnosed in children less than 5 years of age. Neuroblastoma is a more common malignancy in the first year of life whereas Wilms tumor is more common between 2 and 3 years of age.

Which prevention measures assist in decreasing exposure to lead? Select all that apply. Wipe feet before entering home. Wash toys and pacifiers frequently. Use only warm faucet water for consumption. Avoid folk remedies or cosmetics that contain lead. Ensure child's diet contains sufficient iron and calcium.

Wipe feet before entering home. Wash toys and pacifiers frequently. Avoid folk remedies or cosmetics that contain lead. Ensure child's diet contains sufficient iron and calcium Rationale Prevention measures that assist in decreasing exposure to lead include wiping the feet before entering the home, especially if the person works in an occupation in which lead is used, and washing toys and pacifiers frequently. In addition, it is important to avoid folk remedies or cosmetics that contain lead. Furthermore, it is important to ensure that the child's diet contains sufficient iron and calcium. It is important to use only cold water for consumption (drinking, cooking, etc.) because hot water dissolves lead more quickly than cold water and thus contains higher levels of lead.

How would the nurse instruct the parent to care for the eye of a child who has bacterial conjunctivitis? Apply continuous warm compresses to the eye. Use topical corticosteroids to reduce inflammation. Wipe from the outer canthus toward the opposite eye. Wipe from the inner can thus downward and outward away from the opposite eye.

Wipe from the inner can thus downward and outward away from the opposite eye. Rationale To remove secretions from the eye, the nurse should instruct the parent to wipe from the inner canthus of the eye downward and outward away from the opposite eye. Applying continuous warm compresses on the eye can promote bacterial growth. Topical corticosteroids are avoided because they reduce ocular resistance to bacteria. Wiping from the outer canthus of the eye toward the opposite eye may spread the infection to the other eye.

Which method for administering a chelating agent to reduce pain in young patients is appropriate? With procaine Just under the epidermis In the same site each time Only via the intravenous route in children with cerebral edema

With procaine Rationale A local anesthetic such as procaine is injected along with a chelating agent to reduce pain. In order to reduce injection pain, the nurse would administer the chelating agent deeply into a large muscle. It is important to rotate the site of administration to prevent the formation of fibrotic tissue that will result in further pain. The chelating agent would not be administered via the intravenous route in children with cerebral edema.

Which is an example of an activity that can help the child who is going through the dying process? Select all that apply. Providing the child with audiobooks. Writing a letter to the child's mother and father. Journaling about a favorite memory with the family. Watching favorite TV shows on Netflix during treatments. Creating a scrapbook of pictures from the last family vacation.

Writing a letter to the child's mother and father. Memories can be created for the family through the writing of letters by the child. A child life specialist can assist with this activity if the child requires assistance with writing. Journaling about a favorite memory with the family. Journaling provides memories for the child and family member to read throughout the dying process. It is a lasting memory for the family members after the child has died. Creating a scrapbook of pictures from the last family vacation. Memories can be created for the child and family through creating a scrapbook of pictures from past family events.

Clinical manifestations differentiate bacterial conjunctivitis from viral conjunctivitis. Which one of the following is present with bacterial conjunctivitis but not usually found with viral conjunctivitis? a. Child awakens from crusting of eyelids. b. Child has increase in water drainage from eyes. c. Child has inflamed conjunctiva. d. Child has swollen eyelids.

a. Child awakens from crusting of eyelids.

In hospitals, which of the following is the most significant source of MRSA and the major mode of transport? a. Patient to patient via the hands of the health care provider. b. Patients coming in direct contact with other patients. c. Failure of hospital personnel to wear face masks when working with patients in an airborne infection isolation room. d. Indirect contact transmission from hospital equipment.

a. Patient to patient via the hands of the health care provider.

Which of the following statements about rotavirus disease or immunization against rotavirus is correct? a. Rotavirus is one of the leading causes of diarrhea in infants and young children. b. Rotavirus is one of the leading causes of otitis media in young children. c. One vaccine for rotavirus is RotaTeq, which is approved for children 32 weeks of age or older. d. Rotavirus vaccine usually causes only mild reaction, redness, and soreness at the injection site.

a. Rotavirus is one of the leading causes of diarrhea in infants and young children.

Meningococcal conjugate vaccines (MCV4) are not recommended for which of the following populations? a. Routinely for children ages 9 months to 10 years. b. Children aged 2 years to 18 years who travel to or reside in countries where N. meningitides is hyper endemic or epidemic. c. Children and adolescents 11 to 12 years of age. d. College freshmen living in dormitories.

a. Routinely for children ages 9 months to 10 years.

hereditary retinoblastomas are almost always considered to be transmitted as: a. a germline mutation b. a somatic mutation c. a chromosomal aberration d. an autosomal recessive trait

a. a. a germline mutation

a complete remission of leukemia is determined by (select all): a. absence of clinical s and sx b. presence of more than 9% blast cells in the bone marrow c. WBC count of 10 to 12 d. presence of less than 5% blast cells in the bone marrow

a. absence of clinical s and sx d. presence of less than 5% blast cells in the bone marrow

the nursing intervention that would be most helpful for the child who has stomatitis from chemotherapy would be: a. an anesthetic preparation without alcohol b. viscous lidocaine c. lemon glycerin swabs d. a mild sedative

a. an anesthetic preparation without alcohol

seven year old andy was caught taking a playmate's toy. which of the following is an important understanding of this behavior? a. at this age andy's sense of property rights is limited, and he took the item simply because he was attracted to it b. if andy is caught and punished and promises not to do it again, he will keep his promise c. this stealing act is an indication that something is seriously lacking in ady's life d. andy will learn the importance of respecting others' property if the parents unexpectedly give away an item of andy'

a. at this age andy's sense of property rights is limited, and he took the item simply because he was attracted to it

according to the recent studies, the best place for the infant car restraint is in the: a. back seat of the car, facing the rear b. back seat of the car, facing front c. front passenger seat of the car with an air bag, facing front d. front passenger seat of the car without an airbag, facing back

a. back seat of the car, facing the rear

which of the following statements related to bullying is correct? a. bullying usually occurs in school hallways and playgrounds where supervision is minimal but peers are present to witness attack b. bullying or cyberbullying affects 50% of elementary school students either as the bully or the victim c. the victim bullying is usually male, depressed, has poor academic performance, and poor relationships and communication with parents d. victims of bullying are at higher risk for development of future problems of school dropout, unemployment and participation in criminal behavior

a. bullying usually occurs in school hallways and playgrounds where supervision is minimal but peers are present to witness attack

discharge planning for children with lead poisoning includes a. confirmation that the child will be discharged to a home without lead hazards b. immediate referral for development and speech therapy c. visiting nurse care for continuation of IV treatment at home d. explanation that there is little need for follow up because lead levels have returned to normal

a. confirmation that the child will be discharged to a home without lead hazards

When assisting parents in supporting their dying child, the nurse should stress the importance of honesty; if parents are honest and openly discuss their fears, the child is more likely to: a. discuss his or her fears. b. ask fewer distressing questions. c. lose his or her sense of hope. d. do all of the above

a. discuss his or her fears.

The American Academy of Pediatrics has recommended vitamin A supplements for certain pediatric patients with measles. Correct dosage of vitamin A and instructions to parents of these children include: i. single oral dose of 200,000 IU in children 1 year old. ii. single oral dose of 100,000 IU in children 6 to 12 months old. iii. dosage may be associated with vomiting and headache for a few hours. iv. safe storage of the drug to prevent accidental overdose. a. i, ii, iii, and iv. b. i, ii, and iv. c. i, iii, and iv. d. ii and iv.

a. i, ii, iii, and iv.

For which of the following populations is pertussis vaccine currently recommended? i. All children from 6 weeks to the seventh birthday. ii. Children ages 11 to 12 years who have completed the DTaP/DTP childhood series. iii. A second booster for previously immunized adolescents and adults. iv. Children ages 7 through 10 years who are not fully vaccinated for pertussis (i.e., did not receive 5 doses of DTaP, or who received 4 doses of DTaP with the fourth dose being administered on or after the fourth birthday). v. Pregnant adolescents between 27 and 36 weeks' gestation, if not previously administered. a. i, ii, iv, and v. b. i, iii, and iv. c. ii, iii, and v. d. i, ii, iii, and iv.

a. i, ii, iv, and v.

Which of the following strategies are effective in helping an infant sleep well at night? i. Place infant to sleep in its own bed. ii. Give the infant a bottle if it awakens during the night. iii. Establish and enforce a bedtime routine. iv. Allow the infant to watch a movie before bedtime. v. Allow the crying infant to sleep with parents or sibling. vi. Avoid picking up and rocking the infant who awakens during the night. a. i, iii, and vi. b. ii, iii, and v. c. ii, iii, and vi. d. i, ii, and vi.

a. i, iii, and vi.

Primary prevention of communicable disease is best accomplished by: a. immunization. b. control of the disease spread. c. adequate water supply. d. implementing good hand-washing practices among hospital personnel.

a. immunization.

which one of the following actions taken by the nurse is the LEAST likely to prevent recurrence of poisoning? a. in the emergency department, begin a discussion of ways to injury-proof the home b. do a home visit to assess safety before the child is discharged c. administer questionnaire for poison prevention to the parents when the child is d/c'd d. advise parents to kneel down to child's level when determining what products need to be placed out of reach

a. in the emergency department, begin a discussion of ways to injury-proof the home

When the parent of a child who is dying tells the nurse the child is in pain, even when the child is on regularly scheduled pain medication, the nurse should be : a. institute breakthrough pain control measures. b. continue to administered pain meds on a regular preventive schedule. c. educate parent that pain is a physical process. d. educate parents that the child is probably in less pain than the parents think.

a. institute breakthrough pain control measures.

Current research supports the notion that: a. involvement in the experiences of the dying sibling is beneficial. b. siblings of children who died in the hospital reported readiness for the death. c. protecting the sibling of the dying child from the death rituals is beneficial. d. it is better for the sibling to remember the dying child as he or she was when alive.

a. involvement in the experiences of the dying sibling is beneficial.

Rubella vaccine administration: a. is given as protection for the unborn child rather than for the recipient of the immunization. b. is recommended for all children beginning at 4 to 6 years of age. c. is given to all pregnant women if not previously immunized. d. is not given to children whose mother is currently pregnant.

a. is given as protection for the unborn child rather than for the recipient of the immunization.

a candidate for bone marrow transplantation is the child who: a. is unlikely to be cured by other means b. has acute leukemia c. has chronic leukemia d. has a compatible donor in his or her family

a. is unlikely to be cured by other means

diagnostic evaluations for lead poisoning include a. led concentration from venous blood spicimen to confirm diagnosis b. recommended universal screening for all children & those w ages 6 yrs or older given priority c. identifying children at high risk for anemia since these children will most likely have higher lead levels d. understanding that a blood level for lead in the 3-5mg/dL range is normal and the child should be rescreened in 1yr unless exposure status changes

a. led concentration from venous blood spicimen to confirm diagnosis

which on of the following carcinogenic agents has been implicated in the development of childhood cancer? a. low doses of radiation b. excessive sun exposure c. exposure to cigarette smoke d. vitamin K given at birth

a. low doses of radiation

the child who receives a bone marrow transplant will require: a. meticulous personal hygiene b. multiple peripheral sites for IV therapy c. less chemo before the transplant d. a room with laminar air flow

a. meticulous personal hygiene

blood test levels for lead

action for blood lead level of 5 mcg or < educate, rescreen in 1 year action for blood lead level of 5-14 mcg educate, developmental/behavioral surveillance, social services if necessary, follow up within 1 mo then q3-4 mo action for blood lead level of 15-19 mcg same as previous plus contact social services, professional environment clean up, follow 20-44 mcg guidelines if BLL remains 15 on two samples 3 mo apart action for blood lead level of 20-44 mcg consider chelation therapy, refer to clinic specializing in lead therapy action for blood lead level of 45-69 diagnostic testing in 24-48 hours, evaluate management in 48 hours, chelation therapy, ensure aggressive environmental intervention, follow up once per month blood lead level of 70+ immediately provide diagnostic testing and initiate chelation does chelation counteract any effects of lead no

corrosive poisoning tretment

activated charcoal contraindicated for corrosive because it may infiltrate burned tissue water or milk contact poison control center; if their suggested treatment is not available, dilute corrosive with contraindicated gastric emptying by emesis or lavage for corrsives is neutralize is contraindicated with corrosives in case of reaction maintain patent airway, administer analgesics, and oral fluid when tolerated corrosive ingestive treatment esophageal stricture may require dilations or surgery after corrosive ingestion

treatment for aspirin toxicity

activated charcoal immediately if bowel sounds present (unless CI by altered mental status) q4h until charcoal is in stool lavage will not remove concretions of aspirin sodium bicarbonate transfusions are used to correct metabolic acidosis with aspirin toxicity urinary alkalaization may be effective in enhancing elimination of aspirion hypokalemia may interfere with achieving urinary alkalization fluid overload and pulmonary edema be aware of risks with aspirin toxicity hyperpyrexia with aspirin toxicity use external cooling for anticonvulsants, provide oxygen and ventilation, and administer vitamin K treatment for aspirin ingestion toxicity vit k for bleeding

concurrent care

introduction of palliative care at the time of a life threatening diagnosis with increasing support over time as disease progresses

Diagnostic evaluation of neuroblastoma

aimed at locating the primary site of the tumor, analyzing the breakdown products excreted in urine -vanillylmandelic acid -homovanillic acid -dopamine -norepinephrine and permits detection of suspected tumor before and after medical/surgical intervention

Middle childhood

also known as the school years, extends from 6 to 12 years of age. This period begins with entrance into the school environment, which has a significant impact on development and relationships. School-age children are more graceful than they were as preschoolers, and they are steadier on their feet. Their bodies take on a slimmer look with longer legs, varying body proportions, and a lower center of gravity. Posture improves over that of the preschool period to facilitate locomotion and efficiency in using the arms and trunk. Although growth is slower than in previous years, there is a steady gain in height and weight, with maturation of body systems. Primary teeth are lost and replaced by permanent teeth. The most pronounced changes that seem best to indicate increasing maturity in children are a decrease in head circumference in relation to standing height, a decrease in waist circumference in relation to height, and an increase in leg length related to height.

Common nutritional disorders of infancy

and early childhood may result from vitamin and mineral deficiency or excess, inadequate caloric intake or absorption, and food protein intolerance. Vitamin D-deficiency rickets, once rarely seen because of the widespread commercial availability of vitamin D-fortified milk, increased before the turn of the century. Populations at risk include: Children who are exclusively breastfed by mothers with an inadequate intake of vitamin D or are exclusively breastfed longer than 6 months without adequate maternal vitamin D intake or supplementation Children with dark skin pigmentation who are exposed to minimal sunlight because of socioeconomic, religious, or cultural beliefs or housing in urban areas with high levels of pollution or who live above or below a latitude of 33 degrees north and south where sunlight does not produce vitamin D Children with diets that are low in sources of vitamin D and calcium Individuals who use milk products not supplemented with vitamin D (e.g., yogurt, raw cow's milk) as the primary source of milk Children who are overweight or obese Such infants should receive 400 IU of vitamin D. Nutrient consumption varies among vegetarians; therefore, a detailed dietary assessment is essential for planning adequate intakes, particularly in children and pregnant and lactating women. Children and adolescents on vegetarian diets have the potential for lifelong healthy diets and have been shown to have lower intakes of cholesterol, saturated fat, and total fat and higher intakes of fruits, fiber, and vegetables than nonvegetarians.

Bone marrow transplantation (BMT)

another approach to the treatment of childhood cancer. Candidates for transplantation are children who have malignancies that are unlikely to be cured by other means. BMT allows for administration of lethal doses of chemotherapy, often combined with radiotherapy, to rid the body of all cancer cells; then the donor bone marrow or stem cells are infused into the patient.

Coping mechanisms

are behaviors directed at reducing the tension elicited by a crisis. Approach behaviors are coping mechanisms resulting in movement toward adjustment and resolution of the crisis. Avoidance behaviors result in movement away from adjustment or maladaptation to the crisis. The diagnosis of a serious health problem or disability in a child is a major situational crisis affecting the entire family. However, families can experience positive outcomes while they successfully cope with the many challenges that accompany a child with chronic illness or disability.

Plant Alkaloids

arrest cells in metaphase by binding to microtubular protein needed for spindle formation vincristine, vinblastine

postoperative interventions brain tumor

assess neurological and motor function and level of conscious - monitor temp; maintain cooling blanket by the bedside - monitor for signs of respiratory infection - monitor for signs of meningitis ( opisthotonos, kernig's and brudzinski's signs) - monitor for signs of increased ICP - monitor for hemorrhage, posterior blood pooling, mark drainage edges with marker - assess pupillary response, sluggish and dialated report immediately - monitor for colorless drainage of the dressing from the ears and nose ( cerebrospinal fluid) and assess for presence of glucose in the drainage - assess the HCP's prescription for positioning, degree of neck flexion - monitor IV fluids closely - promote measures that prevent vomiting - provide quite environment - administer analegsics - provide emotional support

Bryan, age 6 months, is starting daycare as his mother is returning to work. Bryan has had no immunizations. Which of the following statements provided by the nurse to the mother is the most appropriate at this time? a. "Since Bryan has not started his immunization for streptococcal pneumococci yet, it is best to wait until after he gets established at daycare before beginning his injections." b. "Streptococcal pneumococci are responsible for a number of bacterial infections that are especially problematic for children under 2 years of age who attend daycare. Bryan should start his series of pneumococcal vaccine right away." c. "Why has Bryan not received any immunizations? He is past due. Don't you care about his health?" d. "Pneumococcal vaccine (PCV13) is the only vaccine recommended for Bryan at this time."

b. "Streptococcal pneumococci are responsible for a number of bacterial infections that are especially problematic for children under 2 years of age who attend daycare. Bryan should start his series of pneumococcal vaccine right away."

Bedbugs, although once considered to be practically nonexistent, have remerged within the past decade as troublesome. Which of the following does the nurse recognize as incorrect information about bedbugs? a. They tend to inhibit warm, dark areas such as furniture and emerge at night to feed. b. They act as vectors for disease transmission. c. The treatment of bedbugs should focus on proper identification, treatment of symptoms, and eradication. d. Their bits are often misdiagnosed as scabies, spider bites, or mosquito bites.

b. They act as vectors for disease transmission.

The nurse knows, regarding pertussis, that: a. the incidence has decreased in infants younger than 6 months of age. b. a booster vaccine (Tdap) is now recommended for all children 11 to 12 years of age. c. treatment should begin as soon as exposure is confirmed and includes the antibiotic amoxicillin. d. the disease is not contagious, so close household members do not need treatment.

b. a booster vaccine (Tdap) is now recommended for all children 11 to 18 years of age.

of the following assessment finds, the one that would most likely be seen in a child with leukemia is: a. weakness of the eye muscle b. bruising, nosebleeds, pallor, and fatigue c. wheezing and shortness of breath d. abdominal swelling

b. bruising, nosebleeds, pallor, and fatigue

which of the following accurately describes the expected cognitive development during the concrete operation period of childhood? a. children are able to follow directions but unable to verbalize the actions involved in the process b. children are able to use their thought processes to experience events and actions and make judgements based on what they reason c. children are able to see things from an egocentric outlook that is rigidly developed around the action to be completed d. children progress from conceptual thinking to perceptual thinking when making judgements

b. children are able to use their thought processes to experience events and actions and make judgements based on what they reason

a child who unintentionally contributes to an abusive situation most likely a. fits into that "easy-child pattern" b. has demands, both physical and emotional that are incompatible with parents' ability to meet these needs c. has low self-esteem d. comes from a low socioeconomic background

b. has demands, both physical and emotional that are incompatible with parents' ability to meet these needs

When educating the public about diphtheria vaccine, the nurse recognizes which one of the following as correct? i. Diptheria vaccine is commonly administered in combination with tetanus and pertussis vaccines (DTaP) or DTaP and Hib vaccines for children under 7 years of age. ii. Diptheria vaccine is administered with tetanus and acellular partisan (TDaP) for children 11 years and older. iii. Diphtheria vaccine produces absolute immunity after 3 doses. iv. Several vaccines contain diphtheria toxoid (Hib, meningococcal, pneumococcal), which infers immunity to the disease. a. i, ii, iii, and iv. b. i and ii. c. i, ii, and iii. d. iii and iv.

b. i and ii.

which of the following assessment findings would be expected in an infant with retinoblastoma a. visible red reflex b. leukoria c. nustagmus d. strabismus

b. leukoria

which one of the following would the nurse NOT expect to observe as characteristic of peer group relationships of 8 yr old mark? a. mark demonstrates loyalty to the group by adhering to the secret code rules b. mark demonstrates a greater individual egocentric outlook when compared with other peer group members c. mark is willing to comform to the group's rule of not talking to girls d. mark has a best friend within the peer group with whom he shares secrets

b. mark demonstrates a greater individual egocentric outlook when compared with other peer group members

a 9 year old with leukemia presents with a fever of 2 day; lab shows an ANC of 190mm. nursing care for this child should include: a. placing the child in isolation to prevent contamination of other patients b. practicing meticulous hand washing c. avoiding needle sticks and skin punctures such as IM injections d. immediately placing the child on 100% oxygen by face mask

b. meticulous hand washing

the nurse is to give an injection of the chelation drug ca disodium edentate. which of the following does the nurse recognize as most appropriate a. keeping the child npo 24hrs after administration b. mixing the drug w procaine to lessen the pain associated with the injection c. maintaing seizure precautions at the bedside d. making certain pt has no peanut allergy before injection

b. mixing the drug w procaine to lessen the pain associated with the injection

identify which of the following statements describing fears in the school aged child is true: a. school age children are increasingly fearful of body safety b. most of the new fears that trouble school age children are related to school and family c. school age children should be encouraged to hide their fears to prevent ridicule by peers d. children with numerous fears need continuous protective behavior by parents to eliminate the fears

b. most of the new fears that trouble school age children are related to school and family

burkitt lymphoma is a type of: a. hodgkin disease b. non-hodgkin lymphoma c. acute myelocytic leukemia d. neuroblastoma

b. non-hodgkin lymphoma

intervening therapeutically with terminally ill children and their families requires a. only self-awareness b. nursing practice that is based on a theoretic formulation and emperic observations c. years of experience d. personla experience with death

b. nursing practice that is based on a theoretic formulation and emperic observations

ingestion of injurious agents by children a. occurs most frequently at grandparents' or friend's home b. occurs because infants and toddlers explore their environment through oral experimentation c. has increased despite the use of child-resistant containers d. can be avoided by teaching preschoolers which substances are dangerous

b. occurs because infants and toddlers explore their environment through oral experimentation

studies indicate that toddlers up to 24 months of age are safest when they are a. place in a car restraint seat in the front seat of the car b. placed in a car restraint seat facing the rear in the back seat c. placed in a front passenger seat with a deactivated airbag d. allowed to stand in the back seat holding onto a shoulder belt

b. placed in a car restraint seat facing the rear in the back seat

what period begins toward the end of middle childhood and ends at age 13? a. puberty b. preadolescence c. early maturation d. all of the above

b. preadolescence

treatment for ewing sarcoma usually involves: a. radiation alone b. radiation and chemo c. amputation and chemo d. chemo alone

b. radiation and chemo

burn injuries in the toddler age-group are most the result of a. flame burns from playing with mathces b. scald burn from hot liquid c. hot object burs from cigarettes or irons d. electric burns from electric outlets

b. scald burn from hot liquid

accidental poisoning in toddlers cab be best prevented by a. consistently using safety caps b. storing poisonous substances in a locked cabinet c. keeping ipecac syrop in the home d. storing poisonous substances out of reach

b. storing poisonous substances in a locked cabinet

one of the best ways to prevent drowning in the toddler group is for parent to a. learn CPR b. supervise children within arm's reach whenever they are neat any source of water c. enroll the toddler ina swimming program d. not allow the child to swim

b. supervise children within arm's reach whenever they are neat any source of water

the nurse expects to assist with administration for a specific antidote for poisoning in which one of the following pediatric pts? a. the 8m old child admitted to the emergency deparmtnet after 8-10 holly berries b. the 13 yr old who ingested an overdose of diazepam (valium) c. the 8 yr old who ingested 3 of his mothers birth control pills d. the 6 year old who ingested an overdose of an unidentified corrosive substance

b. the 13 yr old who ingested an overdose of diazepam (valium)

preoperative instructions to prepare parents of a child who is scheduled for eye enucleation should include: a. there will always be a cavity in the skull where the eye was b. the child's face may be edematous and ecchymotic c. the eyelids will be open and the surgical site will be sunken

b. the child's face may be edematous and ecchymotic

which of the following best identifies the spiritual development of school aged children? a. they have little fear of going to hell for misbehavior b. they begin to learn the differences between the natural and the supernatural c. they petition to god for less tangible rewards d. they view god as a deity with few human traits

b. they begin to learn the differences between the natural and the supernatural

a major benefit of using umbilical cord blood for stem cell transplant is: a. stem cells are found in low frequency in newborns b. umbilical cord blood is relatively immunodeficient c. there is a lower risk for acute tumor lysis syndrome d. umbilical cord blood has a greater number of neutrophils

b. umbilical cord blood is relatively immunodeficient

corrosives

batteries, bleach, clinitest tablets, denture cleanest; draine, toilet, and oven cleaners, electric diswater detergent, and mildew remover household bleach frequently, ingested corrosive, but rarely causes serious damage liquid corrosives easily ingested and cause more damage than granular/solid preparations; may be aspirated, causing upper airways injury solid products tend to stick and burn tissues, causing localized damage

occupations and hobbies involving lead

battery and aircraft, manufacturing lead smelting, brass foundry work, radiator repair, construction work, furniture refinishing, bridge repair work, paining contracting, mining, ceramics work, stained glass making, and jewelry making

Hep B

before hospital discharge for preterm - at birth, 1, 2, 6 months IM vastus lateralis - newborn deltoid for older infants and toddlers avoid dorsogluteal birth, 2 months, 6 months When do we give hep B immunizations?

immunization

begins at birth or 2 weeks when are there contraindications for vaccine administration severe febrile illness Immunocompromised child or household member Recently acquired passive immunity Known allergic response Cancer what are nonmedical contraindictations for vaccine administration Parental fears, misinformation, and questions Religious beliefs what are examples of recently acquired passive immunity, that are contraindicated for vaccine admin (blood transfusion, immunoglobulin, or maternal)

The peak age for osteosarcoma

between 10 and 25 years

iron poisoning treatment

bowel irrigation induce vomiting activated charcoal doesnt help chelation therapy

Leukemia

broad term given to a group of malignant diseases of the bone marrow and lymphatic system. It is a complex disease of varying heterogeneity. In children, two forms are generally recognized, acute lymphoblastic leukemia (ALL) and acute myelogenous leukemia (AML). ALL is the most common form of childhood cancer. The most important factors in determining long-term survival for children with ALL are the initial white blood cell count, the patient's age at diagnosis, cytogenetics, the immunologic subtype, and the child's sex.

The HBsAg negative mother of Daniella, a premature infant weighing 4 pounds and born 6 hours ago, asks, "Have you already given Daniella her hepatitis B vaccine?" What is your correct response? a. "Don't worry, we will give it before she leaves the hospital." b. "Daniella is sleeping right now. I will give it as soon as she wakes up." c. "Because Daniella only weighs 4 pounds, she will not receive her vaccine until she is 1 month of age." d. "Because Daniella is premature and weights only 4 pounds, she will be given one vaccine dose now and then she will need another in two months."

c. "Because Daniella only weighs 4 pounds, she will not receive her vaccine until she is 1 month of age."

Which one of the following helping statements would be least therapeutic for the nurse to use with the bereaved family? a. "You can stay with him and hold him if you wish." b. "It must be painful for you to return to the doctor's office without her." c. "Fortunately his suffering is over now." d. "You have been through a very difficult time."

c. "Fortunately his suffering is over now."

a 10 year old child has received a dose of L aspariginase to which he had an anaphylactic reaction a month ago. the boy's mother urges him to get his belongings together so they can leave the outpatient area. The nurse, however, insists that he stay for a period to be observed, because he had a previous reaction. The safe minimum time for observation for this child should be: a. 20 minutes b. 30 minutes c. 60 minutes d. 90 minutes

c. 60 minutes

Anne, an 8-year-old, has been diagnosed with giardiasis. The nurse would expect Anne to have most likely been seen initially with which of the following signs and symptoms? a. Diarrhea with blood in the stools. b. Nausea and vomiting with a mild fever. c. Abdominal cramps with intermittent loose stools. d. Weight loss of 5 lb in the past month.

c. Abdominal cramps with intermittent loose stools.

Which of the following statements about polio vaccine and immunization is not correct? a. Inactivated poliovirus vaccine (IPV) is now recommended for routine childhood vaccination in the US. b. Oral polio vaccine (OPV) has been associated with vaccine-associated paralysis. c. KINRIX contains DTaP, hepatitis B, and IPV and may be used only in children aged 4 years or older as the fourth dose. d. The combination vaccine PEDIARIX (containing DTaP, hepatitis B, and IPV) may be used as the primary immunization beginning at 2 months of age.

c. KINRIX contains DTaP, hepatitis B, and IPV and may be used only in children aged 4 years or older as the fourth dose.

Which of the following statements about varicella vaccine is correct? a. Varicella vaccine is recommended for all children regardless of past disease history. b. A single dose of 0.5 mL of varicella vaccine should be given by deep IM injection. c. The first dose of varicella is recommended for children ages 12 to 15 months, and to ensure adequate protection, a second varicella vaccination is recommended for children at 4 to 6 years of age. d. Varicella vaccine should not be administered simultaneously with MMR

c. The first dose of varicella is recommended for children ages 12 to 15 months, and to ensure adequate protection, a second varicella vaccination is recommended for children at 4 to 6 years of age.

The nurse is conducting an educational session of a child diagnosed with varicella. Which one of the following is not an appropriate comfort measure to include in this session? a. promote skin care including daily bathing and changing clothes. b. Use Caladryl lotion on rash to decrease itching. c. Use hot bath water to promote skin rash healing. d. Keep nails short and smooth to decrease chances of infection from scratching

c. Use hot bath water to promote skin rash healing

Which one of the following does the nurse recognize as contraindicated in providing comfort measures to children with communicable diseases? a. Use of acetaminophen or ibuprofen for control of an elevated temperature in children with varicella. b. Use of diphenhydramine or hydroxyzine for itching. c. Use of aspirin to control elevated temperature and/or symptoms with varicella. d. Use of lozenges and saline rinses in an 8-year-old with sore throat.

c. Use of aspirin to control elevated temperature and/or symptoms with varicella.

Which one of the following techniques would be considered an example of the most therapeutic communication to use with the bereaved family? a. Cheerfulness b. Interpretation c. Validating loss d. Reassurance

c. Validating loss

which one of the following children with ALL has the best prognosis: a. a 11 year old girl with leukocyte count of 30,000 b. a 6 year old boy with a leukocyte count of 120,000 c. a 6 year old boy with a leukocyte count of 30,000 d. a 1 year old girl with a leukocyte count of 120,000

c. a 6 year old boy with a leukocyte count of 30,000

i children 1 to 19 years of age, the most frequent causes of death are a. accidents/trauma, infectious illness, and suicide b. injuries/trauma, cancer, and congenital anomalies c. accidents/trauma, homicide, cancer, and suicide d. prematurity, congenital birth defects, and infectious illness

c. accidents/trauma, homicide, cancer, and suicide

the nurse is planning to advise a school aged child's parents about appropriate physical activity for their child. which fact does the nurse include? a. school aged children have the same stamina and control as 15 y/o b. children are prepared for participation in strenuous competitive athletics c. activities that promote coordination in the child include running and skipping rope d. most children need continued encouragement to engage in physical activity

c. activities that promote coordination in the child include running and skipping rope

delaying vaccinations is usually recommended in the immunosuppressed child because the immune response is likely to be suboptimal, however, it is considered safe to administer: a. any vaccines b. any live attenuated vaccines c. any inactivated vaccines d. the varicella vaccine

c. any inactivated vaccines

ms. jones is a single mother caring for her 10 year old son james. at an office appointment for james, his mother asks the nurse how to prevent her son from becoming involved in gang violence. the best response is which one of the following? a. try to be more of a pal to jason so that he won't seek outside approval b. relax restrictions on james. he needs to increase his independence, and this will show that you trust him c. become aware of any gang related activities in your community and become acquainted with your son's friends d. don't allow james to join any boys only group

c. become aware of any gang related activities in your community and become acquainted with your son's friends

children who are identified as having a difficult or easily distracted temperament; a. rarely pose a problem b. usually exhibit discomfort when introduced to new situations c. benefit from practice sessions before an event d. should not be told when to stop activities since this can trigger a reaction event

c. benefit from practice sessions before an event

the first action parents should be taught to initiate in a poisoning is to a. induce vomiting b. take the child to the family physician's office or ED c. call the poison control center d. follow the instructions on the label of the product

c. call the poison control center

of the following assessment findings in the postoperative care of a child who had surgery to remove a brain tumor, the one with the most serious implication is: a. a comatose child b. serosanguineous drainage on the dressing c. colorless drainage on the dressing d. decreased muscle strength

c. colorless drainage on the dressing

early appearance of secondary sex characteristics of girl during preadolescence may be associated with which of the following feelings? a. satisfaction with physical appearance and higher self esteem b. increase in self confidence and more outgoing c. dissatisfaction with physical appearance and lower self esteem d. increased substance use and reckless vehicle use

c. dissatisfaction with physical appearance and lower self esteem

the nurse while conducting a home visit, finds that the mother of 4 yr old Nathan is using a mercury thermometer to take his axillary temp. which one of the following is the best intervention for the nurse at this time? a. tell mother to stop using mercury thermometers because if the mercury is ingested it can cause mercury toxicity b. explain to the mother that mercury poisoning can cause acrodynia c. explain to mother that mercury thermometers are no longer recommended and if broken, inhaled vapors can cause toxicity d. reassure mother that as long as mercury thermometer is not broken, its ok to continue use

c. explain to mother that mercury thermometers are no longer recommended and if broken, inhaled vapors can cause toxicity

Human papillomavirus (HPV) vaccine is recommended for all of the following populations except: a. female adolescents to prevent HPV-related cervical cancer. b. boys and men (9-26 years) to reduce the likelihood of genital warts. c. female preadolescents ages 7 to 9 years. d. female adolescents who are not sexually active.

c. female preadolescents ages 7 to 9 years.

dillon is a 6 yr old starting in a new neighborhood school. on the first day of school he complains of a headache and tearfully tells his mother he does not want to go school. dillon's mother takes him to school, and the nurse is consulted. the nurse recognizes that dillon is slow to warm up to others and suggests which one of the following? a. put dillon in the classroom with the other children and leave him alone b. insist that dillon join and lead the class song c. include dillon in activities without assigning him tasks until he willingly participates in activities d. send dillon home with his mom because he has a headache

c. include dillon in activities without assigning him tasks until he willingly participates in activities

activated charcoal a. is odorless, tasteless, delivered w fewer complications via gastric lavage b. stimulates gastric mucosa c. is often mixed w diet soda and served through a straw from an opaque contained d. has a bitter taste

c. is often mixed w diet soda and served through a straw from an opaque contained

the nurse is planning an educational session for a group of 9yr olds and their parents aimed at decreasing injuries a. safety rules to prevent burns when dealing with fire b. safety rules to prevent poisonings when dealing with toxic substances c. pedestrian safety rules and skill training programs to prevent motor vehicle accidents d. safety rules for the use of all terrain vehicles encourageing their use only with supervision

c. pedestrian safety rules and skill training programs to prevent motor vehicle accidents

children who have profound anemia during induction therapy should: a. strictly limit their activities b. regulate their own activity with adult supervision c. receive blood transfusions until the Hgb level approaches 10 d. receive chemotherapy until the hbg level approaches 10

c. receive blood transfusions until the Hgb level approaches 10

treatment for wilms tumor is based on clinical stage and histologic pattern and includes: a. chemo and radiation b. radiation alone c. surgery, chemo, and radiation d. surgery alone

c. surgery, chemo, and radiation

jim a 7 year old boy has just been diagnosed with childhood obesity. which one of the following does the nurse recognize as being most likely to contribute to jim's obesity? a. caloric needs are diminished in relation to body size during middle childhood b. when jim entered school he developed an eating style that was increasingly independent from parental influence c. the availability of inexpensive high caloric foods and the tendency toward sedentary activities d. jim is a latchkey child and makes his own breakfast and lunch

c. the availability of inexpensive high caloric foods and the tendency toward sedentary activities

a factor that most influences the amount and manner of discipline and limit setting imposed on school age children is: a. parents age b. parents education c. the childs response to rewards and punishments d. the parents ability to communicate with the school system

c. the childs response to rewards and punishments

Pediatric nurses report that a common obstacle to the provision of good palliative care is: a. parents who do not understand the concept of palliative care. b. communication with practitioners is inadequate. c. uncertainty about the goals of care. d. the cost of providing palliative care is excessive.

c. uncertainty about the goals of care.

a nurse is about to present facts to parents about the ossible death of their chld. in this case, which of the following techniques would be the most effective in promoting communication? a. acknowledging denial in the parents whenever it occurs b. using only medical terms for all explanations c. using body language to communicate caring d. recognizing feelings and reactions but not acknowledging them

c. using body language to communicate caring

on routine physical examination, 2yr old zach is found to have an elevated blood lead level. the most likely cause for this finding is a. zach is allowed to play in the local sandbox at the park b. zach lives in a house build after 1980 c. zach is fed from pottery that family makes themselves d. zach's father is an artist and works at home

c. zach is fed from pottery that family makes themselves

3 chelating agents

calcium disodium edetate (EDTA), british antilewisite, succimer (DMSA) when is british antilewisite used in conjunction with calcium EDTA with high lead levels or encephalopathy possibility of what with chelation rebound BAL is contraindicated in who children with peanut allergies or hepatic insufficiency BAL is administered how deep IM injection, repeated dose over several days to lessen pain from EDTA - anestetic procaine injected, rotation sites, records of I/O, urinalisis chelation therapy removes lead from circulating blood and some tissues, but doesn't stop effects but need multiple treatments chelation therapy need adequate hydration due to filtration through kidneys

what is the patho of pinworms

can be found in the dirt (easily spread) spreads to hands-- children eat with their hands- mouth- GI tract- booty eggs can be i gested or inhaled what is the education for a pt with pinworms good handwashing short nails showers are better than baths tape test 3 days collect in the morning before child washes drug - pyrantel pamoate and albendazole easily transmitted so treat all household members dose should be repeated in 2 weeks wash all clothes in hot water and linins and vaccuum the house hand washing after toileting keep fingernails short dress in one piecer for sleep daily showering not tub

hemorrhagic cystits:

caused by chemical irritation to the bladder prevention of hemorrhagic cystitis: -increase hydration -void immediately when feeling urge -administer drugs early in day -administer mesna Mesna a drug that inhibits urotoxicity of cyclophosphamide

Spinal cord suppression:

children with primary CNS tumors may develop compression or those presence of solid tumors such as neuroblastoma or rhabdomyosarcoma Initial sx of spinal cord suppression: -back pain -sensation change -extremity weakness -loss of bowel and bladder function -respiratory insufficiency Diagnosing spinal cord suppression: MRI (gold standard) Tx of spinal cord suppression: -high doses of steroids to decrease edema -emergency radiation to decrease tumor size

leukocoria

classic sign of retinoblastoma; cat's eye reflex

brain tumor

clinical manifestations - headake uon akening, vomiting not related to feeding, increased ICP diagnosis - clinicl signs and diagnostic imaging (MRI) treatment - surgery, radiation, chemo report dilated unequal pupils - ICP posroperative care - positioning ( no trachtenberg), prevent hypertherpmia with cooling blanket, monitor drainage, regularly pulmonary assessment, test reflexes asap, no positionin againt operative side, head above heart, bring help to position trhe child, npo 24 hrs, stop fluifs if vomits mouth care headache - quite dim environment, restricting visitore, preventing jarring, preventing increased ICP, opioids prevent constipation, care of depressed blink reflex promote retur to optimum functioning

cognitive and behavioral effects of lead poisoning

cognitive and behavioral: aggression, hyperactivity, impulsivity, delinquency, disinterest, and withdrawal long term effects of lead poisoning developmental delays, lower IQ, reading skill deficits, visual spatial problems, visual motor problems, learning disabilities, lower academic success chronic lead poisoning may effect physical growth and reproductive ability worse high dose exposure lead poisoning effects encephalopathy, cognitive impairment, paralysis, blindness, seizures, coma, death

What condition is also known as paroxysmal abdominal pain? Colic Cow's milk allergy Positional plagiocephaly Sudden infant death syndrome (SIDS)

colic Rationale Paroxysmal abdominal pain is also known as colic. Cow's milk allergy is a multifaceted disorder representing adverse systemic and local gastrointestinal reactions to cow's milk protein. Positional plagiocephaly is an oblique or asymmetric head resulting from cranial molding during infancy. Sudden infant death syndrome, or SIDS, is defined as the sudden death of an infant younger than 1 year of age that remains unexplained after a complete postmortem examination. p. 382

wills tumor therapeutic management

combined treatment of surgery (partial to total nephrectomy) and chemo with or without radiation

The essential components of a comprehensive evaluation for childhood cancer include

complete history and review of symptoms, physical examination, laboratory tests, diagnostic imaging, diagnostic procedures (lumbar puncture [LP], bone marrow aspirate, and biopsy), and surgical pathology.

most infant death results from

congenital malformation low birth weight SIDS matrnal complication accidents cord and placenta complications newborn respiratory distress bacterial sepcic neonatal hemorrhage circulatory system disease

Intestinal parasitic diseases

constitute the most common infections in the world. Giardiasis and enterobiasis are the most widespread parasitic infections among children in the United States. Child care centers and institutions providing care for persons with developmental disabilities are common sites for urban giardiasis. The most important nursing consideration is prevention of giardiasis and education of parents, child care center staff, and those who assume the daily care of small children. Attention to meticulous sanitary practices, especially during diaper changes, is essential.

contraindication and precautions for vaccines

contraindications (increased risk for serious adverse reaction) - severe fibrile illness, but not common cold - immunocompromised, altered immune system, presence of recently acquired passive immunity - postpone 3 months - known allergic reaction - pregnancy is contraindication to MMR - breastfeeding is not contraindication to any vaccine NOT CONTRAINDICATIONS - seizures -adverse event following vaccination - penicillin allergy - allergies to duck meat or duck feather - family history of SIDS

Nurses may offer the following assistance in education about death

counseling parents about children's age-specific understanding of death, encouraging parents to help children become familiar and comfortable with loss, taking part in organized death education in schools, and serving as a resource to answer families' and children's questions.

Steroid effects

cushing syndrome appearance: -round face -puffiness -avoid salt and high sodium foods -mood changes from euphoria to depression

hodkin's disease increases in incidence in children between the ages of: a. 1-5 b. 5-10 c. 11-14 d. 15-19

d. 15-19

The perceived inability to use their parents for emotional support is one of the greatest threats to seriously ill children of which age-group? a. Toddlers b. Preschoolers c. School-age children d. Adolescents

d. Adolescents

Measles immunization includes which of the following? a. Given at 12 to 15 months of age with a second dose given at age 4 to 6 years of age and revaccination at 11 to 12 years of age. b. Child vaccinated before 12 months of age should receive two additional doses beginning at 12 to 15 months and separated by at least 4 weeks. c. Revaccination of all individuals born after 1956 who have not received two doses of measles vaccine after 12 months of age. d. All of the above.

d. All of the above.

The nurse is instructing parents on the test-tape diagnostic for enterobiasis. Which one of the following is included in the explanation? a. Use a flashlight to inspect the anal area while the child sleeps. b. Perform the test 2 days after the child receives the first dose of anti parasitic medication. c. Test all members of the family at the same time using frosted tape. d. Collect the tape in the morning before the child has a bowel movement or bath.

d. Collect the tape in the morning before the child has a bowel movement or bath.

Which one of the following interventions is most important for the dying child? a. Emotional support b. Preparing parents to deal with fears c. Relief from pain d. Control of pain

d. Control of pain

Assessment of which of the following is not helpful in identifying potentially communicable diseases? a. Prodromal symptoms. b. Immunization history. c. Past medical history. d. Family history.

d. Family history.

Joey, a 14-year-old adolescent, has recently relocated from Mexico to the US. While helping his dad do farm work, Joey suffered a laceration in the horse barn. Review of Joey's immunization record from Mexico shows that Joey has had 2 previous tetanus and diphtheria vaccine immunization shots, the last one being at the age of 6 years. Which of the following would the nurse in the ER expect to give at this time? a. Tdap. b. DTaP. c. Td booster. d. Tetanus immunization (TIG) human and Tdap.

d. Tetanus immunization (TIG) human and Tdap.

Which of the following is not an influenza vaccine recommendation? a. The vaccine is administered in early fall before the flu season begins and is repeated yearly. b. trivalent, a inactive influenza vaccine is approved for vaccine use in children ages 6 months and older. c. Children with severe egg allergy history should not routinely receive the influenza vaccine. d. The live attenuated influenza vaccine (LAIV) form is recommended for children 2 to 4 years of age with a history of wheezing or asthma.

d. The live attenuated influenza vaccine (LAIV) form is recommended for children 2 to 4 years of age with a history of wheezing or asthma.

In scheduling immunizations, the nurse knows which of the following is correct? a. The beginning primary immunization for infants begins at 2 months of age. b. Children who are born preterm receive only half the normal dose of each vaccine followed by the second half of the vaccine 2 weeks later. c. Children who begin primary immunization at the recommended age but fail to receive all the doses by suggested age need to begin the series again. d. When there is doubt that the child will return for follow-up immunizations according to the optimum schedule, HBV vaccine (HepB), DTaP, IPV (poliovirus vaccine), MMR, varicella, and Hib vaccines can be administered simultaneously at separate injection sites.

d. When there is doubt that the child will return for follow-up immunizations according to the optimum schedule, HBV vaccine (HepB), DTaP, IPV (poliovirus vaccine), MMR, varicella, and Hib vaccines can be administered simultaneously at separate injection sites.

ALL> strategies to have kid to eat when she is nauseated after the chemo. strategies the nurse might suggest include: a. gargling with viscout lidocaine to releive pain b. permitting only nutritious snacks c. starting cory on a regimine of total parenteral nutrition d. administerng an antiemetic drug

d. administerng an antiemetic drug

during the school age years, children learn valuable lessons from age mates. how is this accomplished? a. the child learns to appreciate the varied points of view within the peer group b. the child becomes sensitive to the social norms and pressures of the group c. the child interactions among peers leads to the formation of intimate friendships between same sex peers d. all of the above

d. all of the above

if a child vomits in the postoperative period following surgery for a brain tumor, it may predispose the child to: a. incisional rupture b. increased ICP c. aspiration d. all of the above

d. all of the above

important prognostic factors in determining long term survival for children with ALL include: a.initial white blood cell count b. age and gender c. immunologic subtype, and cytogenetics d. all of the above

d. all of the above

reduction of poisonings in children and infants can be accomplished by a. use of child-resistant containers b. educating parents and grandparents to place products out of reach of small children c. educating parents to relocate plants out of reach of infants, toddlers, and small children d. all of the above

d. all of the above

Interventions to help the family prepare for the care of a terminally ill child include: a. educating the family about complications of overfeeding or overhydration. b. providing a supply of medications to alleviate discomfort. c. encouraging fun and memorable activities. d. all of the above.

d. all of the above.

The sibling of a dying child may feel: a. displaced. b. isolated. c. resentful. d. all of the above

d. all of the above.

Methods to support grieving families at the time of death and in the grieving period after death include: a. encouraging family members to avoid upsetting each other by keeping their feelings to themselves. b. consoling with phrases such as "I know how you feel." c. emphasizing that the painful grieving usually lasts less than a year. d. allowing the family time to stay with the child after the death.

d. allowing the family time to stay with the child after the death.

in obtaining a hx pertaining to an incident of abuse, the nurse should a. expect child to betray the parents by admitting to the abuse received b. expect the child to defend the parents out of a sense of loyalty c. recognize that the child will have a sense of relief after telling someone about the abuse d. avoid biasing the child's retelling of the events

d. avoid biasing the child's retelling of the events

which of the following is most characteristic of the relationship between school aged children and their families? a. children desire to spend equal time with family and peers b. children are prepared to reject parental controls c. the group replaces the family as the primary influence in setting standards of behavior and roles d. children need and want restrictions placed on their behavior by the family

d. children need and want restrictions placed on their behavior by the family

When communicating with dying children, the nurse should remember that: a. older children tend to be concrete thinkers. b. when children can recite facts, they understand the implications of those facts. c. if children's questions direct the conversation, the assessment will be incomplete. d. games, art, and play provide a good means of expression.

d. games, art, and play provide a good means of expression

which of the following descriptions of school aged children is most closely linked to eriksons theory? a. children experience relationships with same sex peers b. there is an overlapping of developmental characteristics between childhood and adolescence c. temperamental traits from infancy continue to influence behavior d. interests expand and children, with a growing sense of independence, engage in tasks that can be carried through to completion

d. interests expand and children, with a growing sense of independence, engage in tasks that can be carried through to completion

therapeutic interventions for lead poisoning do NOT include a. removal of source of lead b. improving nutrition c. using chelation therapy d. intravenous administration of dimercaprol

d. intravenous administration of dimercaprol

the resolution of grief usually a. occurs in sequential phases b. is completed in about 2 years c. is completed in about 3 years d. is a timeless process that is never completed

d. is a timeless process that is never completed

the reed sternberg cell is a significant finding because it: a. is absent in all diseases other than hodgkin disease b. is absent in all disease other than lymphomas c. eliminates the need for laparotomy to determine the stage d. is absent in all lymphomas other than hodgkin disease

d. is absent in all lymphomas other than hodgkin disease

rhabdomyosarcoma is a a. malignant bone neoplasm b. nonmalignant soft tissue tumor c. nonmalignant solid tumor d. malignant solid tumor of the soft tissue

d. malignant solid tumor of the soft tissue

the nurse has conducted an assessment of the sleep pattern of janie and found that janie delays going to bed each night. which of the following interventions does the nurse recognize as LEAST helpful a. consistent bedtime ritual, like reading a book before bedtime b. not allowing child to stay up past reasonable hour c. keeping a light on in the room d. taking the child into the parent's bed

d. taking the child into the parent's bed

In regard to whether a child should attend the funeral of a loved one, the nurse should consider: a. the child's age. b. that it will be a frightening experience. c. his or her responsibility to protect the child from distressing events. d. that attending the funeral may be beneficial to the child.

d. that attending the funeral may be beneficial to the child.

toward the end of middle childhood, the discrepancies in growth and maturation between boys and girls becomes apparent. which statement is true? a. there is usually a 4 year difference between the onset of puberty in boys and girls b. the universal age at which the first physiologic signs appear in girls is 9y/o c. the onset of puberty at age 7 is considered normal d. the average age of puberty is 12 years in girls and 14 in boys

d. the average age of puberty is 12 years in girls and 14 in boys

To prevent the spread of contamination from one patient to another after procedures, the most important strategy the nurse can use is to: a. follow disease-specific infection control guidelines. b. wear vinyl gloves. c. avoid wearing nail polish. d. wash the hands routinely after each patient and after removing gloves.

d. wash the hands routinely after each patient and after removing gloves.

the most common signs and symptoms of brain tumor: a. weight gain and hemiparesis b. may be undetectable while the sutures are open c. will be demonstrated as vomiting after feedings and weight loss d. will be demonstrated as headache and vomiting not related to feeding

d. will be demonstrated as headache and vomiting not related to feeding

consequences r/t inadequate sleep

daytime tiredness behavior changes hyperactivity difficulty concentrating impaired learning poor impulse/emotion control strain on family relationship

consequences of inadequate sleep

daytime tiredness, behavior changes, hyperactivity, difficulty concentrating, impaired learning ability, poor control of emotions, strain on family relationships

contact precaution

direct or indirect skin to skin and physical transfer of microorganisms type of precautions using gloves, gown, surgical masks b/c of multi-drug resistant bacteria, C. diff, E. coli, Shigella, hep A, rotavirus, skin infections that are highly contagious, viral or hemorrhagic conjunctivitis, viral hemorrhagic infections (Ebola, Lassa, Marburg) stethoscope most common piece of medical equipment that is potent source of harmful microorganisms & nosocomial infections

brain tumors

directly related to their anatomic location and size and to some extent the child's age. For instance, in infants whose sutures are still open, a bulging fontanel indicates hydrocephalus. Clinical manifestations may be nonspecific in older children. However, the most common symptoms of infratentorial brain tumors are headache, especially on awakening, and vomiting that is not related to feeding. Diagnosis of a brain tumor is based on presenting clinical signs and diagnostic imaging. Treatment may involve the use of surgery, radiotherapy, and chemotherapy. All three may or may not be used, depending on the type of tumor.

Biologic development toddler

during the toddler years is characterized by the acquisition of fine and gross motor skills that allow children to master a wide variety of activities. In general, adult height is about twice the child's height at 2 years of age. Accurate measurement of height and weight during the toddler years should reveal a steady growth curve that is steplike rather than linear (straight), which is characteristic of the growth spurts during the early childhood years. Although most of the physiologic systems are mature by the end of toddlerhood, development of certain areas of the brain is still occurring, allowing for greater intellectual capacity. Locomotion is the major gross motor skill acquired during toddlerhood followed by increased eye-hand coordination. By 12 to 13 months of age, most toddlers walk alone. They are able to jump using both feet and climb stairs by age 2 to 2½ years.

diagnostic evaluation lymphoma

examination, symtoms, masses, lymph noses, spleen, liver CBCbiochemistry profile, erythrocyte sedimentation rate, c-reactive protein serum ferritin lymph node biopsy

neuroblastoma

extracranial childhood solid tumor median age 19 m may be hereditary most common site - abdomem - firm nontende irirregular mass, pain, discomfort, vomiting, anorexia, respiratory compromise, compression of kidneys, urinary frequency or retention, difficulty breathing, seizures, paraplegia, anemia, thrombocytopenia, neutropenia, impaired vision, periorbital ecchymosis diagnosis - ct, mri, examinatio bone marrow for metastasis, biopsy urinary catecholamines silent tumor demonstrate spontaneous regression prognosis depends on age, site, tumor histology, regional lymph node involvement, response to treatment, biologic features treatment - surgery to remove tumor, sometimes radiation postoperatively to shrink it chemo

chemothrapy adverse reaction

extravasation anaphylaxis ( l-aspariginase, bleomycin, ciplastin, etoposide) - urticaria, angioedema, flushing, rashes, difficulty breathing, hypotension, nausea and vomiting observe child for 1 h and keep emergency equipment nearby and emergency drugs - oxygen, epi, antihistamine, vasopressors nursing responsibilities prevention , recognition, preparation for serious reaction if reaction suspected - discontinue, flush and maintain IV with saline, monitor vitals and subsequent responses

emotional neglec

failure to meet the child's needs for affection

child neglect

failure to provide for the child's basic needs and adequate level of care

leading cause of nonfatal injuries in infants

falls

Non-Hodgkin's Lymphoma

favorable prognosis - young age, no midiastasis, low tumor burden, good response to therapy diagnosis - history, examination, blood chemistry, lp, ct, pet, mri, bone marrow, biopsy treated - chemo

iron toxicity

how many stages present in iron toxicity 5 stage 1 of iron toxicity within 6 hours: vomiting, hematemesis, diarrhea, hematochezia (bloody poo), abd pain, tachypnea, tachycardia, hypotension, coma stage 2 of iron toxicity latent period, up to 24 hrs of apparent improvement stage 3 of iron toxicity 12-24 hours: metabolic acidosis, fever, hyperglycemia, bleeding, seizures, shock, death may occur stage 4 iron toxicity 2-5 days: jaundice, liver failure, hypoglycemia, coma stage 5 iron toxicity 2-5 weeks: pyloric stenosis or duodenal obstruction may occur secondary to scarring

transplantation of hematopoetic blood forming stem cells

from bone marrow or cord blood allogeneic - from family ammbers autologous - stored from patient ae given back by IV new cells begin to produce functioning nonmalignant blood cells in order to do this procedure: - radiation therapy and or high dose of chemo to rid the body of malignant cells and to supress immune system to prevent rejection

hydrocarbone poisoning clinical manifestations

gagging chocking coughing burning throat and stomach nausea vomiting lethargy weakness tachycardia cyanosis retractions grunting even small amounts cause bronchitis and pneumonia

physical examination/ cancer

general - orientation, general state of health skin - petechiae or ecchymosis, lesions or sores, presence of blood from gum or nose, color of skin HEENT - microcephaly, bulging fontanels, evidence of infection, proptosis, pupil discoloration, anisocoria, extraocular movement not intact, limited peripheral vision, nystagmus, leukocoria heart - murmur or thrill, peripheral pulses lungs - evidence of infection, rales of rhonchi, decreased breath sounds, dyspnea, tachycardia, tachypnea abdomen - hepatosplenpmegaly, mass, decreased bowel sounds, striae neurologic - altered consciousness, altered sensation, abnormal reflexes, abnormal cerebellar function, unstable gait, dysarthria, cranial nerve dificits lymphatic - enlarged lymph nodes

An apparent life-threatening event (ALTE)

generally refers to an event that is sudden and frightening to the observer in which the infant exhibits a combination of apnea, change in color (pallor, cyanosis, redness), change in muscle tone (usually hypotonia), choking, gagging, or coughing; the event usually involves a significant intervention and even cardiopulmonary resuscitation by the caregiver who witnesses the event. The treatment of the infant with an ALTE depends on the underlying condition but may involve continuous home monitoring of cardiorespiratory rhythms with a multichannel pneumogram and in some cases the use of a pharmacologic agent.

methods to pressure children into sexual activity

gifts or privileges -withholding privileges -"okay to do" -adults meet their needs of warmth or human contact when child is isolated emotionally or socially -"secret between us" -plays on child's fears

Lymphomas:

group of neoplastic diseases that arise from the lymphoids and hematopoietic systems 2 types of lymphomas: 1. Hodgkin 2. Non-Hodgkin Which type of lymphoma is more common in children? Non-hodgkin lymphoma Hodgkin disease originates: in lymphoid system and primarily involves lymph nodes Where can hodgkin disease move to? -spleen -liver -bone marrow -lungs -mediastinum treatment - chemo and irradiationnon

what are the parts of infection control

hand hygiene standard precautions transmission based precautions

sexual abuse

has increased in the past decade. Common forms are incest, molestation, rape, exhibitionism, child pornography, child prostitution, and pedophilia. Most sexual abuse is committed by men and by persons known to the child, with family members constituting up to two thirds of the perpetrators.

assessment of brain tumors

headache that is worse on awakening and improves dueing the day - vomiting that is unrelated to feeding or eating - ataxia - seizures - behavioral changes - clumsiness - awkward gait or difficulty walking - diplopia - facial weakness -monitor for ICP after craniotomy and contact HCP

vavvinations schedule

hep B Which immunizations do we give at birth? hep B, DTap, rota, HiB, IPV, PCV Which immunizations do we give at 2 months? DTap, rota, HiB, IPV, PCV Which immunizations do we give at 4 months? hep B, DTap, rota, HiB, IPV, PCV, influenza Which immunizations do we give at 6 months? MMR, hep A, DTap, HiB, PCV, varicella Which immunizations do we give at 12-15 months? varicella, DTap, IPV, MMR Which immunizations do we give at 4-6 yrs? Tdap, HPV, mening Which immunizations do we give at 11-12 yrs? mening Which immunizations do we give at 16-18 yrs?

Recommended routine childhood immunizations include those for

hepatitis A and B virus, diphtheria, tetanus, pertussis, polio, measles, mumps, rubella, varicella, pneumococcal disease, meningococcal disease, influenza, and Haemophilus influenzae type b.

NHL

heterogeneous, exhibiting a variety of morphologic, cytochemical, and immunologic features, not unlike the diversity seen in leukemia. Classification is based on the pattern of histologic presentation (1) lymphoblastic, (2) Burkitt or non-Burkitt, or (3) large cell. The present treatment protocols for NHL include an aggressive approach using irradiation and chemotherapy. Similar to leukemic therapy, the protocols include induction, consolidation, and maintenance phases, some with intrathecal chemotherapy.

sex SIDS

higher in boys

discussing death with children

honesty acurate info provide books about illness clear simple language open communication ask parents what they what kids to know take care of anxiety and fear (like being alone)

Common infectious disorders during early childhood include communicable diseases,

intestinal parasitic infections, conjunctivitis, and stomatitis.

Tumor Lysis Syndrome (TLS)

hyperuricemia - lead to acute renal failure ( allopurinol) hypocalcemia hyperphosphatemia hyperkalemia Risk factors for acute tumor lysis syndrome: -high WBC at dx -large tumor burden -sensitivity to chemo -high proliferative rate clinical symptoms -flank pain -lethargy -N/V -muscle cramps -pruritus -tetany -seizures Tx of acute tumor lysis syndrome: -serum chemistries and ph monitoring -I&Os -aggressive IV fluids

sexual abusers i. can be anyone but most often a male who is known by the victim ii. can hold full time jobs and are active in community affairs iii. are most often not family members iv. often spend time with the victims to gain their trust before initiating any sexual contact v. rarely abuse the younger child if the older child is not present a. ii, iii, iv, and v b. i, iii, and iv c. i, ii, iv, and v d. iii, iv, and v

i. can be anyone but most often a male who is known by the victim ii. can hold full time jobs and are active in community affairs iv. often spend time with the victims to gain their trust before initiating any sexual contact v. rarely abuse the younger child if the older child is not present c. i, ii, iv, and v

infection

if child has an absolute neutrophil count of less than 500 mm, they are at risk for: -overwhelming infection -general malaise -seizures -invasion of organisms producing secondary infections What's common with infection? fever What should you watch for if a child with cancer gets an infection? septic shock What specimens should be obtained if a child with cancer gets an infection? -blood -stool -urine - nasopharengeal cultures -chest x-ray can go to school when ANC above 500 wash hands check fever

the nurse working with the abused child and family i. should view the parent and child as victims ii. should teach the parents through demonstration and example rather than lecture iii. recognizes that parents need to correct abnormal behavior in the child -- "we told you not to go with strangers" iv. recognizes that the goal of the nurse child relationship is to provide a role model for the parents in helping them to relate positively to their child a. i, ii, iii, and iv b. ii, iii, and iv c. i and iii d. ii and iv

ii. should teach the parents through demonstration and example rather than lecture iv. recognizes that the goal of the nurse child relationship is to provide a role model for the parents in helping them to relate positively to their child d. ii and iv

psychosocial development of a toddler

include differentiating self from others, tolerating separation from parent, coping with delayed gratification, controlling bodily functions, acquiring socially acceptable behavior, communicating verbally, and interacting with others in a less egocentric manner. Achievement of toilet training is a major developmental milestone for toddlers. According to Erikson, the major developmental task of toddlerhood is acquiring a sense of autonomy while overcoming a sense of doubt and shame. Several characteristics, especially negativism and ritualism, are typical of toddlers in their quest for autonomy. In Piaget's sensorimotor and preoperational phases of development, a toddler experiments by incorporating the old learning of secondary circular reactions with new skills and applies this knowledge to new situations. There are the beginnings of rational judgment, an understanding of causal relationships, and discovery of objects as objects. Preoperational thought is characterized by egocentrism, centration, global organization of thought processes, animism, and irreversibility. Transitional objects, such as a favorite blanket or toy, provide security for toddlers, especially when they are separated from their parents, dealing with a new stress, or just fatigued.

toddler discipline

include reward, ignoring or extinction, and time-out. One of the major tasks of toddlerhood is toilet training. The child must be able to recognize the urge to let go and hold on and be able to communicate this sensation to the parent. Five markers that signal a child's readiness to toilet train are bladder readiness, bowel readiness, cognitive readiness, motor readiness, and psychologic readiness. According to some experts, physiologic and psychologic readiness are not complete until ages 22 to 30 months.

Parent characteristics that increase the risk for abuse

include young parents, single parents, families with little social support, low socioeconomic status, undereducation, low self-esteem, substance abuse, and lack of knowledge of parenting skills. Abuse in single-parent families is often inflicted by an adult friend or companion. Children more prone to abuse include preterm infants, disabled children, and children younger than 3 years of age.

altered nutrition

increase calories by use of: -whole fat foods -add butter -add sugar -add cheese -high calorie snacks (pb crackers or dried fruits) hight protein and high calorie food whole milk, tofu, full fat yogurt, ice cream Why anorexia? -physical effect of cancer -condition aversion to food from N/V during tx -a response to stress -result of depression -control mechanism when so much has been imposed on the child that is out of their control avoid to give favourite food during chemo, to avoid aversion from food

sleep disturbances are related to

increasing autonomy, negative sleep associations, nighttime fears, inconsistent bedtime routines, lack of limit setting, evening media use, exposure to violent media during the day

molestation

indicent liberties touching fondling

disseminated intravascular coagulation

infection that leads to sepsis from bacteria or fungus and lead to multiple complications including DIC treatment = identifying and treating underlying cause infusing heparin to minimize microthrombi cryoprecipitate to replace fibrogen

Stomatitis

inflammation of the oral mucosa, which may include the buccal (cheek) and labial (lip) mucosa, tongue, gingiva, palate, and floor of the mouth. It may be infectious or noninfectious and may be caused by local or systemic factors. In children, aphthous stomatitis and herpetic stomatitis are typically seen.

injuries in school age general

injury is high in boys death rate higher in girls

Palliative care

involves an interdisciplinary approach to the management of a child's life-threatening or life-limiting illness from diagnosis through death and focuses on preventing or relieving the child's symptoms and support of the child and family. Palliative care seeks to relieve the physical, emotional, social, and spiritual distress produced by life-limiting conditions, to assist in complex decision making, and to enhance the quality of life.

Promoting the family's adaptation to the day-to-day management of the child's condition

involves education about the child's condition, general health care, and developmental needs and about realistic goal setting. Emotional support and assessment of the family also play a pivotal role in adaptation. Encourage families to articulate their goals and approaches to managing their children's condition, which provides vital information on how to intervene more thoughtfully with families based on their treatment approach.

Treatment of leukemia

involves the use of intravenous and intrathecal chemotherapeutic agents. Radiation is sometimes used for resistant central nervous system disease or testicular relapse. Typically, leukemia treatment is divided into phases (1) induction, which achieves a complete remission or clinical disappearance of leukemic cells, (2) intensification, or consolidation, therapy, which further decreases the total tumor burden, and (3) maintenance, which consists of further chemotherapy to ensure the disease stays in remission.

Children's concept of death

is determined by their cognitive ability and their experience with life-threatening illness. Young children see death as temporary and reversible and mainly fear separation. School-age children view death as irreversible but not necessarily inevitable and may fear mutilation. Children beyond 9 to 10 years of age realize that death is irreversible, universal, and inevitable, but they may resist the thought of their own death. Toddlers' egocentricity and vague separation of fact from fantasy make death incomprehensible; they may still refer to a dead person as though the person exists. Because of their sense of precausality and self-power, preschoolers may believe that their thoughts actually cause another person's death. With their reasoning power and fear of the unknown, school-age children may feel intense guilt and responsibility for someone's death. Adolescents have difficulty accepting death because of their preoccupation with developing a sense of identity.

The primary nursing responsibility in care associated with sudden infant death i

is educating the family of newborns about the risks for SIDS, modeling appropriate behaviors in the hospital such as placing the infant in a supine sleep position, and providing emotional support to the family who has experienced a SIDS loss.

toddler nutrition

is important at this stage because eating habits established in toddlerhood tend to have lasting effects in subsequent years. At approximately 18 months of age, most toddlers manifest a decreased nutritional need with a decreased appetite, a phenomenon known as physiologic anorexia. The amount of food consumed by a toddler is not as important as the quality of the food consumed because growth is considerably less than during infancy. Conditions such as obesity and cardiovascular disease can be prevented by encouraging healthy eating habits in toddlers and their families.

Family management style

is the configuration formed by individual family members, the management behaviors they engage in with regard to the chronic condition, and the sociocultural context in which these behaviors occur. Five distinct family management styles have been identified: thriving, accommodating, enduring, struggling, and floundering.

What children are told about their serious illness

is the family's decision and can be based on several general principles regarding developmental age, previous knowledge, and honesty. Siblings have special needs, including the need for information, reassurance about their own health status, assurance that they are not responsible for the illness or death, and support for their own grieving process.

Language toddler

is the major cognitive achievement in toddlerhood. Although the number of words acquired—from about four at 1 year of age to approximately 300 at age 2 years—is notable, the ability to comprehend and understand speech is much greater than the number of words the child can say.

Acetaminophen poisoning

is the most common accidental drug poisoning among children and occurs primarily from acute overdose.

Neuroblastoma

is the most common extracranial solid tumor of childhood and the most common cancer diagnosed in infancy. The signs and symptoms of neuroblastoma depend on the location and stage of the disease. However, neuroblastoma is a "silent" tumor and in more than 70% of cases, diagnosis is made after metastasis occurs, with the first signs caused by involvement in the nonprimary site.

Rhabdomyosarcoma (rhabdo, striated)

is the most common soft tissue sarcoma in children. The initial signs and symptoms are related to the site of the tumor and compression of adjacent organs. All rhabdomyosarcomas are high-grade tumors with the potential for metastases. Therefore, multimodal therapy is recommended for all patients.

Chelation

is the term used for removing lead from circulating blood and, theoretically, some lead from organs and tissues.

A major goal in working with the family of a child with special needs

is to support the family's coping and help them function as best as they can throughout the child's life. Long-term, comprehensive, family-centered approaches extend beyond supporting the child and family during the crucial periods of diagnosis and hospitalization. Comprehensive care involves building parent-professional partnerships that can support a family's adaptation to the many changes that may be necessary in everyday life, defining expectations of and for the child, and providing a long-term perspective.

abusive head trauma symptoms

long term outcomes of AHT seizure disorders, visual impairments, developmental delays, hearing loss, cerebral palsy, motor impairments most often there are no signs of what with AHT external injuries presenting symptoms of AHT vomiting, irritability, poor feeding, and listlessness severe- posturing, seizures, decreased LOC, apnea, bradycardia, death

Gastric lavage

may be performed to empty the stomach of the toxic agent; however, this procedure is associated with serious complications (gastrointestinal perforation, hypoxia, aspiration), and it is no longer recommended in all cases of ingestion.

Special decisions at the time of dying and death

may involve hospital or hospice care, the child's right to die and do not resuscitate (DNR) orders, viewing of the body, organ and tissue donation, autopsy, and siblings' attendance at the funeral. Special needs of the family facing the unexpected death of a child include support while awaiting news of the child's status; a sensitive pronouncement of death; acknowledgment of feelings of denial, guilt, and anger; an opportunity to view the body; closure; and referrals for support.

Special decisions at the time of dying and death

may involve hospital or hospice care, the child's right to die, visualization of the body, tissue donation and autopsy, and siblings' attendance at the funeral. In dealing with stress related to a dying patient, the nurse can cope successfully through self-awareness, knowledge and practice, available support systems, maintenance of general good health, and focus on the positive rewards of involvement with dying children and their families.

Child maltreatment

may take the form of physical abuse or neglect, emotional abuse or neglect, or sexual abuse. Child neglect is the most common form of maltreatment Physical neglect involves the deprivation of necessities, such as food, clothing, shelter, supervision, medical care, and education. Emotional neglect generally refers to failure to meet the child's needs for affection, attention, and emotional nurturance. Parental, child, and environmental characteristics may predispose children to maltreatment. The deliberate infliction of physical injury on a child, usually by the child's caregiver, is termed physical abuse.

giardiasis treatment

metronizadole (flagyl) metallic taste, cause nausea, vomiting not recommended for kinds under 2 yo tinizadole nitazoxanide ( no bitter taste, but take with food) resistant to chlorine

Giardiasis

most common intestinal parasitic pathogen in the us intestinal parasitic disease that has increased among young children who attend day care centers typically b/c of hand-mouth activity & uncontrolled fecal activity disease prevented by always washing hands & fingernails w/ soap & water before eating & handling food, after toileting; avoiding placing fingers in mouth, biting nails; discouraging children from scratching bare anal area; drinking only treated water or bottled water (esp if camping); washing all raw fruits & veggies, food fallen on floor chief modes of transmission - person to person, food and animals. contaminated water ( lakes etc) is common source symptoms - abdominal cramps, bloating, diarrhea diagnosis - microscopic examination of stool and duodenal dluid (aspiration, biopsy, string test)

diphteria

nasopharyngitis obstructive laryngotracheitis upper airway obstruction vaginal, otix, conjuctival, cutaneous lesions vaccine Tdap beofre 11, or dtap before 7 yo boosters every 10 y 2 months, 4 months, 6 months, 12-15 months, 4-6 yrs When do we give DTap immunizations?

school age children

naturalistic physiologic explanations of death realize that it is irreversible fear reason of illness, communicability, functioning and process of dying

Which form of child maltreatment is considered the most common? Neglect Sexual abuse Physical abuse Emotional abuse

neglect Rationale Child neglect is the most common form of maltreatment and is generally defined as the failure of a parent or other person legally responsible for the child's welfare to provide for the child's basic needs and an adequate level of care. Sexual abuse is the employment, use, persuasion, inducement, enticement, or coercion of any child to engage in, or assist any other person to engage in, any sexually explicit conduct or any simulation of such conduct for the purpose of producing a visual depiction of such conduct; or the rape, molestation, prostitution, or other form of sexual exploitation of children or incest with children. Emotional abuse may take the form of rejecting, isolating, terrorizing, ignoring, corrupting, verbally assaulting, or overpressuring the child. The deliberate infliction of physical injury on a child, usually by the child's caregiver, is termed physical abuse. Physical abuse can include anything from bruises and fractures to brain damage.

meningococcla disease

neisseria meningitidis - leading cause of bacterial meningitis children and adolescents 11-12 - should receive single immunization of MCV4 and a booster at 16-18 yo contraindicated of allergy to diphtheria toxoid and ruer latex Men Hibrix protects against meningococcal groups A,C,Y,W-135 as well as Hib and given 2,4,6,12-15 mo

vaccinations reactors possible

neomycin eggs yeast (hepB) latex rubber gelatin preservative thimerosal containing ethyl mercury inactivated antigens side effects - tenderness, erythema, swelling, low-grade fever, behavioral changes (drowsiness, eating less, prolonged or unusual cry) epinephrine for severe reactions

postirradiation somnolence

neurologic syndrome that may develop 5-8 weeks after central nervous system irradiation; characterized by somnolence with or without fever anorexia, and nausea and vomiting; may be an early indicator of long-term neurologic sequelae after cranial irradiation

lead poisoning

neurologic system is of most concern when young children are exposed to lead, since the developing brain is very vulnerable mild to moderate lead poisoning can cause a number of cognitive and behavioral problems in young children, including aggression, hyperactivity, disinterest, and withdrawal lead based nonintact paint in structures built before 1978 remains a frequent source of lead poisoning in children if the venous blood value is below 5mg/dL of lead, the child is considered to have a safe blood lead value lead containing pottery or leaded dishes do contribute to lead poisoning because food does not absorb lead at cellular level, lead doesn't assists the regulating action of calcium primary nursing goal in lead exposure is to prevent the child's initial contact universal screening guidelines for blood lead level testing include all children 1-2yrs of age, between 3-6yrs who have not been previously screened mercury thermometers can cause toxicity if they are broken and vapors are inhaled chelation treatment works by the use of a chemical compound that combines with the metal for excretion a level of lead not harmful to the pregnant woman is also not harmful to the fetus (not true) children who are iron deficient absorb lead more readily than those with sufficient iron store high lead levels are most concern when they occur in children who are ages 12 or under (not true)

nightmare/night terror interventions

nightmare interventions accept dreams as real fear, sit with child, offer comfort, assurance, and sense of protection; avoid forcing child back to their own bed, consider professional counseling of recurrent nightmares unresponsive to above approaches sleep terror interventions observe child for a few minutes; without interfering, until child becomes calm or wakes fully; intervene only if necessary to protect child from injury, guide child back to bed; stress to parents that sleep terrors are normal, common, phenomenon in preschooler that requires relatively little intervention

comparison of nightmares and sleep terror

nightmare time of distress after dream is over, child wakes and cries or calls; not during nightmare self sleep terror time of distress during terror itself, as child screams and thrusts; afterward is calm co-sleeping cultural differences dealing with sleep problems nightmares time of occurrence in second half of night, when dreams are more intense sleep terrors time of occurrence usually 1-4 hours after falling asleep, when non-REM sleep is deepest child's behavior during nightmare crying in younger children, fright in all; behaviors persistent even though child is awake child behavior during during sleep terrors initially may sit up trash, or run n bizarre manner, with eyes bulging, heart racing, and profuse perspiring; may scream, cry, talk or moan; show apparent fright, anger, to obvious confusion, which diapers when child is fully awake responsive to others during nightmare is aware of and reassured by another's presence responsive to others during sleep terrors is not very aware of another's present, is not comforted, and may push every person away and scream and threats more if held or restrained return to sleep after nightmare may be considerably delayed because of persistent fear return to sleep after night terror usually rapidly; often difficult to keep child awake yes description of nightmare possible if old enough description of sleep terror no memory of a dream or of yelling or thrashing

Nausea and vomiting:

occurs shortly after administering chemo and as result of cranial or abdominal irradation -can be profound mild to moderate vomiting - phenothiazine, as well as promethazine, prochlorperazine, trimethobenzamide acute nausea and vomiting - metoclopromide ( but causes extrapyramidal effects, which can be prevented with benadryl) Drug of choice for chemo related nausea: ondansetron (Zofran) -no EPS -can be given ODT or IV -works quickly What else is being used to nausea and an appetite stimulant? synthetic cannabinoids Teaching for antiemetics: -give 30 minutes before chemo regularly x 24 post chemo -give chemo at night with antiemetic and sedative

assessing the family's adjustment to a child's chronic illness

ollowing should be noted: the availability of a support system, the family's perception of the event, their coping mechanisms, their concurrent stressors, and their response to the child. Nurses are positioned to help parents cope with their child's chronic illness or disability in many ways. Nurses must be attentive, offer humanistic support, solicit suggestions for care, facilitate communication, allow verbalization of feelings, and refer to volunteer and community agencies. Nurses can advocate for methods that foster opportunities for parent empowerment. Through ongoing contact with the child, the nurse (1) observes the child's reactions to chronic illness or disability, ability to function, and adaptive behaviors within the environment and with significant others, (2) explores the child's own understanding of his or her illness, and (3) provides support while the child learns to cope with his or her feelings.

Effective teaching for injury prevention / infant

optimally begins in infancy by helping parents understand their child's normal development. Because injuries are a major cause of death during infancy, parents should be alerted to aspiration of foreign objects, suffocation, falls, poisoning, burns, motor vehicle injuries, and bodily damage, as well as preventive actions needed to make the environment safe for infants. The three leading causes of accidental death in infants are suffocation, motor vehicle-related injuries, and drowning. A significant number of infants are injured or die from improper restraint within the vehicle, most often from riding on the lap of another occupant. For infants and toddlers up to 24 months of age, a rear-facing car seat in the back seat of the vehicle provides the best protection.

sources of lead

paint, solder, crystal, battery casings, fishing sinkers, curtain weight, and bullets ceramc ware, water, pottery, pewter, dyes, industrial factors, vinyl mgniblinds, playground equipment, collective tows, some imported tour or children's metal jewelry, artists paints, and pool cue chalk lead can be inhaled or ingested, placental transfer, normal hand to mouth beg=havir, presence of dust

sleep terror

partial awakening from stage 4 sleep characterized by loud screams/thrashes and extreme physiological arousal usually 1-4 hours after falling asleep, may not realize what happened

agents of injested poisons

pharmaceutecal ( analgesics, cough meds, topical preparations, antibiotics, vitamins, GI meds, hormons, antihistamins) - cosmetics and personal care products ( deo, make up, perfume, cologne, mouthwash) - homehold cleaning products ( bleaches, laundry pods, desinfectants) - foreign bodies, toys, -plants

pedophilia

preference for a prepubertal child by an adult as a means of achieving sexual ecxitement

osteosarcoma interventions

prepare for treatment communicate honestly prepare for prothstetic fitting if necessary assist the child in dealing with problems of self-image instruct the child and parents about potential development of phantom limb pain

predisposing factors

preterm, especially lowe birth weight multiple births low apgar score infant of native american or african american ethnicity CNS disturbances and raspiratory disorders recent history if illness ( viral) prone position use of soft bedding overheating cosleeping with adults siblings with SIDS prolonged Q-T interval or other cardiac arythmias stuffed animals in crib lower incidents in breastfeeding kids lower incidents in sleeping with pacifier infants

palliative care

prevents and relievinf child's symptoms support child and family principles: - child and family are the unit of care - inerdisciplinary teamfocus on pysical, emotional, social, spiritual needs goal of care revised based on child's condition

chemo

primary or adjunt therapy works by interfering with the function or production of nucleic acids, DNA< RNA combining drugs works the best: VAC - vincristine (neurotoxic) + doxorubicin (myelosuppresive) + cyclophosphamide (myelosuppresive) venous access devices safe and effective continuous ingusion using syringe pump vesicants - sclerosing agents from infiltration must be given through a free flowing IV line

ewing sarcoma

primitive neuroectodermal tumor of the bone arising in the marrow spaces of the bone rather than from the osseous tissues Ewing sarcoma occurs most often in: shaft of the long bones often in pelvis , femur, tibia, humerus, ulna, scapula, ribs, skulls Tx of Ewing sarcoma: -limb salvage procedure -radiation -chemo Nursing care for Ewing sarcoma: -preservation of the limb if possible -coping if amputation occurs treatment - localizedlesions

treatment for early adolescent sleep problems

provide consistent bedtime routine, ignore seeking behavior, do not allow child to climb in bed with parents, keep light on in room, transitional object, and leave water by bed

Prognosis of neuroblastoma

rognosis is poor because of the frequency of invasiveness of the tumor and because in most cases, a diagnosis is not made until after metastasis has occured The younger the child the better the survival rate

abusive head trauma

shaken baby syndrome, inflicted traumatic head injury, and neuroinflicted brain injury most common cause of abusive head trauma parent frustration with crying, parental stress or depression abusive head trauma causes intracranial bleeding (subdural and subarachnoid)

Superior vena cava syndrome:

space occupying lesions located in the chest, especially from Hodgkins lymphoma and Non-Hodgkins lymphoma will cause airway compromise and respiratory failure Sx of superior vena cava syndrome: -cyanosis of upper region -facial and upper extremity edema -distended neck and chest veins -dyspnea, wheezing, cough management: - airway protection alleviation of respiraory distress

A newborn is diagnosed with gastroesophageal reflux. What is the appropriate position for this newborn to be placed in after nursing to decrease the risk of sudden infant death syndrome? Prone Supine Side-lying Semi-Fowler position

supine Rationale The mother should place the infant in a supine position when the infant sleeps to decrease risk of sudden infant death syndrome (SIDS). Placing the infant in prone position when the infant sleeps may causes oropharyngeal obstruction and rebreathing of carbon dioxide, which are possible causes of SIDS. The side-lying position is not recommended for infants because infants placed in a side-lying position are found to have increased incidence of SIDS. A semi-Fowler position is when a patient is lying in a bed in supine position with the head lifted to approximately 30 degrees. A semi-Fowler position is used to provide maximum lung expansion if the infant has breathing difficulties, but it is not the recommended position when the infant sleeps.

The use of multimodal therapy

surgery, chemotherapy, and radiotherapy; enrollment of large numbers of children in cooperative group clinical trials or protocols; and improvements in supportive care have greatly increased the survival of children with cancer.

sexual abuser

the typical abuser is a male who is unknown to the victim (false) boys are less likely to report both intrafamilial and extrafamilial abuse than are girls many sexual offenders are active in the community and hold full time jobs incestuous relationships between stepfather and daughter is generally shorter and more often reported than the incestuous relationships between biologic father and daughter(false) sexual abuse by relatives who have a strong emotional bond with the victim is the least devastating to the child (false) the youngest daughter is typically the child in the incestuous relationship (false) children may not reveal the truth about the abuse because they fear their parents would not believe them

What are children tx with when they have hemorrhaging?

to prevent bleeding: - platelet concentrates -platelet rich plasma Internal hemorrhage does not occur until.. platelets fall below 20,000 mm Most common forms of bleeding: -epistaxis -gingival bleeding To control a nose bleed: -apply pressure at site without disturbing clot -don't shove things in nose -don't tilt head back With children that are at risk for hemorrhage, teach them: avoid activities that might cause injury or bleeding -biking -skateboarding -roller skating -skiing -climbing trees or play ground equipment -contact spor

droplet

type of precautions using surgical masks b/c of HiB (haemophilus influenzae), Neisseria meningitidis, diphtheria, mycoplasmal pneumonia, pertussis, pneumonic plague, streptococcal pharyngitis, scarlet fever, adenovirus, influenza, mumps, HPV, rubella type of precaution used if droplet >5 mm though coughing, sneezing, or talking and during siction and bronchoscopy travels 3 ft or less through air

prevent complications

varisella complication - body area innervated by a particular segment of the spinal cord, as well as secondary bacterial infection, dipegmintation, scarring. postherpetic neuralgia is uncommon children with hemolitic disease are at risk of aplastic anemia from fifth disease leading to disruption of production of red blood cells and thrombocytopenia and neuropenia prevention of complication - compliance with antibiotics vitamin A

abusive head trauma (shaken baby syndrome)

violent shaking of infant that can cause fatal intracranial trauma

Possible causes of Hodgkin's disease

viral infections Previous exposure to alkylating chemical node biopsy

stomatitis treatment

what are the nursing priorities of stomatitis Relief of pain prevent spread ensure hydration what are pain relief measures for stomatitis acetaminophen, ibuprofen, topical anesthetics, equal parts diphenhydramine and Maalox, anitviral agents for HSV acyclovir careful hand washing younger kids elbow retraints articles from mouth clean thoroughly

time of death SIDS

when sleeping higher in winter incisents

Retinoblastoma

which arises from the retina, is the most common intraocular malignancy of childhood. Retinoblastoma has few grossly obvious signs. Typically, the parents are the ones who first observe a whitish "glow" in the pupil, known as the cat's eye reflex, or leukocoria. Treatment of retinoblastoma is complex. Enucleation may be used to treat advanced disease with optic nerve invasion in which there is no hope for salvage of vision. Irradiation can be used when there is vitreous seeding. Chemotherapy has been used to decrease the tumor size to allow treatment with local therapies.


Conjuntos de estudio relacionados

Med-Surg: Ger:Ch 02: Cross-Cultural Caring and Aging

View Set

Chapter 2: Altered Cellular and Tissue Biology

View Set

Chapter 2: Nature of Insurance, Risk, Perils and Hazards

View Set

Evolve: Maternity - Women's Health/Disorders

View Set

Roblox Robux, Fortnight, Prodigy, ROBLOX JAILBREAK, bb, W, Bye bye, Hide and seek

View Set

Intro to business practice 2 Test 2

View Set

Practice Questions for Micro/macro - Final

View Set